You are on page 1of 169

MTB Allergy and Immunology

MTB Q and A

1. Anaphylaxis Definition: Immediate hypersensitivity 9. Most likely dx and tx:


- pt already sensitized to the antigen, IgE binds to mast cells, Allergic reaction that causes superficial swelling in layers of
release of granules (histamine, prostaglandins, and the skin: Urticaria
leukotrienes) - can be caused also my physical agents (pressure in
dermatographism, cold, vibration)
Defined by severity not the cause of the reaction
2. Common causes of anaphylaxis: 1. Antihistamines: hydroxyzine, diphenhydramine,
fexofenadine, loratidine, cetirizine, ranitidine

2. Leukotriene receptor antagonists: montelukast or zafirlukast


10. Most likely dx:
Recurrent episodes of
Watery eyes, sneezing, itchy nose, itchy eyes
Inflamed, boggy nasal mucosa
Pale or violaceous turbinates
Nasal polyps: Allergic Rhinitis
- IgE dependent trigger of mast cells
- clinical dx, skin test may show allergen-specific IgE levels
Insect bites and stings - nasal smear may show large number of eosinophils
Medications: PCN, phenytoin, lamotrigine, quinidine, rifampin, 11. Allergic rhinitis prevention: 1. Remove offending agent
sulfa 2. Intranasal corticosteroid sprays
Foods 3. Antihistamines: loratidine, clemastine, fexofenadine,
Latex very important cause of anaphylaxis in healthcare brompheniramine
3. Anaphylaxis presentation: Rash 4. Intranasal anticholinergic medications: ipratropium
Hypotension 5. Desensitization to allergens that can't be avoided
Tachycardia 12. Most likely dx:
Respiratory (SOB, wheezing, swelling of lips) Recurrent sinopulmonary infections in adults
Urticaria Frequent episodes of bronchitis, pneumonia, sinusitis, and
4. Initial Tx for Anaphylaxis: 1. Epinephrine otitis media
2. Diphenhydramine (H1-blocker) and ranitidine (H2-blocker)
3. Methylprednisolone or Hydrocortisone Giardiasis
4. Airway protection if needed (intubation or cricothyroidotomy) Spruelike intestinal malabsoprtion
5. Angioedema Definition: Sudden swelling in face, tongue, eyes, Increase in autoimmune diseases (pernicious anemia,
airway seronegative rheumatic diseases): Common Variable
Immunodeficiency
Can be from C1 esterase inhibitor deficiency, often idiopathic 13. B cells in Common variable immunodeficiency: Normal in
6. Most likely dx: number B cells, however poor in function. Decreased amount of
Sudden facial swelling and stridor effective immunoglobulins subtypes: IgG, IgM, and IgA
ABSENCE of pruritis and urticaria: Hereditary Angioedema 14. Common Variable Immunodeficiency INC risk of ____.:
LYMPHOMA
Does NOT respond to glucocorticoids 15. Diagnostic test for CVID: Ig levels decreased and there is a
7. Best initial test for hereditary angioedema: Decreased levels of decreased response to antigen stimulation of B cells
C2 and C4 as well as C1 esterase inhibitor deficiency - DEC in the output of B lymphocytes with a normal number of
8. Tx for hereditary angioedema: B cells as well as normal amounts of lymphoid tissue such as
nodes, adenoids, and tonsils
16. CVID Tx: Antibiotics for each infection as it develops

Chronic maintenance is with regular Infusions of IV


immunoglobulins
17. Most likely immunodeficiency:

Male children
1. Acute therapy tx with FFP or ecallantide (specific therapy for
Increased sinopulmonary infections.
angioedema)
Decrease or absence of tonsils, adenoids, lymph nodes, or
2. Long term management with androgens: danazol and
spleen.
stanozolol
T cells normal: X-linked (Bruton) Agammaglobulinemia
3. Ensure airway protection

Long term IVIG keeps children healthier


Page 1
18. Most likely dx: 27. Primary Immunodeficiency disorders:

Recurrent sinopulmonary infections (as early as 6m)


PCP, varicella, and Candida infections: SCID (both B and T
cell deficiencies evident)
19. SCID Tx: Treat w/antibiotics as infections arise

Bone marrow transplant can be curative


20. IgA Anaphylaxis: When blood is given from a patient who has
normal levels of IgA to a patient with IgA Deficiency
21. Most likely dx:

Sinopulmonary infections
Atopic disease
Anaphylaxis with blood transfusion
Spruelike condition with fat malabsorption
INC risk of vitiligo, thyroiditis, and RA: IgA Deficiency

Tx infections as they arise. IVIG injections can't be given in


sufficient quantity, trace amount of IgA in IVIG may provoke
anaphylaxis.

Only transfuse from IgA deficient donors or with washed blood.


22. Most likely dx and tx:
Recurrent skin infections with staphylococcus: Hyper IgE
Syndrome Syndrome

Prophylaxis antibiotics such as doxycycline or cephalexin are


used.
23. Most likely dx:
Immunodeficiency
Thrombocytopenia
Eczema

T lymphocytes markedly deficient in blood and lymph nodes:


Wiskott-Aldrich Syndrome

Bone marrow transplant is curative


24. Most likely dx:

Lymph nodes with purulent material leaking out


Apthous ulcers and inflammation of nares
Granulomas that may obstruct GI or urinary tract: Chronic
Granulomatous Disease (CGD)
25. Common infections with Chronic Granulomatous Disease
(CGD): Staphyloccous
Burkholderia
Nocardia
Aspergillus
26. Dx Criteria for Chronic Granulomatous Disease: Nitroblue
Tetrazolium detects decrease in respiratory burst that
produces hydrogen peroxide.
- decrease in NADPH oxidase, which generates superoxide

Page 2
MTB Q and A
MTB Cardiology

1. Common GI disorders associated w/ chest pain?: GERD 10. Characteristics of Ischemic pain?
Ulcer dz Duration
Cholelithiasis Provoking factors
Duodenitis Associated symptoms
Gastritis Quality
2. Risk factors for CAD?: DM Location
Smoking tobacco Alleviating factor
HTN Radiation: 1. Duration: Stable > 2 or < 10 min; ACS: > 10 to 30
Hyperlipidemia min
FHx premature CAD first-degree relatives 2. Provoking factors: Physical activity, cold, emotional stress
Age above 45 in men and above 55 in women 3. Associated symptoms: SOB, nausea, diaphoresis,
3. What is the worst risk factor and then most common risk dizziness, lightheadedness, fatigue
factor for CAD?: Worst - DM 4. Quality: Squeezing, tightness, heaviness, pressure, burning,
aching
Most common - HTN (BP > 140/90) 5. Location: Substernal
6. Alleviating factor: Rest
4. Define premature CAD: Male relative under 55
7. Radiation: Neck, lower jaw & teeth, arms, shoulders

Female relative under 65 11. If the case described "chest wall tenderness".
5. Marked elevation in ___ is by the far the most dangerous
What is the most likely dx?
portion of a lipid profile for a patient in terms of risk for
CAD?: elevated LDL
Most accurate test?: Costochondritis
6. ___ is acute myocardial damage most often occurring in
postmenopausal women immediately following an Physical exam
overwhelming, emotionally stressful event.: Tako-Tsubo
12. If the case described "radiation to back, unequal blood
cardiomyopathy
pressure btw arms".
- divorce, financial, earthquake, lightning strike, hypoglycemia
- "ballooning" and LV dyskinesis
What is the most likely dx?
- B-blockers + ACEi
7. Correcting which risk factor for CAD will result in the most Most accurate test?:
immediate benefit for the pt?
A. DM
B. Tobacco smoking
C. HTN
D. Hyperlipidemia: Smoking cessation greatest immediate
improvement in pt outcome for CAD
- w/in 1 year stopping risk CAD DEC by 50%, w/in 2 years risk
reduced by 90%
8. Ischemic pain is NOT described as?: Tender
Positional
Pleuritic (changes w/ respiration)
9. Ischemic pain is described as?: Dull or "sore"

Aortic dissection
Squeezing or pressure-like
CXR w/ widened mediastinum
Chest CT
MRI

TEE confirms the dx

Page 3
13. If the case described "Pain worse w/ lying flat, better w/ 17. If the case described "Sudden-onset SOB, tachy, hypoxia".
sitting up, young < 40".
What is the most likely dx?
What is the most likely dx?
Most accurate test?:
Most accurate test?:

Pericarditis

EKG w/ ST elevation everywhere, PR depression


PE
14. If the case described "Epigastric discomfort, pain better w/
eating". Spiral CT, V/Q scan
18. If the case described "Sharp, pleuritic pain, tracheal
What is the most likely dx?
deviation".

Most accurate test?: Duodenal ulcer disease


What is the most likely dx?

Endoscopy
Most accurate test?:
15. If the case described "Bad taste, cough, hoarseness".

What is the most likely dx?

Most accurate test?: GERD

Response to PPIs, aluminum hydroxide and magnesium


hydroxide, viscous lidocaine
16. If the case described "Cough, sputum, hemoptysis".

What is the most likely dx?

Most accurate test?:

Pneumothorax

CXR
19. The "best initial test" for all forms of chest pain is?: EKG
20. When to order Enzymes (CK-MB/Troponin?: 1. Office
(ambulatory clinic) chest pain for days to weeks : No
enzymes -> transfer to the ED

2. ED chest pain for minutes to hours: YES enzymes, after an


Pneumonia EKG is performed

CXR

Page 4
21. When is a stress (exercise tolerance) testing useful?: 31. When is holter monitoring indicated?: Mainly detects rhythm
disorders (a. fib, flutter, ectopy such as premature beats, vent.
tachy)
- ambulatory EKG monitor 24-hr period
- NOT detect ischemia
32. Medications that lower mortality in chronic angina (3).:
Aspirin
B-blockers
Nitroglycerin
33. Nitroglycerin formulation in chronic stable angina vs acute
coronary syndrome.: Chronic stable angina
Exercise tolerance testing (ETT) used to evaluate chest pain - Oral, Transdermal path
when the etiology is not clear and EKG is not diagnostic
Acute Coronary syndrome
ETT based on two factors: - Sublingual, Paste, IV
- You can read the EKG 34. All pt w/ acute coronary syndromes (ACS) should receive 2
- Pt can exercise (get HR about 85% of maximum) anti-platelet medications immediately upon arrival in the
22. How to quickly calculate maximum heart rate?: Max HR = 220 emergency room. The combo asprin and second agent such
minus the age of patient. as ?: Clopidogrel, prasugrel, ticagrelor (all in P2Y12 receptor on
23. The 2 best ways of detecting ischemia w/o the use of EKG the platelet)
are?: 1. Nuclear isotope uptake: thallium or sestamibi - use of 2 antiplatelet meds does not apply to chronic or stable
- abnormalities = DEC uptake CAD
2. ECHO detection of wall motion abnormalities 35. When angioplasty or stenting is planned, the answer is ___
- dyskinesis, akinesis, hypkinesis (drug) or ___ (drug) for antiplatelet therapy.: Ticagrelor or
24. Ischemia vs infarction on thallium uptake?: Ischemia gives Prasugrel
reversible wall motion or thallium uptake btw rest and exercise - restenosis of stenting is best prevented by these
36. Clopidogrel is used in?: - Combo w/ aspirin in all ACS
Infarction is irreversible or fixed - Aspirin intolerance such as allergy
25. What is the pt cannot exercise what is an alternative method - Recent angioplasty with stunting
INC myocardial O2 consumption?: 1. Persantine (dipyridamole) - Rarely associated with TTP
or adenosine in combo w/ use of nuclear isotopes such as 37. Prasugrel is dangerous in which pts and why?: 75 and older
thallium or sestamibi because of an INC risk of hemorrhagic stroke
- antiplatelet med used in angioplasty and stenting, all acute MI
2. Dobutamine in combo w/ use ECHO 38. Used to inhibit platelets in the rare patient who is intolerant
26. Dipyridamole may promote ___ in asthmatics and should be of BOTH aspirin and clopidogrel.: Ticlopidine (cannot use if
avoided.: Bronchospasm the reason for aspirin or clopidogrel intolerance is bleeding,
27. If pt has reversible ischemia on stress test what should be since it will also inhibit platelets)
done next?: Coronary angiography - causes neutropenia and TTP
- revascularization reversible sects can save the tissue, 39. What is an additional therapy for angina refractory or
prevent infarction resistant through other tx?: Ranolazine
28. ___ is the most accurate method of detecting CAD.: 40. Uses ACEi/ARBS in heart disease?: - Low EF/systolic
Angiography dysfunction (best mortality benefit)
29. Surgically correctable disease generally begins w/ at least - Regurgitant valvular disease
___ % stenosis: 70% 41. Most common adverse effect of ACEi?: COUGH
30. Chest pain diagnosis algorithm: - 7% pts
42. If pts on ACEi presents with hyperkalemia what is the tx?:
Switch ACEi to hydralazine and nitrates
- aldosterone excrete K+ from the distal tubule
- hydralazine direct acting arterial vasodilator, DEC afterload,
clear mortality benefit pts systolic dysfunction, use w/ nitrates
43. The National Cholesterol Education Program recommends
statins for those with CAD at any ___.: LDL level
44. What is the goal for LDL when pt on statin?: LDL <100 or < 70
(those w/ CAD and DM)

Page 5
45. What are some CAD equivalents (goal LDL below 100 and 57. Use CCBs in CAD only with?: - Severe asthma cannot use of
statins should be used to bring the LDL down if above 100)?: - B-blockers
PAD - Prinzmetal variant angina
- Carotid disease (not stroke) - Cocaine-induced chest pain (B-blockers contraindicated)
- Aortic disease (aortic artery) - Inability to control pain w/ maximum medical thearpy
- Stroke 58. Adverse effects of CCB: Edema
- DM Constipation (verapamil most often)
46. What is the most common adverse effect of statins?: Liver Heart block (rare)
dysfunction 59. Coronary artery bypass grafting (CABG) lowers mortality only
- elevation transaminases to a level where need to D/C meds in a few specific circumstances w/ very severe disease such
- All pts started on statins have AST, ALT measured as:: - Three vessels w/ at least 70% stenosis in each vessel
- NO recommendation to routinely test all pts for CPK levels in - Left main coronary artery occlusion
the absence of symptoms (rhabdomyolysis < 0.1% pts) - Two-vessel dz in pts w/ diabetes
47. Statins have an ___ effect on the endothelial lining of the - Persistent symptoms despite maximal medical tx
coronary arteries that gives a benefit that transcends simply 60. Internal mammary artery grafts vs saphenous vein grafts how
lowering LDL number.: Antioxidant long do they last before occluding?: Internal mammary artery
48. When is niacin used in combo statin?: Niacin can be added grafts last an average 10 years before they occlude
statin if full lipid control is not achieved with statins
- statins, exercise, and cessation of smoking tobacco will all Saphenous vein grafts remain patents for only 5 years
raise HDL, niacin raise HDL more
49. Niacin side effects?: - Glucose intolerance *half of vein grafts are patent at 10 years
- Elevation uric acid 61. ___ is the best therapy in acute coronary syndromes
- Uncomfortable "itchiness" from transient release of histamine especially those with ST segment elevation.: Percutaneous
50. Which drug can lower triglyceride levels more than statins?: coronary intervention (PCI) -> angioplasty
Gemfibrozil - does not provide clear mortality benefit in stable patients
- benefit lowering triglycerides alone has NOT proven to be as 62. T/F: Maximal medical therapy w/ aspirin, B-blockers,
useful as the straightforward mortality benefit of statins ACEi/ARBs, and statins has proven equal or even superior
51. Caution the use of fibrates (Gemfibrozil) with statins because benefit compared to PCI in stable CAD.: TRUE
increased?: risk of MYOSITIS 63. Acute coronary syndromes are associated with which heart
52. Cholestyramine SE?: Sig. interactions w/ other meds in the gut, sound?: S4 gallop, because of ischemia leading to
may block absorption noncompliance of the LV
- GI complaints (flatus, constipation) 64. INC in JVP on inhalation is indicative of?: Kussmal sign ->
53. Which lipid lowering agent have no clear benefit to the pt but constrictive pericarditis or restrictive cardiomyopathy
definitely lowers LDL?: • LDL levels are an imperfect marker of 65. Triphasic scratchy sound on auscultation.: Pericardial friction
benefit with cholesterol-lowering therapies rub
66. Acute coronary syndrome diagnostic algorithm:
• Ezetimibe: No better than placebo

• No Change in MI, stroke, or death


54. Lipid lowering medications and their adverse effects:

67. A displaced PMI is characteristic of?: LVH


Dilated cardiomyopathy
- could not occur w/ Acute coronary syndrome
55. T/F: None of the calcium channel blockers (Nifedipine, 68. ST elevation in leads II, III, avF. Anatomical location MI.:
nitrendipine, nicardipine, nimodipine) have been shown to Acute MI, inferior wall
lower mortality in CAD.: TRUE 69. PR interval > 200 milliseconds.: First degree AV block
- may INC mortality with CAD because of raising HR and INC - little pathologic potential, no tx when isolated
myocardial O2 consumption 70. ST elevation leads V2-V4. Location of infarction.: Anterior wall
56. Use which calcium channel blockers in CAD?: Verapamil + of the LV, acute MI
Diltiazem 71. Frequent PVCs should they be treated?: Should NOT be
- do not INC HR, used in those who cannot tolerate B-blockers treated, even when associated with an acute infection.
because of severe asthma - Tx of PVCs only worsens outcomes

Page 6
72. ST depression leads V1 and V2. Anatomical location of the 82. Complications of angioplasty (PCI)?: 1. Rupture of the
MI.: Posterior wall MI coronary artery on inflation of the ballon
- low mortality no tx 2. Restenosis (thrombosis) of the vessel after the angioplasty
73. A 70 y/o woman comes to ED with crushing substernal chest 3. Hematoma at the site of entry into the artery (femoral area
pain for last hour. EKG shows ST segment elevation in V2 to hematoma)
V4. Most appropriate next step? 83. What is the most important factor in DEC the risk of
A. CK-MB level restenosis of the coronary artery after angioplasty?:
B. Oxygen Placement of drug-eluting stent (paclitaxel, sirolimus)
C. NG - inhibit local T cell response reduce rate of restenosis
D. Aspirin 84. Rate of restenosis within 6 mo. of PCI (percutaneous
E. Thrombolytics: D. Aspirin lowers mortality w/ acute coronary coronary angiography aka angioplasty): No stenting: 30-40%
syndrome, important to administer it as rapidly as possible Bare metal stenting: 15-30%
74. A 70 y/o woman comes to ED with crushing substernal chest Drug-eluting stent: < 10%
pain for last hour. EKG shows ST segment elevation in V2 to 85. Absolute contraindications to thrombolytics?: 1. Major
V4. Pt was given aspirin. Most appropriate next step? bleeding into bowel (melena) or brain (any type of CNS
A. CK-MB level bleeding)
B. Oxygen 2. Recent surgery (< 2 weeks)
C. NG 3. Severe HTN (> 180/110)
D. Angioplasty 4. Nonhemorrhagic stroke (< 6 mo.)
E. Trops: D. Angioplasty is associated with greatest mortality 86. Acute coronary syndromes tx and benefits?:
benefit of all the steps
75. Diagnostic tests in acute coronary syndrome time of
abnormality and duration of abnormality.:

87. Man comes to the ED chest pain 1 hour, crushing no change


with resp or position. EKG shows ST segment depression V2
76. What can cause false positive troponin level?: Renal
and V4. Aspirin given. Next step in management?: Heparin
insufficiency, since troponin excreted through the kidney
prevent clot from forming coronary arteries and closing off the
- troponin cannot distinguish reinfarction occurring several days
coronary artery
after first event
88. When are Glycoprotein IIb/IIIa inhibitors (abxicimab,
77. When a pt has a new episode of pain w/in a few days of the
tirofiban, eptifibatide) used in acute coronary syndrome?: Pt
first cardiac event, the management is?: 1. Perform EKG to
who are to undergo angioplasty and stenting, best non-ST
detect new ST segment abnormalities
elevation MI
2. Check CK-MB levels
- no benefit in acute ST elevation infarction
- after 2 days should be lowered, if elevated several days later
89. Summary of tx differences between cardiac events.:
means new ischemic event
78. The most common cause of death in the first several days
after the MI is ___.: Ventricular arrhythmia (vent. tachy, v.
fib)
79. ST segment elevation MI best managed initially with ___.:
Aspirin, either orally or chewed
Clopidogrel (allergy aspirin), prasugrel and ticagtelor
80. Angioplasty (PCI) is superior to thrombolytics in terms of?: -
Survival and mortality benefit
- Fewer hemorrhagic complications
- Likelihood of developing complications of MI (less arrhythmias,
less CHF, fewer rupture of septum, free wall (tamponade) and 90. T/F: LMWH is superior to unfractioned heparin in terms of
papillary muscle rupture (valve rupture) mortality benefit.: TRUE
81. The standard of care for angioplasty (PCI) expected to be
performed w/in ___ of the patient arriving in the ED with
chest pain.: 90 minutes

Page 7
91. In non-ST elevation ACS, when all meds have been given, and 101. Most accurate test for valve and septal rupture?: ECHO
the pt is not better, urgent angiography and possible
angioplasty (PCI) should be done. Pt not better means which * Look for a step-up in oxygen saturation as you go from the
symptoms or findings?: - Persistant pain right atrium to the right ventricle to hand you the diagnosis of
- S3 gallop or CHF developing septal rupture
- Worse EKG changes or sustained vent. tachy
- Rising troponin levels Ex: "42% oxygen saturation found on blood from right atrium
92. Acute coronary syndrome treatment algorithm: and 85% saturation found in right ventricular sample"
102.What is needed when there is acute pump failure from an
anatomic problem that can be fixed in the operating room?:
Intraaortic balloon pump
- contracts and relaxes in sync w/ natural heartbeat
- helps give a "push" forward to the blood
103.Is IABP (intraoartic ballon pump) a permanent device?: NO,
serves as a bridge to surgery for valve replacement or
transplant for 24 to 48 hours
104.What should you look for if suspecting an extension of the
infarction/reinfarction?: Look for:
• Recurrence of pain
93. Sinus bradycardia is very common in association w/ MI • New rales
because of?: Vascular insufficiency of the SA node • Bump up in CK-MBs
94. All symptomatic bradycardia tx?: Atropine then pacemaker if • Sudden onset pulmonary edema
atropine not effective
95. ____ will have cannon A waves and bradycardia.: Actions:
• Repeat EKG
• Re-treat with angioplasty or thrombolytics
• Continue: aspirin, metoprolol, nitrates, ACE, statins
105.What should be done if suspecting aneurysm or mural
thrombus as a complication of acute MI?: ECHO

- Most aneurysms don't need specific therapy


- Mural thrombi are treated with heparin followed by warfarin
106. Most likely diagnosis?
Third-degree (complete) AV block
- atria and ventricles are contracting separately and out of
Bradycardia, cannon "a" Waves: Third degree AV block
coordination with each other
107. Most likely diagnosis?
96. Complication of Acute MI includes RV infarction look for?: -
New inferior wall MI
Sudden loss of pulse, JVD: Tamponde/wall rupture
- Clear lungs on auscultation
- ST elevation RV4 most specific finding 108. Most likely diagnosis?

97. Tx RV infarctions?: High volume fluid replacement


Inferior wall MI in history, clear lungs, hypotension w/
- avoid Nitroglycerin worsen cardiac filling
nitroglycerin, tachy: RV infarction
98. What are the signs of tamponade/free wall rupture as a
109. Most likely diagnosis?
complication of acute MI?: Several days after look for "sudden
loss of pulse"
New murmur, rales/congestion: Valve rupture
- Lungs clear, cause of pulseless electrical activity
110. Most likely diagnosis?
Dx: emergency ECHO, emergency pericardiocentesis is done on
the way into the operating room to repair it New murmur, increase in oxygen saturation on entering the
right ventricle: Septal rupture
99. What is the tx of vent tachy or v. fib as a complication of
acute MI?: Can cause sudden death, tx with electrical shock 111.Pt is being prepared for D/C home post acute MI in ICU what
(cardioversion/defibrillation) tests should be done?: Stress test prior to D/C
- EKG - determine if angiography is needed, need for revascularization
- DO NOT do a stress test is pt is still symptomatic this pt
100. Sign of valve or septal rupture as a complication of acute
needs angiography
MI?: New onset of murmur and pulmonary congestion
- MR at the apex radiation to axilla
- Ventricular septal rupture heard lower left sternal border

Page 8
112.Postinfarction routine medications the pt should be sent 121. What is the most likely diagnosis for dyspnea?
home with?: • Aspirin
• Beta blockers (metoprolol) Pallor, gradual over days to weeks.: Anemia
• Statins 122. What is the most likely diagnosis for dyspnea?
• ACE inhibitors
- Best for anterior wall infarctions because of high likelihood Pluses paradoxus, DEC heart sounds, JVD: Tamponade
of developing systolic dysfunction 123. What is the most likely diagnosis for dyspnea?

* Clopidogrel or prasugrel or ticagrelor: those intolerant of Palpitations, syncope: Arrhythmia of almost any kind
aspirin or post-stinting
124. What is the most likely diagnosis for dyspnea?
*ARBs those w/ a cough on ACEi
113.Should pts post infarction be sent home with anti-arrhythmic Dullness to percussion at bases.: Pleural effusion
meds?: DO NOT use amiodarone, flecainide, or any rhythm-
125. What is the most likely diagnosis for dyspnea?
controlling meds to prevent the dev. vent. tachy or fibrillation
- Prophylactic antiarrhythmics INC mortality
Recent anesthetic use, brown blood, not improved with
114.Sexual issues postinfarction?: 1. Do not combine nitrates w/ oxygen, CTAB, cyanosis.: Methemoglobinemia
sildanefil: hypotension can result
126. What is the most likely diagnosis for dyspnea?
2. ED postinfarction most commonly from anxiety, B-blockers
can cause ED
Burning building or car, wood-burning stove in winter, suicide
3. The pt does not have to wait after an MI to engage in sexual
attempt.: CO poisoning
activity, if pt symptoms free can start whenever
4. If post-MI stress test is normal, pt can reengage in any form 127.___ is the most important test of CHF and best initial test.:
of exercise program as tolerated, including sex transthoracic ECHO, evaluate EF
115.Most common cause of CHF.: HTN -> cardiomyopathy 128.What is the most accurate test for Ejection fraction?:
Abnormality of myocardial muscle Multiple-gated acquisition scan (MUGA) or
Nuclear ventriculography
CHF = most common cause of being admitted to the hospital
Transesophageal echocardiography (TEE):
Infarction -> dilation -> regurgitation -> CHF - More accurate for valves, not necessary evaluate CHF
116.Infarction, cardiomyopathy, and valve disease account vast 129.When should you answer "nuclear ventriculography" as a
majority of CHF/systolic dysfunction what are the less diagnostic test?: • Rarely needed
common causes?: Less common causes are: • Person receiving chemotherapy with doxorubicin
• Alcohol • Trying to give max dose to cure lymphoma
• Postviral (idiopathic) myocarditis • But not cause cardiomyopathy
• Radiation
• Adriamycin (doxorubicin) use * Nuclear ventriculogram gives precise evaluation of wall motion
• Chagas disease and other infections abnormalities
• Hemochromatosis (also causes restrictive cardiomyopathy) 130.When should you answer BNP as a test for CHF?: • Acute
• Thyroid disease SOB
• Peripartum cardiomyopathy • Etiology unclear
• Thiamine deficiency • You can't wait for ECHO
117.In addition to dyspnea on exertion with CHF look for?: 1. • Normal BNP excludes CHF!
Orthopnea (worse when lying flat, relieved when sitting up or 131. Tests used to determine etiology of CHF:
standing)
2. Peripheral edema
3. Rales on lung exam
4. Jugulovenous distention (JVD)
5. Paroxysmal nocturnal dyspnea (PND) (sudden worsening at
night, during sleep)
6. S3 gallop rhythm
118. What is the most likely diagnosis for dyspnea?

Slower, fever, sputum, unilateral rales rhonchi.: Pneumonia


119. What is the most likely diagnosis for dyspnea?
132.Tx systolic dysfunction (low EF) CHF.: • ACE inhibitors or
Sudden onset, clear lungs: PE angiotensin receptor blockers (ARBs)
120. What is the most likely diagnosis for dyspnea? • B-blockers
• Spironolactone, Eplerenone
Circumoral numbness, caffeine use, history of anxiety.: Panic • Diuretics
attack • Digoxin
Page 9
133.Which B-blockers should be used tx systolic dysfunction 145. CXR of pulmonary edema shows?:
CHF?: Metoprolol - beta-1 antagonist
Bisprolol - beta-1 antagonist
Carvedilol - nonspecific beta blocker has alpha-1 receptor
blocking activity
134.Most common adverse effect spironolactone?: Hyperkalemia
Gynecomastia

- Eplerenone does not have antiandrogrenic effect, switch if pt


develops gynecomastia on spironolactone
135.T/F: Diuretics control the symptoms of CHF, but they do no
lower mortality.: TRUE
136.Diuretic use in CHF.: • ED: Acute pulmonary edema • Vascular congestion
• Office: Combination with ACEi or ARB • Filling of blood vessels toward head (cephalization of flow)
• Furosemide, torsemide, or bumetanide equal - Usually flow mostly at base because of gravity
• Spironolactone, although a diuretic, is not used at doses • Enlargement of heart
where it has a diuretic effect • Pleural Effusion
137.Does Digoxin lower mortality CHF?: NO 146.What will the ABG/oximetry show in pulmonary edema?: •
- used to control symptoms of dyspnea and will DEC frequency Hypoxia expected
of hospitalization • Respiratory alkalosis -> hyperventilation
• CO2 leaves more easily than oxygen enters
138.T/F: No positive inotropic agent (digoxin, milrinone,
amrinone, dobutamine) has been proven to lower mortality.: 147.Which test is most likely to alter acute management in
TRUE suspected pulmonary edema?: EKG
• A-fib, Atrial flutter, or V-tach as the cause of pulmonary
139.Which devices improve mortality in CHF?: 1. Implantable
edema
defibrillator: for those with Ischemic CM & EF < 35%.
Remember: Arrhythmia and sudden death are MCC of death in
What to do first?
CHF
• Synchronized cardioversion
• Restore atrial systole = Return atrial contribution to cardiac
2. Biventricular pacemaker: pts with dilated cardiomyopathy
output
w/ EF < 35% & wide QRS > 120 ms with persistent symptoms
• Defers/delays need for cardiac transplantation 148.If acute pulmonary edema is from arrhythmia fix it fast with
• Symptoms markedly improved ___?: CARDIOVERSION!!!!

140.When maximal medical tx (ACEi, BB, spironolactone, 149.What test should be done all pts with pulmonary edema to
diuretics, digoxin) and possibly the biventricular pacemaker determine if there is systolic or diastolic dysfunction?: ECHO
fail to control symptoms of CHF, then the only alternative is?: 150.What is the best initial therapy for acute pulmonary edema?:
Seek cardiac transplantation Remove large volume of fluid from vascular space w/ a loop
141.Which tx have a morality benefit in systolic dysfunction?: • diuretic (IV furosemide)
ACEi/ARBs 151.Majority of pt in acute pulmonary edema can be managed
• B-blockers with preload reduction such as?: O2
• Spironolactone Loop diuretics such as furosemide or bumetinide
• Hydralazine/nitrates Morphine
• Implantable defibrillator Nitrates
142.Do CCBs (calcium channel blockers) provide a benefit in
systolic dysfunction?: NO, some CCBs can raise mortality • Removing 1 to 2 liters of fluid is best
• Nesiritide does NOT work better than other agents
143.Tx of Diastolic dysfunction (CHF w/ preserved EF)?: • Beta
blockers have clear benefit 152.What do you do if the questions say: "Preload reduction
• Digoxin, spironolactone clearly has no benefit hasn't been effective?" for tx of acute pulmonary edema?: •
• Diuretics control symptoms of fluid overload Dobutamine in ICU
• Amrinone and milrinone
144.Pt presents with acute SOB w/ rales, JVD, S3 gallop, edema,
- Phosphodiesterase inhibitors that perform the same role
and orthopnea.: Pulmonary edema
- Increase contractility, DEC afterload
- Ascites & enlargement of liver/spleen from chronic passive
congestion of right side of heart
No benefit of digoxin in acute setting, too slow
153. Tx acute pulmonary edema afterload reduction agents?: •
ACEi and ARBs :
- Used on discharge, long-term use with systolic dysfunction
(low EF)

• Nitroprusside and IV hydrazine in ICU

Page 10
154.Regurgitation valve heart disease most commonly due to?: 161. What EKG finding is common mitral stenosis?:
HTN, ischemic heart disease
155.Right sided vs left sided lesions heart valves and changes
with inspiration and exhalation.: • Right-sided heart lesions
(tricuspid and pulmonic valve) INC with INHALATION
• Left-sided lesions (mitral and aortic valve) INC with exhalation
156.Best initial test for valvular heart disease?
Most sensitive and specific?
Most accurate?: 1. ECHO - initial
2. Transesophageal echo more sensitive and more specific than
transthoracic
3. Catheterization - most accurate • Atrial rhythm disturbance, a.fib, very common
157.Tx of valvular heart disease.: Diuretics • LA hypertrophy: Biphasic P wave: V1 and V2
- All forms associated with fluid overload 162. What CXR findings common mitral stenosis?:
- All benefit from diuretics
- Meds alone can't improve stenotic lesions

MS - dilated with balloon


AS - surgical removal
Regurgitant lesions respond to vasodilator therapy w/
ACEi/ARBs, nifedipine, hydrazine

*endocarditis prophylaxis is not indicated for any of these valve


disorders unless the valve has been replaced or there has been
previous endocarditis
158.Most pts with __ are immigrants to the US coming from
geographic regions in which acute RF is still common.: Mitral
stenosis
- look for pregnancy and immigrant history
Pregnancy: Left Atrial Hypertrophy
• 50% increase in plasma volume • Straightening of L heart border
• Contraction of uterus "squeezes" 500 mL extra into central • Elevation of L main- stem bronchus
circulation • Second "bubble" behind heart
- Presents in young adult pts 163.Mitral stenosis treatment?: 1. Diuretics + sodium
159.Unique presenting features of mitral stenosis along with restriction:When fluid overload present
SOB and CHF.: 2. Balloon valvuloplasty with a catheter
3. Valve replacement: only when a catheter procedure cannot
be done or fails
4. Warfarin for a. fib to an INR of 2 or 3
5. Rate control: digoxin, beta blockers, or diltiazem/verapamil
164.Causes aortic stenosis: Congenital bicuspid valve
INC calcification as people age
165.Pt murmur presents with angina, syncope, and CHF. Which
murmur?: Aortic stenosis
- Angina: most common presentation
- CHF: poorest prognosis w/ 2 year average survival
166.• Systolic, crescendo-decrescendo
- Dysphagia: Left atrium (LA) presses on esophagus • Peaks in diamond-shape mid-systole
- Hoarseness: LA presses on laryngeal nerve • Heard best at 2nd right intercostal space
- Atrial fibrillation & stroke from enormous LA • Radiates to carotids:
- Hemoptysis
160.Murmur is in diastole, after an opening snap. Squatting and
leg raising INC the intensity from INC venous return to the
heart. Which murmur?:

Aortic stenosis
- valsalva and standing improve murmur
Mitral stenosis

Page 11
167. CXR in aortic stenosis will show?: 174.Causes of aortic regurgitation?: 1. MI
2. HTN
3. Endocarditis
4. Marfan syndrome or cystic medial necrosis
5. Inflammatory disorders such as ankylosing spondylitis or
Reiter syndrome
6. Syphilis
175.Besides CHF, AR has a large array of relatively unique
physical findings such as?: • Wide pulse pressure
• Water-hammer (wide, bounding) pulse
• Quincke pulse (pulsations in nail bed)
• Hill sign (BP in legs as much as 40 mmHg above arm BP)
• Head bobbing (de Musset sign)
176.• Diastolic, decrescendo murmur
• Heard best: Lower L sternal border
• Valsalva & Standing: Softer
LVH
• Handgrip (increases afterload): Worse: Aortic regurgitation
168. EKG aortic stenosis show?:
177. EKG and CXR for Aortic regurgitation would show?: LVH
178.Tx Aortic regurgitation: 1. ACEi/ARBs or nifedipine as
vasodilator INC forward flow of blood, delay progression

2. Digoxin & Diuretics: Little benefit

3. Surgical valve replacement:


- Acute valve rupture (MI)
- Replace valve before LV dilates excessively
LVH - EF<55%
- S wave in V1 plus R wave in V5 greater than 35 mm - LVESD>55mm
169.Tx of aortic stenosis?: Replacement: -> only truly effective 179. Causes of MVP.: Is common 2%-5% of the population, women
therapy for AS
• Diuretics CHF but pts don't tolerate volume depletion well Marfan
Ehlers-Danlos syndrome
Balloon valvuloplasty : NOT routine for AS
180.This murmur differs because symptoms of CHF are absent,
• AS calcification doesn't improve well with balloon valvuloplasty
pts have atypical chest pain, palpitations, panic attack, most
• Only if surgery isn't an option
often asymptomatic.: MVP
• Unstable/fragile patients
181.• Midsystolic click
170.Causes Mitral regurgitation?: HTN, endocarditis, MI w/
• When severe associated with murmur: MR
papillary muscle rupture, or any other reason that the heart
• Valsalva & Standing worsen
dilates will lead to MR
• Squatting & Handgrip improve (diminish):
171.Pansystolic (holosystolic) murmur, obscuring both S1 and
S2. Radiates to axilla. Handgrip, squatting, and leg raising
worsen murmur of ___.:

MVP
Mitral regurgitation 182.Which diagnostic test should be done for MVP?: • ECHO:
Best choice
172.All left-sided murmurs except ____ will increase with
• Catheterization: Rarely, if ever, done
expiration.: Mitral valve prolapse (MVP) and hypertrophic
• Valve replacement: Rarely needed
obstructive cardiomyopathy
183.Tx MVP: 1. Beta blockers: when symptomatic
173.Mitral regurgitation tx?: 1. ACE or ARBs are best: DEC rate
2. Valve repair with catheter, place clip to tighten valve
of progression
3. Stitches valve to tighten leaflets
2. Digoxin & diuretics for symptomatic CHF
• Surgical repair rarely necessary
3. Valve replacement: indicated when heart dilates
4. Endocarditis prophylaxis NOT recommended
- Don't wait for left ventricular end systolic diameter (LVESD) to
become large 184.Best initial test for suspected cardiomyopathy symptoms of
- Replace when: LVESD > 40 mm or EF < 60% SOB worse w/ exertion, edema, rales, JVD.: ECHO
4. Valve repair: - EKG + CXR should be performed nothing specific on these
-Placing a clip or sutures across valve to tighten tests to confirm diagnosis
185. All cardiomyopathies tx with?: Diuretics
Page 12
186.Which murmurs do not INC with expiration?: HOCM 198.Systolic anterior motion (SAM) of mitral valve is classic for
(hypertrophic obstructive cardiomyopathy) ____. It contributes to obstruction.: HOCM
MVP 199.___ is most accurate test to determine precise gradients of
187.In addition to previous MI and ischemia, dilated pressure across the chamber in HOCM.: Catheterization
cardiomyopathy can be from?: - Alcohol 200.Which two drugs are always wrong in tx of HCM?: Digoxin +
- Postviral myocarditis Spironolactone
- Radiation 201. EKG findings HOCM:
- Toxins such as doxorubicin
- Chagas disease
188.Tx dilated cardiomyopathy which drugs used to lower
mortality and which to control symptoms?: Multiple meds
mortality:
- ACEi (or ARBs)
- Beta blockers (metoprolol, carvedilol)
- Spironolactone (or eplerenone)

Symptom Control ONLY: Nonspecific ST and T wave changes are common.


- Diuretics & Digoxin
189.In dilated cardiomyopathy if the QRS is wide (more than 120 LVH is common. EKG can be normal in a quarter
milliseconds) what should be used to improve both symptoms
and survival?: Biventricular pacemaker Septal Q waves in the inferior and lateral leads are common in
HOCM.
• Automated implantable cardioverter/defibrillator has mortality 202.HOMC specific non-pharm therapy ?: 1. Implantable
benefit defibrillator: HOCM w/ syncope
190. Most common cause of Hypertrophied Cardiomyopathy?: HTN 2. Ablation of septum:
191.Hypertrophied cardiomyopathy vs HOCM?: HCM: Reaction to • Catheter placing absolute alcohol in muscle
stress on the heart such as high BP • Causes small infarctions
• Heart hypertrophies to carry load 3. If symptoms persist: Surgical myomectomy
• Develops difficulty "relaxing" in diastole 203. Differences in therapy btw HCM and DCM?:
• Can't relax = Can't receive blood
• Patient becomes SOB

HOCM:
• Genetic disorder
• Abnormal shape of septum
• Asymmetrically hypertrophied septum and valve leaflet blocks
blood leaving the heart
192.Differences between HCM and other forms of
cardiomyopathy?: S4 gallop
204.This type of cardiomyopathy combines worst aspects of
Fewer signs of heart failure such as ascites or hepatomegaly or dilated and hypertrophic cardiomyopathy, heart neither
splenomegaly contracts nor relaxes, infiltrated with substances creating
immobility.: Restrictive cardiomyopathy
193.Patient presents with dyspnea, chest pain, syncope,
symptoms worse with increase in heart rate (exercise, 205.Name some causes of restrictive cardiomyopathy?: •
dehydration, diuretics), patient is a young athlete that is at Sarcoidosis
risk for sudden death. Diagnosis?: Hypertrophic obstructive • Amyloid
cardiomyopathy • Hemochromatosis
• Endomyocardial fibrosis
194.Which things can worsen HOCM symptoms?: Anything that
• Scleroderma
DEC LV chamber size
- ACEi, ARBs, digoxin, hydrazine, Valsalva, standing suddenly 206.Pt presents with dyspnea, signs RHF, pulmonary HTN, and
kussmaul sign. Dx?: Restrictive cardiomyopathy
195.Tests HOCM?: ECHO best initial
- septum 1.5x thickness of the posterior wall
Kussmaul sign: INC in JVP on inhalation
196.What is the best initial therapy for HOCM and HCM?: B-
207.Best initial test for restrictive cardiomyopathy?: ECHO
blockers
- amyloid presents w/ speckling of the septum
- other agents negative inotropes verapamil and disopyramide
- EF may be normal or elevated
can be useful
208. EKG findings in restrictive cardiomyopathy?: LOW voltage
197.What is contraindicated in HOCM that can be used in HCM?:
Diuretics
- DEC LV chamber size worsen HOCM

Page 13
209.Most accurate test for restrictive cardiomyopathy?: 216.Which disorder can cause pericarditis?: Systemic lupus
Endomyocardial biopsy erythematosus:
- rarely done Most common connective tissue disorder
- dx made from biopsies elsewhere
210.Treatment of restrictive cardiomyopathy?: Tx underlying But...
cause • Wegener granulomatosis
• Diuretics relieve some pulmonary HTN and signs of right HF • Goodpasture syndrome
• No other clear therapy • Rheumatoid arthritis
211. Murmurs and the effects of maneuvers.: • Polyarteritis nodosa and others
...can cause pericarditis
217.Pt presents with sharp chest pain, changes with respiration,
changes with position of body, worsening by lying flat,
improved by sitting up.: Pericarditis
218. EKG findings pericarditis:

More blood INC all murmurs except MVP and HOCM

Standing and valsalva DEC venous return to the heart

Standing or Valsalva = diuretic use


- stenotic and regarg murmurs all tx w/ diuretics and/or salt
restriction Diffuse ST elevation in all leads, PR depression
212. Handgrip will worsen which murmurs?: Worsens AR and MR 219. ___(Drug), DEC the recurrences of pericarditis.: Colchicine
220.Tx pericarditis.: Tx underlying cause
• Handgrip INC afterload, contraction of arm muscles • Majority: Idiopathic presumed viral
compresses arteries -Coxsackie B virus
• DEC emptying of heart • Treated with NSAIDs (e.g., ibuprofen, naproxen)
• Opposite of ACE inhibitor • Colchicine DEC recurrences

Handgrip = Fuller left ventricle If answer has NSAID and colchicine = correct
213.Amyl nitrate will improve which murmurs?: Improve AR and 221. Any cause of pericarditis can extravasate enough fluid to
MR cause ___.: Pericardial tamponade
• Direct arteriolar vasodilator, DEC afterload - compression of the R side of the heart, walls are thinner, as
• Simulates ACEis/ARBs on heart little as 50 mL of fluid accumulating acutely can cause
• Valvular disease treated with ACEi/ARB will improve with amyl tamponade
nitrate - pericardium stretch accomodate ~ 2 L
222.Tx pericardial tamponade due to trauma with a bleed into the
• Amyl nitrate = ACEi = Emptier left ventricle
pericardium.: Emergent thoracotomy
Effect of Handgrip vs Amyl nitrate on intensity (loudness) of
214.
223. Pericardial tamponade which PE finding most associated?:
murmurs?:

Pulsus paradoxus
215. Most common infection of the pericardium is?: VIRAL - DEC > 10 mmHg in blood pressure on inhalation
224.What is the most likely diagnosis pt presents with
hypotension, tachy, distended neck veins, clear lungs?:
Pericardial tamponade

Page 14
225. EKG findings pericardial tamponade?: 232.Which physical findings is most likely associated with pt
with constrictive pericarditis?: 1. Kussmaul sign: INC in JVD
on inhalation (normally the neck veins should go down on
inhalation)

2. "Knock": extra heart sound in diastole from ventricular filling,


as the heart fills to max, it hits stiff, rigid pericardium with a
"knock"
233. The best initial test for constrictive pericarditis?:

Electrical alternans (different heights of QRS complexes btw


beets found on EkG)
226. CXR findings pericardial tamponade?:

CXR
- calcification + fibrosis
234.Which tests are the most ACCURATE for constrictive
pericarditis?: CT scan or MRI
- would not be done if CXR done first
235.Why should an ECHO be performed in constrictive
pericarditis?: Excludes RV hypertrophy or cardiomyopathy as
cause

• Myocardium moves normally with constrictive pericarditis


236.Tx constrictive pericarditis?: 1. Diuretics:
Enlarged cardiac shadow expanding in both dxns ("globular - Used first to decompress filling of heart
heart") - Relieves edema and organomegaly
227. ECHO findings pericardial tamponade?:
2. Surgical removal of pericardium
237.___ is the stenosis of peripheral arteries with the same
causative factors as coronary and carotid disease such as
DM, hyperlipidemia, HTN, and tobacco smoking.: Peripheral
artery disease (PAD)
238.Pt presents with leg pain in the calves on exertion (walking
up or down hills), relived by rest.: PAD
- severe disease associated with loss of hair follicles, sweat
glands, sebaceous glands
- skin becomes smooth and shiny
239.___ pain is worse when walking down hills, because of
RA and ventricular diastolic collapse
leaning back.: Spinal stenosis
228.Right heart catheterization findings pericardial tamponade?:
240.What is the best initial test for PAD?: Ankle-brachial index
Equalization of pressures in diastole
(ABI)
229.Tx pericardial tamponade?: 1. Pericardiocentesis: needle - ratio of the BP in the ankles to the brachial arteries
drainage will rapidly reexpand the heart - Normally equal or slightly greater in ankles because of gravity
2. IV fluids
3. A hole or "window" placed into pericardium for recurrent If difference btw them is>10% (ABI<0.9)*, disease is present
cases
241.What is the most accurate test for PAD?: Angiogram, but not
230.Should diuretics be given as tx for pericardial tamponade?: necessary unless specific revascularization will be done
NO!
242.The best initial treatment PAD: - Aspirin
- Smoking cessation
Diuretics will DEC intracardiac filling pressure and may markedly
- Cilostazol: Single most effective mediation
worsen collapse of right side of heart
* surgery is done to bypass stenosis if med tx fails
231.Any cause of pericarditis with calcification and fibrosis
243. Is there a routine screening test for PAD?: NO
prevent filling of the right side of the heart. Can be chronic
such as in TB.: Constrictive pericarditis
There is no routine screening for PAD since there is no mortality
- also have signs of R heart failure
benefit to be obtained
- Edema, ascites, enlargement liver and spleen, JVD

Page 15
244.In all major vascular disease, control each of the following 254. Development of a right-to-left shunt from pulmonary HTN.:
variables?: BP
LDL below 100
Diabetes
245. Do calcium blockers help with PAD?: NO
246.Pt presents with pain in between the scapulae and there is a
difference in blood pressure between the arms.: Aortic
dissection
247.What is the most accurate test for aortic dissection?:
Angiography
- most invasive
- potential complication of allergy
- renal failure
248.Tx aortic dissection?: "Most important step" is: Control BP
- B-blockers: DEC "shearing forces" that worsen dissection, Eisenmenger syndrome
must start before nitroprusside to protect against reflex tachy - ventricular septal defect who has significant L to R shutting
of nitroprusside, which worsens shearing forces leads to pulmonary HTN -> when severe HTN -> shunt reverse to
- Nitroprusside R to L
- Surgical correction - worsen with pregnancy since pregnancy INC plasma volume by
249.When should AAA have surgical repair?: AAA > 5 cm in 50%
diameter
- surgical or catheter directed repair of the lesion is indicated
250.Which heart disease in pregnancy is the most dangerous to a
pregnant woman?: Peripartum cardiomyopathy w/ persistent
ventricular dysfunction
- if they become pregnant again, very high chance of markedly
worsening her cardiac fxn
251.Antibodies made to myocardium in some pregnant women,
LV dysfunction often reversible and short term what is this
called?: Peripartum cardiomyopathy
- unknown why antibodies made against the myocardium in
some pregnant women
- develops after delivery
- repeat pregnancy w/ peripartum cardiomyopathy = enormous
antibody production against myocardium
252.If LV dysfunction in peripartum cardiomyopathy does not
improve, then the person must undergo?: Cardiac
transplantation
253.Tx peripartum cardiomyopathy: ACEi/ ARB - acceptable since
dev. after delivery most cases
B-blockers
Spironolactone
Diuretics
Digoxin

Page 16
MTB Q and A
MTB Dermatology

1. ABCDE of 6. Basal Cell


melanoma. Carcinoma

Asymmetry Waxy lesion that is shiny like a pearl


Border irregularity
Color irregularities Rate of metastases 0.1 percent.
Diameter > 6 mm
Evolution (changing appearance over Shave bx
time)
2. Dx test melanoma Full thickness biopsy Tx surgical removal (Mohs micrographic surgery)

3. Tx malignant Sx removal w/ significant removal of 7. Kaposi's


melanoma normal skin surrounding the lesion Sarcoma
- Interferon injection is helpful in
widespread disease
4. Squamous cell
carcinoma

Sunlight Reddish/purplish (vascular) lesions found on


Greatly INC by organ transplant 2/2 long- skin predominantly of patients with AIDS (in the
term use of immunosuppressive drugs past mostly in older men of Mediterranean origin)
- HHV-8 oncogenic
Ulcer that does not heal and continues to - can also be found GI tract and lungs
grow - AIDS through sexual contact -> KS
8. Best 1. Tx the AIDS with antiretrovirals and the
Bx and remove. therapy for majority of KS will disappear as CD4 improves
5. Malignant Kaposi's 2. Intralesional injections of vincristine or
melanoma has a Sarcoma interferon are very successful
strong tendency to 3. If these fail, use chemo with liposomal
metastasize to the doxorubicin
___.
* KS is not routinely treated with surgical
treatment

BRAIN

Page 17
9. Actinic 13. Bullae that easily
Keratosis rupture (thin
walled)
Involvement of the
mouth
Fluid loss and
infection, act like
a burn
+ Nikolsky sign

Pemphigus Vulgaris
- denuding of skin from mild pressure
- w/o treatment it is fatal
Premalignant skin lesions from high-intensity sun - autoantibodies desmosomes
exposure in fair-skinned people 14. Causes of Idiopathic
- Small risk of squamous cell cancer, slow pemphigus ACE inhibitors
progression vulgaris Penicillamine
Tx: Removal by curettage, cryotherapy, laser, or Phenobarbital
topical 5-fluorouracil Penicillin
Local imiquimod also effective that is used in tx
15. Tx of Pemphigus 1. Systemic steroids (prednisone)
mollusk contangiosym and condyloma acuminatum
Vulgaris 2. Azathioprine or mycophenolate to
10. Common in the wean the patient off steroids
elderly. 3. Rituximab (anti-CD20 antibodies) or
Hyperpigmented IVIG in refractory cases
lesions referred
16. Most likely dx:
to as "liver
Age 70-80
spots"
Bullae are thicker
"Stuck on"
walled and much
appearance
less likely to
rupture
Mouth rarely
involved
(-) Nikolsky sign

Bullous Pemphigoid
Seborrheic keratoses Mortality much less likely than
- no premalignant potential pemphigoid vulgarism
- remove via cryotherapy, sx, or laser for cosmetic - hemidesmosomes
reasons 17. Most accurate test Biopsy with immunofluorescent
11. Nikolsky's Sign Pemphigus vulgaris for Bullous antibodies
Association Staphylococcal scalded skin syndrome Pemphigoid
Toxic epidermal necrolysis 18. Tx for Bullous 1. Prednisone
12. Most accurate Skin biopsy showing autoantibodies on Pemphigoid 2. Azathioprine, cyclophosphamide, or
test for immunofluorescent staining mycophenolate to wean patients off
Pemphigus steroids
Vulgaris
Mild: erythromycin, dapsone, and
nicotinamide (not niacin)
19. Pemphigus Associated with other autoimmune
Foliaceus diseases, ACE-I, and NSAIDs.

Thinner walled than even Pemphigus


Vulgaris so lesions don't remain intact for
long. No oral lesions.
20. Dx and Tx of Dx with biopsy
Pemphigus Tx with steroids
Foliaceus

Page 18
21. Most likely dx: 29. Very severe hypersensitivity SJS
Nonhealing reaction involving the mucous - Tx: IVIG
blisters on the sun membranes. - steroids not clearly
exposed parts of Sloughs off respiratory epithelium beneficial
the body (hands and may lead to respiratory
and the face) failure. Less worse than Toxic
Epidermal Necrolysis
Hyperpigmentation <10-15 percent of total
of the skin body surface area
Hypertrichosis of <5-10 percent mortality
the face
30. Toxic Epidermal Necrolysis (TEN) Most serious version of
Porphyria Cutanea Tarda (PCT)
cutaneous
22. Porphyria Cutanea - Liver disease (hepatitis C, alcoholism) hypersensitivity
Tarda in those - Estrogen use - rash with mucosal
with a history of ? - Iron overload (hemochromatosis) membrane involvement
23. Diagnostic Test for INC uroporphyrins in a 24-hour urine with Nikolsky signs
Porphyria Cutanea collection 30-100 percent of body
Tarda (PCT) - PCT is a hypersensitivity of the skin to surface
abnormal porphyrins when they are 40-50 percent mortality
exposed to light
Tx: IVIG
24. Porphyria cutanea Uroporphyrin decarboxylase
tarda is a 31. Most likely dx: Erythema Nodosum
deficiency of ___ Painful, red, raised nodules
activity. appear on the anterior surface of
the lower extremities
25. Management of Stop drinking EtOH, stop estrogen use Nodules tender to palpation
Porphyria Cutanea Use barrier skin protection Nodules do not ulcerate
Tarda Use phlebotomy to remove iron Nodules last about 6 weeks
26. Most likely dx: Urticaria 32. Erythema Nodosum can be 2/2 Pregnancy
Evanescent Recent streptococcal
wheals and hives infection
Localized, Coccidioimycosis
cutaneous Histoplasmosis
anaphylaxis Sarcoidosis
without IBD
hypotension or Syphilis
hemodynamic Hepatitis
instability Enteric infections
Itching is (Yersina)
prominent
33. Best initial test for fungal Potassium hydroxide test
27. Most likely dx: Morbilliform Rash (allergic rash) infection of the skin.
Rash resembling - KOH can dissolve
measles Lymphocyte mediated and treated with epithelial cells and
Generalized antihistamines collagen but not fungus
maculopapular
eruption that 34. Most accurate test for fungal Culture the fungus (takes
blanches with infection 6 weeks to grow though)
pressure 35. Tx of tinea that does not involve Topical antifungal
Reaction appears the hair or nails. agents:
a few days after - Clotrimazole
medication - Ketoconazole
28. Widespread, small Erythema multiforme - Econazole
"target" lesions, - May be also from herpes or - Miconazole
most are on the mycoplasma - Nystatin (yeast
trunk. - Prednisone may benefit some patients infections only)
No mucosal - Ciclopirox
membrane
involvement.

Page 19
36. Medications for tinea Oral terbinafine or itraconazole. 45. Most likely dx:
capitis (hair infection) or - Griseofulvin has less efficacy
onychomycosis (nail compared to the above drugs Both dermis
infection) and epidermis
involvement
37. Terbinafine danger Potentially hepatotoxic
Fever, chills,
Check liver function tests
bacteremia
periodically
invades
38. Ketoconazole danger When used systemically: dermal
Hepatotoxicity and lymphatics
gynecomastia Bright red,
39. General tx for bacterial Dicloxacillin, cephalexin angry, swollen Erysipelas
skin infections (Keflex), or cefadroxil appearance to
the face
40. Patient allergic to PCN Rash - cephalosporins safe to Streptococcus
(rash v. anaphylaxis) use
46. Tx for Mild disease: oral meds
Anaphylaxis - macrolides Erysipelas, - Dicloxacillin, cephalexin, cefadroxyl
(erythromycin, azithromycin, cellulitis, - PCN allergy: erythromycin, clarithromycin, or
clarithromycin) or newer folliculitis, clinda
fluoroquinolone: levofloxacin, furuncles, and - MRSA: doxycycline, clinda, TMP/SMZ
gatifloxacin, moxifloxacin carbuncles
Severe disease (fever present): IV meds
41. Indication for IV MRSA Suspicion - Oxacillin, nafcillin, cefazolin
Vancomycin - PCN allergy: clinda, vanco
Nursing home patient or - MRSA: vanco, linezolid, daptomycin,
someone who has been in the tigecycline, ceftaroline
hospital a long time
47. ___ infection
42. Most likely dx: Impetigo of the soft
Superficial bacterial - Contagious and autoinoculable. tissue of the
infection - More often Staphylococcus, skin, extends
Weeping sometimes streptococcus from dermis
Oozing pyogenes into
Honey-colored subcutaneous
Draining tissue.
43. Tx Impetigo 1. Mild disease with topical
agents: Skin warm,
Cellulitis
- Mupirocin swollen,
- Retapamulin tender.
- Bacitracin
2. Severe disease with oral Leg involved >
agents: arms.
- Dicloxacillin and cephalexin 48. The most Inject sterile saline into the skin and aspire it
3. Community-acquired MRSA accurate test for culture
with: for cellulitis. - Staph more common > Strep
- Doxycycline
- Clindamycin 49. Staphylococcal
- TMP/SMZ Infection
occurring
44. T/F: Skin infections with TRUE around a hair
group A beta hemolytic follicle
Step can cause
glomerulonephritis, but not
rheumatic fever.

Folliculitis < Furuncles < Carbuncles

Page 20
50. Treatment for folliculitis Topical mupirocin 60. Other tx options for Herpes Besides acyclovir, gabapentin,
Zoster TCAs, and topical capsaicin
51. Treatment for furuncles and Systemic antistaphylococcal
carbuncles antibiotics (dicloxacillin or 61. Most likely dx and initial Primary syphilis
cefadroxil) test: Darkfield examination
Ulceration with heaped up
PCN rash: use indurated edges, painless
cephalosporins
62. Most likely dx: Secondary Syphillis
Anaphylaxis rx use:
Copper-colored,
- Mild: macrolides,
maculopapular rash intense
clindamycin, doxycycline,
on palms and soles of feet.
TMP/SMX
- Severe: vancomycin,
Mucous patch, alopecia
linezolid, daptomycin,
areata, and condyloma lata
tigecycline, or ceftraoline
63. Most likely dx: Lyme Disease
52. Most common organisms in Streptococcus and Clostridia
Joint disease
Nectrotizing Fasciitis
Neurological or cardiac If rash described, go straight to
53. Most likely dx: Necrotizing Fasciitis disorder tx with oral doxycycline,
Very high fever Erythematous rash with amoxicillin, or cefuroxime
Portal of entry into the skin central clearing (large)
Pain out of proportion to the
64. Most likely dx: Toxic Shock Syndrome
superficial appearance
Fever > 102 F
Bullae
Systolic blood pressure <
Palpable crepitus
90
54. Therapy and best Surgical debridement Desquamative rash
confirmation of Necrotizing Vomiting
Fasciitis Involvement of the mucous
membranes of the eye,
55. Tx for Necrotizing Fasciitis Beta lactam/beta lactamase
mouth, and genitals
inhibitior
65. Tx of TSS Vigorous fluid resuscitation
Ampicillin/sulbactam (Unasyn) Pressors, such as dopamine
Ticarcillin/clavulanate
(Timentin) Antistaphylococcal
Piperacillin/tazobactam medications: oxacillin, nafcillin,
(Zosyn) cefazolin

If Strep A (pyogenes) For MRSA: Vancomycin or


confirmed, tx is clindamycin linezolid
and PCN
66. SSSS vs TSS SSSS looks similar to TEN,
56. Paronychia Infection and Tx Loculated under the skin, including Nikosky sign
surrounding a nail
TSSS same skin involvement
Tx with small incision to allow plus life-threatening multiorgan
drainage and involvement:
antistaphylococcal antibiotics - Hypotension
- Renal dysfunction (elevated
57. Most accurate test for Herpes Viral culture (grows in 24-48
BUN and Cr)
Simplex infxn hours)
- Liver dysfunction
58. Tx for Herpes Simplex Oral acyclovir, famciclovir, or - CNS involvement (delirium)
valacyclovir.
TSSS associated with staph
Topical acyclovir - very little attached to a foreign body
efficacy
Acyclovir resistant herpes -
foscarnet
59. Immunocompromised or Acyclovir, valacyclovir or
primary infection as an adult: famciclovir (otherwise not
Herpes Zoster tx. treated)

Page 21
67. Tx SSSS and TSS Supportive care and 75. Pruritus and scratching with scaly Atopic dermatitis
antistaphlococcal medications rough areas of thickened skin (eczema)
- Oxacillin and nafcillin most (lichenified) face, neck, and skin - IgE levels are elevated
effective folds flexure areas (popliteal area
- Cefazolin behind the knee).
- Antibiotics do not reverse the
76. Superficial skin infections with Staphylococcus
disease but kill the Staph that is
___ commonly occur with atopic
producing the toxin
dermatitis (eczema).
68. Most likely dx: Psoriasis
77. Tx Atopic dermatitis (eczema) via 1. Stay moisturized:
Silvery, scaly plaques Koebner phenomenon -
skin care regiments dry skin is more itchy,
develop on the extensor development of lesions to the site
use humidifier
surfaces, not itchy. of an epidermal injury
2. Avoid bathing, soap,
Nail pitting
and washcloths: skin
Koebner phenomenon
hyperirritable
possible
3. Cotton is less
69. Extensive psoriasis is Depression irritating to skin than
associated with ____. < 10% have arthritis wool
70. Psoriasis Tx local 1. Topical high-potency steroids: 78. Tx Atopic dermatitis (eczema) via 1. Topical
disease fluocinonide, triamcinolone, medical therapy. corticosteroids (flares
betamethasone, clobetasol of diseases)
2. Vitamin A and Vitamin D 2. Tacrolimus and
(calcipotriene) ointment help get pimecrolimus (T cell
the patient off steroids (can inhibiting agents longer-
cause skin atrophy) term control and help
3. Coal tar preparation get the pt off steroids)
4. Pimecrolimus and tacrolimus 3. Antihistamines
(face and penis) alternative to - Mild disease:
steroids for delicate areas nonsedating drugs
(cetirizine,
71. Psoriasis Tx w/ 30% + 1. UV light
fexofenadine,
involvement = extensive 2. Antitumor necrosis factor
loratadine)
disease (TNF) inhibitors (etanercept,
- Severe disease:
adalimumab, infliximab),
hydroxyzine,
miraculous in efficacy in severe
diphenhydramine,
disease
doxepine
3. Methotrexate: used last SE
4. Antibiotics such as
liver and lung, drug of last resort
cephalexin, mupirocin,
except in psoriatic arthritis
retapammulin when
72. What should be checked PPD prior to use impetigo occurs
before giving TNF - TNF inhibitors can reactivate TB 5. UV light
inhibitors (etanercept, (phototherapy)
adalimumab, infliximab)?
79. Tacrolimus and pimecrolimus as LYMPHOMA
73. Most severe psoriasis tx Methotrexate, however SE is liver rarely associated with the
fibrosis development of ____.
74. Atopic dermatitis Common skin disorder associated 80. ___ hypersensitivity reaction to Seborrheic dermatitis
with overactivity of mast cells dermal infection pityrosporum (dandruff)
and the immune system ovale (fungal) with noninvasive - Dandruff, flaky skin on
dermatophyte organisms. a red base on scalp,
Hx of: eyebrows, and
- Asthma nasolabial fold
- Allergic rhinitis - INC in AIDS,
- FHx atopic disorders Parkinson disease
- Onset before age 5, very rare
81. Tx of Seborrheic Dermatitis Both topical steroids
to start after age 30
(hydrocortisone,
alclometasone) and
antifungal agents
(ketoconazole) are
useful

Page 22
82. Stasis Dermatitis Hyperpigmentation that is built up 90. Drugs that can cause PCN
from hemosiderin in the tissue. hypersensitivity reactions? Sulfa drugs (thiazides,
Long periods of venous furosemide, and sulfonylurea)
incompetence of lower extremities.

Prevention of progression: elevation


of legs and LE support hose
83. Contact Dermatitis Hypersensitivity to soaps,
detergents, latex, neomycin over
area of contact.
Jewelry also possible cause.
Linear streaked vesicles, as in with
poison ivy.
84. Diagnostic testing for Patch testing
contact dermatitis
85. Idiopathic, transient
dermatitis that starts
out with a single
lesions (herald patch)
and then disseminates

Looks like secondary


syphilis but spares
palms and soles
(VDRL/RPR negative)

Pityriasis Rosea
- Mild, self-limited

tx: Steroids + UV light


86. Tx fo Acne 1. Mild: topical antibacterials such as
benzoyl peroxide, if ineffective add
topical abx such as clindamycin or
erythromycin
2. Moderate acne: add topical vitamin
A derivatives such as tretinoin,
adapalene, or tazarotene to topical
abxs, if no response use oral abxs
such as minocycline or doxycycline
3. Severe: oral vitamin A, isotretinoin
to oral abxs
87. Isotretinoin cause Hyperlipidemia
___.
88. When giving vitamin A if female, pregnancy test
derivatives what test - only give to patients on suitable
should be order? hormonal and barrier birth control
89. Oral and vaginal Best initial test: KOH
candidiasis dx, tx. Most accurate: Fungal culture
Tx: topical antifungal

Page 23
MTB Emergency medicine
MTB Q and A

1. 46-year-old man has intermittent episodes d. EP studies 6. Adverse effects of Oral mercury - neurologic problems
of palpitations, lightheadedness, and near- - useful in mercury poisoning (nervous, jittery, twitchy, hallucinatory)
syncope. His EKG is normal. The echo detecting a
shows an EF of 42%. Holter monitor shows source of Inhaled mercury vapor - lung toxicity
several runs of wide complex tachycardia ventricular presenting as interstitial fibrosis
lasting 5-10 seconds. arrhythmia
7. After rate control Anticoagulation with WARFARIN,
- if an induce
(rate under dabigatran, or rivaroxaban.
Which of the following is most likely to sustained
100/min), in A. fib
benefit this patient? ventricular
or flutter Don't need to bridge with heparin. Only
tachy, then
use heparin if current clot in atrium.
a. Pacemaker placement person can
b. Digoxin benefit from
1. Slow rate
c. Warfarin implantable
2. Anticoagulant (aspirin for low risk)
d. EP studies defibrillator
e. Swan-Ganz catheter 8. After the patient 1. Open airway: head tilt, chin lift, jaw
has been shown to thrust.
2. 73-year-old man has his third syncopal d. Implantable
be unresponsive, 2. Give rescue breaths if not breathing
episode in last 6 months. An EKG done in defibrillator
and EMS 3. Check pulse and start chest
the field shows VT. His stress test is - will prevent the
activated, the next compressions if pulseless.
normal. next episode of
step is?
What is the most appropriate next step in sudden death or
the management of this patient? syncope 9. All __ present with Spider bites
sudden, sharp pain
a. Metoprolol that the patient
b. Diltiazem may describe as "I
c. Angiography stepped on a nail"
d. Implantable defibrillator or "a piece of
e. EP studies glass was in my
shoe".
3. Acetaminophen Toxicity and Fatality levels Toxicity: > 8 to
10 grams 10. Aspirin and lactate Interferes with oxidative phosphorylation
Fatality: > 12 to production and results in anaerobic glucose
15 grams metabolism (producing lactate)

4. Acutely symptomatic patient with Atropine (initial 11. Aspirin Causes ARDS
bradycardia and signs of hypoperfusion therapy for multisystem Interferes with PT production and raises
treatment. bradycardia) toxicity PT
"best initial Metabolic acidosis from lactate
therapy" 12. Asystole tx

Pacemaker
"most effective
After CPR
therapy"
Epinephrine (or vasopressin)
5. Acute therapy of WPW. Procainamide
or amiodarone - Will shunt blood to critical areas like
useful for both heart and brain
atrial and
13. Atrial fibrillation • Two disorders with nearly identical
ventricular
vs atrial flutter. management
rhythm
disturbances.
• Major points of difference are:
-Flutter is a regular rhythm vs. fibrillation
Only if WPW
is irregular
currently
-Flutter changes to sinus rhythm or
presenting.
deteriorates into fibrillation

Page 24
14. Atrial rhythm acute pulmonary edema 19. Best initial test to
problems can detect TCA toxicity
cause ___ from
loss of atrial
contribution in
those with
cardiomyopathy.
15. Best initial Increased level of free erythrocyte protoporphyrin
diagnostic test
for lead EKG will show widening of QRS complex.
poisoning.
16. Best initial Naloxone with glucose. QT prolongs as well until torsade de
management for pointes.
altered mental If this doesn't work intubate to protect airway, 20. Best initial therapy 100% oxygen to treat smoke inhalation
status of possibly followed by gastric lavage. for burns. and carbon monoxide toxicity.
unclear
21. Best initial tx for Control rate with BB, CCB (diltiazem,
etiology.
atrial fibrillation verapamil), or digoxin
17. Best initial test and atrial flutter.
for digoxin
22. Best initial tx for 100% oxygen
toxicity
carbon monoxide
toxicity If severe - CNS symptoms, cardiac
symptoms, metabolic acidosis, you can
consider hyperbaric oxygen
23. Best intial tx for Fomepizole, inhibiting alcohol
methanol and dehydrogenase and prevents production
ethylene glycol of toxic metabolite.
toxicity
Only dialysis will remove toxins from
body
24. Black widow spider Abdominal pain, muscle pain
bite presentation, Hypocalcemia
lab test, and tx. Tx. Calcium, antivenom
25. Blood color in CO CO - abnormally red
EKG and potassium level. vs. Meth - abnormally brown
Methemoglobinemia
EKG shows downsloping of the ST segment in26.
all Blood gas in Respiratory alkalosis with a decreased
leads aspirin overdose CO2 and bicarb level (because of
18. Best initial test metabolic acidosis rising)
for hypothermia
and most Ex: 7.46, CO2 22, Bicarb 16
common cause 27. Blood gas in CO PO2 is normal because it can't release.
of death. poisoining Because oxygen not released to tissues
you get lactic/metabolic acidosis.

Ex: 7.35, pCO2 26, HCO3 18


28. Brown recluse Local skin necrosis, bullae, and blebs
spider bite No lab abnormalities
EKG presentation, lab
Cardiac arrhythmia test, and tx. Tx with debridement, steroids, and
dapsone
29. Can charcoal be Yes, charcoal won't make N-
used with N- acetylcysteine ineffective.
acetylcysteine?
No contraindication.

Page 25
30. Cathartics Not a good answer (sorbitol). 40. Cyanosis and Methemoglobinemia
normal pO2 =
Speeding up GI transit time does not
41. Death from snake 1. Hemolytic toxin: hemolysis and DIC
eliminate ingestion without absorption
biles is from? and damage to the endothelial lining of
31. Cause of Afib Anatomic cardiac defects dilating the tissues
atrium (thus can't go away with
cardioversion over 90% revert to 2. Neurotoxin: can result in respiratory
fibrillation). paralysis, ptosis, dysphagia, and
- HTN diplopia
- Valvular heart disease
42. Definition of large - Compression of ventricles or sulk
intracranial - Herniation with abnormal
Acute disease normalized
Hemorrhage breathing/unilateral dilation of pupil
spontaneously: Alcohol, caffeine,
- Worsening mental status or focal
cocaine, or transient ischemia, don't
findings
force it.
43. Dog, cat, and Amoxicillin/clavulanate
32. Cause of death in Myocardial infarction.
human bites tx Tetanus vaccination booster if more than
CO poisoning
5 years
CO is like anemia in that it removes
carrying capacity/functional RBCs
Dogs and cats: Pasteurella multocida
33. CHADS2 < 1 or Aspirin with CHADS score 1 or less Humans: Eikenella corrodens
lower "lone atrial
44. Dx Test and Tx for Methemoglobin level
fibrillation" use If CHADS score is 2 or higher, warfarin,
Methemoglobinemia
___ to prevent dabigatran, rivaroxaban, or apixaban
Best initial tx is 100% oxygen
strokes.
34. CHADS Score CHF or cardiomyopathy Most effective therapy is methylene
HTN blue (decreased half life of
Age > 75 methemoglobin)
Diabetes
45. Etiology of death in Airway burn or volume loss.
Stroke or TIA = 2 points
burns
35. Charcoal Superior to lavage and ipecac, toxins in
46. First degree AV
indications blood drop fast
block Tx

Should be given to anyone with a pill


overdose, charcoal is not dangerous in
anyone, can remove toxic substances Extended PR - can treat sx with atropine
even after they have been absorbed 47. First step in Atropine blocks effects of ACh that is
36. Chronic Afib and Lasting more than 2 days organophosphate tx already increased. Dries up respiratory
Cardioversion Routine cardioversion not indicated secretion.

Majority of those cardioverted won't stay Pralidoxime reactivates


in sinus (shocking pt does not cure a acetylcholinesterase, which won't act
dilated left atrium) fast enough in an acute reaction.

37. Chronic atrial anti-coagulated 48. Fluid replacement Fluid replacement (4 mL) x (%BSA
fibrillation should calculation for burned) x weight in kg
be ____ before burns.
cardioversion. Hint: give the largest amount of Ringer
lactate or NS listed as a choice, it is
38. Chronic therapy for Radiofrequency catheter ablation,
probably the right answer
WPW. curative for WPW.
49. Gastric emptying of 1. Caustics (acids and alkali)
EP studies tell you where anatomic any kind is always 2. Altered mental status
defect is. wrong with? 3. Acetaminophen overdose

39. CO poisoning Dyspnea 50. Gastric lavage is 1. Altered mental status: aspiration
presents with? Lightheadedness dangerous in 2
Confusion instances. 2. Caustic ingestion: burning of the
Seizures esophagus and oropharynx
Death from MI

Page 26
51. Gastric lavage When ingestion very recent (within 30 60. Indications for - Head trauma
utility in poison minutes). stress ulcer - Burns
overdose/treatment. prophylaxis - Endotracheal intubation
Gastric lavage can be attempted up to 2 with PPI. - Coagulopathy (platelets <50k , INR over 1.5) with
hours after ingestion. respiratory failure
61. Initial Methanol - retinal inflammation
Gastric lavage is rarely done.
diagnostic
- Removes 50% of pills at 1 hour
abnormality of Ethylene glycol - hypocalcemia, envelope shaped
- Removed 15% of pills at 2 hours
Methanol vs. oxalate crystals in urine
52. Giving fluids and Also not a good answer. Ethylene
diuretics for Glycol
poisoning/overdose. More patients harmed with pulmonary
62. Intubation of Stridor
edema with this method than helped.
burn patients Hoarseness
53. Head trauma with Head CT first, without contrast. indication with Wheezing
LOC management which exam Burns inside the nasopharynx or mouth
findings?
54. Hemodynamically Chest Pain
unstable definition Dyspnea/CHF 63. Ipecac Usage Never in the hospital. Used prior to coming in the
Hypotension hospital.
Confusion
Delays administration of antidotes if given in the
55. Hemodynamically
hospital.
unstable Vtach
* Ipecac is always a wrong answer
64. Lucid interval Second LOC occurring soon after initial LOC.

Both epidural and subdural hematoma associated


with lucid interval.
65. Major bleeding Intracranial hemorrhage
from warfarin
is defined as? Requiring a transfusion
Perform electrical cardioversion several
times, followed by medications such as 66. Management of Airway management and positive pressure
amiodarone, lidocaine, or procainamide. Drowning ventilation

56. Hyperventilation in Hyperventilation -> Decreases pCO2 ->


NOT STEROIDS OR ANTIBIOTICS
Hematoma constriction of cerebral circulation.
-> DEC volume and decreases 67. Management of CPR
pressure. pulseless
activity
Hyperventilation briefly slows herniation 68. Management of
and is a bridge to surgery. VT is based on
57. Hypo/hyperklamia - HYPOkalemia leads to digoxin toxicity the
> digoxin toxicity hemodynamic
Digoxin toxicity can lead to status.
Digoxin toxicity -> HYPERkalemia
hypo/hyperkalemia 1. Pulseless VT: Same way as VF

58. Hypokalemia and Hypokalemia predisposes to digoxin 2. Hemodynamically Stable: Amiodarone,


Digoxin toxicity because of similar binding sites lidocaine, procainamide, THEN cardiovert.
on the sodium/potassium ATPASE.
3. Hemodynamically unstable: Cardiovert several
when less K+ is bound, more digoxin is times, followed by meds such as amiodarone,
bound lidocaine, or procainamide
59. If amount of Get a drug level 69. Mannitol Osmotic diuretic that decreases intravascular
ingestion of volume.
acetaminophen is
unclear, next step? DEC ICP but has only a limited benefit.
70. Medications Digoxin and CCB (DANGEROUS), block normal AV
Contraindicated node and promote alternate pathways
in WPW.
Page 27
71. Methemoglobinemia Oxidized hemoglobin that is locked in the 82. Most likely
ferric state. arrhythmia:

Oxidized hemoglobin is brown and will not Palpitations,


carry oxygen dizziness, Atrial
lightheadedness
72. Most accurate test Digoxin level!
Exercise A.fib (irregularly irregular) most common
for digoxin toxicity
intolerance/dyspnea arrhythmia in the Untied States
73. Most accurate test Lead level Embolic stroke
for lead poisoning. - lead interferes with hemoglobin
83. Most likely dx:
production (sideroblastic anemia)
+/- focal findings
74. Most accurate test +/- lucid interval
for sideroblastic Arterial, biconvex
anemia or lens shaped
hematoma

Prussian blue stain, detects increased


iron built up in RBC mitochondria
Epidural
75. Most accurate test Cardiac electrophysiology (EP) studies Drain large ones
for WPW.
84. Most likely dx:
76. Most accurate test Level of carboxyhemoglobin since
in carbon monoxide routine oximetry will be falsely normal Abdominal pain
toxicity Renal tube toxicity
Low bicarb, low pH (metabolic acidosis), (ATN)
CO high Anemia
77. Most common Atrial tachycardia with variable AV block Peripheral
arrhythmia in neuropathies such
digoxin toxicity as wrist drop
CNS abnormalities
78. Most common CO poisoning
such as memory
cause of death in - gas heaters or wood-burning stoves
loss and confusion Lead poisoning
fires. - automobile exhaust in an enclosed
- lead colic
environment
- sideroblastic anemia
79. Most common Infection - give prophylactic topical
85. Most likely dx and Malignant hyperthermia
cause of death antibiotics (silver sulfadiazine) NOT IV
tx:
several days to antibiotics
Tx: Dantrolene
weeks after burn.
Anesthetics
Infection with staphylococcus
administered
80. Most common Local wound systemically
injury from snake - most bites not deep enough to deliver Elevated
bites is the ___. venom to the bloodstream temperature
- proteases and lipase in the venom Elevated CPK and
damage tissue locally potassium
81. Most common 1. GI problems (N/V, abdominal pain) 86. Most likely dx and Neuroleptic malignant syndrome
presentation for 2. Hyperkalemia (inhibition of Na/K tx:
digoxin toxicity ATPase) Dantrolene or dopamine agonist:
3. Confusion Antipsychotic Bromocriptine or cabergoline
4. Visual disturbance such as yellow medications
halos around objects Elevated
5. Rhythm disturbance (bradycardia, temperature
atrial tacycardia, AV block, ventricular Elevated CPK and
ectopy, and arrhythmias such as a. fib potassium
with a slow rate)
Page 28
87. Most likely Heatstroke 91. Most likely dx:
dx and tx: Head trauma
Tx with IV fluids; evaporation Rarely focal
Exertion No lucid interval
High outside Ecchymoses on CT
temperatures
Elevated
body temp
Elevated
CPK and
potassium
88. Most likely Heat cramps/exhaustion
dx and tx:
Tx with oral fluids and electrolytes
Exertion Contusion
High outside Tx: No specific treatment; observe in
temperatures hospital
Normal body
temp Severe may need surgical debridement
Normal CPK
92. Most likely dx:
and
potassium
Intoxicated person
level
Low body
89. Most likely Concussion temperature
dx and tx: Tx: No specific tx, observe at home for lucid J waves on EKG
Head trauma interval or new focal findings
No focal
finding Wait 24 hours before returning to sports/work
No lucid
interval Hypothermia
Normal CT
93. Most likely dx:
90. Most likely
dx: SVT alternating
Head trauma with ventricular
+/- focal tachycardia
findings SVT that gets
+/- lucid worse after
interval diltiazem or Wolff-Parkinson-White Syndrome
Venous, digoxin - Preexcitation syndrome with early
crescent Observing the depolarization of the ventricle
delta wave on the
EKG
94. Most likely dx: Aspirin overdose

Tinnitus and
hyperventilation
Respiratory
alkalosis
progressing to
metabolic acidosis
Subdural
Rental toxicity and
Tx: Drain large ones
altered mental
status
Increased anion
gap

Page 29
95. Multifocal atrial 104. Pulseless - Tamponade
tachycardia (MAT) electrical activity - Tension pneumothorax
associated with. Tx. caused by? - Hypovolemia and hypoglycemia
DO not give which - Massive pulmonary embolus (PE)
medication? - Hypoxia, hypothermia, metabolic
acidosis
- Potassium disorders, either high or low
105. Pulseless
Electrical
Activity (PEA)
Associated with chronic lung Heart is electrically normal, but no motor
what does the
disease/COPD contraction.
EKG look like?

Treat MAT like AFib, avoid BB (lung Normal EKG, no pulse


disease)
Heart may contract, no blood inside the
Three different p waves chambers = no CO = pulseless.

96. Nerve gas and Through the skin 106. Rabbies vaccine - When an animal has altered mental
organophosphates are needed only if? status/bizarre behavior
absorbed ___.
- If attack was unprovoked by a stray
97. Normally atrium 10-15% to CO normal
dog that cannot be observed or
contributes ___ to CO
diagnosed
and diseased heart can 30-50% in diseased heart
be up to ___. 107. Risk of Left ventricular function is the most
recurrence of important correlate of the risk of
98. Organophosphate/nerve Prevents breakdown of acetylcholine
ventricular recurrence = ECHO
gas etiology
tachycardia.
Causes:
- Salivation 108. Second degree
- Lacrimation Type I
- Diarrhea
- Polyuria
- Bronchospasm, bronchorrhea, and Mobitz I or Wenckebach
respiratory arrest if severe - progressively lengthening that leads to
a dropped beat.
**Nerve gas is more severe and faster
than organophosphates. Commonly a part of normal aging.
Do not tx if asx.
99. Osmolar Gap Increased in methanol, ethylene glycol,
ethanol 109. Second degree
Type II
Expected osmolarity:
Serum osm = 2Na + BUN/2.8 +
glucose/18
Mobitz II - just drops a beat with no
100. Overdose of No therapy possible
lengthening (warning).
acetaminophen more
than 24 hours ago what
Mobitz II can progress into a third degree
is the tx?
AV block. EVERYONE GETS A
101. Patient presents with EKG PACEMAKER even if asx.
pulse of 40. Next step.
110. Similarities with - Produce intoxication
102. Presentation of the Methanol - ocular toxicity Methanol and - Metabolic acidosis
Methanol vs. Ethylene Ethylene Glycol - Increased anion gap
Glycol Ethylene Glycol - renal toxicity - Osmolar gap
103. __ prevents stroke Nimodipine
Treated wtih Fomepizole and dialysis
after subarachnoid
hemorrhage. 111. Sinus NONE, HR < 60
bradycardia tx in
asx pt.

Page 30
112. Sodium Bicarbonate protects heart against 121. SVT Tx
bicarbonate in TCA arrhythmia, has no effect on increased
overdose urinary excretion (as in aspirin)
113. Sources of Methanol - wood alcohol, cleaning
Methanol vs. solutions, paint thinner
Ethylene Glycol
Ethylene glycol - antifreeze
114. Specific types of Salt: acts like CHF - wet, heavy, lungs
drowning salt
1. Valsalva - carotid massage, Valsalva,
versus fresh Fresh: causes hemolysis from
dive reflex, ice immersion
water. absorbing hypotonic fluid into the
2. Adenosine if vagal maneuver don't
vasculture
work
115. Steroids benefit in DO NOT benefit intracranial bleed. 3. BB (metoprolol), CCB (diltiazem) or
intracranial bleed. digoxin if adenosine not effective
DEC edema around mass lesions.
122. Symptoms of - Dyspnea and cyanosis
116. Strongest CNS and cardiac involvement methemoglobinemia - HA, confusion, and seizures
indication for - Metabolic acidosis
digoxin specific
123. TCA toxicity Anticholinergic effects:
antibodies.
symptoms. - Dry mouth
117. Subdural and Can only be distinguished with CT. - Constipation
epidural - Urinary retention
hematoma. Epidural related to head fracture.
124. TCA toxicity Seizures and arrhythmia
118. Subdural and Based on size and sx. symptoms leading
epidural tx to death. - wide QRS
Large hematomas managed with:
125. Third degree AV
1. Intubation and hyperventilation
block Tx
2. Mannitol
3. Drainage
119. A sudden loss of a Asystole
pulse can be V. fib
caused by? V. tachy
Pulseless electrical activity (PEA)
120. Surface area Pacemaker (most effective tx for
percentages of bradycardia)
burn victims
126. Toxic metabolite in Methanol - formic acid/formaldehyde
Methanol vs.
Ethylene Glycol Ethylene glycol - oxalic acid/oxalate
127. Treatment of Snake - Pressure
Bites - Immobilization decreases movement of
venom
- Tx with antivenom

Don't tourniquet, ice, or incision/suction


especially by mouth.
128. Tricyclic overdose Benzodiazepines.
Head - 9%
suppression of
Arms - 9% each
seizures with? So if you reverse benzos with flumazenil
Legs 18% each
and the pt ingestion a lot of TCAs you
Chest or back - 18% each
open them up for seizing.

As an estimate: each hand width is 1% 129. True/False: TRUE


of BSA Alcoholism
decreases the
amount of
acetaminophen
needed to cause
toxicity.
Page 31
130. True/False: True, acute withdrawal can 139. Tx of lead
Benzodiazepine overdose cause seizures poisoning
by itself is not fatal. Do
not give flumazenil.
131. True/False: Steroids TRUE
prevent injury from
caustics.
132. TRUE/FALSE: Warfarin is TRUE
less effective and more
dangerous than
alternatives in preventing
stroke in a.fib.
133. Tx for Caustics (drain Lots of fluid, flush out the Chelating agents.
cleaner, acids, alkali) caustics. - Succimer is the only oral form of lead
chelator.
Do not reverse the agent with - Ethylenediaminetetraacetic acid (EDTA)
alkali or acid. Giving the and dimercaprol (BAL) are parenteral
opposite will cause an agents.
exothermic reaction and make
140. Ventricular
the perforation/damage worse
fibrillation and
pulseless VT best
caustics ingestion causes Unsynchronized
initial therapy.
mechanical damage to cardioversion(defibrillation) followed by
oropharynx, esophagus, and CPR
stomach including perforation
141. VF management Shock, drug, shock, drug, shock, drug,
134. Tx for digoxin toxicity Control potassium and digoxin- CPR at all times in between the shocks.
specific antibodies (remove
142. Volume Lactate ringers/NS
digoxin from circulation)
replacement in
135. Tx for mercury poisoning. No therapy to reverse pulmonary burns. 1/2 required in the first 8 hours
toxicity 1/4 in second 8 hours
1/4 in third 8 hours
Chelating agents: dimercaprol
and succimer can be effective 4 mL for each percent of body surface
136. Tx for post MI ventricular Fix ischemia, do an angiography area (BSA)
tachycardia. for angioplasty or bypass 143. VT with a pulse SYNCHRONIZED CARDIOVERSION
- ventricular arrhythmia is the
144. What to do with N-acetylcysteine
MCC of death in 72 hours
toxic levels of
surrounding an acute MI
acetaminophen
137. Tx of Aspirin Toxicity Alkalizing with sodium (more than 8-10
bicarbonate to increase rate of grams)?
aspirin excretion.
145. When is a When the overdose is from a suicide
138. Tx of hemodynamically Synchronized cardioversion psychiatric attempt, but is the wrong answer on
unstable atrial (prevents deterioration into VT consultation USMLE when specific antidotes and
arrhythmias and VF) indicated in an diagnostic test are needed
overdose?
146. When the answer Acetaminophen
is not clear and Aspirin
the cause of the
overdose is
asked, say ___ or
___ they are, by
far, the most
common cause of
death by
overdose.

Page 32
147. When to give amiodarone/lidocaine (VF). Given after 2 shocks, epi, shock.

Amiodarone first choice or lidocaine.

Give Magnesium with ventricular arrhythmia w/o waiting for a level.


148. When to give epinephrine during arrest (VF)? Two unsynchronized cardioversions, then epinephrine/vasopressin followed by
another electrical shock.
149. When to give precordial thump? Very recent onset of arrest (less than 10 min.) with no defibrillator available

You know its recent because you saw it happen ("witnessed")

.. in short pretty much never.


150. When you don't know what to do in toxicology, Charcoal
give ___.
151. Which drugs can cause methemoglobinemia? - Benzocaine and other anesthetics
- Nitrites and nitroglycerin
- Dapsone
152. Whole bowel irrigation for Placing a gastric tube and flushing out the GI tract w/ polyethylene glycol-electrolyte
poisoning/overdose. solution (GoLYTELY) is almost always wrong

Limited to massive iron ingestion, lithium, and swallowing drug filled packets
(smuggling)
153. Without anticoagulation, there will be about 6 embolic strokes per year for every 100 patients (6%)
___ with atrial fibrillation. - when INR maintained between 2-3 rate 2-3%

Page 33
MTB Endocrinology
MTB Q and A

1. MC causes of pituitary disorders: - Tumors (metastatic cancer, 8. What is the response on pituitary hormones with metyrapone
adenomas, Rathe cleft cyst, meningioma, craniopharyngioma, and insulin?:
lymphoma)
- Trauma
- Radiation
- Infection (TB, sarcoid, histiocytosis)
- Fungi
- Parasites
- Hemochromatosis
2. Deficiency in LH and FSH in women vs men.: Women:
amenorrhea, will not ovulate or menstruate
Metyrapone
• Inhibits 11-beta hydroxylase and DEC cortisol
Men: will not make testosterone or sperm, DEC libido, decease
• Normal: ACTH and 11deoxycortisol levels rise
axillary, pubic, and body hair, ED, decreased muscle mass
3. What is Kallman syndrome and key clinical signs?: DEC GnRH
Insulin stimulation
-> DEC FSH and LH
• Normal: decreased glucose levels raise GH
Anosmia
• Failure of GH to rise in response to insulin indicates pituitary
Renal agenesis in 50%
insufficiency
4. GH deficiency in children.: Short stature
9. How do you replace the following hormones:
Dwarfism
1. Cortisol
5. GH deficiency clinical signs in adults.: Central obesity 2. T3, T4
Increased LDL and cholesterol 3. Testosterone, estrogen
Reduced lean muscle mass 4. GH: 1. Cortisone
6. Diagnostic tests panhypopituitarism.: 2. Thyroxine
3. Testosterone and estrogen
4. Recombinant human growth hormone
10. T/F: Replace cortisone before starting thyroxine.: TRUE
11. Two Modes of DI: Central DI (Low ADH)
- Stroke, tumor, trauma, hypoxia
- Infiltration (sarcoidosis, hemochromatosis)
- Infection

Nephrogenic DI (poor response)


• Chronic pyelonephritis, sickle cell disease
Hyponatremia common 2/2 hypothyroidism and isolated
• Amyloidosis, myeloma
glucocorticoid underproduction
• Lithium
- Potassium levels normal due to aldosterone not being affected
• Hypercalcemia or hypokalemia inhibits ADH effect

MRI detects compressing mass lesions on the pituitary 12. Clinical symptoms of DI: High volume of urine
Increased thirst
7. What tests can you use to check the following hormones:
Hypernatremia 2/2 volume depletion
1. TSH, T3, T4
2. ACTH, cortisol
Severe Hypernatremia
3. LH, FSH, testosterone
- confusion, seizures, coma
4. GH
- Only when volume losses are unmatched by fluid intake
5. Prolactin: 1. TRH
2. cosyntropion or CRH (cosyntropin stimulates the adrenal to
release more cortisol)
3. None
4. GHRH, arginine infusion, IGF
5. TRH

Page 34
13. Diagnostic tests DI.: 19. Best initial test acromegaly.:

Insulin-like growth factor (IGF-1)


• Urine osmolality: Low
• Urine sodium: Low 20. Most accurate test acromegaly.: Glucose suppression test.
• Serum osmolality: High - Normal: glucose should suppress GH levels
• Urine volume: Enormous
MRI should be done after the labs ID acromegaly
14. DI vasopressin stimulation response Central vs.
Nephrogenic.: Central DI: 21. Acromegaly treatment.: 1. Surgery: transphenoidal resection
• Urine volume decrease & urine osmolality increase of the pituitary
2. Medications:
Nephrogenic DI: - Cabergoline: Dopamine agonist inhibits GH release
• No effect of vasopressin on urine volume or osmolality - Octreotide or lanreotide: Somatostatin inhibits GH release
- Pegvisomant: GH receptor antagonist, inhibits IGF release
15. Determining Nephrogenic vs. Central DI and Tx:
from liver
3. Radiotherapy: when there is no response to sx or
medications
22. Causes of Hyperprolactinemia: May have nothing to do with a
pituitary adenoma.

Prolactin can be cosecreted with GH


Hypothyroidism with pathologically high TRH levels

Physiologic causes: pregnancy, intense exercise, renal


insufficiency, INC chest wall stimulation, cutting pituitary stalk
Vasopressin challenge will have no effect on NDI.
Drugs: antipsychotics, methyldopa, metoclopromide, opioids,
Central Tx: Vasopressin
TCA, verapamil
23. What is the only CCB that can raise prolactin levels?:
Nephrogenic Tx: HCTZ, NSAID (prostaglandin inhibitor such as
Verapamil
Indomethacin), Amiloride
24. What is NEVER an initial test for any endocrine problem.: MRI
16. Acromegaly almost always caused by ___.: Pituitary
adenoma 25. Hyperprolactinemia presentation women vs men.:
- overproduction of GH leading to soft tissue overgrowth through
the body
- Part of MEN when combined with parathyroid and pancreatic
disorders (gastrinoma/insulinoma)
17. Acromegaly Clinical Sx: - INC hat, ring, and shoe size
- Carpal tunnel syndrome + obstructive sleep apnea
- Body odor from hypertrophic sweat glands
- Coarse facial features and teeth widening jaw overgrowth
- Deep voice and macroglossia
- Colonic polyps and skin tags
- Arthralgias from joints growing out of ligaments
- HTN for unclear reasons (50%)
- Cardiomegaly and CHF
- ED from INC prolactin consecrated with pituitary adenoma
18. Lab tests acromegaly show?: • Hyperglycemia Women: galactorrhea, amenorrhea, infertility
• Glucose intolerance
• Hyperlipidemia Men: ED, DEC libido, gynecomastia

-> contribute to the cardiac dysfunction

Page 35
26. High Prolactin Lab Workup.: Thyroid Function Test 41. Hyperthyroidism: what is the most likely diagnosis?
Pregnancy Test
BUN/Cr (kidney disease elevates prolactin) High TSH level.: Pituitary adenoma
Liver function tests (cirrhosis elevates prolactin) 42. Dx lab tests hyperthyroidism.:
27. Always exclude __ in any woman with a high prolactin level.:
PREGNANCY
28. MRI in hyperprolactinemia is performed after?: 1. High
prolactin level is confirmed.
2. Secondary causes like medications are excluded; and
3. Patient is NOT pregnant
29. Treatment of hyperprolactinemia.: 1. Dopamine agonists
- Cabergoline is better tolerated than bromocriptine All forms have elevated T4 (thyroxine) level
2. Transphenoidal surgery when NOT responding to medications - only pituitary adenomas have high TSH level, all others TSH
3. Radiation is rarely needed inhibited
30. MC Hypothyroid Cause: Hashimoto's burn out. 43. What is the Tx for:
1. Graves
Other causes: 2. Subacute thyroiditis
Dietary deficiency of iodine 3. Painless thyroiditis
Amiodarone. 4. Exogenous thyroid
5. Pituitary adenoma: 1. Radioactive iodine
31. What is the only process not slowed down by
2. Aspirin
hypothyroidism?: Menstrual flow increases
3. Nothing
32. Tx difference in Hypothyroid with elevated TSH double the
4. Stop use
normal to TSH that is less than double the normal.: Very high
5. Surgery
TSH (more than double normal): Replace with T4
44. Acute Thyroid Storm Tx:
TSH less than double the normal get antithyroid/
peroxidase/antithyroglobulin antibodies. If antibodies are
positive, replace thyroid hormone.
33. Clinical Sx in Hypothyroidism vs Hyperthyroidism:

1. Propranolol: blocks target organ effect and inhibits


peripheral conversion of T4 -> T3

2. Thiourea drugs (methimazole and PTU): block hormone


34. Best initial test hypothyroidism.: TSH production, methimazole preferred
- TSH levels are markedly elevated if gland has failed
35. If TSH level is suppressed in hypothyroidism, measure ___.: 3. Iodinated contrast material (iopanoic acid and ipodate):
free T4 levels blocks T4 to T3 conversion, blocks release of existing hormone
36. Tx hypothyroidism.: Replace with thyroxine (synthroid)
37. Hyperthyroidism: what is the most likely diagnosis? 4. Steroids (hydrocortisone)

Eye Proptosis and skin findings.: Graves 5. Radioactive iodine: ablates the gland for a permanent cure
- TSH receptor antibodies
38. Hyperthyroidism: what is the most likely diagnosis?

Tender thyroid: Subacute thyroiditis


39. Hyperthyroidism: what is the most likely diagnosis?

Nontender, normal exam results.: Painless "silent" thyroiditis


40. Hyperthyroidism: what is the most likely diagnosis?

Involuted gland is not palpable.: Exogenous thyroid hormone


use

Page 36
45. Best Tx for Graves Opthalmopathy: 50. MC Cause of Hypercalcemia: Primary HyperPTH

Other:
Vitamin D intoxication
Sarcoidosis
Thiazide diuretics
Hyperthyroidism
Metastases to bone and multiple myeloma
51. T/F: Primary hyperparathyroidism and cancer account for
90% of hypercalcemia patients.: TRUE
52. Clinical Sx of Hypercalcemia.: Confusion, constipation, stupor
Steroids
and lethargy.
- radiation used if not response to steroids, severe cases may
need decompressive surgery
CV: Short QT syndrome and HTN
46. Thyroid nodules: Bone: Osteoporosis
Renal: Nephrolithiasis, DI, renal insufficiency
53. CV symptoms of hypercalcemia.:

Short QT syndrome and HTN

• 5% women 54. Acute hypercalcemia treated with?: 1. Saline hydration at


• 1% men large volume
• 95% benign 2. Bisphosphonates: pamidronate, zoledronic acid
-Adenoma, colloid nodule, cyst 3. Calcitonin
• Thyroid nodules rarely hyper/hypofunctioning 55. 75-year-old man with history of malignancy admitted with
47. Dx test thyroid nodules.: Thyroid nodule > 1 cm: Biopsy with lethargy, confusion, and abdominal pain. Found to have a
FNA if normal thyroid function (TSH/T4) markedly elevated calcium level. After 3L normal saline and
pamidronate, his calcium level is still markedly elevated the
Nodules in those who are euthyroid should be biopsied. following day.

No need US or radionuclide scanning because cannot exclude What is the most appropriate next step in management?
cancer. a. Calcitonin
b. Zolendronic acid
48. When a patient has a thyroid nodule. What is the next best
c. Plicamycind.
step?: 1. Perform thyroid function testing (TSH and T4)
e. Dialysis
2. If tests are normal, biopsy the gland.
f. Cinacalcet: a. Calcitonin
49. 46-year-old woman with thyroid nodule and normal thyroid - inhibits osteoclasts, onset of action is very rapid,
function testing has a biopsy showing "indeterminate for bisphosphonates take several days to work
follicular adenoma."
56. Tx hypercalcemia caused by sarcoidosis or any
granulomatous disease.: Prednisone
What is the most appropriate next step?
a. Neck CT 57. Causes of primary hyperparathyroidism.: • Solitary adenoma
b. Surgical removal (excisional biopsy) (80%-85%)
c. Ultrasound • Hyperplasia of all 4 glands (15%-20%)
d. Calcitonin levels: b. Surgical removal (excisional biopsy) • Parathyroid malignancy (1%)
- follicular adenoma is a histo reading cannot exclude cancer, 58. Presentation of hyperparathyroidism.: Elevation in calcium
only way to exclude cancer is remove the entire nodule levels often asymptomatic.

When symptomatic, slower manifestations:


• Osteoporosis
• Nephrolithiasis and renal insufficiency
• Muscle weakness, anorexia, N/V, and abdominal pain
• PUD (calcium stimulates gastrin) Page 37
59. PUD and HyperPTH Relationship: Calcium stimulates Gastrin. 70. Define Cushing SYNDROME and Cushing Disease: SYNDROME
60. Dx tests hyperparathyroidism.: INC Ca2+ and PTH - hypercortisolism
LOW phosphate Disease - pituitary overproduction of ACTH
INC chloride 71. Causes of hypercortisolism:

EKG short QT
Elevated BUN/Cr

Alkaline phosphatase INC from the effect of PTH on bone


61. T/F: Bone x-ray is a good way to test for bone effect of PTH.:
FALSE, not a good test for bone effect of high PTH, DEXA
densitometry is better.
62. When is preoperative imaging of the neck with sonogram or
nuclear scanning useful in tx hyperPTH?: Helpful in 72. Clinical presentation of hypercortisolism.: - Fat
determining the surgical approach redistribution: "Moon face," truncal obesity, "buffalo hump," thin
- tx: surgical removal of the parathyroid glands extremities, increased abdominal fat
- Skin: Striae, easy bruising, decreased wound healing, thinning
63. When surgery is not possible to remove parathyroid glands
of skin
give?:
- Osteoporosis
- HTN: INC Na reabsorption in kidney and INC vascular
reactivity
- Menstrual disorders
- Erectile dysfunction
- Cognitive disturbance: from DEC concentration to psychosis
- Polyuria: hyperglycemia + INC free water clearance
73. Best Initial Test for Hypercortisolism (PRESENCE):

Cinacalcet
64. E -/+ Abnormality Causing HypoPTH: Hypomagnesemia needed
for PTH release
65. HypoPTH MCC: 1. Prior Neck Surgery (MCC) - primary
hypoPTH

2. Hypomagnesemia needed for PTH release, low Mg lead to 24 hour urine cortisol (establishes presence)
INC urinary loss of calcium
Alternative: 1 g Dexamethasone suppression test (should
3. Renal failure leading to hypoCa, kidney converts 25- normally suppress morning cortisol level)
hydroxyD to more active 1,25 hydroxy-D
False positives 1 mg overnight suppression test caused by:
Other causes: - Depression
Vitamin D deficiency, genetic disorders, fat malabsorption, low - Alcoholism
albumin states - Obesity
66. For every point decrease in albumin, the calcium level
decreases by ___.: 0.8

low albumin causes decrease in total calcium, but free calcium


is normal
67. Signs of HypoCa: Neural hyperexcitability--
- Chvostek sign (facial nerve)
- Carpopedal spasm
- Perioral numbness
- Mental irritability
- Seizures
- Tetany (Trousseau sign)
68. Dx test hypocalcemia.: EKG shows prolonged QT that may
eventually cause arrhythmia

Slit lamp shows early cataracts


69. Tx hypocalcemia.: Replace calcium and vitamin D
Page 38
74. Best Initial Test for Hypercortisolism (SOURCE): 79. Confirmatory Lab findings in adrenal disorders.:

80. Causes of Addison: 1. Autoimmune destruction (80%)


2. Infection (TB)
1. Check ACTH (elevation indicates pituitary or ectopic) 2. Adrenoleukodystrophy
4. Mets to adrenal gland
2. Dexamethasone can suppress pituitary, MRI needed if ACTH
81. Causes of Acute Adrenal Crisis: - Sudden removal of chronic
doesn't suppress (pituitary lesion)
high dose prednisone (steroid)
- Hemorrhage
3. If MRI does not show clear pituitary lesions, sample the
- Surgery
inferior petrosal sinus for ACTH after stimulation with CRH
- Hypotension
since some pituitary lesions are too small to be detected on
- Trauma
MRI
75. If the ACTH is elevated, and you cannot find a defect in the *less common to have an acute adrenal crisis from loss of
pituitary either by MRI or by sampling of the petrosal sinus, pituitary because aldosterone is not under the control of ACTH
___ looking for ectopic source of ACTH production.:
82. Presentation hypoadrenalism: - Weakness
- Fatigue
- Altered mental status
- N/V, anorexia
- HYPOtension
- HYPOnatremia
- HYPERkalemia
- Hyperpigmentation from chronic adrenal insufficiency develops
over long peered of time

Acute adrenal crisis: profound hypotension, fever, confusion,


Scan the chest coma
- must always confirm the source of hypercortisolism with 83. Electrolyte changes in Hypocortisolism (Addison's):
biochemical testing before perform imaging studies Hypoglycemia
76. Electrolyte abnormalities in Hypercortisolism: Cortisol has an Hyperkalemia
anti-insulin effect. Metabolic Acidosis
Hyperglycemia Hyponatremia
Hyperlipidemia High BUN

Aldosterone like effects of cortisol: Eosinophilia is common in hypoadrenalism


- Hypokalemia 84. The most specific test of adrenal function in hypoadrenalism.:
- Metabolic Alkalosis Cosyntropin test
- synthetic ACTH
Leukocytosis from demargination of WBC. - measure cortisol before and after administration of cosyntropin
77. Tx hypercortisolism: • Surgically remove source of - Normal: rise in cortisol level after giving cosyntropin
hypercortisolism 85. Prior to testing for acute adrenal crisis what should be done
- Transsphenoidal surgery for pituitary sources first?: Treat!
- Laparoscopic removal for adrenal sources 1. Replace steroids with hydrocortisione.
78. How far should you go in the evaluation of an unexpected, 2. Fludrocortisone is a steroid like hormone high in
asymptomatic adrenal lesion found on CT?: 1. Metanephrine of mineralocorticoid and aldosterone-like effect, useful in patient
blood and urine to exclude pheochromocytoma with postural instability.
2. Renin and aldosterone to exclude hyperaldosteronism
3. 1 mg dexamethasone suppresion to r/o subclinical Cushings

Page 39
86. Hyoadrenalism diagnostic test algorithm.: 97. Best initial test Pheochromocytoma: Level of free
metanephrines in plasma

Confirm: 24 hour urine collection of metanephrines more


sensitive than urine vanillylmandelic acid

Other useful tests: epinephrine and norepinephrine direct


measurements
98. Imaging of adrenal glands (CT or MRI) in pheochromocytoma
when should they be done?: After biochemical testing
99. Nuclear isotope scan
87. Sx for Primary Hyperaldosteronism: High BP + Hypokalemia
Detects location of pheochromocytoma that originates outside
88. Etiology primary hyperaldosteronism: Solitary adenoma: 80%
adrenal gland:
Bilateral hyperplasia:15-20%
Malignant: 1%

"The autonomous overproduction of aldosterone despite a high


pressure with a low renin activity"
89. Presentation secondary HTN are more likely in those whose
onset?: 1. Is < 30 or above 60 years of age
2. Is not controlled by 2 anti-HTN meds
3. Has characteristic finds on Hx, PE, labs

HTN, low K+, muscular weakness, DI from the hypokalemia


90. Best initial test primary hyperaldosteronism: Ratio plasma MIBG scanning
aldosterone: plasma renin
100. Pheochromocytoma tx: 1. Phenoxybenzamine: (IV alpha
blocker) best initial tx
A low plasma renin with high aldosterone =
2. Propranolol
Primary hyperaldosteronism
3. Calcium-channel blocker (possible)
91. Most accurate test primary hyperaldosteronism to confirm 4. Laparoscopic removal
unilateral adenoma.: Sample of venous blood draining the
101. Diabetes Dx: >126 on 2 sep occasions (fasting)
adrenal
>200 w/ symptoms (random)
- high aldosterone level
A1C >6.5
92. Role CT scan in primary hyperaldosteronism.: CT scan of the
102. Type 1 DM onset, relationship to obesity and insulin.: - Onset
adrenals should only be done after biochemical testing:
in childhood
- Low K+
- Insulin dependent from early age
- High aldosterone despite a high-salt diet
- Not related to obesity
- Low plasma renin level
- Insulin deficiency
93. Tx primary hyperaldosteronism: 1. Unilateral adenoma is
103. Type 2 DM onset, relationship to obesity and insulin.: - Onset
resected by laparoscopy
in adulthood
2. B/L hyperplasia is treated with eplerenone or spironolactone
- Directly related to obesity
94. SE spironolactone: Gynecomastia and DEC libido because it is - Insulin resistance
antiandrogenic
104. Polyuria
95. ___ nonmalignant lesion of the adrenal medullar Polyphagia
autonomously overproducing catecholamines despite a high Polydipsia
BP.: Pheochromocytoma DEC wound healing: DM
96. HTN that is episodic in nature - DM1 thinner < DM 2
Headache - DM1 more likely to get DKA
Sweating - DM2 less likely to have polyphagia
Palpitations + tremor 105. Best initial drug therapy DM2.: Metformin (blocks
gluconeogenesis) - wt loss
Dx?: Pheochromocytoma
Sulfonylureas - cause wt gain, INC insulin release from
pancrease
106. When NOT to give Metformin?: Renal failure. Can cause
metabolic acidosis.

Page 40
107.___ block the metabolism of incretins, also called glucose 116. Patient presentation DKA.:
insulinotropic peptide (GIP) and glucagon-like peptide (GLP).:

• Hyperventilation, Altered mental status


• Metabolic acidosis with increased anion gap
• Hyperkalemia in blood, but DEC total body potassium because
of urinary spillage
• INC anion gap on blood testing
DDP-IV inhibitors (sitagloptin, saxagliptin, linagliptan,
• Serum is positive for ketones
alogliptan)
• Nonspecific abdominal pain
- incretins (GIP and GLP) INC insulin release and DEC glucagon
• "Acetone" odor on breath
release from the pancreas, secreted in bloodstream when good
• Polydipsia, polyuria
enters the duodenum and metabolized by DPP-IV, incretins with
117.57-year-old man admitted to ICU with altered mental status,
DPP-IV inhibitors increases the half life of incretins
hyperventilation, and markedly elevated glucose level.
108.___ are a direct replacement of incretins.: Incretin mimetics
Which of the following is the most accurate measure of the
(exanatide, lariglutide, albiglutide)
severity of his condition?
- slow gastric motility and decrease weight
109.Glitazones (Thiazoladinedinones) CI: Contraindicated in CHF a. Glucose level
because they cause fluid overload. b. Serum bicarbonate
- Nateglinide and repaglinide are similar to sulfonylureas but do c. Urine ketones
not contain sulfa d. Bloodketones
110.Alpha-glucosidase MOA: Block glucose absorption in the e. pH level on blood gas: b. Serum bicarbonate
bowel. Cause flatus, diarrhea, abdominal pain. Can be used with - if very low, the patient is at risk for death
renal insufficiency. - serum bicarb is a way of saying "anion gap"
- if bicarb low, anion gap INC
Half point decrease in HgA1c. 118. Tx of DKA: Large-volume saline and insulin replacement

Acarbose, miglitol Replace K+ when the K+ level comes down to a level


111.What is the action of pramlintide?: Its an analog of amylin approaching normal
that is secreted normally with insulin, amylin decreases gastric
emptying, decreases glucagon levels and decreases appetite Correct the underlying cause: noncompliance with meds,
112.T/F: Metformin does not cause hypoglycemia.: TRUE, safest infection, pregnancy, or any serious illness
drug to start in newly diagnosed diabetics 119.DM goals A1C <__ %, BP <___. LDL<___.: A1C<7%, BP
113.SE of Exenatide (Incretin/Gliptin family): May cause (<140/90), LDL<100
pancreatitis 120.All patients with DM should receive.: • Pneumococcal vaccine
114.Glargine: Large part of the day (<24 hrs) • Yearly eye exam to check for proliferative retinopathy, which
- steady state added patient not controlled oral hypoglycemic needs laser therapy
agents, more steady state than NPH • Statins if LDL > 100 mg/dL
115. Time durations of the insulins: • ACEi or ARBs if BP is > 140/90 mmHg
• ACEi or ARB if urine tests positive for microalbuminuria
• Aspirin, used regularly in all diabetic patients above the age of
30
• Foot exam for neuropathy and ulcers
121.Microalbuminuria: 30-300 mg/24 hrs
- DM should be screened annually for microalbuminuria
Rapid 3-4 hours
Regular 6-8 hours
Start on an ACE-I or ARB when present. Decrease rate of
NPH 10-20 hours
diabetic nephropathy.
Glargine 24 hours
122.Tx for gastroparesis: Metoclopromide and erythromcyin,
which increase motility.

Gastroparesis: immobility of bowels due to DM DEC the ability of


the gut to sense stretch
Page 41
123. E -/+ disturbances that influence ADH effect on kidney: Hypercalcemia and hypokalemia
124. Pegvisomant Tx: For Acromegaly, GH receptor antagonist, inhibits IGF release from liver.
125. Thyroid Cancer with Increased Calcitonin: Medullary thyroid cancer
126. Metformin MOA: Block gluconeogenesis
127.Sulfonylureas MOA: Not first line.
Increase insulin release from pancreas, driving glucose intracellularly and increasing obesity.
Glinides work the same but don't have sulfa in them.
128.Serum Bicarb and Anion Gap in DKA: Severity of DKA best measured with bicarb. If bicarb is low, patient at risk for death. If bicarb is
low, anion gap is increased.
129. Systemic Effects of Thyroid Adenoma: Bone loss (increased osteoclast activity)
130.Type of retinopathy in DM.: Non-proliferative retinopathy
• Tighter control of glucose
• Aspirin doesn't help retinopathy

Proliferative retinopathy
• Neovascularization and vitreous hemorrhages
• Treated with laser photocoagulation
131.• Decreased sensation in feet
• Main cause of skin ulcers
• Leads to osteomyelitis
• Treatment pain with

Tx with?: Diabetic neuropathy


- damages the vasonervorum that surrounds large peripheral nerves

Tx: Pregabalin, gabapentin, or TCAs

Page 42
MTB Gastroenterology
MTB Q and A

1. Define: 7. In the esophagus, only ___ and ___ are dx by biopsy.: Cancer
Barrett esophagus
Dysphagia 8. Most likely Dx:
Odynophagia -dysphagia first solids, then progressing liquids
Dyspepsia: - Difficulty swallowing -association with prolonged tobacco or alcohol use
- Pain while swallowing ->5-10 years GERD
- Indigestion ->50 y/o: Esophageal cancer
2. Reflux-Dyspepsia-Epigastric Pain -progressive dysphagia is key "from solids to liquids"
+ what symptoms Indicate 9. Esophageal Cancer Dx test:
Endoscopy 1st?: 1. Wt. loss
2. Blood in stool
3. Anemia
4. Dysphagia
5. >50yo
3. Most Likely Dx:
-progressive dysphagia to both solids and liquids at same
time
-no association w/ tobacco or alcohol
-young pt under 50: Achalasia: LES can't relax due to loss of
nerve plexus.
- Aperistalsis of esophageal body Endoscopy w/ BIOPSY
(-can do barium first initial test, but not diagnostic
4. Recent travel to South America and new onset dysphagia?:
-CT/MRI helpful, not diagnostic (never first answer)
Chagas Disease
-PET scan- can detect metastasis and if cancer is resectable)
5. Achalasia Dx Findings:
10. TX esophageal CA: 1. Surgical resection - always the thing to
Barium esophagram:
try
Manometry:
2. Chemo + radiation are used in addition to surgical removal
CXR:
Upper Endoscopy:: 11. ___ is used for lesions that cannot be resected surgically just
to keep the esophagus open for palliation and to improve
dysphagia.:

BE-"bird's beak," - do first

Manometry - INC pressure (most accurate)

CXR - widening of esophagus, never first test


Stent placement
Upper Endoscopy- Normal
6. Tx Achalasia:

1) Pneumatic Dilation
2) Surgical sectioning or myometry
3) Botulinum toxin injection - repeat 3-6 mo.
No Drugs Page 43
12. Chest pain assoc w/ drinking cold liquids, gets better w 17. ___ is often from acid reflux and is associated with hiatal
nitrates, not related to exertion hernia.:
-next best steps in order?
-diagnostic test?:

Schatzki ring
- scarring or tightening (peptic stricture of the distal esophagus
18. Reflux + intermittent dysphagia + no pain
Dx?
associated with?
Tx?:
Esophageal spasm
#1- EKG + stress test = normal
- Esophagram + endoscopy = normal
2. Manometry - Most accurate
- abnormal contraction in various sections of the esophagus
13. DES vs. Nut-cracker esophagus
test?
findings?: Manometry
DES - disorganized contractions
Nutcracker- incr LES amplitude & baseline tone

BOTH have sudden onset of chest pain and can be precipitated


by drinking cold liquids.
-Schatzki's Ring
14. Tx DES and Nut-cracker esophagus: CCBs and nitrates
(scarring & tightening of the distal esophagus)
-assoc w/ hiatal hernia
- PPi can improve number of cases of spastic dz
15. Most common cause: AIDS pts presents with dysphagia? Tx?: -tx: Pneumatic dilation
Steakhouse syndrome = dysphagia from solid food associated
with Schatzki ring
19. Triad:
1. Dysphagia due to esophageal webs
2. Glossitis
3. Iron def anemia
Dx?
-complication?:

Esophageal candidiasis

tx: Oral fluconazole


- if does not improve symptoms perform endoscopy
16. These pills can cause esophagitis if in prolonged contact?:
Doxycycline
Alendronate Plummer-Vinson syndrome
KCl - more proximal than Schatzki ring
- common postmenopausal women

dx: Barium esophagram


-esophageal Squamous cell cancer

Page 44
20. Plummer-Vinson dx test? 29. Dx and Tx Mallory-Weiss Tear?:
Tx?: Barium esophagram
Tx: Iron replacement first, which may lead to resolution of
symptoms
21. Out pouching of the posterior pharyngeal constrictor
muscles.
- Dysphagia
- Halitosis
- Regurgitation of food particles:

Endoscopy

Self-limited
- non penetrating tear of only the mucosa
30. Pt vomiting bright red blood after a night of drinking. no
dysphagia
Zenker's Diverticulum
-next step?
22. Zenker's Diverticulum complication?: Aspiration pneumonia
-TX?: (Mallory-weiss tear)
when contents diverticulum end up in the lung
-EGD
- Right lower or right upper lobe pneumonia
-observe
23. Best diagnostic test Zenker diverticulum? Tx?: - epinephrine - if severe & persistent
31. Tx SEVERE cases of Mallory-Weiss tear?: Injection of Epi or
electrocautery to stop bleeding
32. Full thickness tear esophagus and surgical emergency?:

Barium studies: esophagram


Boerhaave's syndrome
- repair w/ surgery, no medical therapy
- alcoholics
24. Which diagnostic modalities are dangerous in Zenker + Hammen's sign: crepitus, subq air = snap crackle and pop
diverticulum?: NG tube placement or upper endoscopy
33. What is the most common cause of epigastric pain,
- may cause perforation
particularly pts under 50?: Non-ulcer dyspepsia
25. Manometry is the answer on usmle which (3)?: Achalasia
34. Epigastric pain worse with food.: Gastric ulcer
(HIGH LES pressure)
- NSAID abuse
Spasm : DES (Diffuse INC) 35. Epigastric pain better with food.: Duodenal ulcer
- release of bicarb relieving the pain
Scleroderma (LOW LES pressure) 36. Weight loss and epigastric pain, elderly.: Cancer, NSAID
26. Pt presents with reflux and progressive systemic sclerosis.: abuse = gastric ulcer
Scleroderma 37. Epigastric Tenderness, hx gallstones/alcohol use: Pancreatitis
- mechanical immobility of esophagus 38. Bad taste mouth worse laying down, cough, hoarse and
27. Scleroderma reflux. epigastric pain.: GERD
Dx? 39. Diabetes, bloating, N/V after eating epigastric pain.:
Tx?: Manometry (DEC LES pressure) Gastroparesis
40. Epigastric pain and nothing in the history.: Non-ulcer
PPIs dyspepsia
- screen for Barrett's esophagus due to high amount of reflux 41. Best DIAGNOSTIC test for epigastric pain.: Endoscopy
28. Pt presents with upper GI bleeding after prolonged or severe 42. Abdominal pain RUQ list some causes?: Cholecystitis
vomiting or retching.: Mallory-Weiss Tear Biliary colic
- repeated retching => hematemesis of bright red blood or by Cholangitis
black stool Perforated duodenal ulcer

Page 45
43. Abdominal pain LUQ list some cases?: Splenic rupture - 54. Tx alternatives for GERD if medications are not working.:
trauma
IBS - splenic flexure syndrome
44. Abdominal pain RLQ list some cases?: Appendicitis
Ovarian torsion
Ectopic pregnancy
Cecal diverticulitis
45. Abdominal pain LLQ list some causes?: Sigmoid volvulus
Sigmoid diverticulitis
Ovarian torsion
Ectopic pregnancy
46. Abdominal pain mid-epigastrium list some cases?:
Pancreatitis
Aortic dissection
Peptic ulcer disease
47. First line therapy pt presenting with epigastric pain?: PPI
- empiric
- minimum 4 weeks
-Nissen fundoplication: wrapping stomach around esophagus
48. Second line therapy pt presenting with epigastric pain?: H2 -Endocinch: scope to place a suture around the LES
blockers (Ranitidine, nizatidine, cimetidine, famotidine) -Heat/radiation of LES: causing scarring
- Not as effective, but work 70% of pts
55. Most likely Dx:
GI bleeding w/o pain
Liquid antacids
-NSAIDs or Alcohol in Hx: Gastritis
- same efficacy but have reflex rebound acidity and diarrhea
56. How long does it take to develop Barrett's from GERD?: more
49. Uses misoprostol?: Artificial prostaglandin analogue
than 5 years
- Used to tx NSAID-induced gastric damage
- long-standing GERD change to columnar metaplasia
- when PPIs arrived, on USMLE misoprostol always wrong
answer! 57. T/F: Each year about 0.5% of people with Barrett esophagus
progress to esophageal cancer.: TRUE
50. Pt has "epigastric burning pain radiating up into the chest".
- sore throat 58. Dx Barretts Esophagus?: Biopsy is the only way to be certain
- bad taste in mouth (metallic) of the presence of Barrett esophagus
- hoarseness morning 59. Which type of Barretts has the greatest risk of transforming
- cough: GERD into esophageal cancer?: Columnar metaplasia with intestinal
51. When to do 24hr pH in GERD?: if PPI's fail features has the greatest risk
- 24 hour pH monitoring most accurate when hx not clear 60. Tx Barrett alone (metaplasia): PPIs and rescope every 2-3
52. When is endoscopy done in GERD?: 1. Dysphagia or years
odynophagia 61. Tx Low-grade dysplasia LES: PPIs and rescope every 6-12
2. Wt loss mo.
3. Anemia or heme + stool 62. Tx High-grade dysplasia LES: Ablation w/ endoscopy:
4. > 5-10 years of symptoms to exclude Barrett's esophagus photodynamic therapy, radiofrequency ablation, endoscopic
53. All pts should try lifestyle changes in GERD such as?: - Lose mucosal resection
weight if obese 63. Anti-microsomal antibodies: Hashimoto's thyroiditis
- Avoid alcohol, nicotine, caffeine, chocolate, and peppermint 64. Gastritis (aka gastropathy) caused by?: - H. pylori
- Eat within 3 hours of bedtime - Alcohol
- Elevate head 6-8 inches - NSAIDs
- Portal HTN
- Stress
• Burns, trauma, sepsis, multiorgan failure (uremia)
65. GI bleeding w/o associated pain. Look for NSAIDS or
alcoholism in history.: Gastritis
- can presents with almost every degree of bleeding from mild
coffee ground emesis, large volume vomiting red blood, to black
stools (melena)
66. Most accurate test Gastritis: EGD
+ H. pylori testing

* capsule endoscopy is not appropriate for Upper GI bleeding if


endoscopy is one of the choices
67. Atrophic gastris is associated with?: Vitamin B12 deficiency
Page 46
68. Amount of blood loss correlating with type of GI bleed.: 1. 76. Most accurate test PUD?:
Coffee-ground emesis: 5-10 mL
2. Heme (guaiac) positive stool: 5-10 mL
3. Melena: 50-100 mL
69. Testing for H. pylori.:

Upper endoscopy
77. H.pylori more common in DU or GU?

Cancer more common DU or GU?:

70. Tx Gastritis.: PPI


- H2 blockers, sucralfate (aluminum hydroxide complex coats
the stomach, if it is a choice nearly always the wrong answer),
and liquid antacids aren't effective as PPIs
71. Stress ulcer prophylaxis is indicated in?: • Mechanical
ventilation
• Burns - Curling's ulcers
• Head trauma - Cushing's ulcers
• Coagulopathy DU - H.pylori
72. Most common cause PUD?:
GU - cancer
78. Best initial therapy for H. pylori?:

H.pylori PPI + clarithromycin + amoxicillin (metronidazole + tetracycline


NSAIDs can be used if pt does not respond to this tx)

73. Do EtOH or tobacco cause ulcers?: no, they delay healing of


* Retest with stool antigen or urea breath test to confirm cure of
ulcers
Helicobacter
74. T/F: NSAIDs induced PUD produce more bleeding than pain.:
79. Pt has tx resistant ulcer what medication can you add?:
TRUE
Bismuth to a change of antibiotics may aid in resolution of tx-
75. Recurrent episodes of epigastric pain, dull sore, gnawing.: resistant ulcers
80. For those with Gastric ulcer that is refractory to tx what
should be done to r/o cancer?: Repeat endoscopy is done to
exclude cancer as a reason for not getting better
81. Tx failure of PUD stems from these (4) reasons?: 1.
Nonadherence to meds
2. Alcohol
GU = wt loss
3. Tobacco
4. NSAIDs

Page 47
82. Tx Non-ulcer dyspepsia: 90. 1. Gastritis assoc w/ achlorohydria?
2. Cause?
3. Location in stomach?
4. What chemical will be high?: 1. Atrophic
2. Pernicious anemia
3. Body/Fundus
4. Gastrin
(no hydrochloric acid - parietal cells destroyed)
*look for b12 def or anemia
91. Assessing for ___ is the most important initial mgmt of GI
bleeding.: BP
83. Pt has ulcers that are:
- fluid resuscitation
- Large (> 1-2 cm)
- NS or Ringer lactate
- Recurrent after H. pylori eradication
- Distal in the duodenum 92. Most common cause of upper GI bleeding?: Ulcer dz: MCC
- Multiple: Gastrinoma (Zollinger-Ellison Syndrome)
- also caused by: gastritis, esophagitis, duodenitis, cancer, and
84. Gastrinoma is often associated with ___ because acid
varices
inactivates lipase.: Diarrhea
93. Most common cause of lower GI bleeding?: Diverticulosis:
85. Once endoscopy confirms the presence of an ulcer, the most
MCC
accurate diagnosis test for Gastrinoma?: 1. High gastrin
levels off antisecretory therapy (PPIs/H2) with high gastric
- also caused by: Angiodysplasia (AV malformation, AVM),
acidity
polyps or cancer, IBD, hemorrhoids, upper GI bleeding w/ rapid
2. High gastrin levels despite a high gastric acid output
transit from high volume
3. Persistent high gastrin levels despite injecting secretin
(most accurate) 94. Pt is orthostatic they have lost how much blood?
86. Tx Gastrinoma: Local dz - resect
When pulse > 100/min or systolic BP < 100mmHg.: 15-20% of
blood volume lost = Orthostasis
Mets - PPI lifelong
87. Diabetic pt w/ chronic abdominal discomfort
30% blood loss = pulse > 100/in or systolic BP < 100 mmHg
- "bloating"
95. Define orthostasis?: > 10 pt rise in pulse when going from lying
- constipation
down to sitting or standing up
- N/V
- Early satiety
OR
- Anorexia

systolic BP drop 20 pts or more when sitting up


Dx?
Most accurate test? 96. Pt presents with vomiting blood +/- black stool, spider
Medical tx?: Diabetic gastroparesis angiomata, caput medusa, splenomegaly, palmar erythema,
- autonomic neuropathy leading to dysmotility asterixis. Dx.: Variceal bleeding
- inability to sense stretch in GI tract - only form of GI bleeding in which PE helps determine etiology
97. Dx lab tests for acute GI bleed?: Acute
test: Nuclear gastric emptying study - replace fluids and check Hct, platelet count, and coagulation
tests (PT or INR)
Abx: Erythromycin/Metoclopramide => INC gastric motility 98. Tx variceal bleeding.: Octerotide and urgent endoscopy to
88. What is the next best step after diagnosing Zollinger-Ellison?: control bleeding by banding
1. CT/MRI done first -> poor sensitivity 99. When should nuclear bleeding scan test be used?: Endoscopy
2. Somatostatin receptor scintigraphy (nuclear octerotide) unrevealing in a massive acute hemorrhage; lacks accuracy
scan combo with endoscopic US to exclude metastatic dz (do 100. When should angiography be used in dx GI bleeding?:
these if CT or MRI are normal) Specific vessel or site of bleeding needs to be ID prior to sx or
89. Chemical & NT's that stimulate parietal cells to release acid - embolization of the vessel; used only in massive, non
3: Gastrin responsive bleeding
ACh 101. When should capsule endoscopy be used in dx GI bleeding?:
Histamine Small bowel bleeding when upper and lower endoscopy don't
*if high cause PUD show etiology
102. When should MRI/CT abdomen be used in dx GI bleeding?:
Not useful in GI bleeding
103. When should EKG be used in dx GI bleeding?: Shows
ischemia in severe bleeding

Page 48
104.GI bleeding tx?: 1. Fluid replacement 115.T/F: Fat malabsorption frequently presents with wt loss.:
- 1-2 liters an hour NS/Ringer lactate TRUE
2. Packed RBCs 116.All malabsorption diarrhea present with?: All present with
- Hct < 30 in those who are older or suffer from CAD, young pt steatorrhea:
may not need until Hct under 20-25 • Oily, greasy, floating, foul smelling stools
3. FFP - INR > 1.3, High INR = INC mortality
4. Platelets • Deficiency of fat-soluble vitamins (A, D, E, and K)
- < 50,000 when bleeding 117.Causes of malabsorption diarrhea?: Celiac disease (MC)
5. Octreotide for variceal bleeding Whipple's disease
6. Endoscopy to determine diagnosis and administer treatment Chronic pancreatitis
(band varices, cauterize ulcers, inject epinephrine into bleeding
118.Deficiency vitamin D, K, B12 and manifestations clinically in
gastric vessels)
pt with malabsorption diarrhea?:
7. IV PPI for upper GI bleeding
8. Surgery to remove site of bleeding if fluids, blood, platelets,
and plasma will not control bleed
105.What do u do in addition to fluids, platelets, plasma for
esophageal and gastric varices?: 1. Octreotide (somatostatin)
- DEC portal pressure
2. Banding 119.Vitamin B12 needs ___ and ___ to be absorbed.: Intact bowel
3. Transjugular intrahepatic portosystemic shunting (TIPS) - wall and pancreatic enzymes
DEC portal pressure in those not controlled by ocreotide and 120.10% of celiac disease pts what kind of skin condition can be
banding present?: Dermatitis herpetiformis
4. Propranolol or nadolol - prevent future episodes of bleeding 121. Pt presents with Diarrhea:
5. Antibiotics to prevent SBP w/ ascites • Arthralgias
106.T/F: Sclerotherapy is never the right answer if banding is • Ocular findings
technically possible.: TRUE • Neurologic abnormalities (dementia, seizures)
107.Different causes of Diarrhea?: 1. Lactose intolerance • Fever
2. Antibiotic-associated diarrhea • Lymphadenopathy: Whipple dz
3. Malabsorption - PAS positive non-macrophage inclusion
4. Chronic pancreatitis 122. Tx Whipple dz: Ceftriaxone followed by TMP/SMZ
5. Carcinoid syndrome 123.Main distinctions btw chronic pancreatitis and gluten
108.__ has been associated with the highest incident of antibiotic sensitive enteropathy is the presence of ___.: Iron deficiency
associated diarrhea and C. diff.: Clindamycin - need intact bowel wall to be absorbed, but do not need
- antibiotic associated diarrhea may present with blood and pancreatic enzymes
white cells in stool 124.Celiac dz best initial test?: Anti-tissue tranglutaminase
109. Best initial test for antibiotic associated diarrhea? (first)

Most accurate test?: stool C.diff toxin test - Antiendomysial antibody

C. diff PCR (most accurate) - IgA antigliadin antibdoy


110.Best initial therapy for C. diff associated diarrhea?: 125. Most accurate test for celiac dz?: Small bowel biopsy shows
Metronidazole flattening of villi
111.If there's no response to metronidazole tx of c.diff, next step
in management is to?: Switch to oral vancomycin or fidaxomicin
112.Why is IV vancomycin always the WRONG answer for tx of *Whipple dz and tropical sprue also most accurately diagnosed
C. diff diarrhea?: IV vancomycin is always wrong for antibiotic- via biopsy
associated diarrhea since it will not pass bowel wall. 126.Bowel biopsy is essential in celiac dz to exclude?:
113. Antibiotic-associated diarrhea algorithm.: LYMPHOMA
127.What is the most accurate diagnostic test for chronic
pancreatitis?: Secretin stimulation test
-> place an NG tube past pyloris; an unaffected pancreas will
release a large volume of bicarbonate-rich fluids after the IV
injection of secretin

114.Recurrent episodes of C. diff associated diarrhea are best


treated with?: Another course of Metronidazole Page 49
128. Most sensitive diagnostic imaging for chronic pancreatitis?: 140.T/F: IBS is associated with blood or white cells in the stool.:
FALSE, IBS not associated with blood or white cells in the stool
141. Clinical manifestations of both Crohns and UC.:

• Diarrhea + abdominal pain


• Blood in stool -> anemia
• Wt loss
• Fever
Extraintestinal manifestations:
• Arthralgias
1. Abdominal x-ray: 50-60% sensitive for calcification of the • Uveitis,iritis
pancreas • Skin manifestation (erythema nodosum, pyoderma
2. Abdominal CT scan: 80-90% sensitive to pancreatic gangrenosum)
calcification • Sclerosing cholangitis (more frequent in UC)
129. Tx chronic pancreatitis.: Enzyme replacement 142.Risk of cancer associated with CD and UC.: • Both forms of
130.Tx Celiac dz.: Avoid gluten containing foods (wheat, oats, rye, IBD can lead to colon cancer
barley) • Cancer risk INC with duration
• CD involving colon has the same cancer risk as UC
* rice + wine OK 143. Differences between CD and UC.:
131. Tx tropical sprue: TMP/SMX, tetracycline
132.What is the D-xylose test?: • Old test to distinguish
pancreatitis from bowel wall abnormalities
• D-xylose normal in pancreatic disorders
133. Pt presents with intermittent
• Diarrhea:
• Flushing 144.When should screening for colon cancer begin in UC and
• Wheezing CD?: After 8 to 10 years of colonic involvement, with
• CV murmurs - TR: Carcinoid syndrome colonoscopy every 1 to 2 years .
134.Best initial diagnostic test is for carcinoid syndrome?: 145. Most accurate test for CD or UC?:
Urinary 5-hydroxyindoleaceticacid (5-HIAA) test
135.Tx carcinoid syndrome.: Octeotide
- synthetic version of somatostatin used to control the diarrhea
136.Presents like malabsorptive diarrhea except intermittently,
no wt loss, and normal vitamin levels. Dx. Tx.: Lactose
intolerance

Stop eating milk products and consider exogenous lactase


therapy Endoscopy
137. Best initial dx test for lactose intolerance. 146.Antineutrophilic cytoplasmic antibody (ANCA) positive in?:
UC
Most accurate test?: Stool osmolality test (initial) 147.Antisaccharomyces cerevesiae antibody (ASCA) positive in?:
CD
Cessation of symptoms after diet change (most accurate) 148.Acute exacerbations of either CD or UC are treated with
138.Pain syndrome with either diarrhea, constipation, or both. ___.: Steroids (prednisone or budesonide)
Pain is relieved by BM, less at night, relieved by change in 149.Chronic maintenance of remission for CD and UC is ____.: 5-
bowel habit (diarrhea). Not associated with wt. loss.: Irritable ASA derivatives (mesalamine)
bowel syndrome (IBS) - Asacol for UC, Rowasa for UC limited to the rectum
- No specific dx test - Pentasa for CD
139.Tx IBS: 1. Fiber in diet 150. Perianal CD is treated with?: Ciprofloxacin and metronidazole
2. Antispasmodic agents (Hyoscyamine, Dicyclomine) 151.___ and ___ are used to wean pt off of steroids in CD and UC
3. Tricyclic antidepressants (Amitriptyline or SSRI) when the dz is so severe that several recurrences develop as
4. Antimotility agents (Loperamide for diarrhea) the steroids are stopped.: Azathioprine + 6-mercaptopurine
5. Lubiprostone (chloride channel activator that INC BM
frequency), also treats constipation * everyone needs calcium and vitamin D
Page 50
152.Pt with fistulae and severe unresponsive disease with IBD 164.Hereditary nonpolyposis colon cancer syndrome (HNPCC)
tx?: Anti-tumor necrosis factor (TNF) agents (infliximab) comprises of ?: 3 family members
2 generations
Surgery only if there's no response to anti-TNF agents 1 premature (< 50)
153. Surgery role in UC or CD?: UC: can be cured w/ colectomy 165.Screening colonoscopy should begin when if pt has HNPCC?:
Start screening at age 25 with colonoscopy every 1 to 2 years
CD: surgery is used exclusively for bowel obstruction as it 166.Familial adenomatous polyposis (FAP) with the presence of
recurs at the surgical site thousands of polyps with abnormal genetic test know as
154.LLQ pain + tenderness, fever, leukocytosis, palpable mass ____.: Adenomatous polyposis (APC) test
an older pt usually.: Diverticulitis 167.Start screening for FAP with ___ at age ___ every ___ for a
155. Most accurate test Diverticulosis? patient with a family history of FAP.: Sigmoidoscopy at age 12
every year
Tx?: Colonoscopy 168.Pt has a previous non-dysplastic adenomatous polyp pt have
colonoscopy every ___.: 3-5 years
- Barium studies (acceptable not as accurate) 169.Previous history of colon cancer pt should have colonoscopy
every ___.: - 1 year after resection
Tx: fiber, bran, psyllium, methylcellulose - Then at 3 years
156. Best INITIAL test diverticulosis?: - Then every 5 years
170.Multiple hamatomatous polyps, melanotic spots on lips and
skin, INC frequency of breast cancer, INC gonadal and
pancreatic cancer.: Peutz-Jeghers syndrome
171.Colon cancer associated with osteomas, desmoid tumors,
and other soft tissue tumors.: Gardner syndrome
172.Colon cancer associated with CNS malignancy.: Turcot
syndrome
173.Colon cancer in association with multiple hamartomatous
polyps.: Juvenile polyposis
174.For Peutz-Jeghers syndrome, Gardner syndrome, Turcot
CT scan of the abdomen syndrome, juvenile polyposis is there a requirement for early
- 98% accuracy screening?: NO INC frequency of colonoscopy screening
157.Which tests are dangerous in acute diverticulitis?: 175.Most common causes of Acute pancreatitis?: 1. GB stones
Colonoscopy and barium enema 2. ETOH
- INC risk perforation because infection weakens colonic wall
- Drug allergy
158.Tx Diverticulitis?: • Antibiotics that cover E. coli and
- Ductal obstruction, ERCP, CF
anaerobes that are present in bowel such as...
- Trauma
- Ciprofloxacin combo w/ Metronidazole
- Hypertriglyceridemia
- Amoxicillin/clavulanate
- Hypercalcemia
- Ticarcillin/clavulanate
- Infection
- Piperacillin/tazobactam
- Scorpion sting
159.Surgery for diverticulitis in those with?: - No response to - Drug toxicity
medical tx
176.Which drug toxicity can cause acute pancreatitis?:
- Frequent recurrences of infection
Pentamidine
- Perforation, fistula formation, abscess, strictures, or
Didanosine
obstruction
Azathioprine
160.Who is more likely to get a recommendation of surgery for Estrogens
diverticulitis younger or older pts?: Younger pts should have
177.Which drug allergy can cause acute pancreatitis?: Sulfa
the colon resected more often because of the greater total #
drugs such as Furosemide and HCTZ
recurrent episodes that occur
- diverticular dz does not disappear despite tx episodes of 178.Acute epigastric pain, tenderness, and N/V. In severe cases
diverticulitis or the use of fiber in the diet there is hypotension + fever.: Pancreatitis
- pain intensity is subjective and doesn't correlate with degree
161. T/F: Pt with acute diverticulitis should NOT be fed.: TRUE
of organ damage
162.Pt should have a colonoscopy every ___ years beginning at
179.TIP: The pain of ____ goes straight through to the back "like
age __.: 10 years beginning at age 50
a spear" stabbed into the abdomen. ___ pain goes around the
163.Single family member with colon cancer screening?: • Begin side to the back.: Pancreatitis
10 years earlier than the age at which the family member Cholecystitis
developed their cancer or age 40, whichever is younger

• Repeat the scope every 5 years if the family member is under


age 60 Page 51
180.Which of the following is associated with the worst 188. Pseudocysts in acute pancreatitis management?:
prognosis in pancreatitis?
a. Elevated amylase
b. Elevated lipase
c. Intensity of the pain
d. Low calcium
e. C-reactive protein (CRP) rising: d. Low calcium
- severe pancreatic damage DEC lipase production and release
leading to fat malabsorption in the gut
- Ca2+ binds with fat (saponifies) in the bowel, leasing to
calcium malabsorption
181. Best initial tests for pancreatitis?: Amylase and lipase
Pseudocysts are drained with a needle if they are enlarging or
182. Most specific diagnostic test for pancreatitis?: CT scan painful
183.Labs in pancreatitis besides amylase and lipase.: • CBC: 189.Uses ERCP in acute pancreatitis tx?: Remove obstructing
leukocytosis, drop in Hct over time with rehydration stones + dilate strictures
• Elevated LDH and AST
• Hypoxia Place stents
• Hypocalcemia
190. Chronic liver disease manifestations?: 1. Portal HTN leading
• Elevated urinary trypsinogen activation peptide
to varices
184.Disease severity strongly correlates with the degree of ___ 2. Spider angiomata, palmar erythema, testicular atrophy, and
seen on CT scan in acute pancreatitis.: SEVERITY gynecomastia
- > 30% NECROSIS is "extensive" necrosis 3. Renal insufficiency (hepatorenal syndrome)
- Needle biopsy indispensable in determining infection in those 4. Hepatopulmonary syndrome
who have extensive necrosis 5. Thrombocytopenia from splenic sequestration
185. Imaging in acute pancreatitis.: 6. Hypoalbuminemia & edema and ascites
7. Asterixis and encephalopathy
8. Coagulopathy (all clotting factors except VIII are made in
liver)
191.When is paracentesis performed pt presents with ascites?:
New onset ascites

Abd pain + tenderness

Fever
192.Low albumin in the ascitic fluid etiology?: Portal HTN from
cirrhosis
1. CT or MRI scan are best
- Also detect pseudocysts 193. What is the serum ascites albumin gradient (SAAG)?: The
- w/ contrast IV/oral help outline abd structures difference or "gradient" of albumin between serum and ascitic
2. MRCP fluid
- Can help determine etiology (stones, stricture, tumor) 194.When the SAAG (serum ascites albumin gradient) < 1.1 g/dL
- MRCP is diagnostic, ERCP is tx etiology?: Infections (except SBP)
3. Plain X-ray Cancer
- Sentinel loop of bowel (air-filled piece of small bowel in LUQ) Nephrotic syndrome
- Limited utility 195.When the SAAG (serum ascites albumin gradient) > 1.1 g/dL
4. US has very poor accuracy etiology?: Portal HTN
- Overlying bowel blocks precise imaging CHF
186.Acute pancreatitis tx: • NPO (no food) Hepatic vein thrombosis
• IV hydration Constrictive pericarditis
• Analgesia 196.Causative organisms in Spontaneous Bacterial Peritonitis?:
• PPIs pancreatic stimulation from acid entering duodenum Infection w/o perforation of bowel
187.When should antibiotics be use in acute pancreatitis tx?: • >
30% necrosis on CT or MRI, add antibiotics Imipenem or - E.coli (MCC)
meropenem - Other gram - bacilli
- Pneumococcus
• Infected, necrotic pancreatitis should be resected with - Anaerobes (rare)
surgical debridement to prevent ARDS and death 197.Best initial test for Spontaneous Bacterial Peritonitis
(SBP)?: Cell count > 250 neutrophils

Page 52
198.Most accurate test for Spontaneous Bacterial Peritonitis 214. Pt presents with:
(SBP)?: 1. Fluid cx: but takes too long for results (most • Pruritus
accurate) • INC alkaline phosphatase
2. Gram stain is usually negative/LDH nonspecific • INC GGTP
• Elevated bilirubin level: Primary sclerosing cholangitis
Tx: Cefotaxime or ceftriaxone - 80% of PSC occur in association with IBD
199.SBP frequently occurs. When the ascites fluid albumin level 215. Most accurate test PSC?:
is quite low, use prophylactic ___ or ___.: Norfloxacin or
TMP/SMX
200.All variceal bleeds with ascites need __ prophylaxis.: SBP
- Anyone with SBP needs lifelong prophylaxis against
recurrence
201.Tx ascites and edema in cirrhosis.: Spironolactone and other
diuretics
- Serial paracentesis for large-volume ascites
202.Tx coagulopathy and thrombocytopenia in cirrhosis.: FFP
and/or platelets only if bleeding occurs
203. Tx encephalopathy in cirrhosis.: Lactulose and rifaximin MRCP or ERCP (shows beading, narrowing, or strictures in the
biliary system), not liver biopsy
204.Tx varices as primary prevention in cirrhosis.: Propranolol
and banding via endoscopy 216. Medical tx PSC: Cholestyramine or ursodeoxycholic acid

205.Tx hepatorenal syndrome in cirrhosis.: Somatostatin 217.TIP: ___ does not improve or resolve with resolution of IBD.
(octreotide), midodrine Even after a colectomy in UC, the pt may still progress to
needing a liver transplant.: Primary sclerosing cholangitis
206.What is hepatopulmonary syndrome?: Lung dz and hypoxia
entirely on the basis of liver failure 218.Young pt (<40) presents with liver disease and emphysema
- look for orthodeoxia, which is hypoxia upon sitting upright (COPD) who is a nonsmoker. Dx.: Alpha-1-antitrypsin
deficiency
- no specific therapy - tx by replacing the enzyme

207. Most accurate test for alcoholic liver dz?: Liver biopsy 219.Genetic disorder leading to over absorption of iron in the
duodenum. Dx. Which gene?: Hemochromatosis
208.Alcohol and drugs cause liver disease give a greater
- mutation C282y gene
elevation in ___ compared to ___.: AST > ALT alcohol + drugs
Viral hepatitis: ALT> AST 220.Why do men present earlier than women with
hemochromatosis?: Men present earlier than women because
Binge drinking: sudden rise in GGTP menstruation delays the onset of liver fibrosis and cirrhosis

209.___ is the most likely diagnosis when pt presents: 221. Pt in his 50s with mild INC AST and alkaline phosphatase
• Woman in 40s or 50s - fatigue + joint pain (pseudo gout)
• Fatigue and itching - ED
• Normal bilirubin - Skin darkening
• Elevated alkaline phosphatase: Primary Biliary Cirrhosis - Diabetes
(PBC) - Cardiomyopathy: Hemochromatosis

210.The most unique features of primary biliary cirrhosis 222.Which infections occur in hemochromatosis and why?: Vibrio
(PBC)?: vulnificus
Yersina
Listeria

Organisms feed on iron


223.Best initial test Hemochromatosis.: IRON studies
- INC serum iron or ferritin
- DEC iron binding capacity
224.Most ACCURATE test Hemochromatosis.: Liver biopsy
- INC IRON
Xanthelasma/xanthoma EKG show conduction defects
ECHO: dilated or restrictive cardiomyopathy
Osteoporosis 225.Best tx Hemochromatosis.: Phlebotomy
211. Most accurate test for PBC?: Liver biopsy - liver fibrosis can resolve if phlebotomy is begun before
212.Most accurate blood test PBC?: Antimitochondrial antibody cirrhosis develops
- rise bilirubin + IgM do not elevate until disease very far
advanced
213. Tx PBC: Ursodeoxycholic acid

Page 53
226.When is iron chelation therapy used in hemochromatosis 244.• Young women
tx?: 1. Cannot be managed with phlebotomy • Signs of liver inflammation
2. Are anemic and have hemochromatosis from over transfusion • Positive ANA
such as thalassemia • Anti-smooth muscle antibodies positive: Autoimmune
hepatitis
*Deferasirox + deferiprone (oral), deferoxamine (IV) - liver-kidney microsomal antibodies
227.How do you diagnose Hemochromatosis w/o liver biopsy?: - high gamma globulin (IgG)
abnormal MRI combined with abnormal genetic test for - anti-smooth muscle antibodies
hemochromatosis HFE C282y gene - anti-liver/kidney microsomal antibodies
228.Both Hep B and C are associated which 3 complications?: 245.Most accurate test autoimmune hepatitis and tx?: Liver
Cirrhosis biopsy
Liver cancer - Prednisone and/or azathioprine
Polyarteritis nodosa 246. Extremely common cause of mildly abnormal liver function
229.Chronic hepatitis B diagnosis?: Surface antigen positive > 6 tests: Nonalcoholic steatohepatitis (NASH) or nonalcoholic fatty
mo. liver dz
- most cases e-antigen positive - biopsy shows macrovesicular fatty deposits w/o hx alcohol
- Hep B DNA PCR best way determine viral replication activity use
230.T/F: Most hepatitis C is acute form.: FALSE, 80% have 247.Causes of NASH: Obesity
chronic infection DM
- Hep C PCR RNA viral real Hyperlipidemia
231.Tx Acute Hep C: Interferon Corticosteroid use
Ribavirin
Telaprevir/Boceprevir
232.Tx chronic Hep B with any one of the following agents?:
Adefovir
Lamivudine
Telbivudine
Entecavir
Tenofovir
Interferon
233.Tx chronic Hep C use a combo of?: Combo: Ledipasvir +
sofosbuvir
- add interferon, ribavirin, boceprevir when tx fails
234. Genotype 1 Hep C tx?: Ledipasvir + sofosbuvir, both orally
235. Genotype 2 and 3 Hep C tx?: Sofosbuvir and ribavirin, orally
236.SE interferon: Arthralgias, thrombocytopenia, depression,
leukopenia
237. SE Ribavirin: Anemia
238. SE Adefovir: Renal dysfunction
239.Decrease in ceruloplasmin causes buildup of copper.: Wilson
Dz
240.Pt presents with neuro symptoms: psychosis, tremor,
dysarthria, ataxia, or seizures. Coombs negative hemolytic
anemia. RTA or nephrolithiasis.: Wilson Dz
- also features of cirrhosis and hepatic insufficiency
241.Best initial test Wilson Dz: Slit-lamp examination for Kayser-
Fleischer rings
242.Most ACCURATE diagnostic test for Wilson Dz.: Abnormally
increased amount of copper excretion into urine after giving
penicillamine.
243. Tx Wilson dz: • Penicillamine will chelate copper and remove
it from the body

• Zinc: interferes w/ intestinal copper absorption

• Trientine: an alternate copper-chelating compound

Page 54
MTB Gynecology
MTB Q and A

1. Tx for PMS and PMDD: DEC caffeine, alcohol, cigarettes, and 12. Bleeding after intercourse: Postcoital bleeding
chocolate and should exercise - cervical cancer
- cervical polyps
If sxs severe give SSRIs - atrophic vaginitis
2. How long do menopause symptoms usually last?: 12 months 13. Dx tests abnormal uterine bleeding.: - CBC (Hgb and Hct see if
- menstrual irregularity, sweats + hot flashes there is a drop)
- mood changes - PT/PTT (coagulation disorder)
- dyspareunia (pain during sexual intercourse) - Pelvic US to visualize any anatomic abnormality
14. __ unexplained abnormal bleeding.: Dysfunctional uterine
age 48-52 bleeding (DUB)
3. Hormone replacement therapy (HRT) in menopause tx is - occurs when pts are anovulatory
associated with ___ and can lead to ___.: Endometrial - ovary produces estrogen, but no corpus luteum is formed (no
hyperplasia progesterone)
- no withdrawal bleeding
Endometrial carcinoma - endometrium overgrowth until it outgrows the blood supply and
4. Dx menopause and tx.: INC FSH bleeding occurs
15. Dx test dysfunction uterine bleeding.: r/o systemic reasons for
HRT is indicated for short term sxs relief as well as prevention anovulation such as:
of osteoporosis - HYPOthyroidsim
5. Contraindications to HRT tx for menopause.: Estrogen- - HYPERprolactinoma
dependent carcinoma (breast or endometrial cancer)
Endometrial biopsy women over 35 to r/o carcinoma
Hx of PE or DVT 16. Tx dysfunctional uterine bleeding.: 1. OCPs
6. TRUE/FALSE: Postcoital bleeding is cervical cancer until - adolescents and young woman who are anovulatory
proven otherwise.: TRUE - women over 35 who have normal endometrial biopsy
7. Heavy + prolonged menstrual bleeding
2. Acute hemorrhage
"Gushing" of blood - D&C to stop the bleeding

Clots may be seen: Menorrhagia 3. Severe DUB (anemic pt)


- endometrial hyperplasia - not controlled by OCPs
- uterine fibroids - tx endometrial ablation or hysterectomy
- dysfunctional uterine bleeding 17. The disadvantages of diaphragm as contraception.: 1. Need to
- intrauterine device be fitted properly (can change w/ wt gain or pregnancies)
8. Light menstrual flow 2. Proper use of diaphragm requires advanced preparation (6
hours before and left 6 hours after intercourse)
May only have spotting: Hypomenorrhea 3. Improper placement or dislodging of diaphragm reduces
- obstruction (hymen, cervical stenosis) efficacy
- OCPs 18. OCPs reduce the risk of ___, ___, and ___.: Ovarian
9. Intermenstrual bleeding: Metrorrhagia carcinoma
- endometrial polyps Endometrial carcinoma
- endometrial/cervical cancer Ectopic pregnancy
- exogenous estrogen administration
Slight INC risk of thromboembolism
10. Irregular bleeding
- time intervals (irregular) 19. Transdermal patches should not be placed on the ___.: Breast
- duration (frequent) - combo estrogen and progesterone on the skin for 7 days then
- amount of bleeding (excessive): Menometrorrhagia replace with a new one for 3 weeks, followed by patch free week
- endometrial polyps 20. Depot medroxyprogesterone acetate is an IM injection that is
- endometrial/cervical cancer effective contraception for ___ months.: 3 months
- exogenous estrogen administration 21. IUD is placed into the uterus and provides contraception for
- malignant tumors ___ years.: 10 years
11. Menstrual cycles > 35 days long: Oligomenorrhea - copper device
- pregnancy - levonorgestrel
- menopause 22. IUD is associated with ___.: PID when they are placed
- significant weight loss (anorexia) - genital cultures must be done before placement of these
- tumor secreting estrogen devices
23. Post-tubal ligation the risk of pregnancy is low, but if it
occurs there is an INC incidence of ____.: Ectopic pregnancy
Page 55
24. The most common cause of labial fusion is ____.: 21-B- 34. ___ is the most common type of vulvar cancer.:
hydroxylase deficiency
- tx of labial fusion is reconstructive surgery
- labial fusion occurs when excess androgens are present
25. White, thin skin extending from labia to perianal area.: Lichen
sclerosis
- any age can be affected, however, if postmenopausal INC risk
of cancer

tx: topical steroids


26. Patients with chronic irritation develop hyperkeratosis Squamous cell carcinoma
(raised white lesion).: Squamous cell hyperplasia - pruritus, bloody vaginal discharge, and postmenopausal
- any age; pts who have had chronic vulvar pruritus bleeding

tx: Sitz baths or lubricants (relieve the pruritus) PE: small ulcerated lesion to a large cauliflowerlike lesion
27. Violet flat papules.: Lichen planus Bx essential for diagnosis.
- 30-60s
Staging done while the patient is in surgery.
tx: topical steroids 35. Tx squamous cell carcinoma vulva.: Unilateral lesions w/o
28. Tx Bartholin gland cyst.: I&D lymph node involvement is modified radical vulvectomy
- if they continue to recur, then marsupialization should be done
(open space is kept open with sutures) Bilateral involvement is radical vulvectomy, lymph node
- culture fluid for STDs such as Neisseria gonorrhea and involvement need lymphadenectomy
Chlamydia trachomatis 36. ___ invasion of endometrial glands into the myometrium.:
29. Risk factors vaginitis.: 1. Antibiotic use (Lactobacillus
normally keeps the vaginal pH below 4.5)
2. DM
3. Overgrowth of normal flora
30. Vaginal discharge with fishy odor; gray white.

Wet mount shows clue cells.: Bacterial vaginosis


- Gardnerella
Adenomyosis
- age 35-50
tx: Metronidazole and clindamycin
- dysmenorrhea and menorrhagia
31. White, cheesy vaginal discharge.
37. Risk factors for adenomyosis.:

KOH shows pseudohyphae.: Candidiasis


- Candida albicans

tx: Miconazole or clotrimazole, econazole, or nystatin


32. Profuse, green, frothy vaginal discharge.

Saline wet mount shows motile flagellates.: Trichomonas


- Trichomonas vaginalis

tx: Treat both patient and partner with metronidazole


33. Vulvar soreness and pruritus appearing as a red lesion with a Endometriosis
superficial white coating.: Paget disease
- intraepithelial neoplasia most common in postmenopausal Uterine fibroids
Caucasian women
- Bx for diagnosis

tx:
B/L lesion = radical vulvectomy

Unilateral lesion = modified vulvectomy

Page 56
38. Large, globular, and boggy uterus.: 45. Presentation
• Amenorrhea or irregular menses
• Hirsutism and obesity
• Acne
• Insulin resistance (DM2):

Adenomyosis
- MRI most accurate test
- Hysterectomy is the only definitive treatment
39. Endometriosis is most common if a ___ relative has
endometriosis.: First degree relative (mother or sister)
PCOS
- high androgen level and obesity INC in estrogen formation
• Endometrial tissue outside of endometrial cavity
outside the ovary
• Most common sites are ovary and pelvic peritoneum
• Endometriosis occurs in women of reproductive age 46. Dx tests PCOS: 1. Pelvic U/S (B/L enlarged ovaries with
multiple cysts)
40. __ cyclic pelvic pain 1-2 weeks before menstruation and
2. Elevated free testosterone 2/2 androgens
peaks 1 to 2 days before menstruation.: Endometriosis
3. LH to FSH ratio > 3:1
- pain ends with menstruation
- abdominal bleeding is common 47. Tx PCOS: 1. Weight loss (DEC insulin resistance)
- nodular uterus and adnexal mass 2. OCPs (controls androgen levels and prevents endometrial
- dysmenorrhea and dyspareunia hyperplasia) patient does not wish to conceive
3. Metformin (insulin resistance)
41. Dx test endometriosis.: Laparoscopy
4. Clomiphene (infertility)
- rusty or dark brown lesions
- ovary (endometrioma) chocolate cyst
42. Tx endometriosis: 1. Mild disease
- NSAIDs
- Combined OCPs

2. Severe disease
- Danazole
- Leuprolide acetate (leupron)
- Surgery (remove all endometrial implants, infertile pt, total
abdominal hysterectomy and B/L salpingo-oophorectomy)
43. SE Danazole: Androgen derivative
- acne, oily skin, wt gain, and hirsutism
44. SE Leuprolide: GnRH agonist
- continuously given suppresses estrogen
- hot flashes and decreased bone density

Page 57
MTB Hematology
MTB Q and A

1. Causes of low MCV (microcytosis): Iron deficiency 17. Sideroblastic Anemia Causes?: Alcoholic suppression of bone
Thalassemia marrow.
Sideroblastic anemia
Anemia of chronic disease Other causes: lead poisoning, isoniazid, vitamin B6
deficiency.
Alpha thalassemia may have elevated reticulocyte count but
most have decreased. Only microcytic anemia where circulating iron level is elevated.
2. Sideroblastic Anemia MCV: Can either be microcytic or 18. Anemia with target cells (most common).: Thalassemia
macrocytic 19. Low ferritin microcytic anemia.: Iron deficiency
3. Causes of high MCV: B12 and folate defiency 20. High circulating iron microcytic anemia.: Sideroblastic
Sideroblastic anemia anemia
Alcoholism - only form of microcytic anemia in which circulating iron levels
Antimetabolite medications AKA azathioprine, 6-mercaptopurine, are elevated
or hydroxyurea 21. Normal iron studies microcytic anemia.: Thalassemia
Liver disease or hypothyroidism
22. Iron deficiency iron studies.: Low iron
Meds such as zidovudine or phenytoin
INC in TIBC
Myelodysplastic syndrome (MDS)
Low ferritin
4. T/F: Microcytic anemias all give a low reticulocyte count.: Low transferrin saturation (Fe/TIBC)
TRUE
23. Chronic disease iron studies: Low iron
5. Normocytic Anemia cause?: Acute blood loss or hemolysis. DEC TIBC
- Blood loss/hemolysis INC reticulocyte count High ferritin (iron is stored)
6. Hct Transfusion Indication: 25-30 hematocrit in elderly or those 24. RDW and Platelets in Iron Deficiency: RDW INC
with HD Elevated platelets
7. Symptomatic from anemia means?: SOB
Lightheaded, confused, sometimes syncope * most accurate test BM bx rarely done
Hypotension and tachy 25. Most accurate test in sideroblastic anemia.:
Chest pain
8. Each unit of Packed RBCs should raise the Hct by?: PRBC
have double the hematocrit (some plasma removed).
Should improve HCT by 3 points per unit or 1 g/dL of Hg
9. FFP -
MOA?
We don't use it in which diseases?: Replaces clotting factors
in pts with elevated PT, aPTT or INR, or bleeding

NOT used for:


Hemophilia A, B, or vW Disease
10. Cryoprecipitate: Replaces fibrinogen, used in DIC. High levels Prussian blue staining for ringed sideroblasts
of Factor VIII and VWF found in it. 26. Most accurate test for thalassemia.: Hemoglobin
never used first for anything electrophoresis. Normal RDW.
11. Use ___ donor FFP for IgA deficient recipient.: IgA deficient - only 3-gene deletion a-thalassemia is associated with Hgb H
donor FFP for IgA deficient recipient and INC reticulocyte count
12. Platelet infusion is contraindicated in ___.: TTP 27. Most accurate test for alpha thalassemia.: Genetic studies -
13. Give a bleeding pt platelets if the count is?: < 50,000 DNA analysis
14. TIP: Whole blood is never correct. Whole blood is divided into 28. Four genes deleted in alpha thalassemia: Hemoglobin barts
either PRBCs or FFP.: TRUE (gamma-tetrad), CHF causes death in utero.
15. Iron Needs: 1-2 mg normally
Menstruating 2-3 mg
Pregnant 5-6

Duodenum can absorb only 4 mg day


16. Hepcidin levels are ___ in anemia of chronic disease.: LOW
- hepcidin regulates iron absorption

Page 58
29. Tx for: 38. B12 and Folate common lab abnormalities?: Megaloblastic
anemia
Iron deficiency Increased LDH and indirect bilirubin levels
Chronic disease Decreased reticulocyte count
Sideroblastic anemia Hypercellular bone marrow
Thalassemia: 1. Iron deficiency - oral ferrous sulfate, IM iron Macroovalocytes
2. Chronic disease - correct underlying disease (end stage Increased homocysteine levels
renal failure will respond to EPO) 39. Confirming B12 deficiency: Methylmalonic acid level to confirm
3. Sideroblastic anemia - correct cause, some improve with B6 diagnosis of B12 deficiency
(pyridoxine) 40. Pernicious anemia is confirmed with?: Anti-intrinsic factor and
4. Thalassemia - trait not treated. Beta thalassemia major anti-parietal cell antibodies
(Cooley anemia) needs transfusion lifelong => iron overload
41. Pancreatic Enzymes and B12: Enzymes are needed to absorb
manage with deferasirox or deferiprone, or deferoxamine
B12, look for pancreatic problems leading to B12 defeciency.
30. Beta thalassemia electrophoresis findings?: INC hemoglobin F
and A Pancreatic enzymes needed to remove B12 from the R-protein
31. Oral iron chelators are ___ and ___ for hemochromatosis so it can bind with intrinsic factor.
resulting from transfusion.: deferiprone and deferasirox 42. Danger of folate and B12 replacement which electrolyte
32. First step in assessing Macrocytic anemia (B12 suspicion): abnormality?: Hypokalemia (sudden surge leads to increased
blood cell potassium packaging). Observe and replace.
43. Hemolytic Anemia (general lab findings): Decrease HCT
Increase LDH, reticulocytes, indirect bilirubin
Decreased haptoglobin
Decreased folate
Hyperkalemia from cell breakdown
Slight rise in MCV (reticulocytes are bigger)

*chronic hemolysis is associated with bilirubin gallstones


44. What precipitates an acute painful vasooclusive sickle
crisis?: Hypoxia
Dehydration
Infection/fever
Cold temperatures
Get a peripheral smear to check for presence of 45. Most likely dx:
hypersegmented neutrophils. AA
Severe pain in chest, back, and thigh
Many things raise MCV, but only B12 and folate deficiency and May be accompanied by fever.: Sickle cell disease
antimetabolite meds have hypersegmented neutrophils. - point mutation at position 6 of beta globin chain: valine
33. Anemia with methotrexate use in RA: Folate deficiency replaces glutamic acid

34. Vitamin B12 deficiency caused by?: Pernicious anemia 46. Sickle Cell Complications: Bilirubin gallstones from chronic
Pancreatic insufficiency hemolysis
Dietary deficiency (vegan/strict vegetarian) Osteomyelitis (from Salmonella)
Crohn disease, celiac sprue, tropical sprue Retinopathy
Blind loop syndrome (gastric bypass) Stroke
Diphyllobothrium latum, HIV Increased infection from autosplenectomy
Enlarged heart
35. Folate deficiency is caused by?: Dietary deficiency (goat's milk
Lower extremity skin ulcers
has no folate and has limited iron and B12)
Avascular necrosis of the femoral head (x-ray first test, MRI
Psoriasis and skin loss (or turnover)
most accurate)
Drugs: phenytoin, sulfa
Papillary necrosis kidney chronic kidney damage
36. Celiac disease causes which deficiency?: B12, folate, iron Dactylics in children (inflammation of fingers)
deficiency
37. Most common neurologic deficiency with B12: Peripheral
neuropathy.

Posterior column damage to position and vibratory sensation.


Look for ataxia.

Page 59
47. Best initial test for sickle cell: 55. Most likely dx:
Family history of anemia or hemolysis
Bilirubin gallstones
Intermittent jaundice
Splenomegaly
Recurrent episodes of hemolysis:

Peripheral smear - can see Howell-Jolly bodies


48. Most accurate test for sickle cell: Hemoglobin electrophoresis
49. What lowers mortality in sickle cell disease?: Hydroxyurea in
Hereditary Spherocytosis
prevention
- defect in cytoskeleton of RBC leading to abnormally round
Antibiotics for fever
shape end loss of normal flexibility characteristic of the
50. Sickle Cell Tx: 1. Oxygen/hydration/analgesia biconcave disc
2. If fever or WBC count higher than usual antibiotics:
56. Lab values in Hereditary Spherocytosis: Negative Coombs
ceftriaxone, levofloxacin, moxifloxacin
Low MCV
3. Folic acid replacement (chronic)
High MCHC
4. Pneumococcal vaccination (no spleen!)
+ Osmotic Fragility Test (most accurate test)
5. Hydroxyurea to increase Hgb F
57. Tx of Hereditary Spherocytosis: 1. Chronic folic acid
51. When is exchange transfusion used in sickle cell?: Acute
replacement supports RBC production
chest syndrome
Priapism
2. Splenectomy stops hemolysis but doesn't eliminate
Stroke
spherocytes.
Visual disturbance from retinal infarction
58. Causes of Autoimmune (Warm of IgG) hemolysis.: CLL
52. Sickle patient going from Hct 34 to Hct 22, best initial test?:
Lymphoma
Reticulocytes - suspect Parvovirus B19.
SLE
Reticulocytes are supposed to be high in SC, sudden drop is
Penicillin, alpha-methyldopa, rifampin, phenytoin
B19.
Peripheral smear will show nothing.
Warm is Good = IgG = Warm
59. Autoimmune Hemolysis Dx Test: Coombs test detects IgG
*First clue to parvorivus is sudden drop in reticulocyte level
antibody on surface of RBC.
53. Most accurate test for parvovirus B-19 is?: PCR for DNA
IgG removes small amounts of membrane.
- IVIG best initial tx
60. Fragmented Cells in Autoimmune Hemolysis: The smear does
54. Manifestations of Sickle Cell Trait (AS) pt is heterozygous for
not show fragmented cells in autoimmune hemolysis because
sickle cell gene (AS).: Occasional hematuria
RBC destruction occurs inside the spleen or liver, not in the
May have difficulty concentrating the urine (isosthenuria),
blood vessels.
occasionally hematuria.
61. Tx for Autoimmune Hemolysis
Normal smear result.
1. Initial
No treatment for trait.
2. Recurrent
3. Severe: 1. Best initial - glucocorticoids (prednisone)
2. Recurrent episodes - splenectomy
3. Severe - IVIG
4. Alternates - Rituximab, cyclophosphamide, cyclosporine,
azathioprine, mycophenolate mofetil when splenectomy does not
control the hemolysis
62. Cold Agglutinin associated with?: IgM

Associated with EBV, Waldenstrom macroglobulinemia, or


mycoplasma pneumoniae.
63. Presentation of Cold Agglutinin Disease: Numbness or mottling
of ears, fingers, toes, nose.

Sx resolve with warming up the body part.


Page 60
64. Dx test for Cold Agglutinin: Direct Coombs positive for 75. Defects in paroxysmal nocturnal hemoglobinuria (PNH):
complement Deficiency in complement regulatory proteins CD55 and CD59
(decay accelerating factor). PIG-A (phosphatidylinositol class A)
Cold agglutinin titer is the most accurate test is also defective.
65. Tx for Cold Agglutinin Disease: 1. Stay warm
2. Rituximab, sometimes plasmapheresis Sleep has a relative hypoventilation and leads to mild increase
3. Cyclophosphamide and cyclosporine (immunosuppressive in pCO2 and acidosis.
agents) can decrease antibody. 76. Most common presentation with PNH: Episodic dark urine
(from first urination of the day)
DON'T USE STEROIDS or Splenectomy does not work in cold Pancytopenia and iron deficiency anemia
agglutinin disease! Clots in unusual places (not just DVT or PE)
66. Cold Agglutinin vs. Cryoglobulins: Cryoglobulins also has IgM 77. MCC of death in Paroxysmal nocturnal hemoglobinuria.:
and doesn't deal with steroids, however cryoglobulin disease is Thrombosis of the mesenteric or hepatic veins (large vessels)
associated with Hepatitis C, Joint Disease, and 78. Paroxysmal Nocturnal Hemoglobinuria is a stem cell defect
Glomerulonephritis that may cause these other conditions?: 1. Aplastic anemia
67. G6PD Deficiency Provokers: Sulfa drugs, dapsone, infections, 2. Myelodysplasia
quinidine, fava beans, primaquine, nitrofurantion, oxidative 3. Acute leukemia
stress 79. Diagnostic Test for PNH: Most accurate test is decreased
68. Best initial test for G6PD.: CD55 and CD59. Flow cytometry.
80. Tx for PNH:
1. Initial
2. Cure
3. Decreased RBC Destruction: 1. Initial - Prednisone
2. Cure - allogeneic Bone marrow transplant
3. Decreased RBC Destruction inactivates C5 - Eculizumab
4. Folic acid and replacement with transfusion as needed.
81. Aplastic Anemia Definition: Pancytopenia of unclear etiology
82. Pancytopenia Causes: Radiation/toxins toulene, DDT, benzene
Drugs: Sulfa, phenytoin, carbamazepine, chloramphenicol,
EtOH, chemotherapy
Heinz bodies (methylene blue) and bite cells SLE
PNH
G6PD level is normal after a hemolytic event, check in 1-2 Infections: HIV, hepatitis, CMV, EBV
months B12 and folate deficiency
69. G6PD Demographic: Sudden anemia in an AA or Mediterranean Thyroid-inhibiting medications such as PTU and methimazole
MALE (XR) 83. Most accurate test for Aplastic Anemia: Bone marrow biopsy
84. Presentation in Aplastic Anemia: Fatigue (anemia)
X-linked recessive disorder inability to generate glutathione Bleeding (thrombocytopenia)
reductase and protect the cell from oxidative stress Infection (low white cell count)
70. HUS and TTP Deficiency: Deficiency with metalloproteinase
ADAMTS13 *aplastic anemia is confirmed by excluding all the causes of
71. HUS and TTP Association: HUS - E. coli pancytopenia
TTP - clopidogrel, cyclosporine, ticlopidine, AIDS, SLE 85. Tx of Aplastic Anemia: 1. Allogeneic bone marrow
transplantation (if under 50)
Both have -
Intravascular hemolysis 2. Tx over 50 or no match for donor- tx with antithymocyte
Thrombocytopenia globulin (ATG) and cyclosporine. Tacrolimus is an alternative
Renal insufficiency to cyclosporine
86. Etiology of Aplastic Anemia: T cells attack the patient's own
TTP: neuro disorders, fever, confusion, seizures marrow (believed)
72. Lab values for TTP and HUS: Thrombocytopenia 87. Polycythemia Vera Etiology: JAK2 mutation, all cell lines
Negative Coombs Test effected but RBC most
Normal PT/aPTT - RBC grow wildly despite low EPO level
73. Tx of TTP and HUS: Plasmapheresis (or FFP)
DO NOT GIVE PLATELETS
74. Paroxysmal Nocturnal Hemoglobinuria: Clonal stem cell defect
with increased sensitivity of RBCs to complement in acidosis.

Deficiency of CD55 and CD59.


Page 61
88. Presentation of Polycythemia Vera: Pruritus (Itching) with 99. Auer rods:
showers because of histamine released from basophils.

Hypertension
HA, blurry vision, tinnitus.
Fatigue
Splenomegaly
Bleeding from engorged vessels
Thrombosis from hyperviscosity

* small number of patients can convert to AML


89. Oxygen, MCV, and EPO in polycythemia vera.: Oxygen normal
EPO low
MCV low
90. Most accurate test for Polycythemia vera: JAK2 Mutation Eosinophilic inclusion in AML
91. Tx of Polycythemia vera: 1. Phlebotomy and Aspirin to prevent - M3 or acute promyelocytic leukemia
thrombosis 100. Best initial test for Acute Leukemia: Peripheral smear
2. Hydroxyurea to lower cell count showing blasts
3. Allopurinol or rasburicase to protect against uric acid rise 101. Most accurate test AML.: flow cytometry
4. Antihistamines - AML associated with Myeloperoxidase
102. CBC on Acute Leukemia: Pancytopenia (infection, bleeding,
target is hematocrit < 45%
fatigue)
92. Pt with over 1.5 million platelets, best initial tx.: Pt has WBC can be elevated but are not functioning
Essential thrombocytosis:
103. M3 (promyelocytic leukemia) is associated with?: DIC
104. Tx of AML/ALL: Start with chemotherapy. If prognosis poor - go
If over 60 and symptomatic - give hydroxyurea (Anagrelide
to bone marrow transplant. Otherwise, continue chemotherapy.
used when RBC suppression from hydroxyurea)

Add ATRA (all-trans-retinoic acid) to those with M3


Aspirin used for erythromelalgia
105. The best indicator of prognosis in acute leukemia is ___.:
93. Ruxolitinib inhibits___.: JAK2
Cytogenetics or assessing the specific chromosomal
94. Mutation in Essential thombocytosis.: 50% of ET patients have
characteristics found in each pt
JAK2 Mutation
95. Older person with pancytopenia and bone marrow showing Good cytogenetics = less chance relapse = more chemo
fibrosis: Myelofibrosis Bad cytogenetics = more chance relapse = immediate BMT
- RBC production turns to liver and spleen (enlarging)
106. Addition to ALL Tx: Add intrathecal chemotherapy
96. Tx for Myelofibrosis: Tumor necrosis inhibiting factor (methotrexate) to ALL tx to prevent ALL in the CNS
Thalidomide and Lenalidomide INC bone marrow production
107. Prevention of tumor lysis related rise to uric acid:
Rasburicase
Ruxolitinib inhibits JAK2 and suppresses myelofibrosis
108. Repeated blood transfusions electrolyte -/+ disturbance:
Hypocalcemia
*Pt < 50-55 allogenic BMT is attempted
109. Most likely dx;
97. Peripheral smear for Myelofibrosis:
- Pruritis
- Splenomegaly
- Vague sx of fatigue, night sweats, fever
- High WBC on exam: CML

(high WBC will have ALL neutrophils)


110. Diagnostic test for CML: CHECK LAP (leukocyte alkaline
phosphatase score)
Low in CML
High in reactive leukocytosis
111. Most accurate test for CML: BCR-ABL done by PCR or FISH

9:22 translocation, Philadelphia Chromosome


Nucleated RBCs and tear drop cells
112. Tx of CML: Tyrosine kinase inhibitors - imatinib, dasatinib,
98. M3 AML Translocation: Chromosomes 15 and 17
nilotinib (Best initial tx)
113. Curative tx for CML: Bone marrow transplant (never first
therapy)

Page 62
114.CML Transformation: Greatest of all myeloproliferative 121. Diagnostic Tests in CLL:
disorders to turn into Acute Leukemia (blast crisis)
115.Next step in management in SOB patient with high WBC
count (225,000). Cells predominantly neutrophils with 4%
blasts.: Leukostasis reaction: Leukapheresis so O2 delivery
maintained
116.Myelodysplastic Syndrome (MDS): Pancytopenia with
hypercellular bone marrow.

Typically in pts above 60. This is a preleukemic condition.


117.Cause of death in Myelodysplastic Syndrome: Infection or
bleeding, before leukemia occurs
Elevated WBC
118. Dx tests in Myelodysplastic Syndrome:
Half patients have hypogammaglobulinemia
Anemia and thrombocytopenia from autoimmune IgG warm
antibodies
Smudge cell with crushed fragile nucleus
122.Richter phenomenon in CLL: Conversion of CLL into high-
grade lymphoma, happens in 5% of pts
123.Paradoxical CLL Property: Will destroy RBC and platelets,
however does not make useful antibodies for infection
124. Tx of CLL: Stage 0 (elevated WBC), stage 1
(lymphadenopathy), and stage II (hepatosplenomegaly) - no tx
Stage 3 (anemia) - 4 (thrombocytopenia) - fludarabine,
cyclophosphamide, and rituximab

Choice that lists : Fludarabine, cyclophosphamide, and


rituximab best initial tx for advanced stage disease (III, IV) or
Asx pancytopenia in routine CBC any pt who is symptomatic (extreme fatigue and painful nodules)
= Alemtuxumab (anti-S2) when fludarabine fails
INC MCV, nucleated RBC
125.Tx of Refractory CLL Stage 2-4: Cyclophosphamide (more
Marrow: Hypercellular
efficacious, more toxic)
Ringed sideroblasts
126. Tx Mild CLL: Chlorambucil (elderly)
Severity depends on blasts
5q deletion (better prognosis) 127. Tx severe infections in CLL: IVIG
PELGER-HUET CELLS (bilobed nucleus) 128.Tx autoimmune thrombocytopenia or hemolysis in CLL.:
119.Treatment of Myelodysplastic Syndrome: 1. Transfusion Prednisone
2. EPO 129. Prophylaxis in CLL: PCP (Bactrim)
3. Azacitidine or decitabine 130.Which is less dangerous? Thrombocytopenia and anemia
4. Lenalidomide for those with 5q deletion, decreases from autoimmune effect, or from marrow infiltration with CLL
transfusion dependence. cells?: Autoimmune effect
5. BMT under age 50 - tx with prednisone
131.Most likely dx middle aged man:
Azacitidine also decreases transfusion dependence and INC - Pancytopenia
survival in MDS - Massive splenomegaly
120.CLL symptoms: FATIGUE - Monocytopenia
Lymphadenopathy - Inaspirable "dry" tap despite hypercellular marrow: Hairy
Spleen or liver enlargement Cell Leukemia
Infection from poor lymphocyte fxn 132. Best initial test for Hairy Cell:
Hemolysis sometimes

Usually > 50 many asx

Smear showing hairy cells - B cells with filamentous


Page projections
63
133.Most accurate test for Hairy Cell: Immunotyping by flow 144.Tx of Hodgkins: 1. Stage Ia and IIa: local radiation with small
cytometry (CD11c) cause chemo
134. Tx for Hairy Cell: Cladribine or Pentostatin
135.Non-Hodgkins Lymphoma Definition: Lymphocyte increase in 2. Stage III and IV or anyone with "B" symptoms:
lymph nodes and spleen, most often widespread on ABVD
presentation. Adriamycin (doxorubicin) cannot use if EF < 50%
Bleomycin
136.Presentation of non-Hodgkins Lymphoma: Painless
Vinblastine
lymphadenopathy
Dacarbazine
May involve pelvic, retroperitoneal, mesenteric structures
Nodes not warm, red, or tender 145.Relapse after radiation in Hodkin disease.: Relapse after
B symptoms: Fever, weight loss, drenching night sweats radiation -> tx with chemo

137. Best initial test for NHL: Excisional biopsy


Relapse after chemo -> extra high dose chemo and BMT

NEEDLE ASPIRATION NOT GOOD ENOUGH BC NORMAL 146.Complications of Radiation in Hodgkins: Increase risk for
LYMPHOCYTES. solid tumors: breast, thyroid, lung CA
High LDH levels = worse severity
Screen for breast cancer 8 years or after tx
CBC normal most cases.
Increased premature Coronary artery disease
138.Staging procedures for NHL which modalities are used?: 1.
CT scan chest, abdomen, and pelvis 147.Most useful to determine dosing for chemotherapy in
2. BM bx Hodgkins: MUGA scan or nuclear ventriculogram to determine
cardiac EF since adriamycin (doxorubicin) is cardiotoxic cannot
139.Staging of NHL: Stage I - lymph node group
use if EF < 50%
Stage II - two lymph groups on same side of diaphragm
Stage III - both sides of the diaphragm, opposite side 148.Adverse effects of chemotherapy in Hodgkin's
Stage IV - widespread disease Doxorubicin
Vincristine
140.Tx of non-Hodgkins lymphoma: 1. Local disease (Stage Ia
Bleomycin
and IIa) - local radiation and small chemo
Cyclophosphamide
Cisplatin:
2. Advanced disease (Stage III and IV, any "B" sxs) - combo
chemo with CHOP and rituximab, an antibody against CD20
Cyclophosphamide
Hadriamycin (doxorubicin or hydroxydaunorubicin)
Ovincristine (oncovin)
Prednisone
141.Tx for MALT: Lymphoma of the stomach in association with H.
pylori, so treat H. pylori

Clarithromycin and amoxicillin


142. Unique Findings in Hodgkins:

1. Doxorubicin - cardiomyopathy
2. Vincristine - neuropathy
3. Bleomycin - lung fibrosis
4. Cyclophosphamide - hemorrhagic cystitis
5. Cisplatin - Renal and ototoxicity
149.Multiple Myeloma: Proliferation of plasma cells, unregulated
production of useless immunoglobulin. Usually IgM and IgA.
Centers around cervical area 150.First initial test MM: X-ray of the affected bone -> lytic lesions
Reed-Sternberg cells "punched out"
Lymphocyte predominant has best prognosis 151.SPEP (serum protein electrophoresis) shows which spike in
Lymphocyte depleted worst prognosis Multiple Myeloma: IgG (60%) or IgA (25%)
143. NHL and Burkitt: Worst prognosis M-spike
50% have light chains (Bence-Jones) protein only

Page 64
152. Lab abnormalities in Multiple Myeloma: 164. Best initial therapy for Waldenstrom: Plasmapheresis
165.Long term treatment for Waldenstrom: Rituximab or
prednisone cyclophosphamide
Bortezomib or lenalidomide
166.Platelet vs factor bleeding.: Platelet bleed: Superficial,
epistaxis, gingival, petechiae, purpura, mucosal surfaces such
as gums, vaginal bleeding

Factor bleed: Deep, joints and muscles


167.Most likely dx:
Isolated thrombocytopenia (normal HCT, normal WBC)
Hypercalcemia, elevated BUN and Cr Normal sized spleen: Consider Immune Thrombocytopenic
Bence-Jones protein, Beta2 microglobulin levels Purpura
Smear with rouleaux: form when IgG paraprotein stick to red IMMUNE - ISOLATED
cells
168.Dx Test for ITP: Antiplatelet antibodies (lack specificity)
BONE MARROW BIOPSY W/GREATER THAN 10% PLASMA
Ultrasound or CT scan to exclude enlarged spleen
CELLS
Megakaryocytes elevated in number
DEC anion gap, IgG cationic INC chloride and bicarb
169.Tx of ITP
Elevated total protein, normal albumin
No Bleed > 30k
153.Technetium Bone Scan on MM: Lytic "punched out lesions"
Mild bleed < 30k
don't show up on this scan. Technetium is for 'uptake' so
Severe bleed <10k
osteoblastic diseases are effected.
Recurrent episodes
154.Most likely dx: Splenectomy/steroids not woring: No bleed, count > 30,000 -
Bone pain from pathologic fracture: Multiple myeloma No treatment
Osteoclast activating factor, also reason for hypercalcemia and Mild bleed < 30k - Glucocorticoids
lytic lesions. Severe bleed < 10k - IVIG, Anti-Rho (anti-D)
155.Hyperuricemia, Anemia, and Renal Failure in Multiple Recurrent episodes, steroid dependent - Splenectomy
Myeloma: Hyperuricemia - from increased turnover
Anemia - from infiltration of marrow with massive number of Splenectomy/steroids ineffective - Romiplostim eltrombopag
plasma cells (synthetic thrombopoietin for ITP)
Renal failure - from Bence Jones proteins accumulation. 170.Vaccinations prior to splenectomy: Neisseria meningitidis
Haemophilus influenzae
- renal failure and infection common cause of death in myeloma Pneumococcus
156.Decreased Anion Gap in MM: From IgG being cation. Cl- and 171.Most likely dx:
HCO3- go up. Decreasing anion gap. Epistaxis, gingival, gums bleeding.
157.UA vs. 24 hour urine in MM: UA will show fewer protein Normal platelet count
because it only detects albumin. Markedly worsened after use of aspirin
24 hour urine shows excess Bence Jones. aPTT may be elevated: Von Willebrand Disease
158.Most accurate test for MM: Bone marrow biopsy. (ONLY 172.Dx Test for vWF: VWF antigen level may be decreased
THING THAT CAUSES GREATER THAN 10% plasma cells on BM Ristocetin cofactor assay: detects VWF dysfunction
Bx) Factor VIII activity
159.Best initial tx for MM: Dexamethasone with lenalidomide (or Bleeding time increased (rarely done)
bortezomib) 173. Treatments for vWF: Best initial - DDAVP

*most effective tx under < 70 is an autologous BMT with stem If no response - give factor VIII replacement or VWF
cell support after chemo with lenalidomide and steroids concentrate
160. When to use Melphalan for MM: Older, fragile patients. 174.Hemophilia Presentation: Male patient with joint bleeding
161.MGUS: M-spike with no plasma cell increase in bone marrow PT normal, PTT prolonged
biopsy Mixing studies will correct
175.Most accurate test for hemophilia: Specific assay for VIII and
1% a year transformation into myeloma IX
162.Most likely dx: 176.Tx for hemophilia: Mild - DDAVP
Lethargy Severe - replace specific missing factor
Blurry vision and vertigo 177.Factor XI Deficiency: Typically asymptomatic unless trauma or
Engorged blood vessels in the eye sx INC bleeding. Mixing study will correct.
Mucosal bleeding
Raynaud phenomenon: Waldenstrom Macroglobulinemia FFP can stop bleeding if needed.
163.Diagnostic Testing for Waldenstrom: Cold agglutinin 178.Factor XII Deficiency: Elevated aPTT, no bleeding. No therapy
IgM spike on SPEP needed.
No bone lesions
Page 65
179.DIC Risk Factors (associated diseases): Sepsis 195.HUS Cause, Symptoms, and Organ Manifestations: HUS is
Burns caused by a toxin from E. coli. Symptoms include jaundice from
Abruptio placentae or amniotic fluid embolism intravascular hemolysis (leading to thrombocytopenia as well as
Snake bites anemia). System at highest risk is renal due to injuring
Trauma resulting in tissue factor release endothelial cells in the kidney.
Cancer
180.Diagnostic Tests for DIC: Elevation of both PT and aPTT
Low platelet count
Elevated d-dimer and fibrin split products
Decreased fibrinogen level (consumed)
181.Tx for DIC: If platelets < 50k - replace platelets and clotting
factors with FFP.

Cryoprecipitate if FFP does not control bleeding.


182.Most common cause of thrombophilia: Factor V Leiden
mutation.
- use warfarin INR 2-3 for 6 mo.
183.Heparin Induced Thrombocytopenia occurs ___ after using
heparin. There is a drop in ____.: 5-10 days after using
heparin. DROP in platelets (30% drop). Venous clots more
common.
184. Dx test for HIT: ELISA for platelet factor 4 antibodies
185.Tx for HIT: 1. Stop heparin
2. Start direct thrombin inhibitors: argatroban, lepirudin,
bivalirudin, and fondaparinux
3. Warfarin NOT to be used until a thrombin inhibitor is on board.
Fondaparinux is safe.
186.Do not trasfuse platelets with those with?: HIT can worsen
the thrombosis
187. Antiphospholipid Syndromes: Lupus anticoagulant

Anticardiolipin antibody - associated with multiple spontaneous


abortions
188.Thrombophilia + abnormal PTT: Antiphosphospholipid
Syndromes are the only diseases where this happens.
189.Best initial test for Antiphospholipid syndrome: Mixing study,
PTT remains elevated.
190.Most accurate test for lupus anticoagulant.: Russel viper
venom test
191. Tx for Antiphospholipid syndrome (APL): Heparin or Warfarin.

APL syndrome may require lifelong anticoagulation with only 1


clot.
192.Schilling Test Function: To determine if B12 defeciency is due
to pernicious anemia or malabsorption syndrome
193.Most likely dx:
Isolated thrombocytopenia without anemia or leukopenia.
What should you screen for with this dz?: Idiopathic
thrombocytopenic purpura (HIV and Hep C). Isolated
thrombocytopenia is presented in 5-10% of chronic HIV pts.
194.Most likely dx:
Increased hemoglobin concentration
Splenomegaly
Jaundice
Anemia
Coombs-negative hemolytic anemia: Hereditary spherocytosis.
Check for osmotic fragility with acid glycerol lysis test and
eosin-5-maleimide test.

Page 66
MTB Infectious Diseases
MTB Q and A

1. Antibiotic class with the greater efficacy than other classes: 12. Bugs resistant to all forms of cephelosporins: Listeria, MRSA,
Beta-lactams and Enterococcus
2. List Betalactam antibiotics?: Penicillins 13. General cephalosporin coverage in all classes: Covers Strep
Cephalosporins A, B, C, viridans, E. coli, klebsiella, proteus mirabilis
Carbapenams 14. Allergies to PCN and Cephelosporins: PCN Rash - can give
Aztreonam cephalosporins
3. Penicillin coverage (bugs): Strep viridans
Strep pyogenes PCN anaphylaxis - give a non-beta-lactam antibiotic
Oral anaerobes 15. First generation Cephalosporins: Cefazolin, Cephalexin,
Leptospira Cephradrine, Cefadroxyl
Syphillis 16. 1st generation Cephs used to treat: 1. Staphylococci:
4. Ampicillin and Amoxicillin coverage (bugs): Same as PCN: methicillin sensitive = oxacillin sensitive = cephalosporin
Strep viridans sensitive
Strep pyogenes 2. Streptococci (except Enterococci)
Oral anaerobes 3. Gram negative like E. coli but not Pseudomonas
Leptospira 4. Osteomyelitis, septic arthritis, endocarditis, cellulitis
Syphillis 17. 2nd generation Cephs: Cefotetan, Cefoxitin, Cefaclor, Cefprozil,
+ Cefuroxime, Loracarbef
E. coli - add coverage for anaerobes and more gram - bacilli along with
Lyme same coverage as 1st gen cephalosporins
Gram negatives
18. Cefotetan and cefotoxin treat ?: Best initial therapy for PID
5. HELPS coverage by Amoxicillin: H. influenza, H.pylori combined with doxycycline
E.coli
19. Cefotetan and cefoxitin adverse effect: Increase risk of
Listeria
bleeding (deplete prothrombin) and give a disulfiram reaction
Proteus
with EtOH
Salmonella
20. Cefuroxime, loracarbef, cefprozil, cefaclor treat ?:
6. Penicillins/Amoxicillins are the best initial treatment for
Respiratory infections such as bronchitis, otitis media, and
which infections?: Otitis media
sinusitis
Dental infections
Endocarditis 21. 3rd Gen Cephs: Ceftriaxone, cefotaxime, ceftazidime
Lyme disease 22. Ceftriaxone in neonates: Should be avoided because of
UTI impaired biliary metabolism
Listeria 23. Ceftazidime coverage: Covers pseudomonas
Enterococcal infections 24. Ceftriaxone coverage: Meningitis
7. List penicillinase resistant penicillins (PRPs): Oxacillin CAP
Cloxacillin Gonorrhea
Dicloxacillin Lyme involving the heart and brain
Nafcillin 25. Cefotaxime uses?: Superior to ceftriaxone in neonates
8. Penicillinase resistant penicillins (PRPs) are used to treat: - SBP
Skin infections: cellulitis, impetigo, erysipelas 26. 4th generation Ceph and Use: Cefepime
Endocarditis
Meningitis Used to treat neutropenia, fever, ventilatory associated
Bacteremia from staph pneumonia
Osteomyelitis and Septic arthritis if sensitive organism 27. 5th generation Ceph and Use.: Ceftaroline
9. Methicillin adverse reaction and is never the right answer - gram negative bacilli and MRSA, not Pseudomonas
why?: Allergic interstitial nephritis --> Renal failure 28. Carbapenams: Imipenam, Meropenam, Ertapenem (does not
10. Piparacillin, ticarcillin, azlocillin, mezlocillin coverage and cover pseudomonas), Doripenam
best initial therapy for?: Gram negative coverage of E. coli and
Proteus, along with Pseudomonas cover gram - bacilli, many that are resistant, anaerobes,
streptococci, staph = tx neutropenia and fever
Cholecystitis 29. Monobactams: Aztreonam
Pyelonephritis USED FOR GRAM NEGATIVES (including pseudomonas)
Bacteremia No cross rxn with PCN
Hospital acquired or ventilator associated pneumonia
30. Name fluoroquinolone.: Ciprofloxacin
Neutropenia and fever
Gemifloxacin
Levofloxacin
Used in combo with beta-lactamase inhibitor such as
Moxifloxacin
Tazobactam or Clavulanic acid.
11. The only cephalosporin that can cover MRSA.: Ceftaroline Page 67
31. Best initial therapy for community acquired pneumonia.: 46. MRSA Best Initial Tx: Vancomycin
Fluoroquinolones Linezolid: can have a reversible bone marrow toxicity
32. Fluoroquinolone for cystitis and pyelonephritis: Ciprofloxacin Daptomycin: elevates CPK
33. Fluoroquinolone for GI/Diverticulitis: Cipro, Gemi, and Tigecycline
Levofloxacin can be used but need metronidazole for anerobes. Ceftaroline
Telavancin, Dalbavancin, Ortavancin, Tedizolid
Moxifloxacin is the only one that doesn't need metronidazole 47. Minor MRSA infections of the skin area tx with?: TMP/SMX
can be used as a single agent for tx diverticulitis Clindamycin
34. Fluoroquinolones treat (bugs): Gram negative bacteria, Doxycycline
pseudomonads Linozolid
35. Quinolone adverse effects: Bone growth abnormalities in 48. Anaerobes (above and below diaphragm): Above: Clindamycin
children or Pen
Below: Metronidazole or beta-lactam/lactamase combo
Tendonitis and Achilles tendon rupture
*piperacillin, carbapenems, and 2nd gen cephalosporins also
Gatifloxacin removed because of glucose abnormalities cover anaerobes.

36. Aminoglycosides treat (bugs): Gram negative bacilli infections 49. Gram Negative Bacilli Common infections: Bowel (peritonitis,
in bowel, bladder, and blood diverticulitis)
UTI (pyelonephritis)
Synergistic with beta lactams for enteroccci and staphylocci Liver (cholecystitis, cholangitis)
37. Aminoglycosides can they treat anaerobes?: Can't work 50. Agents that cover gram negatives (E.coli, Klebsiella, Proteus,
because aminoglycosides need oxygen Pseudomonas, Enterobacter, Citrobacter) that can cause
infection bowel, urinary tract, and liver.: Quinolones
38. Aminoglycoside SE: Nephrotoxic and Ototoxic
Aminoglycosides
39. Indication to use Nitrofurantoin: Cystitis, especially in pregnant Carbapenaems
women Piperacillin, ticarcillin
40. Doxycycline Use: Chlamydia Aztreonam
Lyme disease of skin and joints Cephalosporins
Rickettsia 51. CNS infections general symptoms.: Fever, N/V, HA, seizures
MRSA of skin (cellulitis)
52. Most likely dx:
Primary or secondary syphilis if PCN allergy
1. Stiff neck, photophobia, meningismus + general CNS
Borrellia, Ehrlichia, Mycoplasma
2. Confusion + general CNS sx
41. Doxycycline SE: Yellowing of teeth in children 3. Focal neurologic findings + CNS sx: 1. Meningitis
Fanconi syndrome (RTA type II proximal) 2. Encephalitis
Photosensitivity 3. Abscess
Esophagitis/ulcers
53. Common causes of Meningitis in Adults: S. pneumonia (60%)
42. Trimethoprim/Sulfamethoxazole (Bactrim) Use: Cystitis Group B strep
Pneumocystis pneumonia treatment and ppx H. influenza
MRSA of skin and soft tissue (cellulitis) Neisseria meningitidis
43. Bactrim SE: Rash, hemolysis with G6PD deficiency Listeria

Bone marrow suppression because of folate antagonist Staphylococcus in those w/ recent Nsx
44. Mouth and GI abscesses coverage.: Beta-lactam/beta- 54. CNS Sx (meningitis) + AIDS < 100 CD4: Cryptococcus
lactamase inhibitor combinations: 55. CNS Sx + rash shaped like a target, joint pain, facial palsy:
Lyme disease
- Amoxicillin/clavulanate
56. CNS Sx + rash moving from arms/legs to trunk: Rickettsia
- Ticarcillin/clavulanate
(rocky mountain spotted fever)
- Ampicillin/sulbactam
- Piperacillin/tazobactam 57. CNS Sx + Pulmonary TB: Tuberculosis

45. Best initial therapy for gram positive organisms: staph and 58. CNS Sx in an adolescent with a petechial rash.: Neisseria
strep.: Oxacillin, cloxacillin, dicloxacillin, nafcillin (PRP) 59. Best initial and most accurate test for meningitis suspicion:
LP
1st gen Cephs: Cefalexin, cefazolin 60. Bacterial meningitis LP: 1000s cell count, neutrophils
Elevated protein
Fluoroquinolone's Decreased glucose
Usually stain available
Macrolides (azithromycin, clarithromycin, erythromycin) are 3rd 61. Cryptococcus, Lyme, Rickettsia LP: 10-100s cell count,
line agents and tend to be more toxic lymphocytes
Protein and glucose possibly elevated
Culture and stain negative
Page 68
62. Tuberculosis LP: 10-100s lymphocytes 76. Most accurate diagnostic test for sinusitis.: Sinus biopsy or
ELEVATED protein aspirate
glucose may be low
Negative culture Only needed bx when:
63. Viral LP: 10-100s lymphocytes Infection frequently recurring
Glucose and protein usually normal No response to antibiotics
Negative culture 77. Streptococcal Pharyngitis Criteria: Pain with swallowing
64. When would a Head CT be needed before LP?: Suspicion of a Enlarged lymph nodes
space occupying lesion Exudate in the pharynx
Answer head CT first when any of the following is present: Fever
- Papilledema No cough and no hoarseness
- Seizures
- Focal neurological abnormalities (90% chance its strep pharyngitis when these features present)
- Confusion interfering with the neurologic examination 78. Best initial test for pharyngitis.: Rapid strep test
- small vesicles or ulcers: HSV or herpangina
*Contraindication to immediate LP giving antibiotics is the best - membranous exudates: diphtheria, Vincent angina, or EBV
initial step 79. Best initial therapy for strep throat: PCN or Amoxicillin
65. When is bacterial antigen testing useful?: When the patient - strep pharyngitis is tx to prevent rheumatic fever
has received antibiotics prior to LP and the culture may be 80. PCN allergy for strep: Cafalexin if allergy was a rash
falsely negative.
66. Best initial treatment for bacterial meningitis: Ceftriaxone, If anaphylaxis, use clindamycin or macrolide
vancomycin, and steroids 81. Flu like sx within 48 hours, next step?: Nasal swab
67. Bacterial meningitis with Listeria risk which abx should be - detect the antigen associated with influenza
added?: Add ampicillin 82. Tx of flu within 48 hours: Neuraminidase inhibitors shorten
course; oseltamivir, zanamivir
Risks: Elderly, neonates, steroid use, AIDS, 83. Best initial test for infectious diarrhea: Blood or fecal
immunocompromised (alcoholism), pregnant leukocytes (won't get a specific organism)
68. Additional precautions/treatments for neisseria meningitis: Lactoferrin is a better answer.
Respiratory isolations
MOST ACCURATE: stool culture
Rifampin, ciprofloxacin, or ceftriaxone to close contacts to 84. Blood and WBC in Stool DDx: Salmonella: poultry
decrease nasopharyngeal carriage Campylobacter: most common cause
69. Most common neurologic deficit of untreated bacterial E. coli - HUS O157:H7
meningitis?: 8th cranial nerve deficit or deafness Shigella - also associated with HUS
70. Most common cause for encephalitis: Herpes simplex Vibrio parahaemolyticus - shellfish and cruise ships
- Head CT first since presence of confusion Vibrio vulnificus - shellfish, hx of liver dx, skin lesions
- Pt has acute onset fever and confusion Yersenia - high affinity for iron, hemochromatosis, blood
71. Most accurate test for herpes encephalitis: PCR of CSF transfusions
72. Best initial tx of herpes encephalitis. ___ is used for C. difficile - WBC and RBC in stool
resistant herpes.: Acyclovir best initial tx 85. No blood in stool with infectious diarrhea: Viral
Giardia: camping/hiking and unfiltered fresh water
Foscarnet is used for acyclovir-resistant herpes Cryptosporidiosis: AIDS less than 100 CD4 (detect with
73. Most sensitive physical finding for otitis media.: Immobility modified acid fast stain)
of tympanic membrane Bacillus cererus - vomiting
Staphylococcus - vomiting
74. Most accurate diagnosis for otitis media: Tympanocentesis,
only chosen with multiple recurrences or if no response to 86. Scombroid Diarrhea: Most rapid onset, wheezing, flushing
multiple antibiotics rash.
Found in fish
75. Best treatment for otitis media: Amoxicillin
Treat with antihistamines

-or- (if no response or recently tx with Amoxicillin) 87. Diarrhea Tx: Mild: Oral fluid replacement

Amox/clavulanate Severe: Fluid replacement and ciprofloxacin


Azithromycin, clarithromycin 88. Severe infectious diarrhea means which symptoms?:
Cefuroxime, loracarbef HypOtension
Tachy
Fever
Abd pain
Bloody diarrhea
Metabolic acidosis
89. Treatment for Giardia Diarrhea: Metronidazole, tinidazole
Page 69
90. Cryptosporidiosis Diarrhea Tx: Treat underlying AIDS, 110.The key points regarding hepatic C treatment?: 1. Acute
nitazoxanide hepatitis C is treated!
91. Tx of Viral, B.cereus, Staph Diarrhea.: Fluid support as 2. Hepatitis C is the only form of acute hepatitis to be treated!
needed 3. Everyone born between 1945-1965 should be tested for hep
92. Which hepatitis is typically worse in pregnancy, especially C regardless of risk factors
among pts from East asia?: Hepatitis E 4. Cure rate are exceeding 90% for the first time ever
5. These advances prevent the need for liver transplant
93. Dx test for hepatitis: Increased direct bilirubin
Increased ALT:AST ratio 111. Ribavirin SE: Anemia
Increased alkaline phosphatase 112. Urethritis vs. Cystitis: URETHRAL DISCHARGE (Urethritis)
94. Most likely association with increased mortality in hepatitis?:
Prothrombin time (PT) Both have increased urinary frequency and burning
- INC risk fulminant hepatic failure and death 113. Best initial test for urethritis: Urethral swab for Gram stain.
95. Best initial diagnostic test for Hep A-E: IgM for acute infection
and IgG for resolution (hep C check PCR for RNA level) Urine testing for nucleic acid amplification can also detect
gonorrhea and Chlamydia.
Hep B and C viral levels via PCR indicate improvement of 114.Most accurate test for urethritis: Urethral culture, DNA probe,
treatment or nucleic acid amplification test
96. First to be abnormal after acquiring Hep B.: Surface antigen 115.Causes of Urethritis: N. gonorrhoeae (intracellular gram -
97. Directly correlated with amount of DNA polymerase and Hep diplococci)
B (viral load): Envelope antigen (E-antigen directly correlated Chlamydia trachomatis
with degree DNA polymerase) Mycoplasma genitalium
Ureaplasma
98. Vaccination which serologic pattern is positive hepatitis B.:
positive surface antibody 116.Tx for Urethritis: Ceftriaxone + Azithromycin
Ceftriaxone + Doxycycline
99. When does transmission of Hep B end?: Loss of surface
antigen (NOT gain of antibody).
*Cefiximine no longer used for gonorrhea
100. Patient with the most benefit to anti-viral therapy for
117.Cervical discharge and inflamed "strawberry" cervix. Dx.
Hepatitis?: Patient with greatest viral load as indicated by e-
Tx.: Cervicitis
antigen
- Swab for nucleic acid amplification
101. Which of test is the best indicator that a pregnant woman - Tx: Ceftriaxone + Azithromycin single dose
will transmit infection to her child?: e-antigen/DNA - Doxycycline (equal efficacy to Azithro but harder to use)
polymerase
118.Most likely dx:
Lower abdominal tenderness
Perinatal transmission is the most common method of
Lower abdominal pain
transmission worldwide
Fever
102. Acute Hep C Medications: Interferon, ribavirin, and either Cervical motion tenderness
boceprevir or telepravir Leukocytosis: Exclude pregnancy - woman w/ lower abd pain or
103. Which meds alone can be oral therapy for hepatitis C for tenderness or cervical motion tenderness
genotypes 2 and 3?: Sofosbuvir and ribavirin
104. Hep B Chronic Definition and Rate: 10% of the time Consider PID
Defined as surface antigen for more than 6 months 119. Most accurate test for PID: Laparoscopy (rarely needed)
105. Hep B Chronic Tx: E-antigen present: 120.Tx for PID: Inpatient: Cefoxitin or Cefotetan combined with
doxycycline
Tx with one of the following:
Entecavir, adefovir, lamivudine, telbivudine, interferon, or Outpatient: Ceftriaxone and doxycycline (possibele with Metro)
tenofovir
106. Adverse effects of interferon: Arthralgia/myalgia Anaphylaxis to PCN: Levofloxacin and Metro as an outpatient
Leukopenia and thrombocytopenia Clinda, gent, doxy as Inpatient
Depression and flu-like symptoms 121. Painless ulcer which STD.: Syphills
107. Hepatitis C treat with ___ for genotype 1.: Ledipasvir and 122. Painful ulcer which STD.: Chancroid (Haemophilus ducreyi)
sofosbuvir 123.Lymph nodes tender and suppurating which STD.:
108. Goal of chronic hepatitis therapy: Reduce DNA polymerase to Lymphogranuloma venerum (L-serotype Chlamydia
undetectable levels trachomatis)
124.Vesicles prior to ulcer and painful which STD.: Herpes
Convert those patients with e-antigen to having anti-hepatitis e- simplex
antibody
109. Risk of fibrosis progressing to cirrhosis in Hep C.: If active
viral replication of Hep C, fibrosis will progress to cirrhosis

Page 70
125.Diagnostic Test for Syphillis: Dark-field microscopy (if 138.Which test is the most sensitive for neurosyphilis in CSF?:
positive for spirochete no further testing needed) FTA-ABS 100% sensitive
139.False positive VDRL/RPR: Infection, older age, injection drug
VDRL or RPR (75% sensitive) use, AIDS, malaria, antiphospholipid syndrome, and endocarditis
FTA or MHA-TP (confirmatory)
126.Chancroid (Haemophilus ducreyi) Dx test: Stain and culture *titers of VDRL and RPR are reliable at greater than 1:8
on specialized media 140.Tx Syphilis: Primary and secondary: single IM injection PCN.
127. Lymphagranuloma venerum Dx test: Complement fixation Oral Doxy if penicillin allergy
titers in blood
Tertiary: IV penicillin, desensitize to PCN if PCN allergy
Nucleic acid amplification testing on swab 141.Jarish-Herxheimer Reaction with Syphilis symptoms and
128.Herpes simplex diagnostic test. Best initial test? Most treatment of the reaction with?: Fever and worse symptoms
accurate test?: Tzanck prep best initial after treatment
Viral culture most accurate test
129.Tx for Syphillis: Single dose of IM benzathine penicillin Give aspirin and antipyretics, it will pass
Doxycycline if PCN allergy 142.When to do desensitization with syphillis?: Tertiary
130. Tx for Chancroid: Azithromycin (single dose) symptoms (neurosyphilis) and pregnant women
131. Lymphogranuloma venerum Tx: Doxycycline 143.Removing genital warts (condyloma acuminata) tx: From HPV
132.Herpes simplex Tx: Acyclovir, valacyclovir, famciclovir - detected based on visual appearance
Foscarnet for acyclovir-resistant herpes
Cryotherapy with liquid nitrogen, surgery for large ones
133.Topical or Oral Acyclovir for Herpes: Topical is worthless.
Use oral
Consider podophyllin or trichloroacetic acid
134.A woman comes in with multiple painful genital vesicles.
144.Immunostimulant that can slough off HPV lesions: Imiquimod
What is the next step in management?: Acyclovir orally ->
- also works for actinic keratosis and basal cell cancer
presentation is clear for herpes with multiple vesicles of the
- does not burn or damage skin
mouth or genitals, dx testing is not necessary
145.Most likely dx:
135.Most likely dx:
Found on hair-bearing areas
Painless genital ulcer with heaped up indurated edges and
Causes itching
painless adenopathy: Primary syphilis
Visible on surface
- chancres heal spontaneously even w/o tx, PCN given prevent
Tx with permethrin: Pediculosis (Crabs)
later stages
Lindane also works, but is more toxic
136.Most likely dx
146.Most likely dx:
Rash (palms and soles)
Web spaces between fingers and toes
Alopecia areata
Nipples or near genitals
Mucous patches
Burrows visible (Smaller than pediculosis)
Condyloma lata: Secondary syphillis
Tx with permethrin:
137. Tertiary Syphilis Manifestastions:

Scabies
- scrape and magnify
- widespread disease is "crusted" or hyperkeraototic and
responds to ivermectin, severe disease needs repeated dosing
147.Most likely dx, initial tx:
Dysuria
1. Neurosyphillis Fever
-Meningovascular (stroke from vasculitis) WBC in UA greater than 10: E. coli most common cause
-Tabes dorsalis (loss of position, vibratory sense, incontinence) UTI
-General paresis (memory and personality changes)
-Argyll Robertson pupil (reacts to accommodation, but not Initial tx quinolone for pyelonephritis
light)
2. Aortitis (AR, aortic aneurysm)
3. Gummas (skin and bone lesions)
Page 71
148.Anatomic defects lead to UTIs such as?: Stones 156.Pt presents with blood cultures growth Clostridium septicum
Strictures what should be done?: Colonoscopy
Tumor or BPH - even greater association with colon pathology than strep.
DM bovis
Foley catheter, neurogenic bladder 157.Culture Negative Endocarditis: 1. Oscillating vegetation on
149.Most likely dx and tx: ECHO
Dysuria
Mild or absent fever 2. Three minor criteria:
Suprapubic pain/discomfort: Cystitis - Fever > 100.3
- Risk of injection drug use or prosthetic valve
Nitrofurantoin or fosfomycin - Embolic phenomena
Bacterium (TMP/SMZ) if local resistance low 158.Best initial therapy for endocarditis: Vancomycin and
Ciprfloxacin not used used routinely Gentamicin
Cefixime 159.Culture specific treatment for endocarditis:
150.Men with UTIs have anatomic abnormalities much more often 1. Viridian's
than women. Best initial test and accurate test.: Best initial 2. S. aureus
test: UA with > 10 WBCs 3. Fungal
Most accurate test: Urine cx 4. S. epidermis
151.Most likely dx and tx: 5. Enterococci: 1. Viridans - Ceftriaxone for 4 weeks
Dysuria 2. S. aureus - oxacillin, nafcillin, cefazolin
High fever 3. Fungal - Amphotericin and valve replacement
Flank pain or costovertebral angel tenderness: Pyelonephritis 4. S. epidermidis or resistant - Vanco
5. Enterococci - Ampicillin and gentamicin
UA shows increased WBC 160.Tx of resistant organism in endocarditis?: Add
CT imaging to r/o anatomical problems aminoglycoside and extend the duration
161.Surgery for Endocarditis Indications.: CHF or ruptured valve
Tx with ceftriaxone, ertapenem or chordae tendeneae
Ampicillin and gentamicin until culture results are known Prosthetic valves (rifampin is added)
Ciprofloxacin Fungal endocarditis
152.Most likely dx: Abscess
Tender prostate on examination AV block
Perineal pain: Acute Prostatitis Recurrent emboli while on antibiotics
162.Add __ for prosthetic valve endocarditis with
Increased yield of urine culture with prostate massage Staphylococcus.: Rifampin
163.Single strongest indication for endocarditis surgery: CHF
Tx with ceftriaxone, ertapenem
and acute valve rupture
Ampicillin and gentamicin until culture results are known
Ciprofloxacin 164.Most common causes of culture negative endocarditis.:
Coxiella and Bartonella
Bactrim for 6-8w for chronic prostatitis 165.HACEK organisms for culture negative endocarditis:
153.Perinephric Abscess Suspicion: When pyelonephritis doesn't Haemophilus aphrophilus
get better despite appropriate culture and dosage. Perform Haemophiluls parainfluenza
sonogram or CT scan. Actinobacillus
Cardiobacterium
Drainage and culture mandatory. Eikenella
Kingella
154.Endocarditis complications: Splinter hemorrhages
Janeway lesions (flat and painless), Osler nodes (raised and
Ceftriaxone is used for the HACEK groups
painful), Roth spots on eyes
Brain (mycotic aneurysm) 166.Indications for Prophylaxis in Endocarditis: 1. Significant
Kidney hematuria and glomerulonephritis cardiac defect - prosthetic valve, previous endocarditis, cardiac
Conjunctival petechiae transplant, unrepaired cyanotic heart disease
Splenomegaly
Septic emboli to the lungs 2. Bacteremia risk - dental work with blood, respiratory tract
surgery
Pt presents with fever and new murmur 167.Initial management for Endocarditis PPx: Ampicillin (prior to
155.Best initial test for endocarditis: Blood culture procedure), if PCN allergic - clindamycin, azithromycin,
Transthoracic echocardigoram clarithromycin
Transesophageal echocardiogram

Page 72
168.Lyme disease bacteria and transmission: Bacteria: Borrelia 183.Indication for Viral resistance testing (genotyping) for HIV.:
burgodorferi 1. Prior to initiating antiretroviral meds
Transmission: Ixodes scapulars tick - DEC starting pt on a med that virus resistant to
2. Evidence of tx failure (rising PCR-RNA viral load)
Untx infection: joint pain, cardiac dz, neuro dz 184.Strongest indication for antiretroviral therapy: CD4 below
500
Regions: Connecticut (LYME, Connecticut) 185.When is HIV tx started?: CD4 below 500
169. Most commonly affected joint in LYME disease: Knee Viral load higher than 100k
170.Most common neurologic finding in Lyme disease: 7th cranial Opportunistic infection
nerve or Bells palsy 186.Best initial drug regimen for HIV: Emtricitabine, Tenofovir,
171.Most common cardiac manifestation in Lyme disease: and Efavirenz (Atripla)
Transient AV block 187.HIV first line medications: Three drugs from 2 classes:
172.Most likely dx: Classes: RTI, non-nucleoside RTI, protease inhibitors
Round red lesion with a pale area in the center
Fever Sample:
Joint pain (oligoarthritis) Atazanvir, daunavir, or raltegravir (protease inhibitors)
CNS or PNS sx like meningitis, encephalitis, or nerve palsy combined with Emtricitabine/tenofovir (RTIs)
Myocarditis or ventricular arrhythmia: Lyme disease
173.Serologic testing for Lyme Indications: Indicated when rash *Ritonavir used with other PI to boost their levels
is absent: joint, neuro, or cardiac functions via IgM, IgG, *Elvitegravir is integrate inhibitor used with cobicistat that
ELISA, Western blot, and PCR testing inhibits its metabolism and boosts its level
174.Lyme disease Tx 188.Postexposure prophylaxis HIV: Given for 4 weeks with
Asx combination therapy
Rash - significant needle stick injuries and sexual exposure
Joint/nerve palsy 189.Abacavir hypersensitivity prediction: HLA B 5701 testing -
Cardiac: 1. Asx tick bite - none dangerous with this mutation
190.Adverse side effects of HIV Medications:
2. Rash/joint/nerve palsy - Doxycycline or Amoxicillin (pt is Zidovudine
pregnant) or Cefuroxime Stavudine and didanosine
Abacavir
3. Cardiac or neuro findings (not nerve palsy) - IV Ceftriaxone Protease inhibitors
175.A single dose of doxycycline is indicted w/in 72 hours of Indinavir
Ixodes scapulars tick bite when?: 1. Ixodes scapularis clearly Tenofovir: 1. Zidovudine- anemia
identified as the tick causing the bite 2. Stavudine and didanosine - peripheral neuropathy and
2. Tick attached for longer than 24 to 48 hours pancreatitis
3. Engorged nymph-stage tick 3. Abacavir - hypersensitivity, Stevens-Johnson reaction
4. Endemic area 4. Protease inhibitors - hyperlipidemia, hyperglycemia
176. Normal level of CD4 cells: 600-1000 5. Indinavir - nephrolithiasis
177.Infection with profound immunosuppression when the CD4 6. Tenofovir - renal insufficiency
count drops below?: 50 191. HIV drugs not safe during pregnancy: Efavirenz
178. PCP in HIV: When CD4 count below 200 192.HIV medication for infant: Zidovudine during delivery and 6w
179.HIV Increased Infections (still w/CD4 above 200): Shingles after to preven transmission
(varicella zoster) 193.Pregnant HIV patients: Antiretrovirals throughout entire
Herpes simplex pregnancy
TB 194.Indication for C-section in HIV mother: Viral load greater than
Oral/vaginal candidiasis 1000
Bacterial pneumonia CD4 count less than 350
Kaposi sarcoma 195.Infection Risk with Hemochromatosis: Listeria
180.Best initial test for HIV: ELISA Yersinia
Confirmed with Western Blot Vibrio vulnificus
181.Best initial test for HIV in infants: PCR (ELISA unreliable 196.Most likely dx and tx:
because maternal HIV antibodies may be presents for up to 6 Gram negative bacilli
mo. after delivery) or viral culture Intubated ICU Patient
182.Utility of PCR-RNA level in HIV: Measures response to Fever
therapy Leukocytosis: Pseudomonas, treated with cefepime, zosyn,
Detects tx failure (rising levels are bad) aztreonam, or gentamicin.
Diagnosing HIV in babies 197.CF Pneumonia in Young and Old (+Tx for each): Young - S.
Aureus, tx with vancomycin
*Undetectable levels below 50/microL indicate CD4 likely rise Old - Pseudomonas, tx with ceftazidime, amikacin, or cipro.
Page 73
198. Tx of Pasteurella (Cat Bite): Amoxicillin/clavulanic acid
199.Oral Leukoplakia: White patch that can't be wiped off, risk factors similar to squamous cell carcinoma. Induration and ulcerated areas
need a biopsy.
200.Kaposi Sarcoma Lesions: Caused by HHV 8 in HIV patients, turn from light brown to violet. Manifest on oral mucosa, face, genitals, and
legs.

Page 74
MTB Nephrology
MTB Q and A

1. Best initial test in Nephrology UA (dipstick is positive, 14. Tuberculosis UA Persistent WBC on UA with
microscopic analysis) negative cultures.
BUN
15. ___ is common for mild IgA nephropathy
Cr
recurrent hematuria.
2. Tamm-Horsafall Protein Normal protein secreted
16. Normal UA has ___ RBC/high < 5 RBCs
less than 30-50 mg per 24
power field.
hours
17. Hematuria is an indication of? • Stones in bladder, ureter,
3. Severe proteinuria Glomerular damage
or kidney
- very large amount of
• Hematologic disorders that
proteins
cause bleeding
4. Increased protein secretion Standing (orthostatic (coagulopathy)
(normally) from? proteinuria) and physical • Infection (cystitis,
activity pyelonephritis)
- transient proteinuria is • Cancer of bladder, ureter,
usually benign or kidney
- persistent proteinuria may • Cancer tx
need kidney bx (cyclophosphamide gives
hemorrhagic cystitis)
5. Normal protein per 24 hour is < 300 mg
• Trauma: simply "banging"
___. - 24 hour urine harder to
the kidney or bladder
collect thus rarely
makes them shed red cells
performed
• Glomerulonephritis
6. Assessing Proteinuria UA • False positive tests for
Serum Protein/Cr ratio hematuria on dipstick are
Biopsy determines cause caused by hemoglobin or
7. TRUE/FALSE: Protein/Creatinine TRUE myoglobin in urine
ratio is superior in accuracy to - faster, easier to perform 18. Woman is admitted with trauma a. Microscopic examination
24-hour urine. P/Cr ratio and dark urine. The dipstick is of urine
8. Urine dipstick for protein Albumin markedly positive for blood. - hemoglobin and myoglobin
detects only ___. What is the best initial test? make the dipstick positive
for blood, but no red cells
9. Microalbuminuria is ___ mg/24 30-300 mg/24 hours a. Microscopic examination of seen on micro exam of the
hours. - tiny amounts of protein urine urine
too small detect on UA b. Cystoscopy
- diabetics importance c. Renal ultrasound
long-term microalbuminuria d. Renal/bladder CT scan e.
leads to worsening renal Abdominal X-ray
fxn f. Intravenous pyelogram
10. Tx for Microalbuminuria in ACE-I 19. Intravenous Pyelogram (IVP) Never the right answer,
Diabetes - decrease progression of because contrast is usually
proteinuria and delay renal toxic
development of renal
insufficiency 20. When "dysmorphic" red cells Glomerulonephritis
are described, the correct
11. ___ is especially important in Kidney bx answer is ___.
kidney disease in a diabetic
patient with no ophthalmic 21. Cystoscopy is the most Bladder
findings. accurate test for the ___.

12. Can Bence-Jones proteins in NO, use 22. Hematuria without infection or Use cystoscopy
myeloma be detected on a immunoelectrophoresis prior trauma, renal US/CT
dipstick. shows no etiology, bladder US
may show a mass for possible
13. Eosinophils in urine Allergic or acute interstitial bx.
nephritis.
Detected by Wright and Dx.
Hansel stains
Very specific (not 23. Red Cell casts associated Glomerulonephritis
sensitive) with?

Page 75
24. WBC casts association with? Pyelonephritis 34. Causes of 1. Acute (allergic) interstitial nephritis
intrinsic (Penicillin)
25. Eosinophil casts associated Acute (allergic) interstitial
renal 2. Rhabdomyolysis and Hemoglobinuria
with? nephritis
disease 3. Contrast agents, aminoglycosides, cisplatin,
26. Hyaline casts associated with? Dehydration concentrating amphotericin, cyclosporine, NSAIDs most
the urines and the normal common toxin causing AKI from ATN
Tamm-Horsfall protein ppt 4. Crystals from hyperuricemia, hypercalcemia,
or concentrates into a cast or hyperoxaluria
27. Broad, waxy casts associated Chronic renal disease 5. Proteins from Bence-Jones proteins from
with? myeloma
6. Poststreptococcal Infection
28. Granular "muddy brown" casts Acute tubular necrosis,
associated with? collections of dead tubular 35. Acute
cells kidney
injury
29. AKI Categories 1. Prerenal (decreased etiologies
perfusion)
2. Postrenal (obstruction)
3. Intrinsic renal disease
(ischemia and toxins)
30. Cause of Prerenal AKI 1. Hypotension (systolic <
90) from sepsis,
anaphylaxis, bleeding,
dehydration 36. AKI • AKI usually = asx rise in BUN and creatinine;
2. Hypovolemia (diuretics, presentation when symptomatic:
burns, pancreatitis) - N/V, fatigue/malaise, weakness
3. Renal artery stenosis - SOB, edema (fluid overload)
4. Relative hypovolemia
from poor pump (CHF, • Very severe disease presents with:
tamponade) - Confusion
5. Hypoalbuminemia - Arrhythmia from hyperkalemia and acidosis
6. Cirrhosis - Sharp, pleuritic CP from pericarditis
7. NSAIDS (constricting 37. Presentation Enlargement (distention) of the bladder and
afferent arteriole) of postrenal massive diuresis after Foley (urinary) catheter
8. ACE-I (dilating efferent azotemia. placement are specific for urinary obstruction
arteriole)
38. Best initial BUN and creatinine
31. Causes of Postrenal AKI 1. Prostate hypertrophy or test for AKI.
cancer
39. Best initial Renal sonogram
2. Stone in the ureter
imaging test - contrast should be avoided in renal
3. Cervical cancer
for AKI. insufficiency
4. Urethral stricture
5. Neurogenic (atonic) 40. Dx. Prerenal azotemia
bladder BUN: Cr - Clear history of hypoperfusion or hypotension
6. Retroperitoneal fibrosis ratio > 20:1
(bleomycin, methylsergide, UNa < 20.
radiation in Hx) FeNa < 1%
Urine
32. Prerenal and postrenal Reversible causes
osmolality
azotemia combined account for
(UOsm) >
80% of acute kidney. The
500.
majority are
(reversible/irreversible).
33. MCC of intrinsic renal disease. Acute tubular necrosis
(ATN) from toxins or
prolonged ischemia of the
kidney

Page 76
41. Dx. Postrenal azotemia 47. Patient presents with fever and acute c. Contrast media
BUN: Cr ratio > 20:1 LLQ abdominal pain. Blood cultures - rapid onset of
Distended bladder or massive grow E. coli and Candida albicans. injury, Creatinine
release of urine with catheter Patient started on vancomycin, rises the next day
placement metronidazole, gentamicin, and
B/L or Uni hydronephrosis on amphotericin. CT scan reveals Gentamicin,
sonogram (US). diverticulitis. After 36 hours, her Vancomycin,
creatinine rises dramatically. amphotericin
42. Dx. ATN
Which of the following is most likely usually take 5 days
BUN: Cr < 20: 1 - body inappropriately
the cause of her renal insufficiency? and multiple doses
UNa > 20. loses sodium and water in
to result in
FeNa > 1%. the urine
a. Vancomycin nephrotoxicity.
Urine osmolality (UOsm) < 300. - urine cannot be
b. Gentamicin
concentrated since the
c. Contrast media
tubule cells are damage
d. Metronidazole
- urine osm same as blood
e. Amphotericin
= isosthenuria
48. How to prevent contrast induced Saline solution (1
43. 20-year-old African-American b. Avoid dehydration
nephropathy? to 2L) hydration
man has screening test for - sickle cell trait defect in
prior and during
sickle cell. He's found to be renal concentrating ability
angiography
heterozygous (trait or AS) for or isosthenuria, painful
- most beneficial in
sickle cell. crisis rarely occur in sickle
preventing contrast
cell trait
induced
What is the best advice for him?
nephrotoxicity
a. Nothing needed until he has a Hydroxyurea: Only if > 4
painful crisis pain crises per year. 49. A patient with mild renal insufficiency c. Urine sodium 5
c. Hydroxyurea undergoes angiography and develops 2 (very low), FeNa <
b. Avoid dehydration mg/dL rise in creatinine from ATN 1% , urine specific
d. Folic acid supplementation despite the use of saline hydration gravity 1.040 (very
Indicated if hemolysis before and after procedure. high)
e. Pneumococcal vaccination What do you expect to find on
laboratory testing? - Contrast causes
44. Tests for AKI of unclear etiology - UA (first)
spasm of afferent
- Urine Na
a. Urine sodium 8 (low), FeNa >1% , arteriole leads to
- FeNa
urine specific gravity 1.035 (high) INC reabsorption of
- Urine Osm
b. Urine sodium 58 (high), FeNa >1% , Na (and thus water)
45. Urine Sodium/Fractional Activates aldosterone urine specific gravity 1.005 (low) -> very
Excretion with Hypoperfusion which increases serum Na, c. Urine sodium 5 (very low), FeNa < concentrated urine
(Pre-Renal) thus leading to decreased 1% , urine specific gravity 1.040 (very (INC specific
Na excreted. high) gravity)
d. Urine sodium 45 (high), FeNa >1% ,
Also, more water is urine specific gravity 1.005 (low) *specific gravity
reabsorbed leading to correlates with
increased concentration. urine osmolality.
46. ATN Damaging Kidney Function Can't reabsorb Na, so more 50. Contrast induced nephropathy lab Is considered an
Na is excreted. values ATN, but has a pre-
renal profile in lab
Can't reabsorb water, so values.
more water is released,
leading to an abnormally
low Uosm (<300 mOsm/kg)

Page 77
51. A patient with b. Hyperuricemia - 2 days after chemo 57. Low magnesium risk and ATN Can cause an increased
extremely severe the Cr rises risk of aminoglycoside or
myeloma with a cisplatin toxicity
plasmacytoma is Lysis of cells with nuclear material cause
58. Causes rhabdomyolysis. Trauma, prolonged
admitted for hyperuricemia from tumor lysis
immobility, snake bites,
combination syndrome. Leads to an increase in Cr.
seizures, and crush
chemotherapy.
injuries
Two days later Tx for myeloma would decrease both
creatinine rises. calcium and Bence-Jones because they 59. The best initial test UA (dipstick AND
What is the most are produced from leukemia cells. rhabdomyolysis. microscopic analysis)
likely cause? - Blood is positive on
a. Cisplatin dipstick but NO RBCs are
b. Hyperuricemia seen on microscopic exam
c. Bence-Jones 60. Most specific lab Urine test for myoglobin
proteinuria rhabdomyolysis.
d. Hypercalcemia
e. Hyperoxaluria 61. Rhabdomyolysis Labs - Increased CPK, urine test
for myoglobin.
52. What can prevent Allopurinol - Hyperkalemia(release of
tumor lysis Hydration K+ from damaged cells)
syndrome? Rasburicase - Hyperuricemia (from
nucleic acid release
Can prevent renal failure in metabolized to uric acid)
chemotherapy patients from tumor lysis - Hypocalcemia occurs
syndrome from increased calcium
53. Toxic overdose of binding to damaged muscle
___ leads to low - Damage from muscles
calcium and renal releases phosphate
failure. 62. Rhabdomyolysis Tx - Saline hydration
- Mannitol (osmotic
diuretic)
- Bicarbonate (driving
potassium back into cells,
may present ppt of
Ethylene glycol precipitates oxalate myoglobin in kidney tubule)
within kidney tubules causing ATN
** Concept hydration and
Oxalate crystals are envelope-shaped mannitol INC urine flow rate
crystals. to DEC amount of time
contact btw myoglobin and
Calcium is low because it ppts calcium tubular cells, myoglobin
oxalate. severe oxidant stress on
tubular cells
54. Methanol toxicity Causes inflammation of the retina NOT
renal toxicity 63. A man comes to the ED after a b. EKG
triathlon followed by status - detect life-threatening
55. 3 things increase 1. Hypoperfusion of kidney epilepticus. He takes hyperkalemia and check
risk of toxic/insult 2. Underlying renal insufficiency simvastatin at triple the for arrhythmia due to
ATN. - HTN recommended dose. His rhabdomyolysis INC K+
- DM muscles are tender and urine is due to release from
3. Older age dark. IV fluids are started. damaged cells since 95%
- lose 1% of renal function every year What is the next best step in of K+ is intracellular
past age of 40 management?
56. Slow onset (5-10 Drug related injury:
days) causes of - Aminoglycosides, amphotericin, a. CPK level
ATN. cisplatin, vancomycin, acyclovir, b. EKG
cyclosporine d. Urine dipstick
- Dose dependent: c. Potassium replacement
DEC Mag -> INC risk for aminoglycoside e. Urine myoglobin
and cisplatin

Page 78
64. Ineffective at reversing ATN WRONG 69. Cholesterol
ANSWERS: emboli
- Low dose presentation,
dopamine dx tests, most
- Diuretics accurate test,
- Mannitol tx.
- Steroids

No therapy
proven to benefit
ATN: correct the
underlying cause From cholesterol plaques around the aorta,
of ATN, give broken off during catheter procedures. Can
hydration and lead to blue/purplish lesions, livedo
correct electrolyte reticularis, and ocular lesions. Lodge in
abnormalities kidney cause -> AKI

65. When Is Dialysis the Answer in ATN? - Fluid overload


Dx: Eosinophilia, low complement, elevated
- Encephalopathy
ESR, and Eosinophiluria
- Pericarditis
Most accurate: biopsy of skin lesion
- Metabolic
(cholesterol crystals)
acidosis
Tx: No specific therapy
- Hyperkalemia
70. Acute (allergic) Antibodies and eosinophils attack cell
AEIOU: Acidosis interstitial lining of the tubules as a reaction to drugs
Electrolytes nephritis (70%)
Intoxications definition and
Overload of causes. Cause: PCN, sulfa drug (furosemide,
volume Uremia thiazides), phenytoin, rifampin, quinolones,
allopurinol, PPIs
66. Patient develops ATN from gentamicin. c. Furosemide
She's vigorously hydrated and treated - causes
These medications can also cause SJS,
with high doses of diuretic, low-dose ototoxicity by
Toxic epidermal necrolysis, hemolysis
dopamine, and calcium acetate as a damaging the hair
phosphate binder. Urine output cells of the 71. Allergenic Skin
increases, but she still progresses to cochlea, resulting substances Kidney
end-stage renal failure. She also in sensorineural affect which Red cells
becomes deaf. hearing loss, organ
What caused her hearing loss? related to how fast systems?
its injected 72. Dx. Acute interstitial nephritis
a. Hydrochlorothiazide - no proven Acute renal - white and red cells in urine
b. Dopamine benefit in tx ATN failure (rising - elevated BUN/Cr < 20:1
c. Furosemide BUN + Cr)
d. Chlorthalidone Fever
e. Calcium acetate Rash
67. Dx. Hepatorenal Arthralgia
Severe liver disease (cirrhosis) syndrome Eosinophilia
New-onset renal failure with no other - renal failure 2/2 and
explanation liver disease eosinophiluria
Very low urine sodium (< 10-15) - lab values fit 73. Most accurate Most accurate test is Hansel or Wright stain
FeNa < 1% with prerenal test acute showing eosinophils present.
Elevated BUN:Cr ratio > 20:1 azotemia interstitial
68. Tx of Hepatorenal Syndrome - Midodrine nephritis.
- Octreotide 74. Tx acute AIN resolves on its own usually removal of
- Albumin interstitial drug or controlling infection, severe disease
nephritis. manage with dialysis

If Cr continues to rise after stopping the


drug, give glucocorticoids (prednisone,
hydrocortisone, methylprednisone)

Page 79
75. Analgesic Vascular insufficiency from NSAIDs 80. Goodpastures
Nephropathy constricting the afferent arteriole and
inhibiting prostaglandins.

Asx in healthy patients, in elderly have


underlying renal insufficiency from
DM/HTN then NSAIDs can tip them into 1. Lung and kidney involvement, no URI
renal insufficiency involvement (Wegeners).
2. Best initial test is antiglomerular
Can also present with papillary necrosis. basement membrane antibodies
76. Papillary Sloughing off of renal papillae. Look for 3. Most accurate test is lung or kidney biopsy
Necrosis extra NSAID use with Hx of sickle cell (linear deposits).
disease, diabetes, urinary obstruction, or 4. Anemia from chronic hemoptysis
pyelonephritis.
Tx with plasmapheresis and steroids,
Most accurate test: CT scan (bumpy white cyclophosphamide can be helpful
contour of interior) 81. IgA MCC of acute glomerulonephritis
Nephropathy Look for an Asian patient with recurrent
Best initial test: UA (urine culture (Berger episodes gross hematuria 1-2 days after
negative) may show necrotic kidney Disease) URI (synpharyngitic).
tissues (renal papillae)
PSGN follows pharyngitis by 2w
No treatment
77. Differences IgA levels increased in 50%, bx most
between accurate
Pyelonephritis
and Papillary Severe proteinuria treated with ACE-I and
necrosis steroids
- more proteinuria = worse prognosis
82. Most likely PSGN
dx: - follows throat infection of skin infection
Dark-cola (impetigo) 1-3 weeks after Strep.
78. Tubular diseases colored urine
vs Glomerular Oliguria Dx:
diseases HTN 1. UA shows glomerulonephritis
Periorbital 2. Antistreptolysin O (ASO) titers and anti-
edema DNAse antibody titers
Low
complement Most accurate is biopsy (not routinely done)
levels.
79. General 1. UA with Hematuria Tx: Antibiotics and diuretics
Glomerular 2. Dysmorphic red cells 83. Alport Congenital Type IV collagen defect leading
Disease Facts 3. Red cell casts Syndrome to:
4. Urine sodium and FeNa low - Sensorineural hearing loss
5. Proteinuria - Visual disturbance from loss of the
7. Bx most accurate for diagnosis collagen fibers that hold the lens of the eye
8. Chronic in place
9. Steroids often given
10. Additional immunosuppressive meds No specific tx.
(cyclophosphamide, mycophenolate) are
frequently used

Page 80
84. Polyarteritis Systemic vasculitis that spares the lungs. 91. Most common DM
Nodosa Associated with Hep B. causes of nephrotic HTN
(PAN) syndrome.
Multiple system manifestations:
92. Causes of 1. Cancer (solid organ) - membranous
- GI - mesenteric vasculitis pain worse with
Nephrotic 2. Children - minimal change disease
eating
Syndrome 3. Injection drug use and AIDS - focal
- Neurologic - mononeuritis multiplex
segmental
STROKE IN A YOUNG PERSON look into
4. NSAIDs - minimal change disease and
vasculitis
membranous
- Skin: digital gangrene and livedo reticularis
5. SLE - any of them
- Cardiac disease 1/3 of patients
93. Nephrotic CHF edema -> edema of dependent
85. Initial test Best initial: Angiography shows aneurysmal
syndrome edema areas (legs)
for PAN dilation of the renal, mesenteric, or hepatic
vs CHF edema.
artery in association with new onset HTN
Nephrotic syndrome -> edema
everywhere
Most accurate is biopsy (skin, nerve, muscle).
94. Infections in Increased infections because of urinary
Tx: Prednisone and cyclophosphamide they nephrotic loss of Ig (immunoglobulins) and
lower mortality syndrome complement.

86. Lupus Biopsy needed for staging


Clots are more common as well
Nephritis Dx
because of loss of antithrombin, protein
and Tx Mild: glucocorticoids
C and S.

Severe (proliferative membranous 95. Dx Tests for Best initial is UA


nephropathy): glucocorticoids combined with nephrotic
either cyclophosphamide or mycophenolate Syndrome Urine albumin/creatinine spot urine
ratio gives a good idea about loss.
87. Large Amyloid
kidneys on HIV nephropathy
Renal biopsy the most accurate
sonogram PCKD
and CT scan Diabetes
UA can show maltese crosses, lipid
88. Amyloidosis deposits in sloughed off tubular cells.
96. Nephrotic Hyperproteinuria (> than 3.5 grams per
syndrome 24 hours)
definition
Hypoproteinemia

Abnormal protein produced in association with: Hyperlipidemia - rise because


Myeloma lipoprotein signals that turn off
Chronic inflammatory disease production are lost in the urine
RA
IBD Edema
Chronic infections
Iron + Zinc are low because carrier
Biopsy most accurate test, will see green proteins lost in urine
birefringence with Congo red staining
97. Nephrotic 1. Initial - glucocorticoids,
89. Tx of Melphalan and Prednisone Syndrome Tx cyclophosphamide for no response
Amyloidosis 2. ACE-I/ARBs for proteinuria
90. Nephrotic Nephrotic Syndrome occurs when proteinuria is 3. Edema managed with salt-
Syndrome so massive that the liver can no longer restrictive diets and diuretics
and the increase production of albumin to compensate 4. Manage hyperlipidemia with statin
Liver for urinary losses.

This proteinuria leads to:


- Edema
- Hyperlipidemia
- Thrombosis (you lose C and S first, and
antithrombin)

Page 81
98. Causes of ESRD. - Diabetes + HTN (most 103. Tx of ESRD 1. Anemia - EPO
common causes) Manifestations 2. Hypocalcemia + osteomalacia -
- Rapidly progressive vitamin D + Calcium
glomerulonephritis (can cause 3. Bleeding - DDAVP to improve platelet
ESRD over weeks) function use only when bleeding
- Any form of tubular or 4. Pruritis - Dialysis and UV light
glomerular damage can cause 5. Hyperphosphatemia - oral binders
ESRD 6. Hypermagnesemia - restrict high
magnesium foods, laxatives, and
ESRD = chronic renal failure antacids
need dialysis or renal 7. Atherosclerosis - Dialysis
transplant 8. Endocrinopathy - diaphysis, estrogen
and testosterone replacement
99. TRUE/FALSE: Peritoneal TRUE
dialysis and hemodialysis 104. Tx of Oral phosphate binders
are equally effective at hyperphosphatemia. - prevent phosphate absorption from the
removing wastes from the bowel
body.
Use:
100. Uremia Definition - Metabolic acidosis
- Calcium acetate, calcium carbonate
- Fluid overload
- Sevelamer and Lanthanum when
- Encephalopathy
hypercalcemia
- Hyperkalemia
- Pericarditis
NEVER use aluminum containing
phosphate binders (aluminum causes
Indication for dialysis
dementia)
101. Manifestations of Renal 1. Anemia: loss of EPO,
105. HLA identical,
Failure (ESRD) normochromic normocytic
related donor
2. Hypocalcemia - no
kidneys, last ___ on
conversion to 1,25-dihydroxy
average.
by the kidney
3. Osteodystrophy -
secondary hyperPTH
demineralization of Ca (weak,
soft bones)
4. Bleeding - platelets don't
24 years
work in a uremic environment
5. Infection - neutrophils don't 106. HUS association. Children
degranulate (like platelets) E.coli 0157:H7 and Shigella
6. Pruritis - urea causes skin - usually resolves spontaneously (mild)
itching - severe HUS = urgent plasmapheresis or
7. Hyperphosphatemia - FFP
Secondary hyperPTH and 107. TTP vs. HUS
inability to excrete
8. Hypermagnesemia - loss
of excretory ability
9. Accelerated
atherosclerosis and HTN -
WBC don't work well in a
uremic environment and don't
BOTH associated with:
remove plaques well
- Intravascular hemolysis (schistocytes,
10. Endocronipathy - ED, low
helmet cells, and fragmented red cells)
T in men, anovulation in
- Renal insufficiency
women
- Thrombocytopenia
102. Most common cause of Cardiac disease
death in those on dialysis TTP has neurological symptoms and
with CKD. fever.

PT/aPTT normal

Page 82
108. TTP Associations HIV, cancer, cyclosporine, ticlopidine, 117. Response to ADH
and Tx and clopidogrel in central vs
nephrogenic DI.
tx with plasmapharesis or infusion of
fresh frozen plasma (FFP)

Steroids DO NOT HELP


109. Comparing benign
(simple) cysts
and potentially
malignant renal CDI: sharp DEC in urine volume, INC
cysts osmolality

NDI: No change in urine volume or


osmolality with ADH administration
118. Tx diabetes 1. Fluid loss: Correct the underlying
110. Dx. PCKD (polycystic kidney disease) insipidus cause of fluid loss
Pain 2. CDI: Replace ADH (vasopressin also
Hematuria known as DDAVP)
Stones 3. NDI:
Infection - correct K+ and Ca2+
HTN - Stop lithium and demeclocycline
111. Most common Renal failure from recurrent episodes of - Give HCTZ or NSAIDs for those still
cause of death pyelonephritis and nephrolithiasis have NDI despite these interventions
PCKD (polycystic 119. Correcting Cerebral edema
kidney disease). hypernatremia - worsening confusion and seizures
112. Polycystic kidney - Liver cysts (most common site outside too quickly what
disease the kidney) is the
extrarenal - Ovarian cysts consequence?
symptoms - MVP 120. Hypervolemia
- Diverticulosis Hyponatremia
- Cerebral aneurysm (often don't rupture) Causes
113. Causes of - Lithium
Nephrogenic DI - Demeclocycline
- CKD
- Hypokalemia
- Hypercalcemia
114. High-volume diabetes insipidus (DI)
nocturia is the
- CHF
first clue to the
- Nephrotic Syndrome
presence of ___.
- Cirrhosis
115. Sx DI and - Confusion
hypernatremia. - Disorientation Instrinsic volume depletion leads to
- Lethargy increased ADH, makes hypoNa since
- Seizures perfusion is more important than Na.

• If uncorrected, severe hypernatremia


causes coma and irreversible brain
damage
116. Diabetes Water restriction test
insipidus best
initial test. DI urine volume will stay high.
Urine osmolality (urine sodium) stays low

Page 83
121. Hypovolemic - Sweating 132. If hyponatremia corrected
Hyponatremia - Burns too quickly
Causes - Fever
- Pneumonia (insensible losses
from hyperventilation)
- Diarrhea
- Diuretics

All are causes of hypernatremia from


decreased water solution, however
replacement with free water turns it
into hyponatremia.

Addison's causes hyponatremia as


well through loss of aldosterone
Central pontine
122. Euvolemic - Pseudohyponatremia myelinolysis/osmotic
Hyponatremia (hyperglycemia) demyelination
Causes - Psychogenic Polydipsia
- Hypothyroidsm - thyroid needed to - Goal: increase in Na is 0.5 to 1
excrete water mEq/hour (12 to 24 mEq/day)
- SIADH (lung/brain disease)
133. Hyperkalemia - Hemolysis
123. For every 100 mg/dL 1.6 mEq/L decrease in Na+ (pseudohyperkalemia) - Fist clenching
of glucose able - Thrombocytosis or
normal there is ___ leukocytosis will leak out of
mEq/L sodium cells in the lab specimen
124. Look for a history of bipolar disorder 134. Name some causes of - Renal failure
___ to suggest Hyperkalemia (decreased - Aldosterone decrease
psychogenic excretion)? - ACEi (ARBs)
polydipsia. - Type IV RTA (hyporeninemic,
125. Causes of SIADH SSRIs hypoaldosteronism)
Sulfonylureas - Spironolactone and eplerenone
Vincristine - Triamterene amiloride
Cyclophosphamide (potassium sparing diuretics)
TCA - Addison's Disease
Small cell lung cancer 135. Hyperkalemia from - Tissue destruction (hemolysis,
Pain tissues rhabdo, tumor lysis)
126. TRUE/FALSE: Sxs of TRUE, very fast the patient may - Decreased insulin: insulin
hyponatremia depend seize drives K+ into cells
on how fast it - Slow drops may be asymptomatic - Acidosis - cells pick up H+
occurs. even if the levels are very low ions and release K+
- Beta blockers and digoxin
127. Most accurate test ADH level is high (inhibit Na/ATPase that brings
SIADH - high urine osmolality (high sodium > potassium into cell)
40 mEq/L) - Heparin increases potassium
- uric acid and BUN low levels
128. Tx of Hyponatremia Restrict fluids 136. Hyperkalemia Sx - Weakness
(mild) - Paralysis when severe
129. Tx for Hyponatremia Saline and loop diuretic - Ileus (paralyzes gut muscles)
(moderate) - Cardiac rhythm disordesr

130. Tx of Hyponatremia Hypertonic saline, conivaptan, 137. Most urgent test in EKG
(severe) tolvaptan (ADH antagonists urgent severe hyperkalemia.
SIADH tx severe, symptomatic)
131. Tx for Chronic SIADH Demeclocycline (blocks action of
ADH at the collecting duct of the
kidney tubule)

Page 84
138. EKG in severe 142. Hypokalemia Weakness
hyperkalemia patient Paralysis
presentation. Loss of reflexes

Muscular contraction (so severe as to


cause rhabdomyolysis) and cardiac
conduction
143. Hypokalemia EKG
Findings

- Peaked T Waves
- Widened QRS U waves
- Prolonged PR
139. Tx of life- Ectopic PVCs, flattened T waves, and ST
threatening depression
hyperkalemia 144. Hypokalemia K+ oral replacement no maximum rate
(abnormal EKG) treatment
K+ IV must be slow as to not cause
fatal arrhythmia
145. Replace ___ if Magnesium
vigorous oral and - Hypomagnesemia can lead to INC K+
IV replacement of loss in the urine
K+ fails to raise - If Mg is replaced close up magnesium-
1. Calcium gluconate or calcium chloride
the K+ levels. dependent K+ channels and stop urinary
- used if EKG abnormal to protect the
loss
heart
2. Insulin and glucose to drive K+ into 146. Anion gap Na - (Cl + HCO3-)
cells calculation.
3. Bicarbonate to drive K+ into cells but Normal: 6-12
should only be used when acidosis 147. What causes Normal anion gap metabolic acidosis
causes hyperkalemia NAGMA? - RTA
- Diarrhea
If no EKG changes:
1. Remove K+ from the body with sodium * Rise in chloride = hyperchloremic
polystyrene sulfonate (kayexalate) binds metabolic acidosis
K+ in the gut and remove it from the body
2. Other methods: beta agonists 148. Distal RTA Type I DEC hydrogen secretion distally,
(albuterol), loop diuretics, dialysis bicarbonate cannot be generated

140. Hypokalemia Shift into cells: Amphotericin, SLE, Sjogrens damage


Causes due to - Alkalosis (H+ ions come out of the cell tubule.
shift into cells in exchange for K+ entering) Ua = pH > 5.5
via? - Increased insulin
- Beta adrenergic stimulation (accelerates Alkaline urine = INC formation kidney
Na/K+ ATPase) stones from calcium oxalate -> calcified
141. Hypokalemia 2/2 1. Loop diuretics renal parenchyma (nephrocalcinosis)
to Renal Losses 2. INC in aldosterone
- Primary hyperaldosternoism (Conn's) Tx: Bicarbonate
- Volume depletion raising aldosterone 149. Most accurate Infuse acid into the blood with ammonium
- Cushing syndrome test Distal RTA chloride
- Bartter syndrome (Genetic disease type I - cannot excrete acid and urine pH
causing salt loss at loop of Henle) remains > 5.5 in RTA type I
- Licorice
3. Hypomagnesemia (magnesium
dependent K channels increased urinary
loss of K+)
4. RTA I and II

Page 85
150. Proximal RTA Damage to proximal tubule from amyloidosis,156. Causes of AGMA. Lactate
Type II myeloma, Fanconi syndrome, heavy metals
decreases ability to reabsorb filtered bicarb Hypotension of hypoperfusion. Blood lactate level

Body becomes so depleted of bicarb that distal Tx: Correct hypoperfusion


absorbs the rest and urine pH < 5.5.
157. Causes of AGMA. Ketoacids

Chronic metabolic acidosis leads to calcium leaching


DKA, starvation. Acetone level
out and bones become soft (osteomalacia)
Tx: Insulin and fluids
Tx: Volume depletion causes enhanced bicarbonate
reabsorption can be done via Thiazides. 158. Causes of AGMA. Oxalic acid

151. Most accurate Evaluate bicarbonate malabsorption in the kidney by


Ethylene glycol overdose. Crystals on UA
test proximal giving bicarbonate and testing the urine pH
RTA Type II. - urine pH will rise in RTA type II
Tx: Fomepizole, dialysis
152. RTA Type IV More often in diabetes. Hyporeninemia,
159. Causes of AGMA. Formic acid
Hypoaldosteronism (Type IV RTA)
Methanol overdose. Inflamed retina
DEC amount of aldosterone at kidney tubule. Loss of
sodium and retention of potassium.
Tx: Fomepizole, dialysis

Persistently high urine sodium despite sodium160. Causes of AGMA. Uremia


depleted diet, high blood K+ level. UA pH < 5.5
Renal failure. BUN, creatinine
Tx: Fludrocostisone
Tx: Dialysis
153. ___ is the steroid Fludrocostisone
with the highest 161. Causes of AGMA. Salicylates
mineralocorticoid
or Aspirin overdose. Aspirin level
"aldosteronelike"
effect. Tx: Alkalinize urine

154. Types of RTA 162. Arterial blood gas in metabolic - pH < 7.4
acidosis. - DEC pCO2 indicating
respiratory alkalosis as
compensation
- DEC HCO3-
163. Metabolic Alkalosis Causes - GI: Vomiting/NG suction
- Increased aldosterone:
primary
hyperaldosteronism
155. Distinguishing - Diuretics
RTA and Diarrhea - Milk-alkali syndrome: high
volume liquid antacids
- Hypokalemia: H+ ions
move into cells so K+ can
be released
164. ABG in metabolic alkalosis. - pH > 7.4
- INC pCO2 indicating
respiratory acidosis as
Urine anion gap (UAG)
compensation
- INC HCO3-
UAG = Na - Cl
165. TRUE/FALSE: Minute ventilation TRUE
is more precise than
Acid is excreted with chloride, so decreased acid
(RTA) leads to a positive number. (UAG + in RTA)respiratory rate.

Diarrhea has metabolic acidosis and thus increases


amount of Chloride. Leads to negative number. (UAG
- in diarrhea)

Page 86
166. Respiratory Alkalosis - Anemia 176. Best initial therapy Thiazide diuretics
Causes - Anxiety for HTN
- Pain
177. UTIs give ___ Struvite stones
- Fever
stones. (magnesium/ammonium/phosphate)
- Interstitial Lung Disease
- Pulmonary Emboli
Remove them surgically

DEC pCO2 178. How is the etiology 1. Stone analysis


INC minute ventilation of a stone 2. Serum calcium, sodium, uric acid,
Metabolic acidosis with determined? PTH, magnesium, and phosphate levels
compensation 3. 24 hour urine for volume, calcium,
oxalate, citrate, cystine, pH, uric acid,
167. Respiratory Acidosis - COPD/emphysema
phosphate, and magnesium
Causes - Drowning
- Opiate overdose
* Fat malabsorption INC stone formation
- Alpha 1-antitrypsin deficiency
- Kyphoscholiosis 179. Stone management
- Sleep apnea/morbid obesity between 0.5 to 2-3
cm.
INC pCO2
DEC minute ventilation
Metabolic alkalosis as
compensation
168. The most common cause Calcium oxalate
of kidney stones - forms more frequently in
(nephrolithiasis ) is ___. alkaline urine Lithotripsy - stones half way up the
ureters
Risk factor: overexertion of
calcium in the urine Stent - hydronephrosis from stones in
the distal ureter
169. 46-year-old man comes to a. Ketorolac
the ED with excruciating - NSAID provides analgesia
Stones half way down the ureter removed
left flank pain radiating to similar to opiate meds, provide
from below with a basket
groin. He has some blood relief for pain first then obtain
in urine. dx test 180. A man with a b. Hydrochlorothiazide
What is the next step? calcium oxalate - removes calcium from the urine by INC
stone is managed distal tubular reabsorption of calcium
a. Ketorolac with lithotripsy.
b. X-ray Stone is destroyed
c. Sonography and passes. Urinary
d. Urinalysis calcium level is
e. Serum calcium level increased.
Besides increasing
170. Crohns disease causes Oxalate
hydration, which is
kidney stones because of
most likely to
INC ___ absorption.
benefit this patient?
171. What is the most accurate CT scan
test for nephrolithiasis? a. Calcium
172. Best initial tx for acute 1. Analgesics and hydration restriction
renal colic is with? 2. CT and sonography to detect b.
obstruction such as Hydrochlorothiazide
nephrolithiasis (uric acid stones c. Furosemide
are not detectable on x-ray but d. Stent placement
can be seen on CT) e. Increased dietary
oxalate
173. Stones < 5 mm Pass spontaneously
management. 181. How does metabolic Metabolic acidosis removes Ca from
acidosis lead to bones and INC stone formation
174. Stones 5-7 mm Nifedipine and tamsulosin to stone formation? - DEC citrate levels (citrate binds calcium
management. help them pass making it unavailable for stone formation)
175. Management of cystine Managed with surgical removal,
stones. alkalizing the urine

Page 87
182. Stress Older woman with painless urinary 189. Best initial tx for HTN. Lifestyle modifications tried 3-6
incontinence sxs, leakage with cough, laughing, or lifting mo. before meds are started
test, tx? heavy objects
- Weight loss (most effective)
Have patient stand and cough; observe - Sodium restriction
for leaking - Dietary modification (less fat and
red meat, more fish and vegetables)
Tx: - Exercise
1. Kegel exercises - Tobacco cessation does not stop
2. Local estrogen cream HTN, but becomes especially
3. Surgical tightening of urethra important to prevent CV disease
183. Urge incontinence Sudden pain in the bladder followed 190. JNC 8 initial mgt of Thiazides, CCB, or ACEi/ARB
sxs, test, tx? immediately by the overwhelming urge to HTN.
urinate
191. If BP is very high on 2 medications should be used at the
Pressure measurement in half-full
initial presentation onset
bladder, manometry
(above 160/100) tx? - stage 2 HTN

Tx: 192. Pregnancy safe HTN BB


1. Bladder training exercises drugs CCB
2. Local anticholinergic therapy Hydralazine
- Oxybutynin Alpha methyldopa
- Tolterodine 193. Meds for refractory • Central-acting alpha agonists
- Solifenacin HTN. (alpha methyldopa, clonidine)
- Dariferancin • Peripheral-acting alpha antagonists
3. Surgical tightening of urethra (prazosin, terazosin, doxazosin)
184. HTN definition. Systolic BP > 140 mm Hg • Direct-acting vasodilators
Diastolic BP > 90 mm Hg (hydralazine, minoxidil)
194. HTN and CAD tx. Use BB, ACE, ARB
BP measurements must be repeated in a
calm state over time 195. HTN and DM tx. ACE, ARB (goal < 140/90)

185. JNC 8 says for 140/90 196. HTN and BPH tx. Alpha blockers
diabetes the BP 197. HTN, depression, BBs
goal is? asthma avoid.
186. JNC 8 says BP 150/90 198. HTN and BB first.
goal ___ over age hyperthyroidism tx.
of 60.
199. HTN and osteoporosis Thiazides
187. Secondary causes - Renal artery stenosis tx.
of HTN. - Glomerulonephritis
200. HTN crisis defined as? High BP and end organ damage
- Coarctation of aorta: upper extremity
- Confusion
> lower extremity BP
- Blurry vision
- Acromegaly
- CP
- Pheochromocytoma: episodic HTN with
- Dyspnea
flushing
- Hyperaldosteronism: weakness from 201. Best initial therapy for Labetalol or nitroprusside
hyporkalemia hypertensive crisis
- Cushing syndrome or any cause of also acceptable: enalapril, CCBs:
hypercortisolism diltiazem, verapamil, esmolol,
including therapeutic use of hydralazine
glucocorticoids
- Congenital adrenal hyperplasia Any IV meds are acceptable
188. Other test to EKG
perform in HTN UA
patient. Glucose measurement to exclude
concomitant diabetes
Cholesterol screening

Page 88
MTB Neurology
MTB Q and A

1. MCA Stroke Presentation: CL weakness or sensory loss 16. Migraine vs. Cluster HA treatment: Both respond to triptans
and ergotamine. However, Cluster responds to oxygen, lithium,
Homonymous hemianopsia - eyes look towards the lesion and prednisone can reduce cluster headaches.
17. Cluster HA Prophylaxis: Verapamil
Aphasia - if occurs on the same side of as speech center 18. Atrophy of the caudate nucleus: Huntington's neuroimgaging
2. ACA Stroke Presentation: Personality/cognitive defects 19. Enlargement of lateral ventricles: Schizophrenia
(confusion)
20. Decreased volume of hippocampus: PTSD

Urinary incontinence 21. Atrophy of lenticular nucleus: Wilson's disease


22. Ppx for Migraine Therapy: Best preventative therapy is
Leg more > than arm weakness Propranolol
3. PCA Stroke Presentation: IL sensory loss of the face, 9th, and
10th nerves Others: CCB, TCAs, SSRIs, botulinum
23. Trigeminal Neuralgia Tx: Oxcarbazepine or carbamazepine
CL sensory loss of limbs
Baclofen and lamotrigine also help.
Limb ataxia
4. Best initial test in a stroke: CT scan w/o contrast If medications don't work: gamma knife sx or surgical
decompression.
5. Most accurate test in stroke: MRI
24. Postherpetic neuralgia Tx: Tx with antiherpetic medications
6. Tx for nonhemorragic stroke: Less than 3 hours: thrombolytics
(acyclovir, famciclovir, or valganciclovir)

More than 3 hours since onset: ASA


Pain is treated with TCAs, gabapentin, phenytoin. Topical
capsaicin helpful.
Hemorrhagic stroke: nothing
25. Prevention of Herpes Zoster: Zoster vaccine indicated in all
7. If patient already on aspirin at the time of stroke: Add
patients above 60
dipyridamole or switch to clopidogrel
26. Epilepsy: Seizures of unclear etiology
8. Endarterectomy Indications: Stenosis above 70%. No value for
milder stenosis. Carotid angioplasty will always be wrong. 27. Best initial therapy for a persistent seizure.: Benzodiazepine
(Lorazepam, diazepam)
9. Most likely dx:
Visual disturbances
If persists:
Photophobia
Fosphenytoin or phenytoin -> phenobarbital -> succinylcholine,
Aura
veruconium (intubate the pt) -> midazolam or propofol
Relationship to menses
Association with food or emotions: Migraine 28. Phenytoin vs. fosphenytoin: Fosphenytoin has fewer SE, no
hypotension or AV block
10. Most likely dx HA:
Frequent 29. Tx of Absence Seizures: Ethosuximide, if out consider valproic
Short duration acid
High intensity 30. When to discontinue seizure meds?: Wait until the pt is seizure
M > F: Cluster headaches free for 2 years, sleep deprivation EEG best way to tell if there
11. Most likely dx: is possibility for recurrence
Visual disturbance 31. Most likely dx:
Muscle pain Stiff neck
Fatigue Photophobia
Jaw claudication: Giant cell (temporal) arteritis Fever
12. Most likely dx: SEVERE HEADACHE
Obesity Loc in 50%: Subarachnoid Hemorrhage
Venous sinus thrombosis 32. SAH v. Meningitis: SubA will be more sudden and you have
OCPs potential for LoC
Vitamin A toxicity 33. Best initial test for SAH: CT w/o contrast
N/V/visual disturabnce: Pseudotumor cerebri 34. Most accurate test SAH: Lumbar puncture showing blood
13. Physical findings of a red, tearing eye with rhinorrhea; 35. Xanthochromia: Yellow discoloration of CSF from breakdown of
Horner syndrome occasionally.: Cluster headache RBC seen in SAH.
14. Psuedotumor cerebri tx: Weight loss, acetazolamide to 36. Ratio of WBC/RBC in SAH: Normal - one WBC/500-1000 RBC
decrease CSF production. 37. Nimodipine function in hemorrhage: Prevents subsequent
15. Cluster headache tx: triptans, ergotamine, or 100% oxygen as ischemic stroke (CCB)
abortive therapy 38. Embolization vs surgical clipping: Embolization (coiling)
superior in terms of survivability
Page 89
39. Most likely dx: 53. Neurofibromatosis (von Recklinghausen Disease):
Neurofibromas: flesh colored lesions attached to peripheral
Loss of all function except posterior column (position and nerves
vibration sense intact)
8th CN tumors - may need sx decompression preserve hearing
Flaccid paralysis below infarction level
Cutaneous hyperpigmented lesions (cafe au lait spots)
Loss of DTR at level of infarction
Meningioma and gliomas
Loss of pain and temperature: Anterior Spinal Artery Infarction 54. Sturge-Weber syndrome presentation.: Port-wine stain of the
40. Subacute Degeneration of the Cord Causes: From B12 face
deficiency or neurosyphilis Seizures
CNS: homonymous hemianopsia, hemiparesis, mental
Position and vibratory sensation are lost subnormality
41. Tx of Spinal Trauma: Glucocorticoids Skull XR: Calcification of angiomas.
42. Brown-Sequard Syndrome: Unilateral hemisection of spinal
cord. No tx beyond controlling seizures.
Loss of Pain and Temp on CL side from injury 55. Tremor at rest and exertion improved with alcohol: Essential
Loss motor function as well as position and vibratory sense on Tremor -
IL side
Caffeine makes it worse
Tx: Decompress for surgical mass. 56. Best therapy for essential tremor: Propranolol
43. Most likely dx: 57. Parkinsonism: Loss of cells in substantia nigra resulting in
Bilateral loss of pain and temperature across upper back and decrease in dopamine
both arms. 58. Most likely dx:
"Capelike distribution" 50-60
Can cause loss of reflexes and muscle atrophy: Syringomyelia Tremor, muscular rigidity, bradykinesia
- fluid filled, dilated central canal in the spinal cord Shuffling gait
44. Most accurate test for syringomyelia: MRI is the most Cogwheel rigidity
accurate test Hypomimia (limited facial expression)
45. Best treatment for syringomyelia: Surgical removal of tumor if Micrographia
present and drainage of fluid from cavity Postural instability
46. Most likely dx: Orthostatic hypotension: Parkinsonism
N/V/HA 59. Mild Parkinsonism Tx: 1. Anticholinergics (benztropine and
Fever trihexyphenidyl) - SE: dry mouth, worsening BPH, and
Seizures constipation
Focal neurological findings: Brain abscess 2. Amantadine (older patients intolerant of anticholinergics)
No way to diagnose BA from CA without biopsy 60. Anticholinergic adverse effects: Dry mouth, worsening prostate
47. Best initial test for brain abscess: CT or MRI hypertrophy, constipation
48. Most accurate test for brain abscess: Brain biopsy 61. Amantadine MOA: Release dopamine from substantia nigra
49. Tx for Brain Abscess: PCN + Metronidazole + Ceftriaxone for 62. Best initial tx for severe parkinson (inability to care for
empiric therapy. If pt had recent Nsx can use Vanco instead themselves, orthostatic): Dopamine agonists - pramipexole and
PCN due to INC risk staphylococci. ropinirole best initial tx
50. Tuberous Sclerosis neurologic abnormalities: Seizures 63. Most effective medication severe Parkinsonism.:
Levodopa/carbidopa: associated with "on/off" phenomenon
Progressive psychomotor retardation which results in episodes of insufficient dopamine (off)
characterized by bradykinesia and (on) too much resulting
Slowly progressive mental deterioration dyskinesia
51. Tuberous Sclerosis Skin Manifestatsions: Adenoma sebaceum 64. COMT inhibitors: Tolcapone, entapcone - extend duration of
(reddened facial nodules) levodopa/carbidopa by blocking metabolism of dopamine (use
when on-off phenomenon)
Shagreen patches (leathery plaques on the trunk) 65. MAO inhibitors: Rasagiline, selegiline - block metabolism of
dopamine
Ash leaf (hypopigmented) patches
52. Tuberous Sclerosis general manifestations: Neurologic Only agent with possibility of retarding progression of
Skin parkinsonism
Retinal lesions 66. MAOi + tyramine containing foods: Hypertension
Cardiac rhabdomyomas 67. Lewy body dementia: Parkinsonism with dementia

Page 90
68. Shy-Drager Syndrome: Parkinsonism predominantly with 84. Reduces glutamine build up in ALS: Riluzole, may prevent
orthostasis progression of disease
69. Tx of Psychotic SE of Parkinsons Medication: Antipsychotics - 85. Spasticity tx in ALS: Baclofen
Clozapine 86. Most likely dx:
70. Spasticity tx: Baclofen, dantrolene, and central acting alpha Distal weakness and sensory loss
agonist tizanidine may work. Wasting in legs
71. Most likely Dx and Tx: Decreased DTR
"creeply crawly sensation at night in legs." Tremors
Worse with caffeine High arch (pes cavus)
Relieved by moving legs: Restless Leg Syndrome Legs look like inverted champagne bottles: Charcot-Marie-
Treat with dopamine agonists such as pramipexole Tooth Disease - no tx
72. Most likely dx: 87. Most accurate test for Charcot-Marie-Tooth Disease:
Choreaform movement disorder (dyskinesia) Electromyography
Dementia 88. Most common cause of peripheral neuropathy: Diabetes
Behavior changes (irritability, moodiness, antisocial mellitus
behavior) 89. Best initial therapy for Peripheral Neuropathy: Pregabalin or
Onset between 30-50: Huntington's Disease gabapentin, TCAs effective in some people
CAG trinucleotide repeat sequence on chromosome 4 90. Causes Bells Palsy: CNVII palsy
73. Most specific test for HD: Genetic test - CAG trinucleotide - most idiopathic
repeat sequence - Lyme, sarcoidosis, herpes zoster, tumors
74. Tx for Huntington's: Nothing reverses. 91. Presentation Bells Palsy: - Pt cannot wrinkle forehead affected
side
Tetrabenazine for dyskinesia - Difficulty closing the eye
- Paralysis of the entire side of the face is classic
Psychosis tx with haloperidol and quetiapine - Hyperacusis: sounds are extra loud since CNVII normally
75. Tx of Tourette's Disorder: 1. Fluphenazine, clonazepam, or supplies stapedius muscle, which acts as shock absorbed on
pimozide ossicles of the middle ear
2. Methylphenidate and ADHD tx are intrinsic to Tourette mgt - Taste disturbances: CNVII supplies sensation anterior 2/3
INC attention and concentration, some people make the tongue
tics/symptoms worse 92. Best initial tx for Bells Palsy: 60% resolve, otherwise use
76. Intranuclear opthalmoplegia: Inability to adduct one eye with prednisone (acyclovir sometimes added but it does not clearly
nystagmus in the other eye. Characteristic of MS help)
77. Most accurate test in MS: MRI (also initial test)
78. CSF findings in MS: Oligoclonal bands, mild elevation of protein * usually no test done if asked for a test pick electromyography
and nerve conduction studies
79. Acute exacerbation of MS tx: High-dose steroids
93. GBS associated with what infectious disease?: Campylobacter
80. Drugs to prevent relapse of MS: Glatiramer (copolymer 1)
jejuni infection
(top choice)
Beta-interferon (top choice) 94. GBS Antibodies: Attack myelin sheaths of peripheral nerves
Fingolimod (oral) 95. Ascending weakness + loss of reflexes Dx.: GBS
Natalizumab - ascending weakness to the diaphragm can lead to respiratory
Mitoxantrone muscle weakness
Azathioprine 96. Most specific test for GBS: Nerve conduction studies/EMG
Cyclophosphamide 97. GBS CSF: Increased protein with normal cell count
81. Which MS medication causes worsening neurological deficits 98. Death from GBS: Dysautonomia and respiratory failure
with chronic use?: Natalizumab, alpha-4 integrin inhibitor. - PFTs tell who might die from GBS
Occasionally associated with progressive multifocal 99. Tx of GBS: IVIG OR plasmapheresis (don't combine)
leukoencephalopathy (PML)
100. Most likely dx:
82. Most likely dx: Double vision
Weakness in 20-40s Difficulty chewing
Difficulty chewing and swallowing Dysphonia
Decreased gag reflex (frequent aspiration) Weakness of limb muscles (worse at the end of day)
UML and LML Normal pupillary response
No sensory loss Ptosis: Myasthenia Gravis- severe myasthenia affects
Sphincters spared: ALS (Amyotrophic Lateral Sclerosis) - respiratory muscles
Electromyography reveals loss of neural innervation in multiple
101. Best initial test for Myasthenia Gravis: Acetylcholine receptor
muscle groups, INC CPK levels
antibodies
83. Most common cause of death in ALS: Respiratory failure -
tracheostomy and maintenance on a ventilator, BiPAP helps Next, check anti-MUSK antibodies (muscle specific kinase)
with respiratory difficulties 2/2 muscle weakness
Page 91
102.Uses Edrophonium MG: Edrophonium: short acting inhibitor of acetylcholinesterase, temporary bump up acetylcholine levels is
associated with clear improvement in motor fxn that lasts for a few min.
103. Myasthenia Gravis associated with (chest): Thymoma => CT with contrast is the best

Get a Chest XR/CT/MRI


104. Most accurate test for Myasthenia Gravis: Electromyography showing decreased strength with repetitive stimulation
105.Best initial treatment for Myasthenia Gravis: Neostigmine or pyridostigmine
* Azathioprine, tacrolimus, cyclophosphamide, mycophenolate used in order to get the pt off steroids before serious adverse effects
occur
106. Next step if Neostigmine/pyridostigmine fails in MG: Thymectomy in patient under 60 years of age

Prednisone over 60 years of age


107. Acute Myasthenia Crisis Tx: IVIG or plasmapharesis
108.Cluster Headache PPx and Acute Attack Tx: PPx: Verapamil, lithium, and ergotamine
Acute Attack: 100% O2 and sumatriptan
109.Central retinal vein occlusion vs central retinal artery occlusion: CRVO - blood and thunder appearance on fundoscopic exam: optic
disk swelling, retinal hemorrhage, dilated veins and cotton wool spots.

CRAO - fundoscopic exam shows pallor of optic disk, cherry red fovea, and boxcar segmentation of blood in retinal veins.
110. Tx of Glaucoma and contraindicated medicine: Can be given mannitol, acetazolamide, timolol, pilocarpine.

Atropine should be avoided since it can dilate the pupil and increase pressure.
111. Myasthenia Gravis Initial Mgmt: Diagnose with electromyography, then complete a CT scan for thymoma.
112.Dx testing dementia: MRI of brain
VDRL and RPR to exclude syphilis
B12 with possible methylmalonic acid level
Thyroid function test
113. Tx Dementia: Donepazil, rivastigmine, and galantamine are equal in efficacy. All INC acetylcholine levels.

Memantine
114.Lewy Body Dementia: Associated with PD
Tx both PD and Alzheimer's dz w/ Levodopa/Cabidopa
115.Emotional and social appropriateness are lost. Memory deteriorates later. No special tx beyond acetylcholine meds. Which
dementia?: Frontotemporal dementia
116.Rapidly progressive dementia
Monoclonic jerks
Normal head MRI or CT
CSF 14-3-3 protein: Creutzfeldt-Jakob Dz
- Biopsy is most accurate
- No specific tx

Page 92
MTB Obstetrics
MTB Q and A

1. Morning sickness is caused by an INC in ___ produced by the 10. Softening to the midline of the uterus
placenta.: beta-HCG @ 6 weeks (first trimester)
- 12th to 14th week of pregnancy Sign.: Ladin sign
2. ___: fertilization to eight weeks 11. Blue discoloration of the vagina and cervix.
___: eight weeks to birth @ 6-8 weeks (first trimester)
___: birth to one year old: Embryo Sign.: Chadwick sign
Fetus 12. Small blood vessels/reddening of the palms.
Infant First trimester.
3. Number of days/weeks since the LMP (usually 2 weeks longer Sign.: Telangiectasis/palmar erythema
than DA).: Gestational age (GA) 13. The "mask of pregnancy" is a hyperpigementation of the face
4. Nagele rule?: LMP - 3 months + 7 days + 1 year = estimated day most commonly on the forehead, nose, and cheeks; it can
of delivery worsen with sun exposure.
5. Trimester breakdown.: @ 16 weeks (second trimester)
Sign.:

6. Term lengths.:

Chloasma
14. A line of hyperpigmentation that can extend from xiphoid
process to pubic symphysis.
Second trimester.
Term: Sign.:
- early term: between 37-38 weeks
- full term: 39-40 weeks
- later term: 41 weeks
7. Gravity/Parity breakdown.:

Gravity = # of times pt pregnant


Parity = what happens to the pregnancy
Linea nigra
1. Full-term births
15. Both urine and serum testing are based on beta-HCG,
2. Preterm births
doubling every ___ for the first 4 weeks.: 48 hours
3. Abortions (both spontaneous and induced)
4. Living children (if pt has a multiple gestation pregnancy, one 16. At ___ weeks the beta-HCG peaks, and levels will typically
birth results in 2 children) drop in the second trimester.: 10 weeks
8. One of the first signs of pregnancy that is seen on PE is the 17. In the third trimester the beta-HCG levels will increase
___ sign.: Goodell sign slowly to level of ___.: 20,000-30,000 IU/mL
- softening of the cervix at 4 weeks 18. Urine pregnancy tests are positive ___following the first day
9. Softening of the cervix. of LMP.: 4 weeks
@ 4 weeks (first trimester)
Sign.: Goodell sign
Page 93
19. At ___ weeks or a beta-HCG of ___ IU/mL, a gestational sac 23. Renal changes in pregnancy.:
should be seen on U/S.:

5 weeks
1000 to 1500
20. Presence of gestational sac: 1. INC size of kidney and ureters, INC risk of pyelonephritis
from compression of the ureters by the uterus
2. INC GFR 2/2 a 50% increase in plasma volume
- DEC in BUN/Cr
24. Hematology changes in pregnancy.: 1. Anemia form INC
plasma volume by 50%
2. Hypercoagulable state
- No INC in PT, PTT, or INR
- INC fibrinogen
- Virchow triad elements (Stasis, Hypercoagulability,
Endothelial damage)
25. Between ___ weeks, U/S should be done to confirm
Transvaginal US at 4-5 weeks when hCG is 1500 gestational age and check for nuchal translucency.:
21. Cardiac changes in pregnancy.:

11-14 weeks
- most accurate way of establishing gestational age at 11-14
weeks is U/S
INC CO (results in INC HR)
Slightly lower BP (lowest around 24 to 28 weeks) 26. Fetal heart motion is seen by ___ weeks.: Seen on US by 5-6
weeks
22. GI changes in pregnancy.: 1. Morning sickness: N/V caused
by INC estrogen, progesterone, and HCG made by the placenta 27. Fetal heart sounds can be heard at how many weeks?: Doppler
2. GERD: LES has DEC tone (progesterone) US at 8-10 weeks
3. Constipation: motility in large intestine DEC 28. Fetal movements ___ weeks.: 16-20 weeks
29. First Trimester Routine Tests:

1. Anemia - CBC
2. Blood type, Rh and antibody
3. GU Screening (PAP, UA, Urine Cx)
4. Immunization of Rubella and Hep B
5. Syphilis Status
6. HIV
7. Chlamydia/Gonorrhea
8. US and blood tests to evaluate for Down syndrome

Page 94
30. First Trimester Optional Tests: TB - positive test, rule out 37. Glucose load testing vs glucose tolerance testing.: Glucose
active disease with Chest XRAY load: Fasting or nonfasting ingestion of 50 g of glucose, and
+PPD/-CXR: INH and B6 for 9 months serum glucose check 1 hour later
+PPD/+CXR: Triple therapy if sputum positive. Avoid
streptomycin because of ototoxicity to fetus Glucose tolerance: fasting serum glucose, ingestion of 100 g of
glucose, serum glucose checks at 1,2, and 3 hours. Elevated
Triple 21 early testing in high risk: glucose during any 2 of these tests is gestational diabetes.
- B-hCG 38. Done at 10-13 weeks in advanced maternal age or known
- PAPP-A genetic disease in parent. Obtains fetal karyotype.:
- Fetal nuchal translucency
31. Most common cause of abnormal MS-AFP: Dating error. Check
with an obstetric ultrasound.
32. Second trimester optional tests to look for genetic and
congenital problems.: 15-20 weeks
Triplescreen:
- Maternal serum alpha-fetoprotein (MSAFP)
- beta-HCG
- Estriol

Quad: Chorionic villus sampling


Add Inhibin A in high risk patients - catheter into intrauterine cavity to aspire chorionic villi from
placenta (can be done transabdominally or transvaginally)
33. Increased MSAFP (maternal serum alpha-fetoprotein):
39. Amniocentesis done after ___ weeks.: Done at 11 to 14 weeks
for advanced maternal age or known genetic disease in parent.
Obtains fetal karyotype (advanced maternal age).
- needle transabdominally into amniotic sac and withdraw
amniotic fluid
40. When is fetal blood sampling done?: Percutaneous umbilical
blood sample
- done in pts with Rh isoimmunization and when a fetal CBC is
Dating error needed
NTD - needle transabdominally into the uterus to get blood from the
Abdominal wall defect umbilical cord
34. Second trimester tests: 1. Auscultation of fetal HR 41. Ectopic pregnancy most commonly occurs in the ___.:
2. 16-20 weeks: quickening (feeling fetal movt for the first time)
- multiparous women feel quickening earlier than primiparous
women
3. 18-20 weeks: routine US for fetal malformation (anatomic
ultrasound)
35. Third trimester routine tests: Visits are every 2-3 weeks until
36 weeks, after 36 weeks every week visits

1. Week 27: CBC (if Hgb < 11, replace iron orally w/ stool
softeners)
ampulla of the fallopian tube
2. Weeks 24-28: Glucose load (if glucose > 140 at one hour,
perform glucose tolerance test) 42. Risk factors for ectopic pregnancy.: • Previous ectopic
3. Week 36: pregnancies (strongest risk factor)
- Cervical cultures for Chlamydia and gonorrhea • Pelvic inflammatory disease (PID)
- Rectovaginal cultures for group B streptococcus (tx if • Intrauterine devices (IUD)
possible for CG and ppx abxs during labor for GBS +) 43. Presentation:
36. ___ contractions occur during the third trimester.: Braxton- - Unilateral lower abdominal or pelvic pain
Hicks - Vaginal bleeding
- if they become regular the cervix should be checked to rule - If ruptured, can be hypotensive with peritoneal irritation:
out preterm labor before 37 weeks Ectopic pregnancy
- beginning at 37 weeks the cervix should be examined at every
visit

Page 95
44. Dx tests in ectopic pregnancy.: 48. Surgical treatment of ectopic pregnancy.:

1. Salpingostomy (preserve fallopian tube by cutting a hole in


it)
1. Beta-hCG 2. Salpingectomy (removal of the whole fallopian tube)
- Confirms pregnancy
2. Vaginal U/S * Mothers who are Rh negative should receive anti-D Rh
- Locates implantation site immunoglobulins (RhoGAM)
3. Laparoscopy 49. Abortion is defined as a pregnancy that ends before ___
- Invasive test and treatment to visualize and remove the weeks or a fetus less than ___ grams.: before 20 weeks
ectopic pregnancy < 500 grams
45. Tx ectopic pregnancy ruptured vs not ruptured.:
* Almost 80% of spontaneous abortions occur prior to 12 weeks
gestation
50. Chromosomal abnormalities in the fetus account for 60-80%
of spontaneous abortions. However, maternal factors that INC
risk of abortion include:: 1. Anatomic abnormalities
2. Infections (STDs)
3. Immunological factors (antiphospholipid syndrome)
4. Endocrinological factors (uncontrolled hyperthyroidism or
diabetes)
Unstable patients (low BP, high HR) 5. Malnutrition
46. Ectopic pregnancy medical treatment algorithm.: 6. Trauma
7. Rh isoimmunization
51. Sxs abortion.: • Cramping abdominal pain
• Vaginal bleeding
• Hypotension
• Tachycardia
52. Dx tests abortion.: • CBC (blood loss evaluation, need for
transfusion)
• Blood type and Rh screen (should blood need to be
transfused, evaluation of need for anti-D Rh immunoglobulin
• Pelvic ultrasound
1. Give dose methotrexate (folate receptor antagonist) follow 53. US shows no products of conception.: Complete abortion
beta-HCG to see if there is 15% decrease in 4-7 days. - f/u: in office
2. If no decrease give second dose
54. US shows some products of conception.: Incomplete abortion
3. No change -> surgery
- D&C/medical
47. Exclusion criteria for Methotrexate in tx ectopic pregnancy.:
55. US shows products of conception intact, but intrauterine
1. Immunodeficiency: avoid methotrexate
bleeding present and dilation of cervix.: Inevitable abortion
2. Noncompliant pt
- D&C/medical
3. Liver disease: Hepatotoxicity (baseline liver disease INC risk
of toxicity) 56. US shows products of conception intact, intrauterine
4. Ectopic is 3.5 cm or larger bleeding, no dilation of cervix.: Threatened abortion
5. Fetal heartbeat auscultated - bed rest, pelvic rest
57. US shows death of fetus, but all products of conception
present in uterus.: Missed abortion
- D&C/medical
58. US shows infection of the uterus and the surrounding areas.:
Septic abortion
- D&C and IV abx such as levofloxacin and metronidazole
59. Medical tx of abortions.: Giving medications that induce labor
(misoprostol PGE1 analog)
- help open the cervix and expulse the fetus
Page 96
60. TRUE/FALSE: Fertility drugs increase multiple gestations.: 66. Presentation:
TRUE - Contractions (abd pain, LBP, or pelvic pain)
61. • Exponential growth of uterus - Dilation of the cervix
• Rapid weight gain by mother - Occurs between 20 and 37 weeks:
• Elevated beta-HCG and Maternal Serum Alpha-Fetoprotein
(MSAFP) (levels higher than expected for estimated
gestational age): Multiple gestations
- U/S done to visualize the fetuses
62. Monozygotic vs. Dizygotic twins.:

Preterm labor
67. Evaluation of preterm labor presentation.: Evaluate fetus for
weights, gestational age, and presenting parts (cephalic versus
breech)
63. Complications of multiple gestations.: 1. Spontaneous
abortion of one fetus 68. Circumstances in which preterm labor should NOT be stopped
2. Premature labor and delivery with tocolytics and delivery should occur?:
3. Placenta previa
4. Anemia
64. 28-year-old woman 28th week of pregnancy with severe lower
back pain. The pain is cyclical and is increasing in intensity.
On physical examination she seems to be in pain. T 98.9 F,
HR 104 bpm, BP 135/80 mmHg, RR 15/min. Cervix is 3 cm
dilated.
Which of the following is the most likely diagnosis?
a. Premature rupture of membranes Membranes
b. Preterm labor • Maternal severe HTN, cardiac disease
c. Cervical incompetence • Maternal cervical dilation > 4 cm
d. Preterm contractions: b. Preterm labor • Maternal hemorrhage (abruptio placenta, DIC)
- combo of contractions and cervical dilation • Fetal death
65. Risk factors for preterm labor.: • Chorioamnionitis
• Preterm rupture of membranes (34 weeks), fetal distress
• Intrauterine growth restriction with reverse diastolic flow
69. What should be given to the mother to stop delivery in
preterm labor?:

• Premature rupture of membranes


• Multiple gestation
• Previous history of preterm labor
• Placental abruption
• Maternal factors 1. Corticosteroids (Betamethasone) - mature fetus lungs (INC
- Uterine anatomical abnormalities (bicornate uterus) surfactant), effect w/in 24 hours last 7 days, DEC risk of RDS +
- Infections (chorioamnionitis) neonatal mortality
- Preeclampsia 2. Tocolytics (Magnesium sulfate, CCB, Terbutaline)
- Intraabdominal surgery - follow corticosteroid adm allow time for steroids to work, slow
progression of cervical dilation by DEC uterine contractions

Page 97
70. SE Magnesium sulfate: Flushing 80. What exam is contraindicated in third-trimester vaginal
HA bleeding?: Digital vaginal exam
Diplopia - may lead to INC separation between placenta and uterus,
Fatigue resulting in severe hemorrhage, same with transvaginal US
- use transabdominal ultrasound first
*Magnesium toxicity may lead to respiratory depression and 81. Presentation
cardiac arrest, check DTR often • PAINLESS vaginal bleeding
• Usually presents > 28 weeks: Placenta previa
*Most commonly used tocolytic • Transabdominal ultrasound
71. SE CCB: HA -Placenta location
Flushing 82. Types of placenta previa:
Dizziness
72. SE Terbutaline: Beta-adrenergic receptor agonist
- Maternal effects include INC HR leading to palpitations and
hypotension
73. ___ presents with a history of gush of fluid from the vagina.:
Premature rupture of membranes
1. Complete - complete covering of the internal cervical os
- The rupture of chorioamniotic membrane before onset of labor
2. Partial - partial covering
74. Dx premature rupture of membranes:
3. Maginal - placenta is adjacent to the internal os (often
touching the edge of the os)
4. Vasa previa - fetal vessel is present over the cervical os
5. Low-lying placenta - placenta that is implanted in the lower
segments of the uterus but NOT covering the internal cervical
os (more than 0 cm but less than 2 cm away)
83. Tx placenta previa: Done when large-volume bleeding or a drop
in Hct
1. Strict pelvic rest (no intercourse)
2. Immediate c-section delivery
- Unstoppable labor (cervix dilated more than 4 cm)
Sterile speculum examination - Severe hemorrhage
- Fluid pools in posterior fornix - Fetal distress
- Fluid turns nitrazine paper blue 3. Type and screen blood, CBC, and PT
- When dry, fluid has ferning pattern 4. Bethamethasone as prep for delivery
75. Premature rupture of membranes leads to?: • Preterm labor 84. Name different placenta invasion types?:
• Cord prolapse
• Placental abruption
• Chorioamnionitis (do fewer exams DEC risk chorioamnionitis)
76. Premature rupture of membranes treatment?: 1.
Chorioamnionitis = delivery now
2. Fetus term, no chorioamnionitis, wait 6-12 hours for
spontaneous delivery, no spontaneous delivery then induce
labor
3. Preterm fetuses w/o chorioamnionitis (bethamethasone,
tocolytics, ampicillin, and 1 dose azithromycin)
- if PCN low risk allergy, cefazolin and 1 dose of azithromycin
- high risk of allergy use clindamycin and 1 dose of azithromycin
1. Placenta accreta: abnormal adheres to the superficial uterine
77. ___ abnormal implantation of the placenta over the internal
wall
cervical os.: Placenta previa
2. Placenta increta: attaches to myometrium
78. INC risk placenta previa with?: 1. Previous C-sections 3. Placenta percreta: invades into uterine serosa, bladder
2. Previous uterine surgery wall/rectum wall
3. Multiple gestations * If the placenta cannot detach from the uterine wall after
4. Previous placenta previa delivery of the fetus, the result is => hemorrhage + shock, pts
79. Third trimester bleeding ddx?: • Vulva (trauma, varicose veins) often require hysterectomy
• Vagina (lacerations) 85. Risk factors for placenta invasion?: Placenta previa
• Cervix (polyp, cervicitis, carcinoma) Prior uterine scars
• Uterine
86. ___ premature separation of the placenta from the uterus.:
-Uterine rupture
Placental abruption
-Placenta previa
- tearing of the placental blood vessels and hemorrhaging into
-Vasa previa
the separated space
-Placental abruption
Page 98
87. What can happen if the placental abruption is large enough?: - 93. Presentation:
Premature delivery - Sudden onset of extreme abd pain
- Uterine tetany - Abd bump in abd
- DIC - No uterine contractions
- Hypovolemic shock - Regression of the fetus: fetus was moving toward delivery,
88. Precipitating factors in placental abruption?: 1. Maternal HTN but is no longer in the canal because it withdrew into the abd:
(chronic, preeclampsia, eclampsia)
2. Prior placental abruption
3. Maternal cocaine use
4. Maternal external trauma
5. Maternal smoking during pregnancy
89. Presentation:
- Third-trimester vaginal bleeding
- Severe abd pain (uterine tenderness)
- Contractions
- Possible fetal distress: Placental abruption
- dx: transabdominal US
90. Types of placental abruption.:

Uterine rupture
94. Tx uterine rupture: 1. Treatment
- Emergent laparotomy and delivery
- Repair of uterus or hysterectomy
2. Future management
- Early delivery via C-section at 36 weeks
95. Rh antibody screening algorithm.:

91. Tx placental abruption.: 1. Cesarean delivery


- Uncontrollable hemorrhage
- Fetal distress
2. Vaginal delivery
- Placental separation is limited
- Fetal heart tracing is assuring
- Fetal death prior to presentation
92. Risk factors for uterine rupture.:

- 1st pregnancy: mild anemia/hyperbilirubinemia


- 2nd pregnancy: maternal antibodies attack the second Rh
positive baby

Leads to hemolysis of fetal RBCs -> hemolytic disease of


1. Previous C-section newborn (fetal anemia, extramedullary production of fetal
- Classical (longitudinal along uterus): higher risk of uterine RBCs, hemolysis) -> heme and bilirubin levels -> neurotoxicity
rupture from hyperbilirubinemia -> high output cardiac failure ->
- Low transverse (more recent use) hydrops fetalis
2. Trauma (MVA) 96. What are some scenarios for anti-D Rh immunoglobulins
3. Uterine myomectomy (RhoGAM)?: When fetal blood cells may cross into the mother's
4. Uterine over distention blood:
- Polyhydramnios - Amniocentesis
- Multiple gestations - Abortion
5. Placenta percreta - Vaginal bleeding
- Placental abruption
- Delivery
97. Prenatal antibody screening is done at 28 and 35 weeks.
Patients who continue to be unsensitized at ___ weeks should
receive anti-D Rh immunoglobulins prophylaxis.: 28 weeks

at delivery, if the baby is Rh positive, the mother should be


given anti-D Rh immunoglobulins again
Page 99
98. If patient is sensitized Rh and have antibodies to Rh positive 107.HELLP syndrome define.: Hemolysis
bloody when is the patient considered sensitized which titer Elevated liver enzymes
level.: Low platelets
108.Pregestational diabetes complications maternal vs fetal.: 1.
Maternal
- 4x more likely preeclampsia
- 2x more likely spontaneous abortion
- INC rate of infection
- INC postpartum hemorrhage
2. Fetal
- INC congenital anomalies (heart and NTD)
- Macrosomia (shoulder dystocia)
3. Preterm labor
109. Tests for pregestational diabetes.:

Antibody Screen Positive


• Antibody titer (indirect antiglobulin test)
• Titer > 1:4 "Sensitized"
• Titer > 1:16 -> consider treatment algorithm
99. Chronic hypertension is a BP above 140/90 before the patient
became pregnant or before ___ weeks of gestation.: 20 weeks
of gestation
100. Tx pregnant patient with chronic HTN.: Methyldopa
1. EKG
Labetalol
2. 24 hour urine for baseline renal function (Cr clearance and
Nifedipine
protein)
101. Gestational hypertension is BP above 140/90 that starts after 3. HbA1C
___ weeks of gestation. There is no proteinuria and no 4. Ophthalmological exam for baseline eye function and
edema.: after 20 weeks of gestation assessing the condition of the retina
102. Preeclampsia risk factors.: Chronic HTN 110. Complications of gestational diabetes.: 1. Preterm birth
Renal disease 2. Fetal macrosomia
103. Presentation of types of preeclampsia.: 3. Birth injuries from fetal macrosomia
4. Neonatal hypoglycemia (INC fetal insulin)
5. Development of overt Type 2 DM postpartum 4-10x more
likely
111.Gestational diabetes is routinely screened for between ___
weeks.: 24-28 weeks glucose load test
112. Gestational diabetes testing algorithm.:

104. Preeclampsia algorithm.:

1. Glucose load test (50 g of glucose measure 1 hour later)


2. > 140 mg/dL (oral glucose tolerance test fasting ingestion of
100 mg serum glucose test at 1, 2, and 3 hours after ingestion)
3. Elevation of serum glucose at 1, 2, or 3 hours (gestational
diabetes)
*Only definitive tx is delivery.
113.Tx gestational diabetes.: • Diabetic diet and exercise
105. Eclampsia is a ___ seizure in a patient with a Hx of
(walking)
preeclampsia.: Tonic-clonic seizure
• Medical management (fails fasting > 95 and 1 hour
106. Tx eclampsia.: 1. Stabilize mother postprandial greater than 140)
- Seizure control: magnesium sulfate - Insulin with NPH before bed and aspart before meals
- BP control: hydrazine - Oral hypoglycemics (metformin or glyburide)
2. Deliver baby
Page 100
114.Fetuses with intrauterine growth restriction (IUGR) weight in 127. Biophysical profile consists of?:
the bottom ___ for their gestational age.: 10%
115. Symmetric vs asymmetric IUGR:

116.Causes of IUGR: 1. Chromosomal abnormalities


2. NTD
3. Infections (viral, protozoans) 1. NST
4. Multiple gestations 2. Fetal chest expansion (1 + episodes in 30 minutes)
5. Maternal HTN or renal disease 3. Fetal movt (count fetal movt, normal is > 3 in 30 min)
- maternal malnutrition and maternal substance abuse (smoking 4. Fetal muscle tone (fetus flexes an extremity)
is the #1 preventable cause in US) 5. Amniotic fluid index (volume of amniotic fluid)
117.Dx tests IUGR.: US done to confirm the gestational age and *Each category worth 2 points, BBP of 8 or 10 is normal
fetal weight 4-8: inconclusive < 4: abnormal
118.Complications of IUGR: 1. Premature labor 128.Fetal heart rate
2. Stillbirth Normal
3. Fetal hypoxia Brady
4. Lower IQ Tachy: Normal: 110 - 160
5. Seizures Brady: < 110
6. Mental retardation Tachy: > 160
119.Tx IUGR: 1. Quit smoking 129.Normal accelerations are an INC in HR of ___ or more beats
2. Prevent maternal infection with immunizations (but NOT live per minute above the HR baseline for longer than 15-20
immunizations) seconds. If this happens twice in ___ min it is reassuringly
3. Determine optimal delivery time normal.: 15 or more
120.Fetuses with an estimated birth weight over ___ are 20 minutes
considered macrosomic babies.: > 4500 g 130.Early decelerations (decrease in HR that occurs with
121.Risk factors of Macrosomia: 1. Maternal diabetes or obesity contractions) cause?:
2. Advanced maternal age
3. Postterm pregnancy
122.Dx tests in macrosomia.: 1. Fundal height > 3 cm greater
than GA -> US (on PE the fundal height should equal the
gestational age at 28 weeks the fundal height is 28 cm)

2. Ultrasound confirms the estimated weight by


- Femur length
- Abdominal circumference
- Head circumference and biparietal diameter
Head compression
123.Complications of macrosomia.: 1. Shoulder dystocia
131.Variable deceleration (decrease in HR and return to baseline
2. Birth injuries
with no relationship to contractions) cause?:
3. Low Apgar scores
4. Hypoglycemia
124.Tx Macrosomia.: 1. Induction of labor
- Lungs mature before the fetus is above 4500 grams
2. Cesarean delivery
- if the fetus is above 4500 g in weight
125.A reactive nonstress test (NST) is defined as?: 1. Detection
of 2 fetal movements
2. Acceleration of fetal HR greater than > 15 bpm lasting 15-20
seconds over a 20-min period Umbilical cord compression
126.If the nonstress test is nonreassuring the fetus could be
___.: Sleeping
- Vibroacoustic stimulation is done to wakeup the baby

Page 101
132.Late deceleration (most serious and dangerous: decrease in 137. Steps in stage 2 labor: cardinal movements of labor.:
HR after contraction started. No return to baseline until
contraction ends) cause?:

Fetal hypoxia
133.Physiologic changes before labor (3)?: 1. Lightening (fetal
descent into the pelvic brim)
2. Braxton-Hicks contractions (benign contractions that do not
result in cervical dilation) 1. Engagement (fetal head enters the pelvis occiput first)
3. Bloody show (blood-tinged mucus from the vagina that is 2. Descent
released with cervical effacement) 3. Flexion
134. Stages of labor: 4. Internal Rotation
5. Extension
6. External Rotation
7. Expulsion
138.Signs of placental separation include?: 1. Fresh bleeding from
vagina
2. Umbilical cord lengthening
3. Uterine fundus rising
4. Uterus becomes firm
139.Induction of labor medications?: 1. PGE2: cervical ripening
135. Stages of fetal head descent.: (do not give to asthmatic patients, may provoke bronchospasm)
2. Oxytocin (uterine contractions)
3. Amniotomy (puncture of amniotic sac via amio hook)
- inspect for prolapsed umbilical cord before puncturing the
amniotic sac
140.Prolonged latent stage occurs when the latent phase lasts
longer than ___ for primipara and longer than __ for
multipara.: Primipara longer > 20 hours
Multipara longer > 14 hours
141.Causes of prolonged latent stage.: 1. Sedation
2. Unfavorable cervix
3. Uterine dysfunction with irregular or weak contractions
142.Treatment of prolonged latent stage.: Rest + hydration
- most will convert to spontaneous delivery in 6-12 hours
143.___ occurs when there is slow dilation during active phase
Station
of stage 1 labor, less than 1.2 cm per hour in primipara
- where the fetus's head is located in relationship to the pelvis
women, and less than 1.5 cm per hour in multipara.:
- measuring: -3 to + 3
Protracted cervical dilation
136. Cervix is fully dilated at ___ cm.:
144.Causes of protracted cervical dilation.: 1. Power: strength
and frequency of uterine contractions
2. Passenger: size and position of fetus
3. Passage: if the passenger is larger than pelvis =
cephalopelvic disproportion
145.Tx protracted cervical dilation.: Cephalopelvic disproportion is
cesarean delivery
- if the uterine contraction are weak, oxytocin may be given
146.Types of arrest disorder.: 1. Cervical dilation: no cervical
dilation for 2 hours
10 cm 2. Fetal descent: no fetal descent for 1 hour

Page 102
147.Cause of arrest disorders.: 1. Cephalopelvic disproportion 151. Tx shoulder dystocia.:
- half of all arrest disorders
- treat via c-section
2. Malpresentation
- fetus is older > 36 weeks with presenting part something other
than the head, head is not downward
3. Excessive sedation/anesthesia
148. Types of breech presentation.:

1. McRoberts maneuver (maternal flexion of knees into


abdomen with suprapubic pressure: first line tx)
1. Frank breech: fetus's hips are flexed with extended knees 2. Rubin maneuver (rotation shoulders push posterior shoulder
B/L toward fetal head)
2. Complete breech: fetus's hips and knees are flexed B/L 3. Woods maneuver (rotation shoulder pushing posterior
3. Footling breech: fetus's feet are first: one leg (single shoulder toward fetal back)
footling) or both legs (double footling) 4. Delivery of posterior arm
149.Tx breech: 1. External cephalic version (after 36 weeks 5. Deliberate fracture of fetal clavicle
gestation fetus can maneuver itself into cephalic prevention 6. Zavanelli maneuver
before 36 weeks) - push head back into the uterus and perform c-section
2. C-section - high rate of both maternal and fetal mortality
- last maneuver to try
150.Shoulder dystocia risk factors?: When fetus's head has been
delivered but the anterior shoulder is stuck behind the pubic 152.Postpartum hemorrhage is bleeding more than ___ mL after
symphysis delivery.: 500 mL
- early occurs w/in 24 hours of delivery
- Maternal DM and obesity causes fetal macrosomia - late bleeding occurs after 24 hours to 6 weeks later
- Postterm pregnancy allows baby more time to grow 153.Causes of postpartum bleeding.: 1. Uterine atony (80% of
- Hx of prior dystocia hemorrhage=> normally postpartum the uterine contraction
compress the blood vessels to stop blood loss)
2. Laceration
3. Retained products of conception
4.Coagulopathy
154.Risk factors for uterine atony.: 1. Anesthesia
2. Uterine overdistention (twins and polyhydramnios)
3. Prolonged labor
4. Laceration
5. Retained placenta (can occur with placenta accreta)
6. Coagulopathy
155.___ after postpartum hemorrhage presents as inability to
breastfeed.: Sheehan syndrome
156.Tx uterine atony: 1. Examine uterus bimanual exam
2. Assure that there is no rupture of the uterus, no retained
placenta
3. Nothing on exam => Bimanual compression and massage
4. Bleeding continues give oxytocin to make the uterus
contract, constricting the blood vessels and decreasing the
blood flow

Page 103
MTB Oncology
MTB Q and A

1. When is breast cancer typically found?: - Asx women on 16. When to use Trastuzumab in Breast Cancer: Her2/neu positive
screening mammography or breast cancer (abnormal estrogen receptor)
- Palpation of hard, painless mass
- May have retraction of nipple DEC risk of recurrent disease and increase survival
2. Mammography is indicated to screen for breast cancer in the 17. When to give adjuvant chemotherapy with breast cancer?:
general population stating at the age of ___.: 50 Lesions larger than 1 cm
3. Best initial test for breast cancer.: Fine needle aspiration
(FNA) Positive axillary lymph nodes are found with metastasis.
- false negative rate 10% 18. T/F: Use Tamoxifen when multiple first-degree relatives have
- cannot test for estrogen/progesterone/Her 2/neu on the FNA breast cancer. It lowers the risk of breast cancer.: TRUE
4. Core needle biopsy: Larger sample that is more deforming, 19. What lowers mortality in breast cancer?: - Mammography
ability to get receptor information (ER, PR, HER 2/neu). - ER/PR testing then tamoxifen/raloxifene
- Aromatase inhibitors
More deforming than FNA. - Adjuvant chemotherapy
5. Most accurate test for breast cancer.: Open biopsy - Lumpectomy and radiation
- frozen section + immediate resection of the cancer followed by - Modified radical mastectomy
sentinel node biopsy - Trastuzumab (anti-Her2/neu)
6. Benefit of mammography when a nodule has already been - Prophylaxis with tamoxifen (or raloxifene)
found.: 5-10% of breast cancer is bilateral 20. Prostate cancer presentation: Typically Asx
7. When to use ultrasound with breast cancer?: Indeterminate Can have obstructive symptoms and a palpable lesion
mass/lesion.
US tells if cyst or solid. tx: prostatectomy
21. Best initial test and the most accurate test for prostate
Answer US if the lesions: cancer?: Biopsy
- is painful 22. Complications of prostatectomy: ED
- varies in size or pain with menstruation Urinary incontinence
8. When to use PET scan with breast cancer?: Determining
content of lymph nodes that are not easily accessed for ED = major drawback to surgery more likely than radiation to
biopsy. cause ED, radiation leads to diarrhea
23. Gleason Grading:
Cancer INC uptake on PET scan
9. BRCA associated with?: INC risk of breast cancer in families
INC risk of ovarian cancer and pancreatic cancer

*BRCA has not yet been shown to add mortality benefit to usual
management -> but some patients opt for B/L mastectomy
10. When to do sentinel lymph node biopsy?: Done routinely in
lumpectomy or mastectomy

Negative sentinel node eliminates need for axillary lymph


node dissection
11. When are estrogen and progesterone receptors tested?: ER
Higher grade, more aggressive
and PR testing is routine for all patients
- higher grade suggest a greater benefit of surgical removal of
-> hormone manipulation therapy is done if either test is positive
the prostate
12. Surgery in Breast Cancer: Radical mastectomy is never the
24. Hormonal Manipulation in Prostate Cancer: Do not prevent
answer because lumpectomy + radiation is as efficacious and
recurrences
less deforming
13. Hormone therapy in breast cancer: Estrogen antagonists: Flutamide, GnRH agonists, ketoconazole, and orchiectomy
Tamoxifen, raloxifene (better premenopausal) can help control size and progression of metastases.

Aromatase inhibitors: Anastrozole, letrozole, exemestane SHRINK already present lesions.


(slight superiority in efficacy, better postmenopausal)
25. Not beneficial in prostate cancer: - Screening imaging study -
14. Side effects of tamoxifen (estrogen antagonists): Endometrial prostate US used to localize lesions to biopsy when PSA is high
cancer and clots - Lumpectomy
15. Side effects of Aromatase inhibitors: Osteoporosis (inhibit - Chemotherapy
estrogen everywhere, even on bone) - Hormone replacement for recurrence

ALL NOT HELPFUL Page 104


26. PSA Necessity: - No clear mortality benefit with PSA 38. Nonseminoma vs. Seminoma: Nonseminoma has increased
- PSA is not to be routinely offered to patients, if the patient is alpha fetoprotein
requesting PSA then do the test
- A normal PSA, does not exclude the possibility of prostate All have HCG elevation
cancer 39. Staging in testicular cancer which test?: CT scan of the
- Above age 75, do not do even if asked abdomen, pelvis, and chest

the higher the PSA, the greater the risk of cancer Metastasis through retroperitoneum and up to the chest.
- PSA corresponds to the volume of cancer 40. Tx of Testicular Cancer: 1. Local disease: Orchiectomy +
27. Elevated PSA Algorithm: radiation
2. Widespread disease: Orchiectomy + chemotherapy

Testicular cancer is one of the only malignancies in which


chemotherapy can cure widely metastatic disease, including
spread into the brain
41. Seminomas are sensitive to?: Seminomas: sensitive to chemo
and radiation

Nonseminomas: sensitive to chemo


28. Lung Cancer that is not resectable: - B/L disease or lymph 42. Advanced Cervical Cancer Tx: Hysterectomy
nodes involved on opposite side 43. Prevention of Cervical Cancer: HPV Vaccine between 11 and
- Malignant pleural effusion 26
- Heart, carina, aorta, or vena cava involved 44. Pap smear routine: Starts at 21, repeat every three years to
65.
* Small cell cancer is considered unresectable in 95% of cases
because it is metastatic or spread outside one lung Or every 5 years if cotested with HPV
29. Screen for lung cancer annually with low-dose chest CT in 45. Low grade and high grade dysplasia on Pap smear is followed
those with?: 30 pack-year smoking history up with?: Colposcopy for biopsy
46. Atypical squamous cells of undetermined significance
Age 55-80 (ASCUS) Positive next step.: HPV testing
30. What is the screening test for ovarian cancer?: There is NO
screening test if HPV positive, do colposcopy
31. Presentation of ovarian cancer.: Woman < 50 with INC 47. If no HPV with ASCUS: Repeat pap smear testing in 6 months
abdominal girth but who is still losing weight.

BRCA is associated with ovarian cancer.


32. Initial test for Ovarian Cancer:

Ultrasound or CT scan
33. Most accurate test for ovarian cancer: Biopsy
34. Usefulness of CA-125 in ovarian cancer.: Used only for follow-
up treatment, not for screening
35. Tx of Ovarian Cancer: Remove all locally metastatic disease
and give chemotherapy
36. Testicular Cancer Presentation: Painless lump in scrotum that
does NOT transilluminate.

INC with history of cryptorchidism


37. Diagnosis of Testicular Cancer: Removal of the whole testicle
with inguinal orchiectomy. DO NOT CUT THE SCROTUM.

NEVER BIOPSY THE TESTICLE!


Page 105
MTB Ophthalmology
MTB Q and A

1. Comparison of Viral and Bacterial Conjunctivitis.: 4. Glaucoma:


Presentation.
Eye findings.
Most accurate test.
Best initial tx.: Pain.

Fixed midpoint pupil.

2. Conjunctivitis: Tonometry.
Presentation.
Eye findings. Acetazolamide, mannitol, pilocarpine, laser trabeculoplasty
Most accurate test. 5. Abrasion.
Best initial tx.: Presentation.
Eye findings.
Most accurate test.
Best initial tx.: Trauma

Feels like sand in eyes.

Fluorescein stain
Itchy eyes, discharge
No specific therapy, patch not clearly beneficial
Normal pupils 6. Chronic glaucoma symptoms.: • Most often asymptomatic
• Diagnosed by screening
Clinical dx • Confirmation with tonometry
• Elevated IOP
Topical antibiotics
7. Tx chronic glaucoma.: 1. Prostaglandin analogues
3. Uveitis: - Latanaprost, Travoprost, Bimatoprost
Presentation. 2. Topical beta blockers
Eye findings. - Timolol, Carteolol, Metipranolol, Betaxolol, Levobunolol
Most accurate test. 3. Topical carbonic anhydrase inhibitors
Best initial tx.: - Dorzolamide, Brinzolamide
4. Alpha-2agonists
- Apraclonidine
5. Pilocarpine
6. Laser trabeculoplasty: performed if medical tx is inadequate
8. Sudden onset of an extremely painful, red eye that is hard to
palpation. Walking into a dark room ppt the pain. Pupil does
not react to light. Dx.:

Autoimmune disease

Photophobia

Slit lamp exam

Topical steroids
Acute angle-closure glaucoma
- cup to disc ratio > 0.3
- tx with tonometry
9. Tx acute angle-closure glaucoma: IV acetazolamide
IV mannitol (osmotic draw of fluid out of the eye)
Pilocarpine, BB, and apraclonidine to constrict the pupil and
enhance drainage

Laser iridotomy
Page 106
10. Infection of the cornea. Eye is very red, swollen, painful. 21. Sudden onset of painless, unilateral loss of vision that is
Dx.: Herpes Keratitis described as "a curtain coming down". Dx.:
- fluorescein staining show dendritic pattern on exam
- do not use steroids, INC viral production
11. Tx Herpes Keratitis: oral acyclovir, famciclovir, or valacyclovir

topical: trifluridine and idoxuridine


12. Do not use ___ in herpes keratitis can make the condition
worse by INC viral production.: STEROIDS
13. Tx cataracts.:

Retinal detachment
- trauma, extreme myopia, diabetic retinopathy
22. Tx retinal detachment: Reattachment by mechanical methods
• Surgery, laser, cryotherapy
• Injection of expansile gas pushes retina back up against globe
of eye
23. MCC of blindness in older person in the U.S.: Macular
degeneration
Surgical removal of the the lens and replace with new lens
- unknown cause
14. Nonproliferative or background retinopathy is managed by - atrophic (dry) and neovascular (wet) type.
____.: controlling glucose
24. • Far more common in older patients
15. Most accurate test diabetic retinopathy.: Fluorescein • Bilateral
angiography • Normal external appearance of eye
16. Tx proliferative retinopathy.: Laser photocoagulation • Central vision lost: Macular degeneration
- VEGF are injected in some patients to control 25. T/F: Atrophic macular degeneration has no proven effective
neovascularization therapy: TRUE
17. ___ may be necessary to remove a vitreal hemorrhage 26. ____ type causes 90% of permanent blindness from macular
obstructing vision.: Vitrectomy degeneration: Neovascular or wet type
18. Retinal artery and vein occlusion similarities.: - more rapid, severe
- new vessels grow between the retina and the underlying Bruch
membrane
27. Best initial therapy for neovascular disease.:

Both present with:


- Sudden onset monocular visual loss
- Can't make diagnosis without retinal examination
- No conclusive therapy for either condition

* macula is described as "cherry red" in artery occlusion


because the rest of the retina is pale
19. Tx retinal artery occlusion.: - 100% oxygen
- Acetazolamide
- Ocular massage
- Anterior chamber paracentesis to DEC IOP
- Thrombolytics
20. Tx retinal vein occlusion.: Ranibizumab VEGF inhibitors
- Ranibizumab, bevacizumab, or aflibercept
- Injected directly into vitreous chamber every 4 to 8 weeks

Over 90% stop progression


1/3 improve vision

Page 107
MTB Pediatrics
MTB Q and A

1. Normal vital signs RR: 40-60 BPM 8. Management of


in a newborn. HR: 120-160 BPM Newborn at
Delivery
2. Apgar score:
Newborn
Assessment

- Erythromycin or tetracycline
ophthalmic ointment prevent ophthalmia
Quantifiable measurements for the need
neonatorum (Neisseria or Chlamydia) or
and effectiveness of resuscitation (does
sylver nitrate solution*
not predict mortality)

- 1 mg of vitamin K IM to prevent
1 min score: evaluates conditions
hemorrhagic disease
during labor and delivery
5 min score: evaluates response to 9. All neonates must - PKU
resuscitation efforts be screened for - Congenital adrenal hyperplasia (CAH)
these disease - Biotinidase
3. TRUE/FALSE: A low FALSE, not associated
prior to - B-thalassemia
score on Apgar is
discharge: - Galactosemia
associate with
- Hypothyroidism
future cerebral
- Homocysteinuria
palsy.
- CF
4. At day 1 the most Chemical irritation
10. Phenylketonuria AR genetic disease deficiency of enzyme
likely cause of
phenylalanine hydroxylase (PAH) that
conjunctivitis in
leads to mental retardation
newborn is ___.
- tx: special diet low in phenylalanine first
5. From days 2-7 most Neisseria gonorrhoeae 16 years of patient's life
likely cause of
11. A condition Congenital hypothyroidism
conjunctivitis in
affecting 1 in
newborn ____.
4,000 infants that
6. Conjunctivitis after Chlamydia trachomatis can result in
more than 7 days cretinism.
post delivery is
12. Before discharge Every child gets a hepatitis B vaccine,
most likely due to
of a newborn but only those with HBsAg-positive
____.
which vaccination mothers should receive the hep B
7. Conjunctivitis after Herpes infection is given? immunoglobulin (HBIG) in addition to the
3 weeks or more is vaccine
most likely due to
13. Transient Hypoxia during delivery cord clamping
___ in a newborn.
polycythemia of stimulates EPO -> INC circulation RBC
newborn cause? - newborn's first breath will INC O2 lead
to DEC EPO which will normalize
hemoglobin
- Splenomegaly normal finding in
newborns
14. Transient Newborns born via C-section may have
tachypnea of the excess fluid in lungs -> hypoxic
newborn cause?
Tachypnea > 4 hours -> consider SEPSIS
evaluate blood and urine culture

LP with CSF analysis and culture done


when newborn displays neuro findings
such as irritability, lethargy, temp
irregularity, and feeding problems
Page 108
15. Transient > 60% of newborns are jaundiced 20. The most common newborn Clavicular fracture
hyperbilirubinemia - due to infant's spleen removing excess fracture as a result of shoulder - X-ray best diagnostic test
of the newborn RBCs that carry Hgb F dystocia (anterior shoulder of - Tx: immobilization,
cause? - physiologic relate of hemoglobin -> rise the baby gets stuck behind the splinting, and PT
in unconjugated bilirubin mother's pubic bone).
16. Three major types 1. Linear: most common 21. Cause of polyhydramnios. Too much fluid 2/2 fetus
of skull fractures 2. Depressed: can cause further cortical not swallowing
in newborn? damage w/o surgical intervention - Neurological Werdnig-
3. Basilar: most fatal Hoffmann (infant unable to
swallow)
17. Scalp injuries in a
- GI (intestinal atresia)
newborn.
22. Cause of oligohydramnios Too little fluid because
fetus cannot urinate

1. Prune belly: lack of


abdominal muscles, so
unable to bear down and
- Caput succedaneum -> swelling of soft urinate
tissues of the scalp that does cross - tx: serial Foley catheter
suture lines placements, high risk of
UTIs
- Cephalohematoma -> subperiosteal
hemorrhage that does not cross suture 2. Renal agenesis:
lines incompatible with life
(Potter syndrome)
18. Waiter's tip
appearance 2/2
3. Flat facies due to high
shoulder dystocia
atmospheric pressure
causing compression of the
Infant unable to
fetus that is normally
abduct the
buffered by the amniotic
shoulder or
fluid
externally rotate
and supinate the 23. Bowel sounds in chest can be Diaphragmatic hernia
arm Duchenne-Erb Paralysis: C5-C6 heard
Air fluid levels seen on CXR Two types:
Dx made clinically - Morgagni: defect in
and retrosternal or parasternal
immobilization is - Bochdalek: defect in
best tx. posterolateral (most
common) most LEFT side
19. Claw hand due to Klumpke Paralysis: C7-C8 +/- T1
lack of grasp 24. ___ defect in which intestines Omphalocele
reflex and organs form beyond the - failure of GI sac to retract
abdominal wall with a sac at 10-12 weeks gestation
Paralyzed hand covering. - Screen AFP and U/S
with Horner - Sx reintroduction of
syndrome (ptosis, contents needed
miosis, and
anhydrosis) Associated Edwards
syndrome (trisomy 18)
Dx. made
25. Congenital weakness where Umbilical hernia
clinically and
vessels of the fetal and infant
immobilization is
umbilical cord exited through Associated with congenital
the best tx.
the rectus abdominis muscle. hypothyroidism

Screen with TSH.


May close spontaneously

Page 109
26. Umbilical hernia Congenital hypothyroidism 31. Painless, swollen fluid filled Hydrocele
protrusion of - 90% close spontaneously at age 3 sac along the spermatic cords - remnant of tunica vaginalis
contents due to - After age 4, sx intervention prevent w/in the scrotum that - Usually will resolve within
congenital bowel strangulation and necrosis transilluminates upon 6 months.
weakens of the inspection. - must differentiate from
rectus abdominis inguinal hernia
muscle is highly
32. Cause of varicocele. Varicose veins in the
associated with
scrotal veins causing
___.
swelling of pampiniform
27. Most common Incorrect dating plexus and INC pressure
cause of elevated
AFP is ___. other causes: NTD, abdominal wall defect - Most common c/o dull
ache and heaviness in
28. ____ is a wall Gastoschisis
scrotum
defect lateral to - multiple intestinal atresias can occur
- Best initial test: PE "bag
midline with - tx: immediate surgical intervention with
of worms" sensation
intestines and gradual introduction of bowel and silo
- Most accurate: U/S
organs forming formation
scrotal sac both sides
beyond the - overly aggressive surgical
showing dilation of vessels
abdominal wall reintroduction of the bowel will lead to
of the pampiniform plexus
with no sac third spacing and bowel infarction
> 2 cm
covering.
29. Most common Tx: indicated for delayed
abdominal mass in growth of the testes or in
children. those with evidence of
testicular atrophy
33. __ absence of one testicle in Cryptorchidism
the scrotum, and it usually - 90% of cases can be felt
found in the inguinal canal. in the inguinal canal
- Orchiplexy is indicated to
bring the testicle down into
the scrotum after the age of
Wilms tumor 1 to avoid sterility
- hemihypertrophy of one kidney due to 34. TRUE/FALSE: Cryptorchidism TRUE
INC vascular demands is associated with an increase
- Aniridia risk of malignancy regardless
- Constipation, abd pain, N/V of surgical intervention.
Dx:
35. Urethral opening on ventral Hypospadias
- Abdominal U/S (initial)
surface
- Contrast-enhanced CT (most accurate)
High association with
Tx: Chemo and radiation
cryptorchidism and inguinal
B/L kidney involvement indicates partial
hernias
nephrectomy
30. ___ are Neuroblastoma Needs surgical correction
statistically the - adrenal medullar tumor
most common - Hypsarrhythmia (EEG) and DO NOT circumcise due to
cancers in infancy opsomyoclonus (jerky eye movements difficulties in sx correction
and the most "dancing eyes" and myoclonic jerks and of hypospadias
common cerebellar ataxia ("dancing feet")
36. Urethral opening on dorsal Epispadias
extracranial solid
surface
malignancy. INC VMA and metanephrines urine
Associated with urinary
collection are diagnostic
incontinence

Evaluate for bladder


exstrophy

Needs surgical correction

Page 110
37. Developmental 1. Sucking reflex: baby will automatically 45. Most common Transposition of the Great Vessels
achievements suck on a nipplelike object cyanotic lesion of - aorta origin from the RV and pulmonary
reflexes 2. Grasping reflex neonates. artery from LV
3. Babinski reflex: toe extension - No oxygenation of blood can occur w/o:
4. Rooting reflex: if you touch a baby's PDA, ASD, or VSD
cheek, the baby will turn to that side
5. Moro reflex: arms spread Early and severe cyanosis is seen,
symmetrically when the baby is scared single S2 heard
6. Stepping reflex: walking-like
46. CXR in
maneuvers when toes touch the ground
transposition of
7. Superman reflex: when held facing
the great vessels.
the floor, arms go out
38. Tetralogy of Fallot 1. Pulmonary stenosis
is characterized 2. Overriding aorta
by? 3. RVH
4. VSD
39. Tetralogy of Fallot chromosome 22
is associated with
chromosome ___
deletions.
40. Presentation of 1. Cyanosis of lips and extremities
Tetralogy of Fallot 2. Holosystolic murmur best heard at
left lower sternal border Egg on a string
3. Squatting after executive activities 47. Tx Transposition • Prostaglandin E1 to keep the PDA
- Causes an INC preload and INC SVR, of the Great open (NSAIDs contraindicated especially
DEC R to L shunting, INC pulmonary Vessels. indomethacin)
blood flow, INC blood O2 saturation
41. Dx of Tetralogy of • Surgery (1/4 survive surgery)
Fallot. 48. Pulsus alternanas Sign of LV systolic dysfunction
49. Pulsus bigeminus Sign of hypertrophic obstructive
cardiomyopathy (HOCM)
50. Pulsus bisferiens AR
51. Pulsus tardes et AS
parvus
52. Pulsus paradoxis Cardiac tamponade and tension
pneumothorax

CXR showing boot-shaped heart

DEC pulmonary vascular marking

Sx intervention only definitive therapy


42. VSDs are common Down (trisomy 21)
in? Edwards (trisomy 18)
Patau (trisomy 13)
43. The most common Endocardial cushion defect at the
cardiac defect in atrioventricular canal
Down syndrome
is?
44. The 3 holosystolic 1. MR
murmurs. 2. TR
3. VSD

Page 111
53. Absent pulses with 57. Dyspnea with respiratory distress VSD
a single S2 - CXR: INC vascular
INC RV impulse High-pitched holosystolic murmur markings
Gray rather than over lower left sternal border. - ECHO: dx and
bluish cyanosis cauterization is
Loud pulmonic S2. definitive
58. __ is the process in which a L to R Eisenmenger syndrome
shunt caused by a VSD reverses (ES)
into a R to L shunt due to
hypertrophy of the RV.
59. Tx VSD - Smaller lesions close
in the first 1 to 2 years
- Larger lesions are
more symptomatic
Hypoplastic LH syndrome require surgical
- LV hypoplasia, MV atresia, aortic valve intervention
lesions
Conservative tx:
CXR: Globular shaped heart with Diuretics and digoxin
pulmonary edema, ECHO most accurate
Left untreated:
Sx or heart transplant complication lead to
54. ___ occurs when a Truncus arteriosus CHF, endocarditis, and
single trunk - severe dyspnea pulmonary HTN
emerges from both - early and frequent respiratory infections 60. The three major types of ASD. 1. Primum defect:
RV and LV and - peripheral pulses bounding, single S2 concomitant MV
gives rise to all heard abnormalities
major 2. Secundum defect:
circulations. CXR: Cardiomegaly, INC pulmonary most common and
markings located in the center of
the atrial septum
Tx: Most severe sequela pulmonary HTN 3. Sinus venous
develop in 4 month. Sx must be defect: least common
completed early to prevent pulmonary
HTN ASD is a hole in the
55. No venous return septum between both
between atria 2x common in
pulmonary veins women > men
and the left 61. Wide fixed splitting of S2. ASD
atrium, - most definitive test is
oxygenated blood cardiac catheterization
instead returns to - ECHO just as useful
the SVC.
CXR: INC vascular
Total Anomalous Pulmonary Venous markings and
Return cardiomegaly
56. __ is the most VSD 62. ASD tx Vast majority close
common spontaneously
congenital heart
lesion. Sx or transcatheter
closure is indicated for
all symptomatic
patients

Dysrhythmias and
possible paradoxical
emboli from DVT later
in life

Page 112
63. When is PDA > 24 hours 71. 3 like appearance
considered or rib notching on
pathologic? < 12 hours of life = normal CXR
64. - Machinery- PDA
like murmur - failure of spontaneous closure of ductus
- Wide pulse - closes when PO2 rises above 50 mm Hg
pressures (low O2 may be due to pulmonary
- Bounding compromise due to prematurity; areas of
pulses high altitude INC occurrence of PDA)

ECHO best initial test


Cardiac catherization most accurate
EKG shows LVH 2/2 high systemic resistance
65. Tx PDA Indomethacin (NSAIDs inhibits Coarctation of the aorta
prostaglandins) to close PDA unless it is
72. Dx. long QT syndrome
needed to live in concurrent conditions such
Hearing loss.
as TOF
Syncope
66. Mitral lesions Axilla Normal VS and
radiate to the exam
___. FHx of sudden
cardiac death
67. Tricuspid and Back
pulmonary 73. Exam question Coarctation of the aorta
lesions states short girl - congenital narrowing of the aorta in the
radiate to the with webbed neck, area of the ductus arterosus
___. shield chest, - associated Turner syndrome
streak gonads,
68. Aortic lesions Neck
horseshoe
radiate to the
kidneys, or
___.
shortened 4th
69. CXR finding: metacarpal think
Pear-shaped ___ cardiac
heart. defect.
74. Presentation:
- Severe CHF and
respiratory
distress w/in first
few months of life
- Differential
pressures and
pulses btw UE and
LE
- Reduced pulses
in the LE and HTN
Pericardial effusion in UE
70. CXR finding:
jug handle
appearance: Coarctation of the aorta
- CXR: rib notching "3" sign
- Cardiac catherization most accurate
test

Tx: Surgical resection of the narrowed


Primary pulmonary artery hypertension segment and then balloon dilation if
recurrent stenosis occurs

Page 113
75. Hyperbilirubinemia 80. Presentation:
is considered "Vomiting with
pathologic when? first feeding"
Hx
polyhydramnios
Recurrent
aspiration
pneumonia

Tracheoesophageal fistula (TEF)


- CXR: gastric air bubble and esophageal
air bubble
1. Appears first day of life - Coiling NG tube, inability to pass the
2. Bilirubin > 5 mg/dL/day stomach
3. Bilirubin rises > 19.5 mg/dL in a term
child CT or esophagram can also be used
4.Direct bilirubin rises above >2 mg/dL 81. Tx TEF
at any time
5. Hyperbilirubinemia persists after the
second week of life
76. Most serious
complication is the
deposition of
- Sx repair in 2 steps to correct the
bilirubin.
congenital anomaly
___ presents with
- Abx coverage for anaerobes due to
hypotonia,
high risk of lung abscess formation 2/2
seizures,
aspiration
choreoathetosis,
- Fluid resuscitation before Sx must be
and hearing loss.
done to prevent dehydration of the infant

Kernicterus 82. ≥ 3 weeks of age


- deposition of bilirubin in basal ganglia Nonbilious
vomiting ->
77. Dx test in - Direct and indirect bilirubin projectile
pathologic - ABO and Rh incompatibility check Small "olive pit"
jaundice in a infant and mother size epigastric
newborn. - Peripheral smear look for hemolysis mass
78. Tx pathologic Blue-green light helps break down Dehydration -
jaundice in bilirubin to exitable components 'contraction
newborn. - consider exchange transfusion if alkalosis'
bilirubin rises 20-25 mg/dL
Dx. Pyloric stenosis
79. If you see Tracheoesophageal fistula
- hypertrophy of pylorus
recurrent
- succussion splash on auscultation
aspiration
- hypochloremic, hypokalemic metabolic
pneumonia in
alkalosis loss of H+ ions in the vomit
newborn consider
___.
"Olive sign" - > palpable mass the size of
an olive felt in the epigastric region

Page 114
83. Best initial test 88. Dx test Choanal
pyloric stenosis. atresia. Tx.
Most accurate test
with 4 signs?

Abdominal U/S shows thickened pyloric


stenosis

Most accurate test: Upper GI series,


which will show 4 signs.
CT scan
1. Mushroom sign: hypertrophic pylorus
against the duodenum
Tx: Sx intervention to perforate the
2. String sign: thin column of barium
membrane and reconnect the pharynx to
leaking through the tightened muscle
the nostrils
3. Railroad track sign: excess mucosa in
the pyloric lumen results in 2 columns of 89. ___ is congenital Hirschsprung disease
barium lack of innervation - Down syndrome association
4. Shoulder sign: filling defect in the of the distal bowel - More common in boys > girls (4:1)
antrum due to prolapse of muscle inward by the Auerbach
plexus. The lack
84. Name and
causes a constant
describe the 3
contracture of
Atresias
muscle tone. This
- defect
is associated
- presentation
frequently with
- test
___.
- tx
90. Pt presents: Hirschsprung disease
Does not pass
meconium over 48
85. Tx pyloric IVF, replace lost electrolytes (K+) hours or fails to
stenosis. pass meconium
NGT to decompress the bowel
Extreme
Sx myotomy constipation
followed by large
86. What is CHARGE Coloboma of the eye, CNS anomalies
bowel obstruction.
syndrome? Heart defect
Atresia of choanae
DRE: tight
Retardation of growth and/or
sphincter, an
development
inability to pass
Genital and/or urinary defects
flatus.
(hypogonadism)
Ear anomalies and/or deafness 91. Dx test X-ray: distended bowel loops with lack
Hirschsprung of air in the rectum, contrast enema
87. Presentation:
disease. Tx. show retention of barium for > 24 hours
Child turns blue
when feeding and
Manometry: high pressure in the anal
pink when crying.
sphincter

Dx: full thickness Bx reveals lack of


ganglionic cells in the submucosa

Choanal atresia tx: 3 stage surgery procedure is curative


92. Imperforate anus Down syndrome
associated - imperforate anus is opening of the anus
with____. is missing and the rectum ends in a blind
pouch with conservation of the sphincter

Page 115
93. Imperforate anus Vertebral anomalies 99. In children, volvulus Midgut
is one of the Anal atresia occurs in the ____ with - Ileum
components of CV anomalites the majority being in the
VACTERL TEF ___.
syndrome what Esophageal atresia
100. Presentation: Volvulus
are the other Renal anomalies
Vomiting, colicky abd
components? Limb anomalies
pain
94. Dx: Complete Imperforated anus Multiple air fluid levels
failure to pass - PE no anus Upper GI series "bird
meconium. - Sx curative beak" site of rotation
95. Pathophysiology of Lack or absence of apoptosis that leads 101. Tx volvulus Best initial therapy: endoscopic
duodenal atresia. to improper canalization of the lumen of decompression and most
the duodenum effective if endoscopy fails is
surgical decompression
96. Duodenal atresia
associated with
Sx or endoscopic untwisting is
___ and ___.
emergent (bowel necrosis w/
perforation can lead to life-
threatening sepsis)
102. Intussusception is Rotavirus vaccine
associated with Henoch-Schonlein purpura
previously used __
vaccine and ___.
Down syndrome 103. Pt presents with colicky Intussusception
Annular pancreas abd pain, bilious - currant jelly stool
97. Pt presents with vomiting, and right
bilious vomiting quadrant sausage-
w/in 12 hours of shaped mass.
birth. 104. Dx test Intussusception. Best initial: U/S show doughnut
sign or target sign, concentric
alternating echogenic (mucosa)
and hypoecogenic (submucosa)
bands

Barium enema is both diagnostic


Duodenal atresia and therapeutic
- X-ray: double bubble sign 105. When is barium enema Child has signs of:
98. Tx duodenal contraindicated in a - Peritonitis
atresia. child as a treatment for - Shock
intussusception? - Perforation
106. Tx intussusception. Fluid resuscitation and balance
electrolytes (K, Ca, Mg) are the
most important initial steps

NGT decompression of the bowl

Carefully observe child 24 hours


post barium enemy since 10%
recur w/in 24 hours

If barium enema is not curative.


IVF then emergent sx intervention to
Replace electrolytes (K+) prevent bowel necrosis
NGT to decompress the bowel

Sx duodenostomy most common sx


procedure and definitive treatment

Page 116
107. Bilious vomiting 113. Pt presents 1. Send stool for blood and leukocyte count
in newborn 3 with to detect presence of invasive toxins
different causes? diarrhea. Dx 2. Stool cultures for O&P
tests. 3. Possible sigmoidoscopy to examine
pseudomembranes in the setting of
c.difficile
114. Tx diarrhea. Mild cases: oral fluids
Severe cases: IV fluids
115. Viral causes
108. Toddler presents of diarrhea.
with painless
rectal bleeding.
Massive frank
bright red blood
per rectum.

116. Presentation: Necrotizing enterocolitis


- Child born - seen in premature infants where the bowel
premature undergoes necrosis and bacteria invade the
with low intestinal wall
birth weight - mortality 30%
- Vomiting
and
Meckel's diverticulum abdominal
- true congenital diverticulum vitelline distention
duct persists in the small intestinal tract, - Fever
contain ectopic gastric tissue 117. Dx tests
-> gastric acid secreted by the ectopic necrotizing
tissue causing searing nearby small bowl enterocolitis.
tissue
109. Dx test Meckel's Most accurate: technetium 99m scan
diverticulum. Tx. (Meckel scan)

Sx removal of the diverticulum only


curative therapy
110. Mickel's 1. Affects 2% of population
diverticulum rule 2. Occurs 2 feet from the ileocecal
of 2s. valve
3. Affects 2 types of ectopic tissues
(gastric and pancreatic)
4. Male patients 2x more affected Abd X-ray: pathognomonic "pneumatosis
5. Patient age <2 intestinal" or air w/in the bowel wall and CT
6. Only 2% of patients symptomatic will reveal air in the portal vein, dilated bowel
7. About 2 inches long loops, and pneumoperitoneum if a perforation
has occurred
111. __ is the second Acute diarrhea
most common
Frank or occult blood can be seen in stool
cause of infant
death worldwide. 118. Tx 1. Feeding must be D/C for bowel rest
necrotizing 2. IV fluids must be started immediately
112. ___ is the Gastroenteritis
enterocolitis. 3. NGT must be placed for bowel
inflammation of
decompression
the GI tract
4. If medical mgt does not lead to resolution,
secondary to
then surgery is indicated to remove the
microbial
affected bowel
infiltrate and
spread.

Page 117
119. When there is Start abx of choice vancomycin, 122. Congenital
confirmed gentamicin, metronidazole adrenal
evidence of - adjunct with serial abd X-rays to hyperplasia (CAH)
necrotizing exclude perforation has 3 forms?
enterocolitis next
step in
management? 1. 21-hydroxylase (90% of the cases)
2. 17-hydroxylase
120. A 10.5-pound
3. 11-beta-hydroxylase
infant is born to a
mother with Type 123. Dx test congenital Dx at birth with serum electrolytes and
I diabetes. Upon b. Blood sugar level adrenal INC 17-OH progesterone level
examination of hyperplasia.
newborn, he is
124. Tx congenital IVF and electrolyte replacement with
shaking, and a
adrenal lifelong steroid maintain adequate
holosystolic
hyperplasia. mineralo/glucocorticoid levels
murmur is heard
over precordium.
Specific psych counseling to aid with
The baby's right
gender ID issues
arm is adducted
and internally 125. Congenial
rotated. His lab Adrenal
findings show an Hyperplasia
elevated differences
bilirubin. between 3 most
Which of the common types.
following is the
most appropriate
next step in
management?
126. Children ___ are
a. IV insulin
highest risk for
b. Blood sugar
Rickets because
level
their bones are
c. Serum calcium
rapidly growing.
levels
d. Serum TSH
e. CT head and
neck
121. 5 Main Findings 1. Macrosomia (all organs large, except
in infants of brain, polycythemia, hyper viscosity,
diabetic mothers. shoulder dystocia, brachial plexus palsy)
2. Small left colon syndrome (smaller
descending colon constipation, dx barium
study, tx with smaller and more frequent
feeds) 6-24 months
3. Cardiac abnormalities (asymmetric - softening of bones and weakening of
septal hypertrophy obliteration of the LV bones, making them more susceptible to
lumen, DEC CO, EKG and ECHO, tx BB fracture
and IVF) - lack of vitamin D, calcium, or
4. Renal vein thrombosis (flank pain, phosphate
bruit, hematuria, and thrombocytopenia) - ulnar/radial bowing and waddling gate
5. Metabolic findings: 127. Cause of rickets. 1. Vitamin D deficient rickets caused by
- Hypoglycemia: seizures, Hypocalcemia: lack of enough vitamin D in the child's
tetany, lethargy, Hypomagnesemia: diet
hypocalcemia and PTH DEC , 2. Vitamin D dependent rickets is
Hyperbilirubinemia: icterus and inability to convert 25-OH to 1,25 (OH)2
kernicterus 3. X-linked hypophosphatemic rickets
occurs when an innate kidney defect
results in inability to retain phosphate,
no phosphate means bone mineralization
is poor and bones are weakened

Page 118
128. Dx tests Rickets 137. Neonate:
- Chorioretinitis
- Hydrocephalus
- Multiple-ring
enhancing lesions
on CT

Rachitic rosy-like appearance on CXR of


the costochondral joints with cupping and
fraying of the epiphyses

Bowlegs
129. Tx Rickets Replacement of phosphate, calcium, and
vitamin D in the form of ergocalciferol or Toxoplasmosis
1,25 (OH)2, calcitrol, and annual blood - Toxoplasma gondii
vitamin D monitoring - Best initial test: elevated IgM to
130. AAP recommends 2 months of age toxoplasma
that infants who - Most accurate: PCR for toxoplasmosis
are exclusively - tx: Pyrimethamine and sulfadiazine
breastfed be 138. Tx toxoplasmosis Pyrimethamine and sulfadiazine
given vitamin D
139. Neonate:
supplementation
- Rash on palms and
beginning at ___
soles
of age.
- Snuffles
131. A child that has Sepsis - Frontal bossing
febrile seizure - CBC w/ differential blood and urine - Hutchinson eighth
what should the cultures nerve palsy
child be - Urinalysis - Saddle nose
evaluated for? - CXR
- LP (if irritability or lethargy is mentioned
= meningitis)
132. Sepsis tx in Ampicillin or Gentamicin
neonate.
133. Most common Group B strep
organisms E.coli Syphilis
causing sepsis in S. aureus - Best initial test: VDRL or RPR
neonate. Listeria monocytogenes - Most accurate: FTA ABS or dark field
microscopy
134. Dx test sepsis in Blood cultures and urine cultures
neonate. CXR
Tx: Penicillin
135. Neonatal sepsis
140. Neonate:
Early vs Late
- PDA
onset causes and
- Cataracts
tx.
- Deafness
-
Hepatosplenomegaly
- Thrombocytopenia
- Blueberry muffin
rash
- Hyperbilirubinemia
136. Name the TORCH Toxoplasmosis Rubella
infections. Other infections such as Syphilis - Maternal IgM status along with clinical
Rubella dx
CMV - Each disease manifestation must be
Herpes simplex virus individually addressed

Tx: Supportive

Page 119
141. Neonate: 146. Neonate:
- - Fever and URI
Periventricular progressing to
calcifications diffuse rash
with - HSV 6 and HSV
microcephaly 7
-
Chorioretinitis
- Hearing loss
- Petechiae CMV
Roseola
- Best initial test: urine or saliva viral titers
- Supportive tx
- Most accurate: urine or saliva PCR for
viral DNA 147. Neonate:
- Fever precedes
142. Tx CMV Ganciclovir with signs of end organ damage
classic parotid
143. Neonate: Herpes gland swelling
- Week 1: - Best initial test: Tzanck smear with possible
shock and DIC - Most accurate: PCR orchitis.
- Week 2:
vesicular skin Tx: Acyclovir and supportive care
lesions
- Week 3 :
encephalitis Mumps
- Paramyxovirus
144. Neonate: - Supportive tx
- Cough
- Coryza 148. __ diffuse Scarlet fever
- Conjunctivitis erythematous
- Grayish eruption that is
macule on concurrent with
buccal surface pharyngitis.
Cause by
erythrogenic
toxin made by
Rubeola or Measles Strep pyogenes
- Paramyxovirus and typically last
- Koplik spot 3-6 days.
- Most accurate: measles IgM antibodies
149. Pentad: Scarlet fever
- Tx: supportive
- Fever - clinical dx
145. Neonate: - Pharyngitis - elevated antistreptolysin O titer, ESR,
- Starts with - Sandpaper rash and CRP
fever and URI over trunk and
and extremities
progresses to - Strawberry
rash with tongue
"slapped - Cervical
cheek" lymphadenopathy
appearance. Dx?
150. Tx Scarlet fever Penicillin
Azithromycin
Cephalosporins

5th disease or erythema infectiosum


- Parvovirus B19
- tx: supportive

Page 120
151. 2-year-old b. Racemic epinephrine 154. Tx croup
brought in by - classic signs of coup
daycare provider - prevent asphyxiation and probable
for severe cough, tracheostomy, administer EPI do DEC
fever, and runny swelling
nose. The child's
cough sounds like
a bark and she's
in obvious
respiratory Mild sx: steroids
distress. Upon
physical Moderate and severe: racemic EPI
examination, the 155. 4-year-old child
child refuses to brought in by
lie flat. A chest daycare provider
X-ray shows a because he's
positive steeple extremely
sign. irritable and
What is the most refuses to eat. He
appropriate next refuses to lean
step in back, speaks in
management? muffled words,
looks extremely
a. Intubate
a. Intubate ill, and is
b. Racemic drooling. Chest
epinephrine radiograph shows
c. Empiric a positive thumb-
antibiotics print sign.
d. Acetaminophen What is the most
e. CT-scan of appropriate next
neck step in
152. Croup in Parainfluenza virus types 1 and 2 management?
infectious upper - RSV second most common cause a. Intubate
airway condition b. Racemic
characterized by epinephrine
severe c. Empiric
inflammation antibiotics
most commonly d. Acetaminophen
caused by ___. e. CT-scan of the
neck
153. Presentation:
- Barking cough 156. Child presents
- Coryza with:
- Inspiratory - Hot potato voice
stridor - Fever
- Difficulty - Drooling in the
breathing lying tripod position
down - Refusal to lie
- Hypoxia flat
(peripheral - Hx not being
Croup immunized
cyanosis and
- classic steeple sign, a narrowing of
accessory
the air column in the trachea
muscle use)
- clinical dx and aided by x-ray if
symptoms are mild Epiglottitis
- hypoxia on presentation - H. influenze type B
- Hot cherry-red epiglottis
- X-ray: thumb print sign

Page 121
157. Tx epiglottitis 1. Intubate the child in the OR 165. Infant Congenital hip dysplasia
2. Administer ceftriaxone for 7-10 days + Ortolani and - Pavlik harness tx
3. Rifampin must be given to all close Barlow maneuver
contacts "Click" and "Clunk"
in the hip
158. Whopping cough is Bordetella pertussis
a form of bronchitis 166. Age 2-8
caused by ___. Painful limp
X-ray shows joint
159. Presentation
effusion and
whooping cough
widening
name the 3 stages.

Legg-Calve-Perthes disease (avascular necrosis


1. Catarrhal stage: severe congestion of the femoral head)
and rhinorrhea - 14 days in duration - tx: Rest and NSAIDs
2. Paroxysmal stage: severe coughing
episodes with extreme gasp of air f/u: surgery on both hips, if one necroses,
(inspiratory whoop) followed by vomiting eventually other will too
- 14 to 30 days 167. Adolescent,
3. Convalescent stage: DEC frequency especially obese
of coughing patients
- 14 days Painful limp
160. Dx Whooping cough 1. Clx dx with whooping inspiration, Externally rotate
vomiting, and burst blood vessels in leg
eyes
2. Butterfly pattern on CXR
3. PCR for nasal secretions or
Bordetella pertussis toxin ELISA
161. Tx whooping cough 1. Erythromycin and azithromycin in
catarrhal stage, not in paroxysmal
2. Isolate the child, and macrolides must
be given to close contacts
3. DTaP vaccine has DEC incidence
162. Dx: Bronchitis Slipped capital femoral epiphysis
- Productive cough - inflammation of airways - Xray shows widening of joint space
lasting 7-10 days. - supportive tx - Tx: internal fixation with pinning
163. Dx: Pharyngitis 168. Which vitamin Vitamin A
- Cervical - inflammation of the pharynx and deficiency? - Pseudotumor cerebri
lymphadenopathy adjacent structures caused by group A - poor night vision - HyperPTH
- Petechiae beta hemolytic strep -
- Fever > 104 and - Rapid DNAse antigen detection test hypoparathyroidism
other URI sxs - oral Penicillin for 10 days or macrolides
169. Which vitamin Vitamin B1 (thiamine)
- Acute rheumatic for penicillin allergy
deficiency? - water soluble, therefore no toxicity
fever and
- Beriberi
glomerulonephritis
- Wernicke's
164. Dx: Diphtheria encephalopathy
- Gray highly - membranous inflammation of the
170. Which vitamin Vitamin B2 (Riboflavin)
vascular pharynx due to bacterial invasion by
deficiency? - water soluble, no toxicity
pseudomembranous Corynebacterium diphtheria
- Angular chelosis
plaques on the - cx of the superficial membrane
- Stomatitis
pharyngeal wall
- Glossitis
- Do not scrape Tx: Antitoxin

Page 122
171. Which vitamin deficiency? Vitamin B3 (Niacin)
- Diarrhea -water soluble, no toxicity
- Dermatitis
- Dementia
- Death
172. Burning feet syndrome which vitamin deficiency? Vitamin B5 (pantothenic acid)
-water soluble, no toxicity
173. What must be given with INH to prevent peripheral neuropahty? Vitamin B6 (pyridoxine)
- -water soluble, no toxicity
174. Which vitamin deficiency? Vitamin C
- Ecchymosis
- Bleeding gums
- Petechiae
175. Vitamin D toxicity findings. Hypercalcemia
Polyuria
Polydipsia
176. Which vitamin deficiency? Vitamin K
- INC PT/INR - toxicity is rare and an upper limit has not been established
- Sxs mild to severe bleeding
- Analogous to warfarin therapy

Page 123
MTB Preventive Medicine
MTB Q and A

1. Which cancer screening method lowest mortality the most? 11. When should patients be screening for DM?: HTN
A. Pap smear Hyperlipidemia
B. Colonoscopy
C. PSA * Screening for diabetes with fasting blood glucose (2
D. Mammography above age 40 measurements over 126 or HbA1c > 6.5%)
E. Mammography above age 50: E. Mammography above age 12. Hypertension Screening: All patients above 18 should have
50 their BP checked every office visit
2. Breast Cancer Screening: Mammography should be done 13. For adults the 2 most beneficial vaccines are?: 1. Influenza
starting at age 40 to 50 every 2 years, reduction in mortality is (either inactivated or live attenuated)
greatest above 50 2. Pneumococcus
14. Influenza and Pneumococcal Vaccine: - Chronic lung, heart,
Screening can stop at age 75. liver, kidney, and cancer conditions (including asthma)
- HIV/AIDs
No proven benefit to routine breast self exam - Immunocompromised (cancer/anatomic asplenia)
3. Which of the following is most likely to benefit an asx patient - Patients on steroids
with multiple first-degree relatives with breast cancer? - Patients with DM
A. Tamoxifen or raloxifene
B. BRCA testing * Do not use liver attenuated vaccines in patient over 50 or with
C. Aromatase inhibitors (anastrozole, letrazole) those with the above conditions
D. Dietary modifications (low fat, soy diet) 15. Influenza Vaccine Indications: Everyone yearly
E. HER-2/neu testing Healthcare workers
F. Estrogen and/or progesterone receptor testing: A. Pregnant patients
Tamoxifen and raloxifene
16. Pneumococcal vaccine Indications: Everyone > 65
- result in 50-60% reduction in breast cancer when compared to
Cochlear implant
placebo
CSF leaks
- benefit greatest in those with 2 first-degree relatives with
Alcoholics
breast cancer (mother or sister)
One vaccine above 65 only
- Single revaccination after 5 years if the patient is
*Aromatase inhibitors useful in preventing metastasis in those
immunocompromised or the first injection was prior to the age 65
with proven breast cancer
Tobacco smokers
4. Cervical Cancer Screening: 21, every three years until 65
17. Pneumococcal vaccine administration: First give 13 polyvalent

Pap every 3 years age 21-30


After 6-12 months, give 23 polyvalent
Pap + HVP testing every 5 years 30-65
18. Varicella-Zoster Vaccine: Everyone over 60.
ASCUS --> HPV + ---> Colposcopy - prevents post-herpetic neuralgia

5. HPV vaccine females age?: 11-26 19. Hepatitis A and B vaccines benefits which patients?: Children
Adults:
6. Colonoscopy Screening: Starts at 50, every 10 years
- Chronic liver disease
- Men who have sex with men or multiple sexual partners
If family diagnosed, then screen at 40 or 10 years before they
- Household contacts with hepatitis A or B
had the disease
- Injection drug users
7. Uses capsule endoscopy.: Detects small bowel bleeding
20. Hepatitis A vaccine indication.: Travelers to countries of high
- not a cancer screening method
endemicity
8. HNPCC - Lynch Syndrome: Colon cancer in 3 family members
21. Hepatitis B vaccine indication.: ESRD (dialysis)
and 2 generations . 1 family member having it prematurely
Healthcare workers
(before 50).
DM

Screening should begin at 25, done every 1-2 years. 22. Tetanus vaccine: Td (toxoid) every 10 years
One Tdap (tetanus with acellular pertussis) as one of the
9. Prostate Cancer Screening: No recommendation.
boosters
10. Cholesterol and LDL measurements is recommended for
healthy patients when?: 1. Men > 35 Tetanus immune globulin in those never vaccinated
2. Women > 45 Dirty wound: Booster after 5 years
Clean wound: Booster after 10 years
* Lipid screening all patients with DM, HTN, CAD, carotid
disease, peripheral vascular disease, aortic disease

Page 124
23. Meningococcal Vaccine: Routine at age 11
- Asplenia
- Terminal complement deficiency
- Military recruits
- Residents of college dormitories
- Travelers to Mecca or Medina in Saudi Arabia for Hajj (pilgrimage)
24. Osteoporosis Screening: DEXA scan at age 65
- hip fractures in elderly women has an extremely high risk of mortality
25. Abdominal Aortic Aneurysm Screening: All men > 65 with a smoking history screen once with US to exclude aneurysm

AAA repair > 5 cm

Screen 65-75 with FHx of AAA


26. Smoking Cessation: Most effective method is bupropion and varenicline.

Page 125
MTB Psychiatry
MTB Q and A

1. Defects in BOTH intellectual functioning (cognitive abilities) 13. For dx ADHD how long symptoms have to last and symptoms
AND social functioning, more common boys, highest usually appear before age?: present > 6 mo.
incidence in school-age children.: Intellectual disability before the age of 12
(formerly mental retardation)
2. IQ range 50-55 to 70. *sx may persist into adulthood
Degree of mental retardation? 14. Diagnosis ADHD requires?: Symptoms presents in at least 2
Level of functioning?: MILD areas - such as home and school
- Reaches sixth grade level of education
- Can work and live independently home: interrupt others, fidget in chairs, run or climb
- Needs help in difficult or stressful situations excessively, unable to engaged in leisure activities, talk
3. IQ range 30-40 to 50-55. excessively
Degree of mental retardation?
Level of functioning?: MODERATE school: unable to pay attention, make careless mistakes in
- Reaches second grade level of education schoolwork, do not follow though w/ instructions, difficulties w/
- May work with supervision and support organizing tasks, easily distracted
- Needs help in mildly stressful situation 15. First line tx ADHD: Methylphenidate
4. IQ range 20-25 to 35-40. Dextroamphetamine
Degree of mental retardation?
Level of functioning?: SEVERE - work well in reducing the symptoms of inattention and
- Little or no speech hyperactivity since affect noradrenergic and dopaminergic
- Very limited abilities to manage self-care pathways of attention
5. IQ range below 20. 16. Side effects of first line tx ADHD: Methylphenidate,
Degree of mental retardation? Dextroamphetamine
Level of functioning?: Profound
- Needs continuous care and supervision Insomnia
6. Tx Intellectual disability: - Genetic counseling, prenatal care, DEC appetite
safe environments for expectant mothers GI disturbances
- If due to med cond. -> effective tx for disorder INC anxiety
- Special education to improve level of functioning HA
- Behavioral therapy to help reduce negative behaviors 17. Second-line tx ADHD: Atomoxetine- NE reuptake inhibitor fewer
7. ___ characterized by problems in social interactions, SE and less risk of abuse
behavior, and language that tend to occur in children younger
than age < ___ and impair daily functioning.: Autism spectrum Clonidine, Guanfacine- alpha-2 agonists - enhance cognition
disorder and attention in prefrontal cortex
<3 18. TIP: ____ is usually chosen over the first-line tx for ADHD,
8. Deficits in autism spectrum disorder?: - Lack of social given the SE profiles of the other txs.: Atomoxetine
connection 19. This disorder is usually by age 8, boys > girls before puberty,
- Poor eye contact equal incidence after:
- Problems with language, relationships, understanding others - argue w/ others, lose temper easily, blame others for
- Stereotyped or repetitive movts mistakes
- Inflexibility - problems w/ authority figures (parents/teachers)
- Unusual interest in sensory aspects of the environment - behaviors during interactions w/ others not siblings:
9. Goal Tx autism spectrum disorders: - Improve pts ability to Oppositional defiant disorder
develop relationships, attend school, and achieve independent - teach parents child management skills and how to lessen
living oppositional behavior
20. This disorder seen more frequently in boys and in children
Behavioral modification programs that help w/ language and whose parents have antisocial personality disorder and
ability to connect w/ others alcohol dependence. Dx before age 18 years.: Conduct
10. If a pt has autism spectrum disorder and is aggressive which disorder
med can be used?: Antipsychotic meds such as Risperidone - rules are broken, aggression toward others bullying, cruelty to
animals, fighting, or using weapons
11. When should deafness be r/o in pt suspected to have autism
- destroy property via vandalism, fires
spectrum disorder?: If the parents report that a child does not
- steal from others, lie to get goods
respond when his or her name is called
- violate laws (truancy, running away, breaking curfew)
12. ADHD characterized by?: Inattention
21. Tx conduct disorder: Behavioral intervention using rewards for
Short attention span OR
prosocial and nonaggressive behavior
Hyperactivity that is severe enough to

if aggressive antipsychotic meds can be used


interfere w/ daily functioning in school, home, or work
Page 126
22. Seen in boys age 6-10, is not dx before 6 or after 18, children 33. SE SNRI: HTN
in adulthood develop depression or anxiety. Sweating
34. Which therapy modality has been proven to be effective tx
Features: chronic, severe, persistence irritability w/ temper MDD?: Cognitive therapy
outburst, angry, irritable, sad mood, everyday, out of - reduce depression by teaching pts to ID negative cognitions
proportion to situation: Disruptive mood dysregulation disorder and develop positive thinking
(DMDD) 35. SSRIs should not be taken with ___ as they will cause
- no period lasting > 3 consecutive months w/o all symptoms dramatic INC in serotonin.: MAOi
- severe enough to interfere with home, school, peers
36. What drug can be used for pt w/ depression and neuropathic
23. Tx disruptive mood dysregulation disorder (DMDD): Tx pain?: Duloxetine - approved for depression and neuropathy
individualized to the needs of the particular child and his/her
37. What drug can be used in a pt who presents w/ depression
family
who is fearful of wt gain or sexual SE of SSRI or is a smoker
- individual therapy, work w/ family or school
trying to quit?: Bupropion - fever sexual SE and less wt gain
- use of meds to tx symptoms
than SSRIs
24. Tourette disorder is characterized by the onset of multiple - may also be used as adjunct or replacement tx for SSRI-
tics, lasting more than > ___ before the onset of __. Head induced sexual SE
shaking, grunting, blinking, coughing, and throat clearing.: > - Approved for smoking cessation
1 year - SE: insomnia, agitation
before age 18
38. Pt presents with depressed mood and insomnia which drug
should be used?: Mirtazapine stim. NE and serotonin release-
more common boys begin by age 7
antidepressant and sedative SE
25. Tx Tourette disorder: Dopamine antagonists, antipsychotic
39. Bipolar disorder is a mood disorder where the pt experiences
meds:
manic symptoms that last at least ___ that cause significant
distress in the level of functioning.: at least 1 week
Haloperidol
Pimozide 40. Manic symptoms include?: Distractibility
Risperidone Irresponsibility (excessive invol. pleasurable activities)
Grandiosity (INC self-esteem)
* clonidine, alpha 2 agonist can also be used Flight of ideas (racing thoughts)
Activity (goal-directed)
26. Mood disorders in Major Depressive Disorder present with at
Sleep (decrease)
least __ course of symptoms.: 2 week that is a change from
Talkativeness (pressured speech)
previous level of functioning
27. Symptoms Major Depressive Disorder: Depressed mood OR * starts w/ depression and INC energy despite lack of sleep
anhedonia (absence of pleasure) + 4 others:
41. Dx Bipolar disorder make sure condition is not secondary to
?: Drug use -> cocaine or amphetamine
Sleep
Interest (loss)
obtain good hx and drug screen
Guilt (worthlessness)
Energy (lack) 42. Manic vs hypomanic symptoms?: Manic
Concentration (poor) - Last > 1 week
Appetite (wt changes) - Affect functioning
Psychomotor: agitation (anxiety) or retardation (lethargy) - Severe enough to warrant hospitalization
Suicidality
Hypomanic
28. R/o which diagnosis if suspect major depressive disorder?:
- Less than 1 week
HYPOthyroidism
- Do not severely affect functioning
29. Most common neuro associations w/ major depressive - Not severe enough to warrant hospitalization
disorder?: Parkinson dz
43. Bipolar disorder type I includes __ and ___.: Mania and
Neurocognitive disorders
depression
30. First line tx MDD.: SSRI (Fluoxetine, paroxetine, sertraline,
44. Bipolar disorder type II includes __ and ___.: Hypomania and
citalopram, escitalopram)
depression
- mild SE
- less toxic in OD than other antidepressants 45. In acute mania of bipolar depression use ___ as tx?: Lithium,
valproic acid, and atypical antipsychotics as first-line
31. How should SSRI's be managed in MDD tx?: - If NO effect after
4 weeks, switch to other SSRI 46. If acute mania w/ severe symptoms use which drugs?:
- If some improvement is noted, but not full response, INC dose atypical antipsychotics due to shorter onset of action
SSRI 47. If bipolar depression tx?: Lithium, quetiapine, lurasidone, or
- Although TCA can be used, their lethal potential thus not used lamotrigine
routinely 48. ___ is approved for bipolar depression and is the only
32. Second-line tx MDD.: SNRI (Venlafaxine, duloxetine, medication in Pregnancy Category B indicated for the
desvenlafaxine) disorder.: Lurasidone
Page 127
49. If kidneys are compromised do not use ___ tx bipolar 63. Tricyclic antidepressants (amitriptyline, nortryptyline,
disorder.: Lithium imipramine) SE?: Hypo/HTN
50. Persistent depressive disorder is characterized by the Dry mouth
presence of depressed mood that last most of the day and is Constipation
present almost continuously. Symptoms present more > ___.: Confusion
2 years (1 year in children/adolescents) Arrhythmias
51. Tx persistent depressive disorder.: Antidepressant meds and Sexual SE
psychotherapy Wt gain
GI disturbances
52. Cyclothymia is characterized by the presence of ___ and
mild___.: Hypomanic episodes and mild depression 64. MOAi (phenelzineu, isocaboxazid, tranylcapromine) SE?:
- sx > 2 years Monitor diet, given food rich tyramine = HTN
Safe foods white wine + processed cheese
53. Tx Cyclothymic disorder: Lithium, valproic acid or
antipsychotic meds, and psychotherapy
Unsafe foods: red wine, aged cheese, and chocolate :(
54. Atypical depression is characterized by ___ changes such as
65. SSRI (fluoxetine, paroxetine, sertaraline, citalopram,
INC sleep, INC weight, and INC appetite, and interpersonal
escitalopram, fluovoxamine) SE?: HA
rejection sensitivity that results in significant social or
Wt changes
occupational impairment.: Reverse vegetative changes
Sexual SE
- mood worse evening
GI disturbances
- "extremities feeling heavy"
66. SNRI (venlafaxine, duloxetine, desvenlafaxine) SE?: HTN
55. Tx Atypical depression: SSRI (fluoxetine, sertraline,
Blurry vision
paroxetine, citalopram, escitalopram)
Wt changes
Sexual SE
or MAOis (phenelzine, isocaboxazid, and tranylcypromine)
GI disturbances
56. Major Depressive Disorder with seasonal pattern is?:
67. Bupropion SE?: INC risk seizures
Seasonal changes in mood during fall and winter
- wt gain, INC sleep, lethargy 68. Trazodone SE?: Priapism
- tx: Phototherapy, Bupropion, SSRI 69. Mirtazapine SE?: Wt gain
Sedation
Phototherapy: 12-18 in. from source 10,000 lux white 70. Lithium SE?: Tremors
fluorescent light w/o UV wavelengths > 30 min. each morning, Wt gain
keep eyes open (don't stare at the light lol) GI disturbances
57. Immediately after birth up to 2 weeks, sadness, mood Nephrotoxic
lability, tearfulness, no negative feelings mother toward Teratogenic
child.: Postpartum blues "baby blues" Leukocytosis
- tx: supportive, usually self-limited DI
58. Depressive disorder with peripartum onset, w/in 1-3 months
after birth, depressed mood, weight changes, sleep Severe toxicity: confusion, ataxia, lethargy, abnormal reflexes
disturbances, and excessive anxiety. May have negative 71. Valproic acid SE?: Tremors
feelings toward baby.: Depressive disorder w/ peripartum onset Wt gain
- tx: antidepressant meds GI disturbances
59. During pregnancy up to 4 weeks after birth, depression, Alopecia
mania, hallucinations, delusions, and thought of harm, may Teratogenic
have thoughts to harming baby.: Bipolar disorder w/ peripartum Hepatotoxic
onset or brief psychotic disorder w/ peripartum onset
- tx: antipsychotic meds, lithium, and possibly antidepressants Monitor levels: toxicity cause HypoNatremia, coma, or death
60. How long does feeling of bereavement (grief) usually last 72. Lamotrigine SE?: SJS
after the death of a loved one and pt has feelings of sadness, 73. Electroconvulsive therapy SE?: HA, transient memory loss
worrying about deceased, irritability, sleep difficulties, poor - considered the best/single most effective tx for depression
concentration, and tearfulness?: Less than 6 mo. to 1 year, 74. Symptoms Serotonin Syndrome?: Cognitive: agitation,
can go on longer confusion, hallucination, hypomania
61. Tx Bereavement: Psychotherapy
- Pharm is the wrong answer! Autonomic: sweating, hyperthermia, tachy, nausea, diarrhea,
62. When to pick a diagnosis of major depression over shivering
bereavement?: - Thoughts of death
- Morbid preoccupation w/ worthlessness Somatic: tremors, myoclonus
- Marked psychomotor retardation 75. Tx Serotonin syndrome: Stop SSRI meds
- Psychosis
- Prolonged functional impairment Symptomatic tx of fever, diarrhea, HTN
- Symptoms last longer than 2 weeks and adversely affect
functioning Cyproheptadine (serotonin antagonist)
Page 128
76. More than 1 day but less than 1 month, delusions, 94. Onset of symptoms of hours to days with muscle spasms,
hallucinations, disorganized speech, grossly disorganized or such as torticollis, laryngeal spasms, occulogyric crisis.:
catatonic behavior.: Brief psychotic disorder Acute dystonia
- tx: antipsychotic meds 95. Tx acute dystonia: Benztropine
77. More than 1 mo. but less than 6 mo., delusions, Trihexyphenidyl
hallucinations, disorganized speech, grossly disorganized or Diphenhydramine
catatonic behavior, and negative symptoms (flat affect, poor 96. Weeks of generalized restlessness, pacing, rocking, inability
grooming, social withdrawal): Schizophreniform disorders to relax.: Akathisia
- tx: antipsychotic meds - reduce dose antipsychotic meds, beta blockers, switch to
78. More than 6 months of delusions, hallucinations, disorganized atypical meds, BZD
or catatonic behavior, and negative symptoms. severely 97. Rare before 6 mo. abnormal involuntary movt of head, limb,
affects level of functioning.: Schizophrenia and trunk. Perioral movt most common.: Tardive dyskinesia
- tx: antipsychotic meds - switch to atypical antipsychotic, clozapine least risk
79. Schizophrenia diagnosis symptoms must be present for ___.: 98. Muscular rigidity, fever, autonomic changes, agitation, and
6 mo. + affect functioning obtundation as SE of antipsychotic use.: Neuroleptic malignant
- impairs judgement, behavior, and ability to interpret reality syndrome
80. T/F: Schizophrenia is an equal incidence in men and women 99. Tx neuroleptic malignant syndrome (2).: Dantrolene or
but it affects women earlier due to earlier age of onset.: Bromocriptine
FALSE 100. Delusional disorder last ___ and no impairment in ____ .:
more than 1 mo. non-bizarre delusions
Affects MEN earlier due to age of onset
81. What screening test should be done if thinking schizophrenia No impairment of level of functioning (still obeys law, goes to
dx?: Urine tox screen work, and pays bills even if they believe the country is about to
- r/o cocaine or amphetamine use be invaded)
82. Tx Schizophrenia: Hospitalize pts who are acutely psychotic 101. Tx delusional disorder: Atypical antipsychotic

Ensure pt safety use atypical antipsychotic as first-line agent Psychotherapy to promote reality testing
(risperidone, olanzapine, quetiapine, ziprasidone, aripirazole, 102. ___ is the experience of intense anxiety along w/ feelings of
paliperidone, asenapine, iloperidone, or lurasidone) dread and doom.: Panic disorder
83. In an emergency situation when IM medication is needed in 103. Panic disorder requires at least which 4 symptoms of
schizophrenia pt use?: Short-acting meds such as olanzapine autonomic hyperactivity?: Diaphoresis
or ziprazidone Trembling
Chest pain
Haloperidol still used, but has more SE, so if given the choice Fear of dying
pick atypical Chills
84. If noncompliant with schizophrenia meds consider?: Long- Palpitations
acting antipsychotics such as risperidone or paliperidone as SOB
first line
85. __ is used only when patients do NOT respond to an adequate - Nausea, dizziness, dissociative symptoms, paresthesias
trial of typical or atypical antiphycotics for tx schizophrenia; 104. Panic disorder symptoms duration?: last less than 30 min
never use as first line tx.: Clozapine may be accompanied by agoraphobia (fear of places where
86. Olanzapine SE: Greater incidence of DM + wt gains escape is felt to be difficult)

avoid in diabetic and obese pts - typically in women, occur anytime, no specific stressor
87. Risperidone SE: Greater incidence of movt disorders 105. Which disorders should be r/o if suspect panic disorder?:
88. Quetiapine SE: Lower incidence of movt disorders; appropriate Thyroid dz
use in pts w/ existing movt disorders Hypoglycemia
89. Ziprasidone SE: INC risk of prolongation of QT interval; avoid Cardiac dz
in pts with conduction defects 106. Tx Panic disorder?: SSRI (fluoxetine, paroxetine, sertraline)
90. Clozapine SE: High risk of agranulocytosis, need to monitor Pts may benefit from BZDs (alprazolam, clonazepam, or
CBC on reg, never use as first-line tx given SE profile lorazepam)
- Begin w/ both, then taper, and D/C BZD given the potential for
91. Aripiprazole SE: Partial dopamine agonist, approved as adjunct
abuse
tx for major depressive disorder
92. Lurasidone SE: Only antipsychotic in Pregnancy category B * behavioral and individual therapy are helpful in in conjunction
safer use in pregnant pts w/ meds (not as sole tx)
93. ___ are the least likely antipsychotics to cause wt gain, 107. If a single panic attack is the diagnosis what is the tx?: BZD
diabetes, and metabolic syndromes.: Aripiprazole
108. Fear of an object, such as animals, heights, or cars.:
Ziprasidone
Specific phobia
Page 129
109.Fear of a situation, such as public restrooms, eating in 123.SE BZDs (diazepam, lorazepam, clonazepam, alprazolam,
public, or public speaking. These involve situations where oxazepam, chlordiazepoxide, temazepam, flurazepam):
something potentially embarrassing may happen.: Social Sedation
phobia Confusion
110.Tx phobias: - Behavioral modification (systematic Memory deficits
desensitization) Respiratory depression
- Relaxation techniques (breathing, guided imagery) INC addiction potential
111.___ are used only for performance anxiety such as stage 124.SE Buspirone: HA
fright. They are given 30-60 minutes before the performance.: Nausea
B-blockers (atenolol or propranolol) Dizziness
112.Obsessions vs Compulsions?: Obsessions -> Thoughts that 125.Lorazepam specific use in anxiety?: Emergency situations
are intrusive, senseless, and distressing to the pt, INC anxiety, can be given IM
fear of contamination 126.Clonazepam specific use in anxiety?: May be used if
addiction is a concern given its longer-half life
Compulsions -> Rituals, counting, checking, performed to 127.Chlordiazepoxide, oxazepam, lorazepam specific use in
neutralize obsessions, time consuming and tend to lower anxiety?: Tx of alcohol withdrawal
anxiety
113.OCD can coexist with ___.: Tourette disorder - Lorazepam + Oxazepam drug of choice in pts w/ liver
- OCD more frequently seen young pts problems
114.Tx OCD: SSRI tx of choice (fluoxetine, paroxetine, sertraline, 128.Alprazolam specific use in anxiety?: Panic attacks and panic
citalopram, fluvoxamine) disorder
129.Flurazepam, temazepam, triazolam use in anxiety?:
Behavioral therapy - > exposure and response prevention Approved as hypnotics (rarely used)
115. Tx Hoarding disorder: SSRI tx of choice 130.____ is a BZD antagonist used only when the overdose is
acute and you are certain there is no chronic dependence.:
Pts benefit from behavioral modification techniques or Flumazenil
psychotherapy (CBT) 131.Flumaxenil can cause ___ in BZD dependent pts.: SEIZURES
116.Tx choice body dysmorphic disorder.: SSRI combined w/ - can cause acute withdrawal, which can be tremor or seizure
individual psychotherapy similar to delirium tremens (alcohol withdrawal)
117. PTSD vs acute stress disorder.: PSTD - symptoms > 1 mo. 132.Pt presents talkative, sullen, gregarious, moody,
disinhibited tx of intoxication?: Mechanical ventilation if severe
Acute stress disorder - symptoms > 2 days and max 1 mo., 133.Pt presents with tremors, hallucinations, seizures, delirium
occur w/in 1 mo. of traumatic event tremors tx?: BZD
118.What should be r/o in dx of PTSD or acute stress disorder?: - thiamine, multivitamins, folic acid
Depression 134.Pt presents with euphoria, hypervigilance, autonomic
Substance abuse hyperactivity, wt loss, pupillary dilation, perceptual
119. Tx PSTD or acute stress disorder.: Paroxetine and Sertraline disturbances tx?: Intoxication amphetamines + cocaine
first line, Prazosin used to reduce the incidence of nightmares (synthetic forms "bath salts")

Relaxation techniques and hypnosis tx: Antipsychotics and/or BZDs and/or antihypertensives
135.Pt presents with anxiety, tremulousness, HA, INC appetite,
Psychotherapy after traumatic events allow for coping depression, risk of suicide withdrawal from which drugs?
techniques and acceptance of event Tx.: Withdrawal amphetamines + cocaine (synthetic forms "bath
120.Generalized anxiety disorder when pts experience excessive salts")
anxiety and worry about most things, lasting more than ___.:
6 mo. tx: Bupropion and/or bromocriptine
- typically anxiety is out of proportion to the event 136.Pt presents with impaired motor coordination, slowed sense
- women c/o feeling anxiety as long as they can remember of time, social withdrawal, INC appetite, conjunctival
121.GAD accompanying symptoms?: Fatigue injections, tx?: Cannabis (synthetic form K2 and spice)
Difficulty concentrating
Sleep problems tx: consider use antipsychotic is pt is psychotic
Muscle tension 137.Pt presents with irritability, anger, anxiety, sleep problems,
Restlessness restlessness, appetite problems withdrawal from and tx?:
122. Tx GAD: SSRI (fluoxetine, paroxetine, sertraline, citalopram) Cannabis (synthetic form K2 and spice)

Venlafaxine and buspirone also effective tx: symptomatic

Psychotherapy and behavioral therapy are beneficial, not


considered first line agents in most pts
Page 130
138.Pt presents with ideas of reference, perceptual 150.Pts believe that they have some specific disease despite
disturbances, possible increased psychosis, impaired constant reassurance.: Illness anxiety disorder (formerly
judgement, tremors, incoordination, dissociative symptoms hypochondriasis)
intoxication tx?: Hallucinogens (LSD, Psilocybin (shrooms)) 151.Typically affects voluntary motor or sensory functions that
are indicative of medical condition but are usually caused by
tx: antipsychotics and/or BZDs and/or talking down psychological factors. Can be associated with "la belle
139.Pt presents with belligerence, apathy, aggression, impaired indifference", where the patient is unconcerned about his or
judgement, stupor, or coma after intoxication w/inhalants tx?: her impairment.: CONVERSION
Antipsychotics 152.When a caretaker fakes signs and symptoms in another
140.Pt presents with apathy, dysphoria, pupillary constriction, person (usually a child) in order to assume the sick role, the
drowsiness, slurred speech, coma, or death tx?: Opiates dx is ____.: Factitious disorder imposed onto others
(synthetic desomorphin, krokodil) 153.When signs and symptoms are faked in oneself, may inflict
life-threatening injuries on themselves in order to get
tx: Naloxone attention?: Factitious disorder imposed onto self
141.Pt presents with fever, chills, lacrimation, abdominal 154.Dx factitious disorder: Typically, women who have a hx of
cramps, muscle spasm, diarrhea tx?: Opiates (synthetic being employed in healthcare
desomorphin, krokodil) withdrawal
Men have physical symptoms
tx: Clonidine, methadone, or buprenorphine
142.Pt presents with belligerence, psychomotor agitation, pts ultimate goal gain admission into hospital
violence, agitation, violence, nystagmus, HTN, seizures tx?: - must r/o that there is no legitimate medical illness before dx
Intoxication: PCP (phencyclidine) factitious disorder
155.Tx factitious disorder: No specific therapy have been proven
tx: Antipsychotics and/or BZDs and/or talking down to be effective
143.Pt presents with irritability, aggression, mania, psychosis
after use anabolic steroids tx intoxication?: Antipsychotics - when child is involved, CPS should be contacted
144.Pt presents with withdrawal anabolic steroids w/ 156.___ characterized by conscious production of signs and
depression, HA, anxiety, INC concern over body's physical symptoms for obvious gain, such as avoiding work, evading
state tx?: SSRIs criminal prosecution, or achieving financial gain. Not a
145.If suspect someone is an alcoholic of the CAGE test how mental illness.: Malingering
many responses must be positive and indicate further 157.Diagnosis/Management Malingering: Frequently seen military
assessment is warranted?: + 2 out of 4 questions or prisoners
- Diagnosed when there is a discrepancy btw the pt's complaints
C: Have you ever tried to cut down on your drinking? and the actual physical or lab findings
A: Have you ever gotten annoyed by others who have criticized - Lack of cooperation from pt is characteristic of malingering
your drinking
G: Have you ever felt guilty about your drinking? * If medical eval reveals malingering, then confront the pt w/ the
E: Have you ever used alcohol as an eye-opener? outcome
146.Tx substance related disorders?: Detoxification: usually 5-10 158.___ characterized by maladaptive reaction to an identifiable
days, mostly in hospital setting to assure safe detox stressor such as loss of job, divorce, or failure in school.
Symptoms usually occur 3 mo. of the stressor and remit w/in
Rehab: usually 28 days or more, with a focus on relapse 6 mo. of removal of the stressor.: Adjustment disorder
prevention techniques - anxiety, depression, disturbances of conduct
- AA, Narcotic anonymous 159.Tx of choice adjustment disorder: Psychotherapy
147.Pharm tx for substance related disorder detox/rehab?: - both individual and group therapy have been used
Disulfiram (acetaldehyde dehydrogenase inhibitor) - reduce 160. Which personality disorder?
desire to drink
Suspicious, mistrustful, secretive, isolated, and questioning
Naltrexone (opioid receptor antagonist) - help prevent relapse of the loyalty of family and friends.: Paranoid
alcohol or drug abuse 161. Which personality disorder?

Acamprosate - reduce desire to drink Choice of solitary activities, lack of close friends, emotional
148.Characteristics of somatic symptom disorder?: Presence of coldness, no desire for or enjoyment of close relationships.:
1or more somatic symptoms that are distressing and cause Schizoid
impairment of functioning 162. Which personality disorder?
- pt has excessive thoughts, feelings, or behaviors related to
somatic symptoms Ideas of reference, magical thinking, odd thinking, eccentric
behavior, INC social anxiety, brief psychotic episodes.:
> 6 mo. most commonly young woman Schizotypal
149. Tx Somatic symptom disorder: Psychotherapy
Page 131
163. Which personality disorder? 175.___ characterized by frequent binge eating, lack of control
over eating episodes, compensatory behavior of purging,
Must be center of attention, inappropriate sexual behavior, misuse of laxatives and diuretics, fasting, and excessive
self-dramatization, use physical appearance to draw attention exercise.: Bulimia nervosa
to self.: Histrionic - later in adolescence, normal wt, hx of obesity
164. Which personality disorder? 176.Tx Bulimia nervosa: Does not require hospitalization unless
severe electrolyte abnormality
Failure to confront to social rules, deceitful, lack of remorse, Psychotherapy
impulsive, aggressive toward others, irresponsible, must be SSRI
over 18.: Antisocial 177.Features of binge eating disorder: Recurrent episodes of
165. Which personality disorder? binge eating that occur at least 3 times per week for more than
3 mo.
Unsubtle relationships, impulsive, recurrent suicidal
behavior, chronic feelings of emptiness, inappropriate anger, Overweight pts, lack sense of control over eating
dissociative symptoms when severly stressed, brief
psychotic episodes.: Borderline Eat faster than usual during episodes, eating until
166. Which personality disorder? uncomfortably full, eating large amount of food in the absence
of hunger, eating alone, and feeling disgusted w/ oneself post
Grandiose sense of self, belief they are special, lack eating episode
empathy, sense of entitlement, require excessive 178. Tx Binge eating disorder: Topiramate proven effective, SSRI
admiration.: Narcissistic limited benefit
167. Which personality disorder?
Psychotherapy indicated, CBT, interpersonal psychotherapy,
Unwilling to get involved with people, views self as socially and dialectic behavioral therapy
inept, reluctant to take risks, feelings of inadequacy.: 179.When is designation of eating disorder not otherwise
Avoidant specified (NOS) used?: When pts do not meet criteria either
168. Which personality disorder? anorexia nervosa or bulimia nervosa

Difficulty making day-to-day decisions, unable to assume Examples:


responsibility, unable to express disagreement, fear of being - criteria for anorexia presents in girls but menstruation normal
alone, seeks relationships as source of care.: Dependent - anorexia pt w/ normal wt
169. Which personality disorder? - use of compensatory behavior after eating normal amount of
food
Preoccupied with details, rigid, orderly, perfectionistic, 180.Characterized by excessive daytime sleepiness and
excessively devoted to work, inflexible.: Obsessive abnormalities in REM sleep, begins young adulthood.:
compulsive Narcolepsy
170.Tx personality disorders?: Psychotherapy - sleep studies are indicated for dx
- meds if mood or anxiety symptoms are present 181.Tx narcolepsy: No therapy curative
171.Which personality disorder may have short-lived psychotic - Forced naps during the day
symptoms that are brief and usually occur after stressful - Modafinil: used to maintain alertness
situations?: Borderline + Schizotypal - Methylphenidate + dextroamphetiamine
- Gamma-hydroxybutyrate (GHB) given bedtime induce
172.Anorexia nervosa signs.: Severe wt loss
symptoms of narcolepsy and contain them at night
Hypotension
Bradycardia 182.Psychiatric and physical symptoms of narcolepsy.: 1. Sleep
Lanugo hair attacks: episodes of irresistible sleepiness and feeling
Edema refreshed upon awakening
EKG: rhythm changes K+ deficiency 2. Cataplexy: sudden loss of muscle tone, considered
pathognomic and may ppt by loud noise or emotions
- teenage girls age 14-18 3. Hypnogogic and hypnopompic hallucinations: hallucinations
that occur as the pt is going to sleep and waking up
173. Most common cause of death anorexia nervosa?: Arrhythmia
4. Sleep paralysis: patient awake but unable to move, occurs
174.Tx Anorexia nervosa: Hospitalization to prevent dehydration, upon awakening
starvation, electrolyte imbalance and death
183.Polysomnogram in sleep apnea show?: Episodes of apnea
lasting more than 10 sec accompanied by DEC arterial
Psychotherapy, behavioral therapy
oxygenation, bradycardia, and INC diaphragmatic effort

SSRI have been used to promote wt gain 184.Medical complications sleep apnea?: Arrthymias
Pulmonary HTN
Occasionally death

Page 132
185.Tx Sleep apnea: Nasal continous positive airway pressure 203.Risk factors Suicide: Men
(CPAP) Older adults
Wt loss Social isolation
Corrective surgery Presents of psychiatric illness/drug abuse
Avoidance of sedatives and alcohol, which worsen the condition Perceived hopelessness
186.Tx insomnia: Sleep hygiene techniques such as going to bed Previous attempts
and waking up at the same time 204.Tx suicide: hospitalize pt
take all threats seriously
Avoiding caffeine

Avoiding daytime naps

Use bed only for sleep, not for reading, watching TV


187.Medical tx insomnia: Zoldipem
Eszopiclone
Zaleplon
188.Gender identity usually established by the age of ___.: 3
- based on person's sense of maleness or femaleness
189.Sexual identity is based on?: person's secondary sexual
characteristics
190.Gender role defined as?: Based on external patterns of
behavior that reflex inner sense of gender identity
191.Tx impotence: R/o medical cause or medication,
psychotherapy, couples sex therapy
- persistent or recurrent inability to attain or maintain an erection
until completion of the sexual act
192.Tx premature ejaculation: Psychotherapy, behavioral
modification techniques, SSRI
193.Tx genitopelvic pain disorder (formerly dyspareunia) and
penetration disorder (vaginismus)?: Psychotherapy

+ dilator therapy for penetration disorder


194.These disorders focus on humiliation, non consenting
partners, or use of nonliving objects. More frequent in men,
recurrent, sexually arousing must occur more than 6 mo.
cause distress, do not dx if done in experimentation.:
Paraphilic disorders
195. Recurrent urge to expose oneself to strangers: Exhibitionism
196.Recurrent use of nonliving objects to achieve sexual
pleasure: Fetishism
197.Recurrent urge or behavior involving the act of humiliation:
Masochism
198.Recurrent urge or behavior involving acts which physical or
psychological suffering of victim is exciting.: Sadism
199.Recurrent urge or behavior involving cross dressing for
sexual gratification, usually found heterosexual males.:
Transvestic fetishism
200.Rubbing, usually one's pelvis or erect penis, against a non
consenting person for sexual gratification: Frotteurism
201. Tx paraphilias: Individual psychotherapy

Behavioral modification techniques such as aversion


conditioning

Antiandrogens or SSRI to reduce sexual drive


202.Disorder characterized by persistent discomfort and sense
of inappropriateness regarding pts assigned sex.: Gender
dysphoria
tx: sexual reassignment surgery, individual psychotherapy
Page 133
MTB Radiology
MTB Q and A

1. ___ best initial test for pulmonary complaints.: CXR 12. ___ is the best initial test for osteomyelitis.: X-ray of the
- Cough, SOB, CP (pleuritic), sputum, hemoptysis bone
- elevation of the periosteum
PE: rales, rhonchi, wheezing, dullness to percussion, superior - takes 2 weeks to show abd with osteomyelitis
vena cava syndrome - get MRI or nuclear bone scan if x-ray does not show
2. ___ films are the standard of care when a CXR is done, patient osteomyelitis
able to stand up.: PA films 13. T/F: Skull x-rays are rarely correct for any question.: TRUE
3. __ films of unstable patients too sick to stand up, usually ICU 14. Non-contrast head CT is best initial test for?: 1. Severe head
patients with respiratory complaints.: AP films trauma (LOC or altered mental status)
4. ___ x-rays are done to evaluate pleural effusions found on PA 2. Stroke
films.: 3. Any form of intracranial bleeding including subarachnoid
hemorrhage
15. Contrast head CT is best initial test for?: Cancer and infection
will enhance with contrast

* cannot distinguish between neoplastic disease and abscess


by CT or MRI, but the head CT with contrast is the best initial
test for any form of intracranial mass lesion
16. Do not order contrast with severe ___.: Renal failure
17. What should be given to a patient with mild renal
insufficiency that is getting contrast?: Hydrate with saline and
possibly use bicarbonate or N-acetylcysteine
18. Stop __ prior to using contrast.: Metformin
19. Abdominal CT should be performed with both?: IV and oral
contrast
- oral outlines abdominal structure that are pressed against
each other
20. Abdominal CT is good for?: 1. Retroperitoneal structures
Decubitus films (pancreas since sonogram cannot see the pancreas through the
- patient on the side effusion in the chest is confirmed when abdominal wall)
freely mobile and forms a layer on the side of the x-ray 2. Appendicitis and other intraabdominal infections
5. ___ is the best initial test of a dissection of the thoracic 3. Most accurate test for nephrolithiasis
aorta.: Widening of the mediastinum on PA film 4. Masses w/in abdominal organs such as liver and spleen
5. Diverticulitis
6. When apical lordotic films might be done, ___ of the chest is
generally the best initial study.: CT scan of the chest 21. Most accurate test for nephrolithiasis.: Abdominal CT
- Apical lordotic films used to be initial test for TB but now no 22. ___ is the most accurate test for diverticulitis.: Abdominal CT
longer 23. When is chest CT the answer on the test?: 1. Hilar nodes such
7. ___ is done to help ID the precise location of an infiltrate as sarcoidosis
found on a PA film.: Lateral CXR 2. Mass lesions such as cancer
8. Best initial test for an effusion since they detect as little as 50 3. Cavities
to 75 mL of effusion.: Lateral CXR 4. Interstitial disease: parenchymal lung disease
- PA chest shows an effusion only if 200 to 300 mL of fluid 5. PE: spiral CT or CT angiogram
accumulates 24. ___ is the most accurate test for all CNS diseases with the
9. Best indication for abdominal x-ray.: Ileus - multiple air-fluid exception of looking for hemorrhage.: MRI
levels in the small bowel - contrast detects cancer and infections
Small bowel obstruction 25. MRI is the answer on the test?: 1. Demyelinating diseases
10. Abdominal x-rays are not accurate which cases?: Not such as MS
accurate for stones of the kidney ~ miss 20% stones 2. Posterior fossa lesions in the cerebellum
3. Brainstem
Not reliably find air under diaphragm because it does not always 4. Pituitary lesions
visualize the top of the diaphragm especially in a tall person 5. Facial structures such as the orbits and sinuses
6. Bone lesions, particularly osteomyelitis
11. For perforation of the bowel, get a ___ x-ray.: upright chest x-
7. Spinal cord and vertebral lesions
ray not abdominal x-ray
26. T/F: With cancer and infection the radiologic test is never the
most accurate test: biopsy is.: TRUE

Page 134
27. When is ultrasound the answer?: 1. Gallbladder disease, 32. ___ another test for fever of unknown origin. Tagged WBC
including the ducts for stones and obstruction scan used to detect infection.:
2. Renal disease, although CT is more sensitive for
nephrolithiasis
3. Gynecologic organs: uterus, ovaries, adnexa
4. Prostate evaluation (transrectal approach)
28. Endoscopic ultrasound (EUS) is the most accurate method in
assessing?:

Indium scan
- patient's WBCs tagged with indium
33. Low-probability scans with V/Q scanning still have a clot in
1. Pancreatic lesions, particularly in the head __ % of cases and high-probability scans do not have a clot in
2. Pancreatic and biliary duct disease __ % cases.: 15%
3. Gastrinoma localization (Zollinger-Ellison syndrome) - V/Q no longer the standard of care in detecting PE, spiral CT
29. ____ is the only functional test of the biliary system that (CT angiogram) is confirmation of PE
allows detection of cholecystitis.: 34. ___ is the most accurate method to measure ejection
fraction.: Multi-gated acquisition scan (MUGA) or nuclear
ventriculography

HIDA (hepatobiliary) scan


30. ___ is good as a sensitive test to detect occult metastases
from cancer.: Bone scan
31. ___ scan for a fever of unknown origin.:

Gallium scan
- follows iron metabolism and is transported on transferrin
- gallium INC uptake w/ infection and in some cancers because
of INC iron deposition

Page 135
MTB Rheumatology
MTB Q and A

1. ___ is a chronic, slowly progressive, erosive damage to joint 8. Etiology Gout: 90% of cases in men
surfaces; this loss of articular cartilage causes INC pain with Overproduction:
minimal or absent inflammation.: Osteoarthritis - Idiopathic
2. The incidence of degenerative joint disease is directly - INC turnover of cells (cancer, hemolysis, psoriasis, chemo)
proportional to INC ___ and ___ to the joint.: INC age and - Enzyme deficiency (Lesch-Nyhan syndrome, glycogen
trauma storage disease)
3. TRUE/FALSE: Obesity increases degenerative joint disease.:
TRUE Underexcretion:
- Renal insufficiency
4. Osteoarthritis is most commonly symptomatic in ___ joints.:
- Ketoacidosis or lactic acidosis
- Thiazides and aspirin
9. Common presentation gout.: Man develops sudden,
excruciating pain, redness and tenderness of the big toe at
night after binge drinking with beer

Fever is common
10. Composition of Tophi: Deposits of urate crystals with foreign
body reaction.

- Weight-bearing joints (knee, hip, ankle) From longstanding gout form in cartilage, SQ tissues, bone, and
- Hand DIP most often kidney.
- Crepitations 11. Chronic gout findings.: - Tophi
- Stiffness short duration < 15 min. - Uric acid kidney stones: 5% to 10% patients
- Long asx periods between attacks are common
DIP enlargement: Heberden nodes 12. Most accurate diagnostic test for gout:
PIP enlargement: Bouchard nodes
5. Lab test for Osteoarthritis.: Lab tests are normal
6. The most accurate test for osteoarthritis.:

Arthrocentesis (esp. to rule out infectious pathology)


- aspiration of the joint shows needle-shaped crystals with
negative birefringence on polarized light

WBC joint fluid INC: 2000-50,000/microL (neutrophils)

X-ray of the affected joint


- Joint space narrowing
- Dense subchondral bone
- Bone cysts
7. Tx Osteoarthritis.: 1. Weight loss and moderate exercise
2. Acetaminophen: best initial analgesic
3. NSAIDs: used if symptoms aren't controlled with
acetaminophen; toxicity - GI bleeding
4. Capsaicin cream
5. Intra-articular steroids if other medical tx does not control
pain
6. Hyaluronan injection into join
7. Joint replacement if disease is severe

Page 136
13. Dx lab test or imaging in gout.: 22. Risk factors for Pseudogout.: MC Rist factors:
- Hemochromatosis
- HyperPTH

Also associated with:


- DM, hypothyroidism, Wilson disease
23. CPPD (Pseudogout) presentation.: - CPDD differs from gout in
that large joints such as the knee and wrist are affected, but
not particularly the first MCP of the foot
- It differs from DJD in that DIP and PIP aren't affected
24. Lab tests pseudogout.:

- Uric acid levels: elevated at some point in 95% of patients


- Single level during an acute attack normal in 25%
- Acute attacks = elevated ESR and leukocytosis

X-rays:
• Normal in early disease
• Erosions of cortical bone happen later
14. Acute attack gout treatment.: 1. NSAIDs superior to • Uric acid levels normal
colchicine as "best initial therapy" • X-ray: calcification of cartilaginous structure
2. Corticosteroids injection: single joint, oral: multiple joints.
Steroids (e.g., triamcinolone) is answer when: Synovial fluid: WBC 2000-50,000/microL
- No response to NSAIDs 25. Athrocentesis revealing positively birefringent crystals in
- Contraindication to NSAIDs such as renal ________________ disease.:
insufficiency
3. Colchicine is used in those who cannot use either NSAIDs or
steroids.
15. Rx for Hypertension in gout patient: Losartan (ARB) -also
lowers uric acid; STOP thiazides
16. Colchicine toxicity: Diarrhea and bone marrow suppression
(neutropenia)
17. Chronic management of gout.: 1. Diet
- DEC consumption of alcohol, particularly beer; lose weight
- DEC high-purine foods (meat and seafood)
CPPD disease (rhomboid shaped crystals)
2. Stop thiazides, aspirin, and niacin
3. Colchicine is effective at preventing second attack of gout. 26. Best initial drug for CPPD (Pseudogout) disease.: 1. Best
Prevent attacks brought on my use of probenecid and initial therapy: NSAIDs
allopurinol. 2. Severe disease not responsive to NSAIDs give intra-
4. Allopurinol decreases production of uric acid. Febuxostat is articular steroids (e.g., triamcinolone)
used if allopurinol is contraindicated. Febuxistat is xanthine 3. Colchicine helps prevent subsequent attacks as prophylaxis
oxidase inhibitor. between attacks
5. Pefloticase dissolves uric acid. 27. Dx:
6. Probenecid and sulfinpyrazone INC excretion of uric acid in Older, slow, worse with use.
the kidney (uricosuric). Rarely used. DIP, PIP, and knees
18. Probenecid, NSAIDs, and sulfinpyrazone are contraindicated Synovial fluid: < 200 WBCs, osteophytes, and joint space
in renal insufficiency. ___ is safe with renal injury.: narrowing: DJD
Allopurinol 28. Dx:
19. Adverse effects of chronic treatment of gout.: Men, acute, binge eating.
Hypersensitivity (rash, hemolysis, allergic intestinal nephritis) 1st big toe.
occurs with uricosuric agents and allopurinol
Synovial fluid: 2000-50,000 WBC, negatively birefringent
20. Allopurinol major toxicity.: Stevens-Johnson rash >> Toxic
needles.: Gout
epidermal necrolysis
21. Pseudogout is deposition of?: Calcium pyrophosphate from
calcium-containing salts depositing in articular cartilage

Page 137
29. Dx: 42. Most accurate test for back pain with compression or
Hemochromatosis, HyperPTH infection suspected.:
Wrists and knees

Synovial fluid: 2000-50,000 WBC, positively birefringent


needles.: CPPD (Pseudogout)
30. Dx:
Young, female, morning stiffness better with use
Multiple joints of hands and feet

Synovial fluid: anti-cyclic citrulinated peptide (anti-CCP) in


blood 10,000-20,000 WBC or fluid: Rheumatoid arthritis
31. Dx:
High fever, very acute.
Single hot joint.

Synovial fluid: > 50,000 neutrophils, culture of fluid.: Septic


MRI
arthritis
32. Patient has lumbosacral strain what is the tx.: No imaging - do CT myelogram (intrathecal contrast must be given to INC
studies, no treatment beyond NSAIDs accuracy) if MRI is contraindicated (pacemaker)
33. Signs of cord compression with high fever and elevated ESR 43. MRI indications in case of pain upon straight leg raise.: Do
in _______ pathology.: S. aureus epidural abscess MRI if accompanied by neuro deficits (paralysis, weakness)
34. Management of patient with a Hx of cancer presenting with
sudden onset neurologic deficit (sensory level).: Suspect cord "no MRI" for just LBP and positive SLR alone
compression (neurologic emergency) 44. Most likely diagnosis of back pain:
- hyperreflexia found below the level of compression
35. Sensitive clinical sign of cord compression.: Point tenderness Hx of cancer.
over vertebra. Vertebral tenderness, sensory level, hyperreflexia.: Cord
36. Pain upon straight leg raise clinical significance.: Disk compression
herniation - 50% chance only (can be present for other reasons 45. Most likely diagnosis of back pain:
too)
Fever, high ESR.
* SLR is pain going down the buttock and below the knee when Vertebral tenderness, sensory level, hyperreflexia.: Epidural
the leg is raised above 60 degrees. abscess
37. No pain upon straight leg raise clinical significance: Disk 46. Most likely diagnosis of back pain:
herniation excluded - high sensitivity of 95% (negative PPV)
38. Dorsiflexion of foot affected in ____________ nerve root Bowel and bladder incontinence, ED
compression: L4 B/L leg weakness, saddle area anesthesia.: Cauda equina
- Knee jerk reflex affected. 47. Most likely diagnosis of back pain:
- Inner calf sensory area affected.
39. Dorsiflexion of toe affected in ___________ nerve root Under age of 40, pain worsens with rest and improves with
compression.: L5 activity
- Inner forefoot sensory affected. DEC chest mobility: Ankylosing spondylitis
40. Eversion of foot affected in ______________ nerve root 48. Most likely diagnosis of back pain:
compression: S1
- Ankle jerk reflex affected. Pain/numbness of medial calf or foot.
- Outer foot sensory affected. Loss of knee and ankle reflexes, positive straight leg raise.:
41. Best initial test for back pain with compression, infection Disk herniation
suspected, and fractures.: Plain x-ray 49. Rx for Cord compression: Systemic glucocorticoids, chemo
for lymphoma, radiation for many solid tumors.

Surgical decompression if steroids and radiation are not


effective.
50. Initial treatment for epidural abscess: Steroids to control
acute neurologic deficits

Vancomycin/Linezolid until sensitivity is found; then switch to


oxacillin/nafcillin + Gentamicin >> surgical drainage (if large
collection of infected material to cause neuro defects or does
not respond to abx alone) Page 138
51. Tx cauda equina syndrome.: Surgical decompression 58. Fibromyalgia clinical picture.:
52. Tx for disk herniation (sciatica): NSAIDs with ordinary mobility
>> Steroid injection(resistant to conservative management) >>
Surgery (rarely needed only if focal neuro deficits develop or
progress)
53. Man with a history of prostate cancer comes to the
emergency department with severe back pain and leg
weakness. He has tenderness of the spine, hyperreflexia, and
decreased sensation below his umbilicus.
What is the most appropriate next step?

a. Dexamethasone
b. MRI
c. X-ray
d. Radiation Young woman
e. Flutamide - Chronic MSK pain and tenderness with trigger points of focal
f. Biopsy: a. Dexamethasone tenderness at the trapezius, medial fat pad of the knee, and
- when there is an obvious cord compression, the most lateral epicondyle
important step is steroids urgently to DEC the pressure on the
cord *Pain at many sites (neck, shoulders, back, and hips)
- prevent permeant paralysis with steroids most important to do associated with:
first - Stiffness, numbness, and fatigue
- HA
54. Important diseases to rule out in suspected spinal stenosis.:
- Sleep disorder
Peripheral arterial disease (vascular studies and physical
findings are normal) 59. Dx test fibromyalgia.: No test to confirm fibromyalgia.
- Trigger points at predictable points
55. Spinal Stenosis clinical picture: Person > 60yrs
- All labs are normal such as ESR, C-reactive protein, RF, and
- Pain while walking radiating to buttocks and thighs B/L
CPK levels
- Worse on extending back/walking downhill
- Relieve in cycling/bending forward (flexion) 60. Best initial treatment for Fibromyalgia: 1. Best initial therapy:
- Normal pedal pulses and ankle/brachial index Amitriptyline
2. Milnacipran (SNRI specifically for Fibromyalgia) and
56. Most accurate test to diagnose spinal stenosis:
Pregabalin
3. Trigger point injections with local anesthetic
61. Steroids use in fibromyalgia.: Steroids are the wrong answer
for fibromyalgia
62. Systemic disease that can cause carpal tunnel syndrome.:
Pregnancy
Diabetes
RA
Acromegaly
Amyloidosis
Hypothyroidism

* carpal tunnel is peripheral neuropathy compression of the


median nerve as it passes under the flexor retinaculum, both
sensory and motor affected
MRI 63. Patient presentation carpal tunnel syndrome.: - Pain in the
57. Spinal stenosis management: 1. Weight loss and pain meds hand affecting the palm, thumb, index finger
(NSAIDs, opiates, aspirin) - Radial half of the ring finger with muscle atrophy of the thenar
2. Steroid injection (lumbar epidural space) eminence
3. PT and exercise such as bicycling or swimming and can put - Pain worse at night and is more frequent in those work
off surgery involves prolonged use of the hands such as typing
4. Surgical correction to dilate the spinal canal is needed in
75% of patients Tinel sign: reproduction of the pain and tingling with tapping or
percussion of the median nerve

Phalen sign: reproduction of sxs with flexion of the wrists to


90 degrees

Page 139
64. Most accurate diagnostic test for carpal tunnel syndrome.: 72. Patellofemoral Syndrome clinical picture: Anterior knee pain
Electromyography and nerve conduction testing at patella that is worse just after starting to walk after prolonged
sitting, improves with walking
DO NOT do wrist MRI
65. Best initial therapy for carpal tunnel syndrome.: Best initial Exam: crepitus, joint locking, and instability.
therapy: Wrist splits to immobilize the hand in position to relive X-rays normal
pressure 73. Causes of Patellofemoral Syndrome: Trauma, imbalance of
quadriceps strength, meniscal tear
Patients should avoid manual activity 74. Surgical indications of Patellofemoral syndrome: NO
Steroids injection (if splints and NSAIDs do not control indication (nothing to fix)
symptoms) - PT helps and strength training with cycling. Knee braces don't
help.
Surgery can be curative cutting the flexor retinaculum 75. Pain of _________ [Tarsal tunnel syndrome/Plantar Fasciitis]
66. Hyperplasia of the palmar fascia leading to nodule formation improves with use.: Plantar fasciitis (tarsal tunnel pain worsens
and contracture of the fourth and fifth fingers.: with use)
- worse pain in the morning, improves with walking a few steps
76. Plantar fasciitis clinical picture.: Very severe pain at the
calcaneus with point tenderness
77. Plantar fasciitis tx.: Stretching exercises, arch support, and
NSAIDs
- Steroid injection is performed if these don't solve the problem
- Surgical release plantar fascia rarely necessary.
78. X-ray of the foot in plantar fasciitis.: Not useful in plantar
fasciitis
- no correlation with the presence of heel spurs
Dupuytren's contracture 79. Morning stiffness of multiple small, inflamed joints is the key
- alcoholism and cirrhosis to the diagnosis.: RA
67. Dupuytren contracture treatment.: Triamcinolone, lidocaine, 80. Presentation RA:
or collagenase injection (helps with early Dupuyten
contracture) may help.

Sx release is performed when function is impaired.


68. Rotator cuff injury patient presentation.: Pain in the shoulder
that is worse at night when lying on the affected shoulder

Severe tenderness at the insertion of the supraspinatus


69. Most accurate test rotator cuff injury.:

- B/L symmetrical joint involvement: PIP joints of the fingers,


MCP joints of the hands, and involve wrists, knees, and ankles
- Morning stiffness lasting at least 30 min., but often much
longer
- Rheumatoid nodules (20%), most often over bony prominences
- Ocularsxs: episcleritis
- Lung: pleural effusion and nodules of lung parenchyma
- Vasculitis: skin, bowel, and peripheral nerves
- Cervical joint: particularly in C1 and C2 which can lead to
subluxation
- Baker cyst may rupture and mimic a DVT
- Pericardititis and pleural disease
- Carpal tunnel syndrome
MRI 81. Dry eyes, mouth, and other mucous membranes.: Sicca
70. Best initial therapy for rotator cuff tear.: NSAIDs syndrome
Rest 82. Episcleritis is a feature of _________ chronic inflammatory
Physical therapy disease.: RA
*Steroid injection relieves pain 83. Vertebral joint commonly affected in RA: C1 and C2 (leading
71. Surgery indications for rotator cuff tears: Complete tear, to subluxation)
failure to respond to NSAIDs, steroids and physical therapy
Page 140
84. Dx test RA.: 1. RF in 70% to 80% 100.Uses of sulfasalazine, leflunomide, abatacept, and anakinra.:
2. anti-CCP (anti-cyclic citrulinated peptide) 95% specific Alternative DMARDs to add to methotrexate if anti-TNF agents
3. X-ray: erotions, osteopenia do not control disease.
4. Elevated ESR and C-reactive protein 101.SE sulfasalzine: - Bone marrow toxicity
5. Anemia: Normocytic - Hemolysis of G6PD deficiency
6. Arthrocentesis: used to exclude crystal disease and - Rash
infection. Modest elevation lymphocytes. 102.Symptom control of RA which drugs.: 1. NSAIDs are best
7. Abnormal X-ray no longer needed to dx RA. initial therapy for RA pain
85. Factors for point-based diagnoses of RA.: total of 6 or more = - work immediately, improve inflammation
RA 2. Steroids - work on pain 2/2 inflammation, used as bridge
- Joint involvement (up to 5 points) waiting for DMARDs to take effect
- ESR or CRP (1 point) 103.Steroids indications in RA: When NSAIDs do not control
- Duration for longer than 6 weeks (1 point) symptoms; while DMARDs take effect (Steroids do not halt
- RF or anti-CCP (1 point) progression)
86. Most common cause of death due to RA: Coronary artery 104. Adverse effects of RA medications chart.:
disease (due to vasculitis)
87. The most important issue in RA is stopping the progression of
the disease. Any patient with erosive disease or x-ray
abnormalities needs at least ___ .: Methotrexate to slow
disease progression
- Erosive disease (joint space narrowing, physical deformities,
X-ray abnormalities)
88. RA
Splenomegaly
Neutropenia
Dx?: Felty Syndrome 105.High fever of unknown origin in a young person accompanied
89. RA with salmon colored truncal rash. Likely diagnosis is
Pneumoconiosis __________.: JRA (joint symptoms are relatively mild)
Lung nodules - fever spikes, chest or abdomen "salmon" colored rash
Dx?: Caplan Syndrome - Splenomegaly, pericardial effusion, mild joint symptoms
90. TRUE/FALSE: Neither NSAIDs nor steroids stop RA from 106.Labs in JRA: No clear dx test
progressing.: TRUE - Anemia, hypoalbuminemia, and leukocytosis
- any patient with erosive RA needs DMARD (disease modifying - ANA normal
antirheumatic agents) - Ferritin elevated, acute phase reactant, rises in inflammation
91. Best initial DMARD in RA.: Methotrexate 107.Tx JRA.: Half of cases improve with aspirin and NSAIDs
92. MTX major toxicity: Liver toxicity - no response then use steroids
Bone marrow suppression - steroid resistant cases are treated with TNF drugs.
Pulmonary toxicity 108.Immunologic labs that form SLE diagnostic criteria: Positive
93. TNF-inhibitors (infliximab, adalimumab, etanercept) ANA, or any of: Anti-dsDNA, Anti-Smith, antiphospholipid
indications in RA.: As a combination with MTX as initial antibodies)
DMARDS, Disease unresponsive to MTX or intolerance of
methotrexate
94. Labs to accompany TNF-inhibitor (infliximab, adalimumab,
etanercept) therapy.: PPD skin test
95. Rituximab indication in RA.: Rituxima used combo with
methotrexate in those not responding to anti-TNF meds
- Developed for non-Hodgkin lymphoma
- Effective in RA as a DMARD by removing CD20 positive
lymphocytes from circulation
96. Hydroxychloroquine indication in RA.: Monotherapy in DMARD
in cases of mild disease in which we wish to avoid toxicity of
MTX
- most often hydroxychloroquine used combo MTX
97. Hydroxychloroquine major toxicity: Retinal damage (do dilated
eye exam)
98. Which RA drugs are safe in pregnancy?: Hydroxychloroquine
Sulfasalazine
99. Rituximab major toxicity: Infection (anti-CD20 antibody -useful
in Hodgkins and in RA)
Page 141
109.Dx of SLE is based on the presence of at least __ of the 11 121.Rx for Lupus nephritis: Steroids, alone or in combo with
known manifestations of the disease.: Cyclophosphamide and Mycophenolate

Lupus nephritis severity determined by bx (differentiate


glomerulosclerosis or scarring of the kidney which does not
respond to tx)
122.Young patient vs old patient death in SLE causes.: Young -
die of infection

Old - accelerate atherosclerosis makes MI most common cause


of death
123.2 main types of anti-phospholipid antibodies: Lupus
anticoagulant
4/11
Anticardiolipin antibodies
1. Malar rash
2. Discoid rash
Antiphospholipid syndrome idiopathic disorder with IgG or IgM
3. Photosensitivity
antibodies made against negatively charged phospholipids
4. Oral ulcers
124.Antiphospholipid syndrome presentation.: Thromboses of
5. Arthritis (90% of SLE), normal x-ray
both arteries and veins as well as recurrent spontaneous
6. Serositis: inflammation of pleura and pericardium gives chest
abortions
pain
- elevation aPTT
7. Renal: Proteinuria, ESRD, membranous glomerulonephritis,
- false positive VDRL or RPR
red cell casts, hematuria
8. Neuro: psychosis, seizures, or stroke from vasculitis 125.Clotting profile in Antiphospholipid syndrome: high aPTT,
9. Hematologic: hemolytic anemia, anemia chronic disease, normal PT, normal INR
lymphopenia, thrombocytopenia 126.Antiphospholipid syndrome have false positive VDRL or RPR
10. Immunologic (lab) abnormalities: anti-dsDNA, anti-Sm, false with normal FTA occurs bc antibody reacts with the reagent
positive syphilis, positive LE cell prep in the lab which is ___.: Cardiolipin
11. + ANA 127.________ antibodies are associated with spontaneous
110.Ocular finding in SLE are not part of the formal diagnostic abortion in antiphospholipid syndrome.: Anti-cardiolipin
criteria including.: Photophobia 128.________ antibodies are associated with arterial/venous
Retinal lesions (cotton wool spots) thromboses in Antiphospholipid sydnrome: Lupus
Blindness anticoagulant
111.Additional findings in SLE not part of the dx criteria.: 129.Best initial test for Antiphospholipid syndrome with clotting
Mesenteric vasculitis abnormality: Mixing studies
Raynaud phenomenon - aPTT remains elevated in antiphospholipid syndrome
Antiphospholipid syndrome 130.Most specific test for lupus anticoagulant.: Russel viper
112.X-ray findings in SLE arthritis.: None (no deformation - just venom test (RVVT) prolonged
pain brings pt. to physician) 131.Tx APL (antiphospholipid syndrome).: An asx APL antibody
113.Diagnostic test for SLE with high NPV.: ANA (sensitive - does not need to be treated
negatives are not false ) - Tx DVT and PE with heparin and warfarin INR 2-3
114.Diagnostic test for SLE with high PPV.: Anti-dsDNA, Anti-Sm - Unclear if need lifelong anticoagulant first thrombotic episode,
(specific -positives are not false) recurrent thrombotic episodes are treated lifelong
115.Complement levels in SLE.: Correlate with disease activity 132.Rx for abortion due to Antiphospholipid syndrome: Cannot
(drop with exacerbations of acute disease); Decrease in C3 stop an imminent abortion;
116. Acute SLE flareup diagnostic labs.: Decrease in complement,
rise in Anti-dsDNA Heparin and aspirin for recurrence
117. Anti-SSA and anti-SSB and SLE.: found in 10-20% cases add 133.When should there be an investigation for anticardiolipid
little to the diagnosis if the DNA is positive antibody as a cause of spontaneous abortion?: 2+ more first-
- more often found in Sjogren syndrome (65% cases) trimester events or a single second-trimester event
118.Hydroxychloroquine indications for SLE: Mildly chronic 134.Scleroderma is diffuse in 20% of cases and limited in 80%.
disease limited to skin and joint Limited scleroderma is also known as ?: Calcinosis
Raynaud
119.Steroids indication in SLE: Bolus prep used in acute flare
Esophageal dysmotility
ups.
Sclerodactyly
Telangiectasia
Lupus nephritis.
135.Dx.
120. SLE drug that controls progression: Belimumab
Young (20-40s) woman (3x more likely than men) with fibrosis
of the skin and internal organs such as lung, kidney, and GI
tract.: Scleroderma (systemic sclerosis)/CREST
Page 142
136.Scleroderma presentation: 1. Raynaud syndrome: increased 150. Dermatomyositis presents with?:
vascular reactivity of fingers beginning with pain and pallor
(white) or cyanosis (blue) followed by reactive hyperemia (red)
2. Skin manifestations: fibrosis of hands, face, neck, and
extremities; telangiectasia and abnormalities of pigmentation
occur
3. GI: esophageal dysmotility with GERD, large-mouthed
diverticuli of small and large bowel
4. Renal: sudden HTN crisis
5. Lung: fibrosis leading to *restrictive lung
disease and pulmonary HTN* 1. Malar involvement
6. Cardiac: myocardial fibrosis, pericarditis, and heart block; 2. Shawl sign: erythema of face, neck, shoulders, upper chest,
lung disease gives right ventricular hypertrophy and back
137.Intestinal manifestations of systemic sclerosis.: Large- 3. Heliotrope rash: edema and purplish discoloration of eyelids
mouthed diverticuli 4. Gottron papules: scaly patches over the back of hands,
138.Renal manifestations of scleroderma/CREST.: Sudden HTN particularly PIP and MCP joints
crisis 151._____________ [Dermatomyositis/Polymyositis] is
139.Most sensitive test for scleroderma/CREST.: ANA (high NPV associated with cancer in 25% of cases.: Dermatomyositis
but non specific) - Ovary
140.Most specific test in diffuse scleroderma: SCL-70 (anti- - Lung
topoisomerase) but present only 30% those with diffuse and - GI
20% in limited - Lymphoma

141.___ antibodies are extremely specific for CREST syndrome.: 152.Best initial test for Dermatomyositis/polymyositis: CPK and
anticentromere aldolase
- ANA is positive but nonspecific
142.Tx slows the underlying disease process of limited
scleroderma.: MTX 153.Most accurate test polymyositis/dermatomyositis.: muscle
- Penicillamine ineffective bx
143.Tx renal crisis in scleroderma.: ACEi (use even if creatinine 154.Anti-Jo antibody positivity clinical significance: Often
is elevated) associated with lung fibrosis
144.Tx esophageal dysmotility in scleroderma/CREST.: PPI for 155.MRI in polymyositis/dermatomyositis.: patchy muscle
GERD involvement, EMG usually abnormal
145. Tx Raynaud in scleroderma/CREST.: CCB 156.Tx polymyositis/dermatomyositis.: 1. Steroids are usually
sufficient
146.Tx pulmonary fibrosis in scleroderma.: Cyclophosphamide
2. When patient is unresponsive or intolerant of steroids, use:
improves dyspnea and PFTs
- Methotrexate
147.Tx pulmonary HTN in scleroderma/CREST.: Bosnian - Azathioprine
ambrisetan (endothelial antagonist) - IVIG
- Mycophenolate
Sildenafil 3. Hydroxychloroquine helps skin lesions
157.__ is idiopathic disorder 2/2 antibodies predominantly
Prostacyclin analogs: iloprost, treprostinil, epoprostenol
against lacrimal and salivary glands.: Sjogren syndrome
148.CREST vs Scleroderma: CREST syndrome is (Calcinosis, - 90% affecting women
Raynaud's, Esophageal dysmotility, Sclerodactlyly,
158. Sjogren syndrome associated with?: - RA
Telangiectasia)
- SLE
- Primary biliary cirrhosis
+ lungs, heart, and kidney involvement = Scleroderma
- Polymyositis
- Hashimoto thyroiditis
CREST can cause primary pulmonary HTN but lungs are normal
159.Hepatobilliary condition associated with Sjogren's: Primary
149.Polymyositis and dermatomyositis presentation.: - Proximal
biliary cirrhosis
muscle weakness (difficulty getting up from a seated position
or walking up stairs)
- Dysphagia (involvement of striated muscles of the pharynx,
difficult to initiate swallowing)

CK-MB may be elevated

Page 143
160.Sjogren clinical presentation.: • Sjögren presents with 169. PFT findings in PAN: NO findings (always spares the lungs)
dryness of mouth and eyes 170.Most common neurologic abnormality in Polyarteritis
• Keratoconjunctivitis sicca nodosa.: Foot drop (also stroke in a young person)
• Need to constantly drink water 171. Most accurate test for PAN.: Bx of symptomatic site
• Dysphagia
172. Angiography findings in PAN.:
• Dental caries and loss of teeth
• Dyspareunia

Less common manifestations are:


- Vasculitis
- Lung disease
- Pancreatitis
- Renal tubular acidosis (20%)
161.Most dangerous complication of Sjogren's: Lymphoma (up to
10%)
162.Best initial test for Sjogren's: Schirmer test (filter paper
against the eye) Beading/abnormal dilation of renal, mesenteric or hepatic artery
- observe amount of tears produced by the amount of wetness 173.Test to differentiate polymyositis/dermatomyositis from
on filter paper Polymyalgia rheumatica: Normal CPK and aldolase in PMR
163.Most accurate test for Sjogren's: Lip or parotid gland bx 174.P-ANCA (anti-myeloperoxidate) is associated with?: Churg-
showing lymphoid infiltration in the salivary glans Strauss
164.Best initial blood test for Sjogren's: SS-A (Anti-Ro) and SS-B 175.Tx PAN: Prednisone and cyclophosphamide
(Anti-La) (also present in SLE -sensitivity also only 65%) - tx hepatitis when found
165. Rose bengal stain in Sjogrens shows.: 176.Dx.
Pt over 50 with:
- Pain and stiffness in shoulder and pelvic girdle muscles
- Difficulty combing hair and rising from chair
- Elevated ESR
- Normochromic, normocytic anemia: Polymyalgia
rheumatica
- CPK and aldolase are normal
- tx: STEROIDS even at low doses good response
177.Dx.
Pt has visual sxs
Jaw claudication (pain in jaw when chewing)
Abnormal corneal epithelium Scalp tenderness
166.Tx Sjogrens: 1. Best initial therapy is to water the mouth HA
- Use frequent sips of water, sugar-free gum, and fluoride DEC arm pulses, bruits near clavicles, AR
treatments Treatment?: Giant cell (temporal) arteritis
2. Artificial tears to avoid corneal ulcers - ESR and C-reactive protein elevated
3. Pilocarpine and cevimeline increase acetylcholine, the main - Tx HIGH dose Prednisone since patient can go blind (not
stimulant to produce saliva reversible)
4. Cevimeline increases rate of saliva production - Most accurate test is biopsy of temporal artery/affected artery
178.Dx.
• No cure Pt has sinusitis.
• Evaluate for lymphoma Otitis media
167.___ is a disease of small- and medium-sized arteries Mastoiditis
leading to a diffuse vasculitis that inexplicably spares the Oral and gingival involvement
lungs. Associated with ___.: Polyarteritis Nodosa (PAN) Skin, joint, eye lesions
- Associated with Hep B and C Renal insufficiency.: Wegener granulomatosis
168.Polyartetitis Nodosa (PAN) clinical presentation.: 1. Renal: - associated with upper and lower respiratory findings in
glomerulonephritis without a biopsy can't be diagnosed association with renal insufficiency
- UA isn't enough to confirm it's PAN 179.Wegener's is associated with positive ________ (Anti-
2. Neuro: any large peripheral nerve can be involved proteinase-3 Ab/Anti-myeloperoxidase Ab): C-ANCA = Anti-
- Peroneal neuropathy leading to foot drop proteinase-3
• Look for a stroke in a young person 180. Best test for Wegener.: Lung bx > renal bx > sinus bx
3. GI: Abd pain worsened by eating from vasculitis of 181. Tx Wegener.: Prednisone and cyclophosphamide
mesenteric veins. Bleeding occurs. N/V common.
182.Clinical picture of Wegener's lung involvement: Unresolving
4. Skin: LE ulcers, livedo reticularis, purpura, nodules, and
pneumonia not better with Antibiotics
rarely gangrene
Page 144
183.Eosinophillia is most strongly associated with _________ 196.What is pathergy?: Sterile skin pustules from minor trauma
vasculitis: Churg Struass Syndrome (also has asthma) (needlestick)
- bx most accurate test - characteristic Behcet syndrome skin lesions
- tx: prednisone and cyclophosphamide 197.Oral and genital ulcers.
184.Pulmonary renal syndrome, Churg-Strauss also have ___ Erythema nodosum like lesions on the skin.
and ___.: Asthma Ocular lesions leading to uveitis and blindness.
Eosinophilia Arthritis.
185.Dx: CNS lesions mimicking MS.
Child with GI tract (pain, bleeding)
Purpura Likely diagnoses.: Behcet Syndrome
Arthralgia - Asian or middle eastern person
Hematuria: Henoch-Schonlein Purpura 198.Tx Behcet syndrome: Steroids
- clinical dx - to wean off steroids use (Azathioprine, cyclophosphamide,
- bx most accurate test colchicine, thalidomide)
186. Leukocytoclastic vasculitis on bx. Answer.: 199.Characteristic clinical picture of seronegative
spondyloarthropathy: - Men <40 with joint pain
- Involvement of spine + large joints
- Enthesopathy (inflammation where tendons and ligaments
attach to bones)
- Uveitis
- HLA-B27
200.T/F: Corticosteroids are a good treatment for seronegative
spondyloarthropathy.: FALSE
201.3 types of seronegative spondyloarthropathy.: Psoriatic
arthritis
Ankylosis spondylitis
Reactive arthritis (Reiter Syndrome)
202.Backache and stiffness in young man that improves with
activity and worsens with rest.
Henoch-Schonlein Purpura (HSP)
- Leukoplastic reactions are painless, palpable purpura of Pain that radiates to the buttocks with flattening of the normal
buttocks and legs lumbar curvature and DEC chest expansion.

187.Lab findings to diagnose HSP: None (IgA levels not reliable) -


Likely diagnoses.: Ankylosing Spondylitis
clinical diagnoses + biopsy
- enthesopathy (inflammation of tendons) occurs in Achilles
188.Tx Henoch-Schonlein Purpura: Most cases resolve tendon
spontaneously
203.Clinical findings of ankylosing spondylitis. Cardiac, eye,
- Steroids are the answer for severe abdominal pain or
joint, and spinal.:
progressive renal insufficiency (do not reverse but DEC
progression)
189.Chronic HCV infection is associated with increased
_________ Ig levels: Cryoglobulins (Cryoglobunemia)
- IgM antibodies
190.Cryoglobulinemia clinical findings: Joint pain
Glomerulonephritis
Purpuric lesions
Neuropathy
191.Tx for Cryoglobulinemia: Treat underlying HCV with
interferon/Ribavirin or telaprevir or boceprevir (steroids NOT
effective)
192.IgM associated with EBV infection: Cold Agglutinins (cause
hemolytic anemia in EBV, mycoplasma, lymphoma) • Transient peripheral arthritis of knees, hips, and shoulders
193.Tx cold agglutinins hemolysis.: Stay warm (50%)
Rituximab, cyclophosphamide, cyclosporine • Cardiac: AV block in 3% to 5%; aortic insufficiency
194. Hepatitis C which complement is decreased.: C4 • Uveitis
• "Bamboo spine" is a late finding with fusion of vertebral joints
195.Tx cryoglobulinemia: + RF factor and cold ppt immune
complexes

Tx: underlying cause Hep C

Page 145
204. Best initial test for Ankylosing spondylitis: 215. Best initial test for Reactive arthritis: No specific test.

Rule out Reactive arthritis and look for underlying cause (IBS,
Chlamydia infection, GI infection)
216.___ is a skin lesion unique to reactive arthritis that looks
like pustural psoriasis.:

X-ray of Sacroiliac joint showing space narrowing


205. Most accurate test for ankylosing spondilitis: MRI
206. Which lab test is elevated in ankylosing spondylitis?: ESR Keratoderma blennorhagicum
207.Bridging syndesmophytes over vertebral bodies seen in 217.TRUE/FALSE: Antibiotics do not reverse reactive arthritis
_________ disease.: Ankylosing spondylitis (causing bamboo once joint pain has started.: TRUE
spine) 218.Tx Reactive arthritis (reiter syndrome): NSAIDs and correct
208.Tx ankylosing spondylitis.: Exercise program and NSAIDs underlying cause
- NSAIDs don't work use anti-TNF drugs such as etanercept, - Sulfasalazine used when NSAIDs do not control it
adalimumab, or infliximab - Steroid injection into the joint may help
209.Findings in psoriatic arthritis: 80% will have preceding 219.Dx.
psoriasis Older woman.
SI joint involvement Vertebral fractures leading to loss of height/wrist fracture.
Sausage digits (from enthesopathy) Spontaneous fractures in wt bearing bones.: Osteoporosis
Nail pitting - screen women above age 65 with DEXA
210. Best initial test for psoriatic arthritis.: 220.Most Accurate test for Osteoporosis: Bone Densitometry
(DEXA)
- T-score compares bone density with the normal density of a
young woman
221.Osteopenia diagnostic criteria.: T-score between 1 to 2.5 SD
lower than normal
222.Osteoporosis diagnostic criteria.: T-score below 2.5 SD lower
than normal
223.Blood tests in osteoporosis.: All blood tests are normal
- Calcium, phosphate, PTH levels are normal
224.Tx Osteoporosis: 1. Vitamin D and calcium are best initial
X-ray of DIP therapy
- showing pencil-in-a-cup deformity 2. Bisphosphonates (alendronate, risendronate, ibandronate)
- bony erosions + irregular bone destruction are used when T score more than 2.5 standard dev. below
normal
211.Labs psoriatic arthritis: INC ESR
3. Estrogen replacement in postmenopausal women
INC uric acid due to INC skin turnover
4. Raloxifene substitute for estrogen in postmenopausal
212.Rx Treatment order for Psoriatic arthritis: 1. NSAIDs best women; reduces the risk of breast cancer and DEC LDL levels
therapy 5. Teriparatide - analogue of PTH stimulates new bone matrix
2. MTX used when severe disease or no response to NSAIDs formation
3. Anti-TNF agents when MTX does not control disease 6. Used as a nasal spray, Calcitonin decreases the risk of
vertebral fractures
Steroids wrong choice
225.SE Bisphosphonates: Jaw necrosis - very rare
213.Reactive arthritis 2/2 which 3 causes.: 1. IBD (equal sex Esophagitis (pill esophagitis)
incidence)
226.SE Teriparatide: Osteosarcoma in rats
2. STI (far greater in men)
- Hypercalcemia
3. GI (Yersinia, Salmonella, Campylobacter)
214.Triad in reactive arthritis.: 1. Joint pain *analog of PTH
2. Ocular findings (uveitis, conjunctivitis)
227. Risk factors for septic arthritis.: Prosthetic joint > RA > OA
3. Genital abnormalities (urethritis, balanitis)
* Risk of infection directly proportionate to the degree of joint
damage
Page 146
228.Etiology of septic arthritis which bug most common.: 241.Adults get osteomyelitis via ___.: Contiguous (nearby)
Staphylococcus - 40% infection
Streptococcus - 30%
Gram-negative rods - 20% Risk factors:
229.Dx. Vascular insufficiency
Joint warm, red, and immobile. - DM, SCD (Salmonella)
Palpable effusion. 242.Children get osteomyelitis through ___ spread.:
Chills and fever due to bacteremia.: Septic arthritis Hematogenous
230.Best initial and most accurate test for septic arthritis.: 243.Clinical picture of Osteomyelitis: Look for diabetic patient
Aspiration of the joint with a needle (artherocentesis) with an ulcer from peripheral neuropathy or vascular disease
Fluid shows: - Warmth, redness, and swelling in the area
- Leukocytosis > 50,000-100,000 (neutrophils) - Draining "purulent sinus tract" in the lesion
- Gram stain + gram positive cocci - Afebrile pt
- Synovial cultures 244. Best initial test for osteomyelitis:
- Blood culture
231.Best initial therapy for septic arthritis: Ceftriaxone and
Vancomycin
232.Therapy for Septic arthritis with culture showing S.aureus:
Oxacillin, Nafcillin, Cefazolin, Piperacillin with Tazobactam (beta-
lactam sensitive)

Linezolid, Daptomycin, Tigecycline, Ceftraoline (resistant)


233.Most common organism for recently replaced artificial joints
is ___.: Staph epidermidis
234.Management of Infected prosthetic joints: Removal first,
Antibiotic for 6-8 wks, replacement

- Prosthetic joint infection if joint is loose, infection is likely


present Best initial : X-ray
TRUE/FALSE: Gonococcal arthritis is more frequent during
235. Normal X-ray, next step MRI
menses.: TRUE - if MRI contraindicated (pacemaker) order bone scan
236. The difference in presentation of Gonococcal arthritis 245. Most accurate test for osteomyelitis: Biopsy
(gonorrhea) vs septic arthritis.: - Polyarticular involvement 246.When to order ESR for osteomyelitis?: To follow response to
- Tenosynovitis (inflammation of the tendon sheath, making therapy
finger movt painful)
247.When to culture and drain in osteomyelitis?: NEVER, cannot
- Petechial rash
distinguish superficial colonization from whatever organs, is
inside the bone causing infection
*this tells you to culture everywhere to reach max sensitivity for
gonorrhea 248.Emperic therapy for osteomyelitis: None (obtain culture on
biopsy first and treat according to sensitivity)
237. Synovial fluid analysis for infectious arthritis:
- Sensitive staph (oxacillin, cefazolin, nafcillin, ceftriaxone)
- MRSA (vanco, linezolid)
- Gram negative bacilli (e.coli) tx ciprofloxacin
249.Toxicity of fluoroquinolones.: Achilles tendon rupture
-They are also contraindicated in pregnancy and children
because they interfere with bone growth
238.Gonococcal arthritis diagnostics: Culture multiple sites to
250.Tx gram negative bacilli septic arthritis.: Quinolones
have max sensitivity
Aztreonam
- Pharynx
Cefotaxime
- Rectum
Piperacillin
- Urethra
Aminoglycosides
- Cervix as well as joint and blood
239.Tx for Gonococcal arthritis: Ceftriaxone, cefotaxime, or
cefrizoxime best empiric for disseminated gonorrhea

Fluoroquinolone's are not the best therapy since 5% resistance


use only if sensitive
240.If recurrent gonorrhea infection is describes, test for ___.:
terminal complement deficiency for MAC

Page 147
MTB Surgery
MTB Q and A

1. The number 1 limiting factor prior to surgery is a history of 15. Trauma assessment how should circulation be maintained?:
___.: CV disease Insert 2 large-bore IVs into the patient and begin aggressive
2. What ejection fraction cut off is associated with an increased fluid resuscitation to prevent hypovolemic shock.
risk for non-cardiovascular surgery?: EF less than 35% 16. What variables are a concern for airway compromise?:
3. Patient recently experienced a myocardial infraction 5 months
ago and wants to schedule a cholecystectomy next week.

Management and why?: Defer surgery for another month and


stress test that patient prior to surgery.

Current guidelines recommend deferring surgery for at least


6 months after an MI followed by stress testing.
4. Patient needs a total knee replacement and has signs of JVD
and lower extremity edema. What is needed in this patient's
surgical management?: Review of medications to ensure the 1. Altered mental status.
patient is on the following drugs for his CHF (all decrease 2. Facial trauma.
mortality). 3. Apnea.
1. ACE inhibitors. 17. When evaluating whether or not to intubate a patient what
2. BB. variables guide your decision making?:
3. Spironolactone.
5. If the patient is under the age of 35 and has no history of
cardiac disease what test if the only one needed prior to
surgery?: EKG
6. What tests should be required for all patients with cardiac
disease prior to surgery?: 1. EKG.
2. Stress testing. (ischemic lesions)
3. ECHO. (structural disease and ejection fraction)
7. For men at what age are they at risk for cardiovascular
complications?: 45 years old
8. For patients with lung disease or those who have a smoking 1. Is there a risk for a compromised airway such as AMS, facial
history what is needed for management prior to surgery?: 1. trauma and apnea?
PFT for vital capacity evaluation. 2. Is there facial trauma?
2. Smoking cessation for 6-8 weeks prior to surgery and use of 3. Is there a cervical spine injury?
nicotine patch. 18. Patient presents after car accident with chest pain, pallor,
9. For patients with renal disease what is necessary in cool extremities, HR 120 bpm, JVD, and BP is 80/40. CXR
management prior to surgery?: 1. Adequate hydration. shows 3 broken ribs on the left side. Which type of shock is
2. Dialysis 24 hours prior to surgery for those who need this?: Cardiogenic shock 2/2 pericardial tamponade
dialysis. - blood collected in pericardial sac leading to RV diastolic
10. Patient with severe claudication age > 70 needs a femoral- collapse and impaired filling
popliteal bypass. What testing needs to be done 19. Define SIRS criteria?: 2 of the following needs to be present
preoperatively?: 1. BMP. for SIRS:
2. EKG. 1. HR > 90 bpm.
3. Thallium stress test. 2. Temp < 36 or > 38.
11. Patient presents to the ER without facial trauma what is the 3. RR over 20 or PCO2 of less than 32.
best way to maintain an airway in this patient?: Orotracheal 4. WBC below 4,000 or above 12,000.
tubes 20. How does one define the following:
12. Patient presents with severe facial trauma and is in need of 1. SIRS
an airway. What is contraindicated and what needs to be 2. Sepsis.
done?: Intubating with an orotracheal tube is contraindicated. 3. Severe sepsis.
4. Septic shock.: 1. SIRS is 2 criteria.
Perform a cricothyroidotomy patient with facial trauma. 2. Sepsis is SIRS and an identified source of infection.
13. Patient presents with a cervical spine injury and needs an 3. Severe Sepsis is sepsis with organ dysfunction.
airway. What do you do?: Intubate with orotracheal tube using 4. Septic SHOCK is severe sepsis with hypOtension.
a flexible bronchoscopy.
14. What is the oxygen saturation goal for a patient?: greater than
90%

Page 148
21. What are signs of shock other than changes in vitals such as 27. What variables are important when evaluating shock?: 1.
tachy and hypotension?: 1. Brain - confusion. Signs and symptoms.
2. Heart - chest pain and SOB. 2. CVP.
3. Liver - INC transaminases. 3. CO.
4. Renal - INC BUN/Cr ratio. 4. Wedge pressure.
5. Blood - INC lactate. 5. Response to fluids.
22. Pale and cool extremities. 28. What is the treatment for hypovolemic shock and neurogenic
DEC CVP, CO, LVEDP/PCWP. shock?: Fluids and pressors.
INC SVR, HR. 29. Motor vehicle collision, abdominal pain that radiates to the
Trauma. back, and ecchymosis on the flank 2 days later. Diagnosis?:
Type of shock. Hemorrhagic pancreatitis - retroperitoneal hemorrhage
Tx.: Hypovolemic 30. Elevated blood pressure and tearing midepigastric pain to
- Causes: Hemorrhage (#1), Dehydration, Burns radiates sharply to the back.: Aortic dissection.
- Tx: Fluids and pressors
31. Brusing around the umbilicus.
23. Pale and cool extremities. Name of finding? Causes?: Cullen sign
DEC CO. 1. Hemorrhagic pancreatitis.
INC CVP, SVR, HR, LVEDP/PCWP. 2. Ruptured AAA.
Type of shock.
32. Bruising in the flank.
Tx.: Cardiogenic - chest pain, SOB, JVD
Name of finding and cause?: Grey Turner sign.
- Causes: MI (#1), CHF, arrhythmia
- Retroperitoneal hemorrhage
- Tx: Treat cardiac problem 33. What signs correlate with splenic rupture?: 1. Kehr sign -
pain in the left shoulder.
24. Warm extremities.
2. Balance sign - dull percussion on the left and shifting
DEC CVP, SVR, CO, LVEDP/PCWP.
dullness on the right.
INC HR.
Type of shock. 34. With abdominal trauma if a bleeding is suspected, how does
Tx.: Neurogenic one evaluate such a problem?: 1. FAST scan first to evaluate
- Causes: CNS damage (cervical/thoracic spinal cord - #1) for an intraabdominal bleed.
2. CT scan to evaluate for a retroperitoneal bleed, especially if a
Signs/Symptoms: splenic rupture is suspected despite a negative FAST scan.
- Warm, flush 35. Hemodynamically unstable patients with abdominal trauma
- Evidence of CNS damage (trauma) need?: Exploratory laparotomy: unstable

Tx: stable: close monitor, abdominal exams, IV fluids


- Aggressive IV fluid delivery and Pressors 36. ___ is the best initial test to evaluate free air under the
25. Warm extremities. diaphragm.: Upright chest x-ray
DEC CVP, SVR, no change in LVEDP/PCWP. - perforation of the bowel
INC HR, CO. 37. Thoracotomy vs thoracostomy: Thoracotomy - making an
Type of shock. incision to gain access to the thoracic organs Thoracostomy:
Tx.: Septic tube placed into the pleural space
- E. coli and S. aureus (most common organisms) 38. Patient undergoes a thoracotomy for CT surgery. What is a
- Warm, flush common complication of this procedure for the first few
- Possible nidus of infection (UTI, pneumonia, wound) days?: Atelectasis and pneumonia because the pain from this
procedure is unbearably difficult.
Tx: 39. Distended bowel with multiple air fluid levels.: Ileus
- Fluids, Abx, Pressors (Dopamine, NE) - abdominal x-ray
26. Shock management algorithm.: - nontechnical etiology for lack of peristalsis in the GI tract
40. JVD, hypotension, muffled heart sounds, and electrical
alternates on EKG.: Pericardial tamponade
41. What are common forms of trauma that lead to pericardial
tamponade?: 1. Broken ribs.
2. Gunshots.
3. Bullets.
4. Knife stabbing.
42. What is the best diagnostic test for pericardial tamponade?:
ECHO
43. Tx pericardial tamponade.: Pericardiocentesis most effective.

Page 149
44. What defines tension pneumothorax when compared to 57. LUQ abdominal pain, causes?: 1. Splenic rupture.
regular pneumothorax?: Tracheal deviation away from 2. IBS - splenic flexure syndrome.
pathologic lung. 58. Midepigastric pain, causes?: 1. Pancreatitis.
2. Aortic dissection.
Both have: chest pain, hyper resonance, and DEC breath 3. Peptic ulcer disease.
sounds, and tracheal deviation 59. RLQ pain, causes?: 1. Appendicitis.
45. What is the management of tension pneumothorax vs regular 2. Ovarian torsion.
pneumothorax?: Both require chest tube placement, but with a 3. Ectopic pregnancy.
tension pneumothorax it is an urgent problem that needs 4. Cecal diverticulitis.
immediate needle decompression prior to chest tube placement. 60. LLQ pain, causes?: 1. Sigmoid diverticulitis.
46. In a trauma setting what could lead to a blunting of the 2. Sigmoid volvulus.
costophrenic angle on chest x-ray and CT scan.: Hemothorax - 3. Ovarian torsion.
blood in the pleural space. 4. Ectopic pregnancy.
- absent breath sounds and dull to percussion 61. Ear pain can be referred to what organ?: Pharynx.
- tx: chest tube drainage and possible thoracotomy
62. Pancreas referred pain location?: Back.
47. An 18 year old presents with severe groin pain after falling
63. Gallbladder referred pain location?: Right shoulder/scapula
on the central bar of the bike. PE shows blood at the urethral
meatus and high-riding prostate. Next step in management?: 64. Crunching heard upon palpation of the thorax, pain radiating
Urethral disruption to left shoulder, odynophagia and severe and acute onset of
- KUB x-ray followed by retrograde urethrogram then can do excruciating retrosternal chest pain. Diagnosis?
foley to aid in urination Test to confirm?
Complications?: Esophageal perforation
48. 75 year old man. Atrial fibrillation, coronary artery disease,
and dyslipidemia. Severe abdominal tenderness. Pain worse
Accurate test: Esophogram using diatrizoate meglumine and
with eating. Likely diagnosis? Next best step?: Acute
diatrizoate sodium (Gastrografin) showing leaking of contrast
mesenteric ischemia.
outside of the esophagus.
- Severe abd pain 10/10 out of proportion to physical findings,
no guarding, soft abdomen, no rebound tenderness
Complication: Mediastinitis
- Angiography or possibly a surgical candidate.
49. Patient has a history of abdominal pain after eating and Tx: Sx exploration with debridement of the mediastinum and
bloody diarrhea. History of diabetes, hypertension and closure of perforation
hypercholesterolemia.
Likely diagnosis? * Barium cannot be used caustic to the tissues!
Best initial test.
65. Where is the perforation in Boerhaave syndrome often located
Most accurate test?
at?: Left posterolateral aspect of the distal esophagus
Tx?: Ischemic bowel
Initial test: CT scan of the abdomen. 66. Mallory-Weiss syndrome is a mucosal tear due to vomiting
Most accurate: Angiography or colonoscopy with bx most common location is ?: at the gastroesophageal junction
(GEJ)
tx: IV normal saline followed by surgical intervention to remove 67. Mallory-weiss syndrome vs Boerhaave syndrome chart.:
necrotic bowel.
50. Watershed areas of the GI tract include are the most common
location for infarction?: Splenic and hepatic flexures.
51. Acute mesenteric ischemia is the acute occlusion of
mesenteric arteries most commonly the ___.: SMA
52. Number 1 risk factor for mesenteric ischemia?: atrial
fibrillation
- can cause emboli to occlude the vessel
53. Labs in acute mesenteric ischemia show?: INC lactic acid and
leukocytosis 68. Risk factors for gastric perforation.: 1. H. pylori
54. Air in bowel wall. Diagnosis? Treatment?: Mesenteric 2. NSAID abuse
ischemia via abd x-ray (most accurate angiography) 3. Burns
4. Head injury
Emergent laparotomy with resection, endovascular therapy 5. Trauma
only if there is a clear reason to avoid sx 6. Cancer
55. Referred pain of the appendix (atypical part of the body): Left 69. What is the problem with concurrent alcohol and smoking use
lower abdominal quadrant. in a patient with peptic ulcer disease?: Prevents healing.
56. RUQ abdominal pain, causes?: 1. Cholecystitis 70. Other than peritonitis what other complications are
2. Biliary colic associated with gastric perforation?: Pancreatitis (gastric
3. Cholangitis juices fries the pancreas) if the ulcer is in the posterior part of
4. Perforated duodenal ulcer the stomach
Page 150
71. Patient presents with acute, progressive worsening abd pain 79. Dx abdominal abscesses.:
that radiates to the right shoulder due to irritation of the
phrenic nerve. Signs of peritonitis (guarding, rebound
tenderness, abd rigidity).: Gastric perforation with fluid
leakage into the abdominal cavity causing peritonitis
72. Initial test gastric perforation.:

CT scan and incision and drainage only tx


- CT or U/S percutaneous drainage
- Abx given prevent bacteremia
80. What do you expect to find on ultrasound findings of someone
with cholecystitis?: 1. Pericholecystic fluid.
2. Thickened gallbladder wall.
81. The most common complication after diverticulitis is ___.:
Abscess formation
Upright CXR shows free air under the diaphragm
82. Pain that radiates to the back? Name 2 emergent conditions:
1. Pancreatitis.
Most acute: CT scan.
2. Aortic dissection.
73. Medical management for perforated ulcer?: 1. NPO
83. What results would be expected on a HIDA scan of someone
- Prevent further extrusion of gastric contents into peritoneal
with acute cholecystitis?: Delayed emptying of the gallbladder.
cavity
2. NGT. 84. What special signs are associated with acute appendicitis?:
3. IV fluids and antibiotics. 1. Rovsing: palpation of the LLQ causes pain in the RLQ
4. Emergent Sx: 2. Psoas: pain with extension of the hip.
- Exploratory laparotomy and repair of perforation. 3. Obturator: pain with internal rotation of the right thigh.
74. If RLQ pain was in a female patient of childbearing age, 85. Appendicitis
ectopic pregnancy, cyst, and torsion must be considered. Etiology.
What should be ordered?: beta-HCG and pelvic sonogram Sxs and symptoms.
Dx tests.
75. Workup for a woman with abdominal pain. Beta-HCG is
Tx.
elevated and pelvic ultrasound shows ectopic pregnancy.
Complications.: Fecolith obstructing the appendiceal orifice,
What do you do?: Emergent surgery.
causing inflammation
76. What should you never do in a patient with acute
diverticulitis?: Barium enema and colonoscopy Anorexia, fever, periumbilical pain w/ RLQ tenderness, elevated
- INC incidence of perforation WBC with left shift
77. What is the algorithm for right lower quadrant pain?:
CT scan most accurate

Laparoscopic sx

Abscess formation and gangrenous perforation


86. Acute pancreatitis
Etiology.
Sxs and symptoms.
Dx tests.
Tx.
Complications.: Alcohol or gallstone obstruction of the duct,
78. Causes abdominal abscess.: Surgical complication (#1) causing inflammation
Inflammatory disease
Trauma Fever, severe midabdominal pain radiating to the back, N/V

CT scan is the best test. Amylase is sensitive and lipase is


specific.

Aggressive IV fluids and NPO.

Hemorrhagic pancreatitis and pseudocyst formation.


Page 151
87. Diverticulitis. 95. What etiologies can cause a bowel obstruction?: 1. Adhesions
Etiology. from previous abd surgery (MCC).
Sxs and symptoms. 2. Hernias.
Dx tests. 3. Crohn disease.
Tx. 4. Neoplasms.
Complications.: Fecal impaction into pseudodiverticula, causing 5. Intussusception.
inflammation 6. Volvulus.
7. Foreign bodies.
Fever, N, most commonly LLQ pain, and peritonitis 8. Intestinal atresia.
9. Carcinoid.
CT scan is the best and most accurate test. 96. What is the best drug to tx obstructions from stool impaction
in patients on chronic opioids?: Methylnaltrexone.
Abx for the first attack, sx resection if it recurs or perforates 97. Signs and symptoms of bowel obstruction.: • Severe, crampy
abdominal pain
Abscess formation. No endoscopy due to risk of perforation. - Colicky in nature
88. Cholecystitis. • N/V
Etiology. • Fever
Sxs and symptoms. • High-pitched, "Tinkling" bowel sounds high pressure in the
Dx tests. bowel
Tx. • Hypovolemia due to third spacing
Complications.: Gallstones occluding the lumen of the cystic 98. Dx test bowel obstruction.: • Labs
duct, causing inflammation of the gallbladder - INC WBC, INC Lactate, DEC pH

Fever, serve RUQ tenderness, Murphy's sign, pain on • Abdominal X-ray


inspiration causing sensation of breathing, N/V - Air-fluid levels, dilated loops of bowel

US will reveal pericholecystic fluid, gallbladder wall thickening, • Most accurate: CT abdomen
and stones. HIDA most accurate test. - transitional zone from dilated loops of bowel with contrast to
an area of bowel with no contrast
Laparoscopic sx or open if there is perforation of the
99. Multiple air-fluid levels with dilated loops of small bowel.:
gallbladder.
Small bowel obstruction
Perforation of the gallbladder. 100. Tx bowel obstruction.: 1. NPO
2. NG tube with suction
89. Is appendicitis a clinical diagnosis?: Usually you can treat
- lowers bowel pressure proximal to obstruction
appendicitis just from the history and physical. However,
3. Medical management
additional testing maybe required if the clinical picture is not
- IV fluids to replace volume lost via third spacing
obvious.
4. Surgical decompression indicated if:
90. Complications of appendicitis?: 1. Abscess. - complete: emergent
2. Gangrenous perforation. - lack of improvement with medical management
91. For cholecystitis what is the most accurate test?: HIDA scan. 101. Fecal incontinence definition?: Continous or recurrent
92. Failure to pass stool and flatus and hyperactive bowel uncontrolled passage of fecal material > 10 mL for at least 1
sounds. Diagnosis?: Small bowel obstruction. month in an individual > 3 years of age.
- abd sx very significant risk factor as adhesions can form from 102. Best initial test for fecal incontinence?
surgery Most accurate test?
93. Mechanism of disease with bowel obstruction?: Occlusion of If there is a history of anatomic injury what would be the next
the lumen, gas and fluid build up, severely increasing pressure best most accurate test?: Initial: Sigmoidoscopy or anoscopy.
within the lumen. Decreased perfusion of the bowel and necrosis
ensues. Most accurate: Anorectal manometry.
94. Can someone have a bowel obstruction even though a small
amount of GI contents is passing?: Yes, this is known as a Hx anatomy injury best test: Endorectal manometry.
partial small bowel obstruction. 103. What is dextranomer/hyaluronic acid injections used for?:
This substance is deposited in the submucosal layer of the GI
walls to help bulk the tissue.
50% of patients with fecal incontinence will get better with this
treatment.
If this fails, colorectal sx is needed.
104. What are some biofeedback measures to help with fecal
incontinence?: Control exercises and muscle strengthening
exercises.

Page 152
105.Mild fractures w/o displacement. Management.: Closed 113.What is the typical presentation for a pathologic fracture?:
reduction Hx of minimal trauma to bone causes a fracture. (older person
106.Severe fractures w/ displacement or misalignment of bone fractures a rib from coughing)
pieces. Management.: Open reduction and internal fixation
107.Skin must be closed and the bone must be set in the Etiologies include the following:
operating room with debridement. Management of which 1. Metastatic carcinoma (usually breast or colon).
fractures?: Open fractures. 2. MM.
3. Paget disease.
108.A fracture, in which the bone get broken into multiple
pieces. Caused by crush injuries.:
Tx: surgical realignment of the bone and tx underlying disease
114.Open fracture tx.: Surgery always the right answer
- fracture causes break in the skin, high rates of bacterial
infection to the surrounding tissue
115.70 year old female patient presents to the ER with right
chest pain. Signs of ecchymosis on the right lateral thoracic
wall and an indentation of rib 6. Patient stated this all started
with a cough. Clinical problem?: Rib fracture likely pathologic.
116.Arm held to the side with externally rotated forearm with
severe pain.
Comminuted fractures Diagnosis?
109.A complete fracture from repetitive insults to the bone in What other conditions need to be evaluated for?:
question. Most common is of the metatarsal in an athlete with
persistent pain.:

Stress fracture
110.When diagnosing a stress fracture does X-ray often show Anterior shoulder dislocation.
evidence of fracture? If not what next?: Usually stress - injury causes strain on the glenohumeral ligaments (most
fractures are not evident on X-ray. common type 95%)
Do a CT or MRI.
111.What is the management of a person with a stress fracture?: X-ray best initial, MRI most accurate
Conservative: rehabilitation, reduced physical activity and r/o: Axillary artery or nerve injury.
casting. Tx: surgery relocation and immobilization
117.Causes of posterior shoulder dislocation?: 1. Seizures.
Surgery is indicated if symptoms are persistent. 2. Electrical burns.
112.A specific fracture of the vertebra in the setting of
osteoporosis.:

Compression fracture
- 1/3 of osteoporotic vertebral injuries are lumbar, 1/3
thoracolumbar, and 1/3 thoracic
Page 153
118.Arm is medially rotated and held to the side. 121.Trigger finger is this a clinical diagnosis? How does it
On exam pulses and sensation is diminished. Diagnosis? present? Treatment?: Yes.
Initial tests and most accurate test? Woman wakes up with severe pain in index finger which is flex
Treatment?: while all other are extended.

Pulling the finger causes a loud popping sound and pains


subsides.

Steroid injections and surgery if steroids fail.


122.40 year old woman wakes up with severe pain in index
finger which is flexed while all other are extended. Pulling
the finger causes a loud popping sound and pains subsides.
Diagnosis? Mechanism of disease?:

Posterior shoulder dislocation.


X-ray followed by MRI (most accurate)

Traction and surgery because pulses are diminished during


physical exam.
119.With a clavicular fracture what needs to be ruled out? What
Trigger finger.
is the treatment?: r/o: Subclavian artery and brachial plexus
injury.
Thickening of tendon sheath restricts gliding motion of flexor
tendons. Nodule may develop on the tendon and lock the finger
Simple arm sling.
in place.

caused by trauma, TTP clavicle region


Tx: Steroid injection, if this fails may need sx to cut the sheath
120.Falling on an outstretched hand and persistent pain in that is restricting the tendon
anatomical snuffbox that occurred today. Diagnosis? X-ray
123.Man over the age of 40 who complains of his hand not being
findings? Treatment?:
able to extend. Diagnosis? Mechanism of disease?
Treatment?:

Scaphoid fracture.
Dupuytren contracture.
X-ray won't show results for 3 weeks.
Palmar fascia becomes constricted.
Thumb spica cast.
Surgery.
124.Define fat embolism syndrome.: 1. Confusion.
2. Petechial rash, usually upper extremity and trunk.
3. SOB and tachypnea with dyspnea

Page 154
125.19 year old breaks her femur 3 days ago during a soccer 134.Tx compartment syndrome.: Medical emergency and
tryout. She is brought to the ER with confusion and shortness immediate fasciotomy but be completed in order to relieve
of breath. Splotchy magenta rash around base of neck and pressure before necrosis occurs.
back. ABG shows PO2 under 60 mm Hg. Diagnosis?: Fat 135. ___ injury is the most common knee ligament injury.: ACL
embolism syndrome. 136.With lateral and medial collateral ligament injury where is
126.What is the time frame of a fat embolism syndrome? the direction of force that caused these injuries?: Opposite to
Treatment?: Within 5 days of a fracture (femur). the ligament because force causes a pivoting motion one one
side causing the extension of the joint space on the opposite
Oxygenation with goal of over PO2> 95%. side, pathologically lengthening it.
137.What type of surgery is done for medial and lateral ligament
Intubation followed by mechanical ventilation if necessary. repairs?: Surgery not arthroscopic repair.
127.What diagnostic tests would support a fat embolism 138.Traumatic injury to the knee. Popping sound upon flexion and
syndrome diagnosis?: 1. ABG will show PO2 < 60 mm Hg. extension.: Meniscal injury
2. Chest x-ray showing infiltrates. - MRI
3. Urinalysis showing fat embolism. - Arthroscopic repair
128.60 year old man complains of bilateral leg pain of several 139.Unhappy triad?: 1. ACL
months. Pain is worse when walking several blocks and 2. MCL
improves when sitting down. Leaning forward alleviates the 3. Lateral or medial meniscus
pain. He is a nonsmoker. Likely diagnosis? Most appropriate
140.Confusion in a 70 year old with 50 pack year smoking
diagnostic step?: Spinal stenosis.
history. Mid abdominal pain. Pale in moderate distress.
Leg MRI.
Pressure of 80 over 55, pulse of 120. Palpable pulsatile mass
129.What disease history is a dead give away to discern in abdomen. Diagnosis?: Ruptured AAA.
pseudoclaudication from claudication?: 1. Bilateral - syncope in the setting of AAA is rupture until proven otherwise
claudication-like symptoms.
141.What are the management criteria of an asymptomatic
2. Relief of symptoms when leaning forward.
abdominal aortic aneurysm?: 3-4 cm: ultrasound every 2-3
130.Treatment for spinal stenosis?: 1. NSAIDs. years.
2. Surgery. 4-5.4 cm: ultrasound or CT every 6-12 mos.
131.Spinal stenosis can present with pain in what parts of the > 5.5 cm, asx: surgery.
body.: 1. Neck pain. 142.Screening guidelines for abdominal aortic aneurysms?: Over
2. Back pain. 65 in a smoker or former smoker will need an abdominal
3. B/L leg/buttock pain. ultrasound to screen AAA
143.What do the following tests provide in the work up of an
* worsen with walking, improve with flexion which opens the
abdominal aortic aneurysm?
spinal canal and relives nerve root compression
CT or MRI?
132.6 P's of compartment syndrome?: 1. Pain: first symptoms Ultrasound?: CT or MRI: information with regards to where the
2. Pallor: lack of blood flow abdominal aortic aneurysm is in relation to the other arteries.
3. Paresthesia: "pins and needles" sensation
4. Paralysis Ultrasound: information on size and a relatively inexpensive
5. Pulselessness: lack of distal pulses means of monitoring the AAA.
6. Poikilothermic: cold to the touch
144. Risk factors for aortic dissection include what?: 1. HTN (#1
risk factor)
*Compression nerves, blood vessels, and muscles inside a
2. Age over 40.
closed space also can be w/in a cast after setting a fracture
3. Marfan syndrome.
133.What are the early signs of compartment syndrome?
What are the late concerning signs of compartment Aortic dissection caused by tear in the intima of the aorta
syndrome?: creates a false lumen.
145.How can aortic dissection present?: 1. Sudden onset of
tearing chest pain that radiates to the back.
2. Asymmetric blood pressure in the right and left arms.
146.In an unstable patient with a clinical presentation of an
aortic dissection what its the best next step? What other
diagnostic options are there too?

What would be the best diagnostic test of choice if the patient


was stable?: TEE because of all the diagnostic tests this is the
quickest (best if pt unstable) another option is MRA
1. Early: Pain, paresthesia, and pallor.
Stable: MRA diagnostic test of choice
2. Late: Paralysis, poikilothermia, and pulselessness.

Page 155
147.What considerations need to be taken when managing an 160.Postoperatively pt is confused they are either ___ or ___.
aortic dissection? Get an ___.:
How does this impact management?: Is this an ascending or
descending dissection?

Ascending dissection: Emergent surgery and BP control.

Descending dissection: BP control.


148.What is the best medications to initially treat an aortic
dissection?: Beta-blockers to treat the HTN followed by
vasodilators such as sodium nitroprusside.
149.By what mechanism would giving vasodilators alone to treat
an aortic dissection a bad idea?: Reflex tachycardia will Hypoxic or Septic
increase the shearing forces of the pathology and worsen the
patient's outcome. ABG, CXR, blood cultures, urine cultures, and CBC
150.What are the 5 W's of postoperative fever. State the timeline 161.Post-op pt has severe hypoxia, tachypnea, accessory muscle
and meaning of the mnemonic.: POD 1-2: Wind - atelectasis or use for ventilation, and hypercapnia.: ARDS
post-op pneumonia. - dx with CXR that shows B/L pulmonary infiltrates w/o JVD (r/o
POD 3-5: Water - UTI. CHF) and tx with PEEP
POD 5-7: Walking - DVT, thrombophlebitis (IV lines), and PE. 162.PE presents with acute onset of CP with clear lung exam.
POD 7: Wound - Wound infections & cellulitis. Best initial diagnostic test?: EKG
POD 8-15: Drug fever or deep abscess. - sinus tachy w/ nonspecific ST segment changes
151.Post-op day 2, patient experiences fever. Next best step?: - follow w/ CT angiogram of chest
Chest x-ray followed by sputum cultures to evaluate for
atelectasis or pneumonia. tx: heparin bridge to coumadin
152.Post-op day 4, patient experiences fever. Next best step?: if pt has second PE on coumadin, IVC filter via inguinal
Urine analysis (look for positive nitrates and leukocyte catherization
esterase) followed by urine culture to evaluate for UTI. 163.Pt is allergic to contrast and high suspicion of PE which
153.Post-op day 6, patient experiences fever. Next best step?: diagnostic modality should be used after EKG?: V/Q scan if
Doppler ultrasound of extremities followed by changing of IV patient has an allergy to IV contrast cannot perform spiral CT
access lines and culture of IV tips to evaluate for DVT or scan
thrombophlebitis.
154.Post-op day 7, patient experiences fever. What are the
possibilities and what do you do?: Wound infection and
cellulitis - exam the wound for erythema, purulent discharge and
or swelling.
155.Tx POD 7 wound infection and cellulitis.: Incision and
drainage if abscess or fluid followed by antibiotics
156.Post-op day 8-15, patient experiences fever. What are
diagnostic causes and what do you do?: Drug fever - review
medications.

Deep abscess - CT scan for examination of a deep fluid


collection.
157.Tx POD 8-15 abscess.: CT guided percutaneous guided
drainage of the abscess, otherwise surgery
158.Patient experiences post-operative pneumonia on day 2.
What could have prevented this outcome?: Incentive
spirometry and vancomycin and tazobactam-pipercillin for HAP.
159.Patient has a DVT on post-operative day 6. What is the
management for this patient?: Heparin 5 days as a bridge to
coumadin for 3-6 mo.

Page 156
MTB Pulmonology
MTB Q and A

1. Reversible airway obstruction secondary to bronchial smooth 9. PFT in asthma show?: 1. DEC FEV1 + DEC FVC with DEC ratio
muscle hyperactivity.: Asthma FEV1/FVC
- airway inflammation, mucus plugging, smooth muscle 2. INC FEV1 of more than 12% and 200 mL with use of
hypertrophy albuterol
- can lead to chronic, irreversible airway obstruction 3. DEC FEV1 of more than 20% with the use of methacholine or
2. Asthma associated with which other comorbid conditions?: histamine
Atopic disorders 4. INC diffusion capacity of the lung for carbon monoxide
Obesity (DLCO)
3. Causes of acute exacerbations of symptoms of asthma?: • 10. Pt asymptomatic what is the most accurate test for reactive
Allergens such as pollen, dust mites, cockroaches, and cat airway disease?: 20% DEC FEV1 w/ use of methacholine or
dander histamine
• Infection - methacholine artificial form acetylcholine used dx testing
• Changes in weather (cold air) 11. Additional testing options asthma?: 1. CBC show INC
• Exercise eosinophil count
• Catamenial (related to menstrual cycle) 2. Skin testing ID specific allergens provoke
• Aspirin (DEC production PGE lung = bronchoconstriction), bronchoconstriction
NSAIDs, B-blockers)* (B2 receptor blockage in lung = 3. INC IgE levels suggest allergic etiology
bronchoconstriction), tobacco smoke - also seen allergic bronchopulmonary aspergillosis
• GERD 12. Asthma treatment step wise approach?: Step 1. Inhaled SABA
4. Pt presents with wheezing, acute onset SOB, cough. Some pts (Albuterol, Pirbuterol, Levelbuterol)
chest tightness, INC sputum production common.: Asthma Step 2. SABA + Low-dose inhaled corticosteroid (ICS) initial
- fever only present sometimes usually viral URI as cause of long term control agent (Beclomethasone, budesonide,
asthma flair flunosolide, fluticasone, mometasone, triamcinole)
5. Which of the following is most likely to be found in an Step 3. Add LABA (Salmeterol or formoterol) to SABA and ICS,
asthmatic patient?: 1. Symptoms worse at night or INC dose ICS
2. Nasal polyps and sensitivity to aspirin Step 4. INC dose ICS to max in addition to LABA and SABA
3. Eczema or atopic dermatitis on PE Step 5. Omalizumab may be added to SABA, LABA, ICS those
4. INC length of expiratory phase of respiration INC IgE levels
- I/E ratio decreases (normal is 1:2) Step 6. Oral corticosteroids (prednisone) added when all other
5. INC use of accessory respiratory muscles (intercostals) therapies are not sufficient
6. The best initial test in an acute 13. What are some alternative long-term control agents asthma?:
exacerbation asthma?: Peak expiratory flow (PEF) or arterial 1. Cromolyn and nedocromil to inhibit mast cell mediator
blood gas (ABG) release and eosinophil recruitment
- ABG if mild, early exacerbation: mild hypoxia, respiratory 2. Theophylline-cardio/neuro tox
alkalosis 3. Leukotriene modifiers: montelukast, zafirleukast, or zileuton
- ABG if severe, late exacerbation: severe hypoxia, respiratory (best w/ atopic pts)
acidosis 14. Adverse effects systemic corticosteroids.: - Osteoporosis
7. Most accurate diagnostic test in asthma?: - Cataracts
- Adrenal suppression and fat redistribution
- Hyperlipidemia, hyperglycemia, acne, hirsutism (women)
- Thinning skin, striae, easy bruising
15. Adverse effects of inhaled steroids?: Dysphonia

Oral candidiasis
16. SE Zafirlukast?: Hepatotoxic

Associated with Churg-Strauss syndrome


17. Which vaccine are given to asthma pts?: Influenza
Pneumococcal vaccine
18. How is the severity of an asthma exacerbation quantified?: 1.
Pulmonary function test (PFTs) DEC PEF (peak expiratory flow) (based height)
- used in stable outpatient setting 2. ABG with an INC A-a gradient
- spirometry show DEC ratio FEV1/FVC 19. Tx asthma exacerbation.: 1. O2
- FEV1 DEC > FVC 2. Albuterol
8. In asthma CXR results?: Normal, some hyperinflation depending 3. Steroids: give immediately
on degree air trapping

Most useful excluding: pneumonia, CHF, pneumothorax


20. When is magnesium useful in acute asthma exacerbation?: 32. Acute exacerbations of COPD what does the ABG show?: INC
Magnesium used when pt not responding to several rounds of pCO2 and hypoxia
albuterol while waiting for steroids to take effect - respiratory acidosis not sufficient metabolic compensation and
- helps relieve bronchospasm bicarb will be elevated to compensate
21. Should Epi be given acute flair asthma?: NO, rarely used and 33. CBC in COPD might show?: INC hematocrit from chronic
only as drug of last resort hypoxia
- no more effective than albuterol, more adverse SE 34. EKG vs ECHO in COPD pt?: EKG:
- Ipotropium should be used but does not work as quickly as - RAH, RVH
albuterol - a. fib, MAT (multifocal atrial tacky)
22. Which drugs are NOT effective in the treatment of acute
exacerbation of asthma?: Theophylline ECHO: RAH, RVH, pulmonary HTN
Cromolyn or nedocromil 35. Which interventions in COPD that improves mortality and
Leukotriene modifiers delays progression of dz?: - Smoking cessation
Omalizumab - O2 therapy pO2< 55 or saturation < 88%; mortality directly
Salmeterol correlated number of hours O2 used (pt pulmonary HTN, high
23. Ipratropium and tiotropium is very effective in ___.: COPD HCT, or cardiomyopathy use pO2< 60 sat <90%)
24. Pt not responding to O2, albuterol, and steroids, develops - Influenza and pneumococcal vaccines
respiratory acidosis (INC pCO2) due to an asthma 36. Which modalities improve symptoms of COPD but do not
exacerbation. Next step?: Consider endotracheal incubation decrease disease progression?: 1. SABA (albuterol)
and mechanical ventilation 2. Anticholinergic agents: tiotropium (long act), ipratropium
25. Productive cough > 3 mo/ year for 2 consecutive years.: (short act)
Chronic bronchitis 3. Steroids
26. What does tobacco smoke do to cause COPD?: Destroys 4. LABA (salmeterol)
elastin fibers 5. Pulmonary rehabilitation
27. SOB worsened by exertion, Intermittent exacerbations with 37. T/F: Inhaled anticholinergic agents are most effective in
INC cough, sputum brought on my infection, "wide expanded COPD.: TRUE
chest wall", muscle wasting and cachexia.: COPD 38. When all medical therapy is insufficient for COPD next step?:
- barrel chest INC air trapping Refer for lung transplantation
- wasting + cachexia: COPD now through to have a component 39. What is the tx for acute exacerbations of chronic bronchitis?:
of systemic inflammation - Combo bronchodilators and corticosteroid therapy (systemic)
28. Best initial test COPD?: - Antibiotics cover (s. pneumo, H. influenzae, Moraxella
catarrhalis)
40. Most effective antibiotics to use to tx acute exacerbations of
chronic bronchitis?: 1. Macrolides (azithromycin,
clarithromycin)
2. Cephalosporins (cefuroxime, cefiximine, cefaclor, ceftibuten)
3. Amoxicillin/clavulanic acid
4. Quinolones: levofloxacin, moxifloxacin, gemifloxacin

If allergy above agents use: Doxy, TMP/SMX


41. Permanent anatomic abnormality that cannot be reversed or
cured, chronic destruction and dilation of large bronchi.:
CXR
- INC AP diameter
- Air trapping and flattened diaphragms
29. Most accurate diagnostic test COPD?: PFT
- DEC FEV1, FEV, FEV1/FVC ratio (<70%)
- INC TLC due to INC RV
- DEC DLCO in emphysema (destruction of alveolar septa where
capillaries are found)
- incomplete improvement with albuterol Bronchiectasis
- little or no worsening w/ methacholine
30. What is the reversibility of COPD with bronchodilators
(albuterol)?: 1. No reversibility (typically irreversible)
2. 50% show some degree of response
3. Complete
31. What is full reversibility in response to bronchodilators?:
Greater than 12% INC and 200 mL INC in FEV1
42. Most common cause Bronchiectasis?: CF: 50% cases 48. ___ is hypersensitivity to fungal antigens that colonize
bronchial tree. Almost exclusively with asthma and atopic
Other causes: disorders. Cough, wheezing, hemoptysis.: ABPA (allergic
- TB (and mycobacterium avium) bronchopulmonary aspergillosis)
- Pneumonia (staph) 49. Asthmatic with recurrent episodes of brown-flecked sputum
- Panhypogammaglobulinemia and immune deficiency and transient infiltrates on X-ray.: ABPA (allergic
- Foreign body or tumors bronchopulmonary aspergillosis)
- Allergic bronchopulmonary aspergillosis (ABPA) 50. ABPA diagnostic test?: • Peripheral eosinophilia
- Collagen- vascular dz such as RA • Skin test reactivity to aspergillus antigens
- Immotile cilia syndrome (Kartagener) • Ppt antibodies to aspergillus on blood test
43. Best initial test Bronchiectasis?: • Elevated serum IgE levels
• Pulmonary infiltrates on chest X-ray or CT
51. Allergic Bronchopulmonary Aspergillosis (ABPA) tx?: 1. Oral
steroids (prednisone) for severe cases
- Inhaled steroids aren't effective for ABPA (cannot deliver
high enough dose of steroid to be effective)
2. Itraconazole orally for recurrent episodes
52. CF is an autosomal ___ disorder of ___ transporter.:

CXR
- dilated, thickened bronchi, sometimes with a "tram-track" which
is thickening of bronchi
44. Most accurate test Bronchiectasis?:

recessive
CFTR
- Bone: clubbing, arthritis
- Spleen: hypersplenism
- Stomach: GERD
53. GI involvement in CF?: 1. Meconium ileus in infants w/
abdominal distention
2. Pancreatic insufficiency (90%) with steatorrhea and vitamin
A, D, E, K malabsorption
High resolution CT 3. Recurrent pancreatitis
- sizable airways are larger in diameter than corresponding 4. Distal intestinal obstruction
bronchial arteries 5. Biliary cirrhosis
54. GU involvement in CF?: 1. Men infertile 95% azoospermia with
* impossible to dx bronchiectasis w/o imaging such as CT
the vas deferens missing in 20%
45. ___ is the only way to determine specify bacterial etiology in 2. Women are infertile because chronic lung dz alters menstrual
Bronchiectasis?: Sputum cx cycle and thick cervical mucus blocks sperm entry
46. Recurrent episodes of very high volume purulent sputum 55. The most accurate test CF?: INC sweat chloride test
production. Hemoptysis can occur. Dyspnea and wheezing - Pilocarpine INC acetylcholine levels with INC sweat
75% cases. Wt loss, anemia of chronic dz, crackles on lung production
exam, clubbing is uncommon.: Bronchiectasis - above 60 meq/mL on repeated testing establishes the
47. Tx Bronchiectasis: 1. Chest physiotherapy ("cupping and diagnosis
clapping" of lung) and postural drainage
- Essential for dislodging plugged-up bronchi
2. Tx each episode of infection as it arises
- Same antibiotics as for COPD exacerbations
- Only difference is inhaled antibiotics seem to have some
efficacy
3. Rotate antibiotics, 1 weekly each month
4. Surgical resection
- Focal lesions
56. ___ accounts for 90% of deaths in CF.: 65. Pt presents with fever, cough, dyspnea, "bronchial" breath
sounds and egophony. Chills or "rigors". Chest pain.:
Pneumonia
- rigors => sign of bacteremia
- chest pain due to inflammation of pleura (pleuritic changing
with respiration)
- abdominal pain due to lower lobe irritation the intestine through
diaphragm
66. Hemoptysis from necrotizing disease, "currant jelly" sputum.
Which pneumonia pathogen?: Klebsiella pneumoniae
67. Foul-smelling sputum, "rotten eggs": Anaerobes
68. Dry cough, rarely severe, bullous myringitis. Which
Lung disease
pneumonia pathogen?: Mycoplasma pneumoniae
57. Young adult pt presents with chronic lung disease, recurrent
69. GI symptoms (abdominal pain, diarrhea) or CNS symptoms
episodes of infection, sinus pain and polyps, hemoptysis,
such as H/A and confusion. Which pneumonia pathogen?:
bronchiectasis, dyspnea.: CF
Legionella
58. In CF which pancreatic cells are spared?: Islets spares, beta
70. AIDS with < 200 CD4 cells which type of pneumonia
cell function normal until much later in life
pathogen?: Pneumocystis
59. Sputum culture in CF pts would most likely show?: -
71. Pneumonia infections with a "dry" or nonproductive cough?: •
Nontypable Haemophilus influenzae
Mycoplasma
- Pseudomonas aeruginosa
• Viruses
- Staphylococcus aureus
• Coxiella
- Burkholderia cepacia
• Pneumocystis
• Chlamydia
* repeated sputum cx important since these bugs can be highly
resistant
- involve more the interstitial space and often leave the air
60. Tx CF: 1. Antibiotics are routine
spaces of the alveoli empty -> less sputum production
- Eliminating colonization: difficult
72. Best initial test for all respiratory infections?: CXR
- Sputum culture: essential to guide tx
- Inhaled aminoglycosides: almost exclusively limited to CF 73. Best way to determine specific etiology of CAP?: Sputum
2. Inhaled recombinant human deoxyribonuclease (rhDNase) gram stain and sputum cx
- Breaks down massive amounts of DNA in respiratory mucus 74. The term ___ refers to an organism not visible on gram stain
that clogs up airways and not cultural on standard blood agar.: Atypical pneumonia
3. Inhaled bronchodilators - Albuterol - Mycoplasma
4. Pneumococcal and influenza vaccinations - Chlamydia
5. Lung transplantation - Legionella
6. Ivacaftor Inc activity CFTR in some pts - Coxiella
61. ___ pneumonia occurring before hospitalization or w/in 48
hours of hospital admission.: Community-acquired pneumonia Viruses
(CAP) *30-50% cases of CAP
62. Most common cause of CAP?: Strep. pneumoniae 75. CXR pneumonia shows B/L interstitial infiltrate are seen with
which bugs?: - Mycoplasma
63. Common CAP pathogens and their associated conditions?:
- Viruses
- Coxiella
- Pneumocystis
- Chlamydia

* Nonproductive cough
* X-rays lag behind clinical finding

1st CXR can be falsely negative 10-20%


76. When is sputum culture "adequate"?: > 25 WBC and < 10
epithelial cells
64. ___ are main ways to distinguish pneumonia from
77. When are blood cx positive in pneumonia?: 5-15% cases of
bronchitis?: Dyspnea
CAP, S. pneumoniae blood cx can be positive
High fever
Abnormal CXR
78. What test can be done if pt not responding to treatment and 84. Inpatient tx CAP?: Respiratory fluoroquinolones: levofloxacin
has a pleural effusion?: or moxifloxacin

OR

Ceftriaxone and azithromycin - IV


85. Reasons to hospitalize pt CAP?: 1. Hypotension systolic < 90
2. RR > 30
3. pO2 < 60 mmHg, pH < 7.35
4. Elevated BUN > 30
5. Sodium < 130
6. Glucose > 250
7. Pulse > 125/min
8. Confusion
Thoracentesis 9. T> 104
- look for empyema (infected pleural effusion) 10. > 65 years old or comorbidities such as cancer, COPD,
- empyema acts like an abscess and will improve more rapidly if CHF, renal failure, or liver dz
it is drained with a chest tube 86. T/F: Hypoxia and hypotension as single factor are a reason to
79. Fluid composition empyema.: LDH > 60% of serum hospitalize a patient.: TRUE
Protein > 50% of serum 87. CURB 65 rule?: Confusion
WBC > 1000/microl Uremia
pH < 7.2 Respiratory Distress
BP low
* suggestive of infection age: 65 or greater
80. When is bronchoscopy warranted in tx CAP?: Only if sputum
stain and sx and blood cx don't yield an organism and pts admit for management of CAP
condition is worsening despite empiric therapy *0-1 point: Home
- exception is pneumocystis pneumonia in which noninvasive > 2 pts : admission
test rarely reveals dx 88. Infected pleural effusion or empyema will respond most
81. T/F: S. pneumo can be tested via urine antigen.: TRUE rapidly to drainage by?:
82. Name some specific diagnostic test for organism CAP?
- M. pneumoniae
- C. pneumoniae
- Legionella
- Chlamydia psittaci
- Coxiella burnetii
- Pneumocystic jiroveci (PCP):

Chest tube or thoracotomy


- large effusion hard to sterilize
- each side of the chest can accommodate 2 to 3 liters of fluid
89. Etiology of pleural effusions transudative vs exudative?:

83. Outpatient tx CAP?:

Exudates are caused by infection and cancer.


90. ___ is defined as the presence of both air and fluid within the 99. Ventilator associated pneumonia when should a sputum cx be
pleural space surrounding the lung. An upright chest x-ray obtained?: Nearly worthless
will show air fluid levels. The horizontal fluid level is usually - due to colonization of endotracheal tube (ET)
well defined and extends across the whole length of 100. Give the order of tests that are given in the order from least
hemithorax.: accurate but easiest to do, to the most accurate but most
dangerous for VAP?: 1. Tracheal aspirate - ET tube contents,
inaccurate
2. Bronchoalveolar lavage - bronchoscope can be
contaminated passing nasopharynx
3. Protected brush specimen - bronchoscope tip covered when
passes nasopharynx and uncovered inside lungs
4. Video-assisted thoracoscopy (VAT) - scope in chest wall
biopsy lung sample
5. Open lung biopsy - most accurate diagnostic test for VAP,
much more complicated
101. What is the most accurate diagnostic tool for VAP?: Open
Hydropneumothorax lung biopsy
- chest tube drainage most effective way to remove this - greater morbidity
91. Who should receive the pneumococcal vaccine?: • Everyone > - potential complications because of the need for thoracotomy
65 should receive vaccination with 12 polyvalent vaccine, 102. ___ is air abnormally leaking into the soft tissue of the chest
followed in 6-12 mo. with 23 polyvalent vaccine wall.:
• Chronic heart, liver, kidney, lung (including asthma) dz

Other reasons:
- Asplenia (sickle cell disease)
- Hematologic malignancy (leukemia, lymphoma)
- Immunosuppression: DM, alcoholics, corticosteroid users,
AIDS or HIV positive
- CSF leak and cochlear implantation recipients
92. Pneumococcal vaccination timing pt over 65?: Generally
healthy: single dose at 65

• Use 13 first, then 23 in 6-12 mo.

• If first vaccination was given before 65 or with the other


conditions previously described, second dose should also be Subcutaneous emphysema
given 5 years after the first dose - cause poor placement chest tube
93. Do healthcare works need to get the pneumococcal vaccine?: - damage 2ndary to positive pressure ventilation
NO 103. Tx of VAP: Combine 3 different drugs
94. ___ is pneumonia developing more than 48 hours after 1. Antipseudomonal beta-lactam
admission or after hospitalization in the last 90 days.: - Cephalosporin (ceftazidime or cefepime) or
Healthcare-Associated pneumonia or hospital-acquired - Penicillin (piperacillin/tazobactam or
pneumonia (HAP) - Carbapenem (imipenem, meropenem, or doripenem)
95. HAP more likely which organisms?: Gram negative bacilli 2. Second antipseudomonal agent
- E.coli - Aminoglycoside (gentamicin or tobramycin or amikacin) or
- Pseudomonas - Fluoroquinolone (cirpfloxacin or levoflaxacin)
3. MRSA agent
96. Tx for HAP infection?: - Antipseudomonal cephalosporins:
- Vanco or Linezolid
cefepime/ceftazidime
OR 104. Dangerous SE Imipenem?: Seizures, exerted through kidneys
- Antipseudomonal penicillin: piperacillin/tazobactam - renal failure can cause rise in Imipenem leading to toxicity
OR 105. Why can't daptomycin be used for VAP?: Daptomycin is
- Carbapenems: imipenem, meropenem, or doripenem inactivated by surfactant
106. Aspiration pneumonia happens in the ___ lobe when lying
* Macrolides (azithromycin, clarithromycin) are unacceptable flat?: Right Upper lobe
97. Piperacillin and ticarcillin are always used in combo with?: 107. Lung abscesses occur in pts with?: Large volume aspiration
Beta-lactamase inhibitor such as tazobactram or clavulanic acid of oral/pharyngeal contents usually with poor dentition
98. Pt has fever and/or rising WBC count, new infiltrates on CXR,
purulent secretions coming from the endotracheal tube.: VAP
-> ventilator associated pneumonia
108.Large volume aspiration occurs as a result of ___ that can 115.Most accurate test pneumocystis pneumonia.:
lead to lung abscess formation?: Stroke with loss of gag reflex Bronchoalveolar lavage
Seizures 116. If the sputum stain is positive in PCP what is the next step?:
Intoxication if positive, there is no need to do further testing
Endotracheal intubation
109.Pt that is intubated develops large-volume sputum that is - if sputum is negative, bronchoscopy must be done
foul smelling. Wt loss.: Lung abscess 117.Best initial tx pneumocystis pneumonia: TMP/SMX
- anaerobes - Add steroid to DEC mortality if PCP is severe (pO2 < 70 or
110. Best initial test for lung abscess?: an A-a gradient > 35)

Alternative TMP/SMX if allergy:


Atovoquone if PCP only mild hypoxia
118.If there is toxicity from TMP/SMX when tx PCP what are the
alternative meds?: Clindamycin and primaquine (primaquine
contraindicated pt has G6PD def.)

OR

Pentamidine
119. SE of TMP/SMX: Rash most common

Bone marrow suppression


120.What tx should be started to prevent PCP in those with AIDS
CXR whose CD4 count below 200?:
- cavity with air-fluid level

Chest CT more accurate


111. What is the most specific test for lung abscess?:

* If CD4 count maintained above 200/microl for several months,


prophylaxis can be stopped
121.Dapsone is contraindicated in those with ___.: G6PD
deficiency
Lung biopsy can establish specific microbiology etiology 122.Almost all patients with TB have 1+ established risk
factors?: - Recent immigrants (in past 5 years)
- sputum cx wrong answer, everyone's sputum has anaerobes - Prisoners
from mouth flora - HIV positive
112.Which drugs are the best for lung abscess coverage?: - Healthcare workers
Clindamycin or Penicillin - Close contact of someone with TB - Steroid use
- Hematologic malignancy
113.Patient with AIDS presenting with dyspnea on exertion, dry
- Alcoholics
cough, and fever. Question will often suggest or directly state
- DM
that CD4 count is low (< 200/ L) and that patient isn't on
prophylaxis. 123.Pt is an immigrant that presents with fever, cough, sputum,
wt loss, hemoptysis, and night sweats.: TB
Dx?: Pneumocystis Pneumonia (PCP) - symptoms > 3 weeks duration
- P. jiroveci
- Also occurs in chronically immunosuppressed patients those
on => long term high dose steroids
114.Best initial test for Pneumocystis Pneumonia.: CXR
- B/L interstitial infiltrates

ABG
- look for hypoxia INC Aa gradient

LDH always elevated


- if LDH normal don't answer PCP
124. Best initial test TB?: 138.What is considered a positive PPD test?: • Induration is
counted not erythema
• Induration > 5 mm
- HIV-positive patients
- Glucocorticoid users
- Close contact with active TB patients
- Abnormal calcifications CXR
- Organ transplant recipients

• Induration > 10 mm
- Recent immigrants (past 5 years)
- Prisoners
- Healthcare workers
CXR - Close contacts with TB patients
- cavitary upper lobe infiltrate - Hematologic malignancy, alcoholics, DM
125.Sputum stain and cx specific for TB includes?: culture
specific for acid-fast bacilli (mycobacteria) (Ziehl-Neelsen) • Induration > 15 mm
must be done 3x to fully exclude TB - Those with no risk factors
126.If have 3 negative acid-fast stains for TB but clinical 139.T/F: Everyone with a reactive PPD should have a CXR to
suspicion is high what is the next step?: Bronchoscopy with exclude active dz?: TRUE
BAL or pleural biopsy 140.Two-stage testing PPD.: • If pt never had a PPD skin test
127.T/F: PPD skin test is the best test for TB in symptomatic pt.: before, a second test is indicated within 1-2 weeks if first test is
FALSE, PPD skin test never the best test for TB in symptomatic negative
patient - 1st test maybe falsely negative
128.What is the single most accurate diagnostic test TB?: pleural • If 2nd test is negative: truly negative
biopsy • If 2nd test is positive: first test was false negative

129.When the smear is positive for TB what is the tx?: begin tx


If the first test is positive, a second test isn't necessary
all 4 drugs:
Rifampin 141. Tx for positive PPD.: • After active TB has been excluded with
Isoniazid a CXR, patients should receive 9 months of isoniazid
Pyrazinamide • Positive PPD confers a 10% lifetime risk of TB
Ethambutol • Isoniazid results in 90% reduction in risk; after isoniazid
lifetime risk of TB goes from 10% to 1%
After using RIPE for first 2 months: 142.T/F: Once the PPD is positive it will always be positive in
- Stop ethambutol and pyrazinamide the future.: TRUE
And 143. Does previous BCG have any effect on the tx of TB?: NO
- Continue rifampin and isoniazid for next 4 months
130.When is the tx of TB extended longer than the 6 mo. standard Previous BCG has NO effect on these recommendations.
of care tx?: Extended to 9 mo. for:
- Osteomyelitis If PPD is positive, the patient must take isoniazid for 9 months
- Miliary tuberculosis (wide spread dz) even if he or she had BCG.
- Meningitis 144.What is Interferon gamma release assay test?: Interferon
- Pregnancy or any other time pyrazinamide isn't used gamma release assay is a blood test equal in significance to
131. All TB medications cause?: Hepatoxicity PPD to exclude TB exposure
- Don't stop them unless the transaminases rise 3-5x the upper
limit of normal - no cross-reaction with BCG
132.Red color to body sections which TB drug SE?: Rifampin 145. Qualities of benign and malignant pulmonary nodules?:
- no management, benign finding
133.Peripheral neuropathy which TB drug SE?: Isoniazid
- pyridoxine (VB6) to prevent
134.Hyperuricemia which TB drug SE?: Pyrazinamide
- no tx unless symptomatic
135.Optic neuritis/color vision which TB drug SE?: Ethambutol
- DEC dose in renal failure
136.Which TB drugs should pregnant women not receive?: 146.T/F: Biopsy all enlarging lung lesions, particularly if they
Pyrazinamide are rapidly enlarging.: TRUE
Streptomycin - however, doubling in size < 30 days more likely to be
infectious than malignant (think about clinical scenario to
137.Use of steroids in TB?: • Glucocorticoids DEC risk of
decide)
constrictive pericarditis in those with pericardial involvement
• DEC neurologic complication in TB meningitis
147.What is the management of highly probable malignant lung 158. Cotton exposure.
lesion?: RESECTION
148.Which patients are considered intermediate probability that Name the pneumoconiosis?: Byssinosis "brown lung disease"
the lung lesion is malignant?: - Age range gap (between 30 159. Electronic manufacture.
and 40) OR
- Size (between 1 cm and 2 cm) Name the pneumoconiosis?:
149.What is the management of intermediate probability
lesions?: 1. Sputum cytology
- if positive = highly specific, "most appropriate next step in
management" is resection
--> If negative cytology doesn't exclude malignancy
2. Bronchoscopy/ transthoracic needle biopsy
- Each is "the most appropriate next step" in most patients with
intermediate probability of malignancy
- Bronchoscopy for central lesions
- Transthoracic biopsy for peripheral lesions Berylliosis - biopsy shows granulomas
150.What is the most common adverse effect of a transthoracic 160. Moldy sugar cane.
biopsy?: Pneumothorax
151.PET scan is more accurate with a lung lesion > ___ cm.: 1 Name the pneumoconiosis?: Bagassosis
cm 161.All forms of pulmonary fibrosis, regardless of etiology,
152.PET scan use in lung lesions?: • Tells whether content of present with?: - Dyspnea worsening on exertion
lesion is malignant without biopsy - Hypoxia worsening on exertion
• Malignancy has increased uptake of tagged glucose - Fine rales or "crackles" on exam
• Sensitivity of PET scan is 85 - 95% - Loud P2 heart sound (if pulmonary HTN present)
• Negative scan points away from malignancy - Clubbed fingers
153.What is VAPS in management of intermediate probably 162. Best initial test interstitial lung disease?: CXR
lesions?: • VATS is both more sensitive and more specific 163.What is more accurate than a CXR but not as accurate as
than all other forms of testing lung biopsy for interstitial lung dz?:
• Frozen section in operating room allows for immediate
conversion to an open thoracoscopy and lobectomy if
malignancy is found
154.Benign pulmonary nodules often represent scar from
previous infection such as?: 1. Immigrant think TB
2. SW US, think coccidioidomycosis
3. Ohio River Valley, think histoplasmosis
155.Name some specific causes of pulmonary fibrosis?: 1.
Idiopathic; interstitial pulmonary fibrosis
2. Radiation
3. Drugs: bleomycin, busulfan, amiodarone, methylsergide,
nitrofurantoin, cyclophosphamide High resolution CT
156. Sandblasting, rock mining, tunneling. - MOST accurate: LUNG BIOPSY
164.What would the PFT show in interstitial lung disease?: •
Name the pneumoconiosis?: Silicosis FEV1, FVC, TLC, and RV all , but since everything is decreased
157. Shipyard workers, pipe fitting, insulators.
FEV1/FVC ratio will be normal
Name the pneumoconiosis?:
• DLCO DEC in proportion to severity of alveolar septal
thickening
165.Tx interstitial lung disease?: - White cell or inflammatory
infiltrate; prednisone should be used (berylliosis most likely to
respond to steroids)
- Mostly fibrosis (e.g., IPF), steroids typically NOT effective

Asbestosis
166.Young African American woman with SOB on exertion, fine 173.Bronchioalveolar lavage in sarcoidosis shows?: Elevated
rales on lung exam, w/o wheezing, erythema nodosum level of Helper cells
(painful nodules on shins), lymphadenopathy.: 174.PE derives from DVT of the large vessels in?: legs in 70% of
cases and pelvic veins in 30%
175. Virchow's triad:

Sarcoidosis - noncaseating granulomas


Thrombophilia -> antiphospholipid syndrome
- > parotid gland enlargement
- facial palsy
Stasis: hip replacement surgery
- heart block, restrictive cardiomyopathy
176. Best initial test for PE?:
- CNS involvement
- iritis and uveitis
167.TEST: Answer ___ when a CXR or CT shows hilar adenopathy
in a generally healthy African American woman.:

Sarcoidosis
168. Best initial test suspect sarcoidosis?:
CXR
EKG
ABG

next best step CT angiogram (spiral CT scan)


177. Most accurate test PE?:

• Hilar lymphadenopathy > 95% with sarcoidosis

• Parenchymal involvement can also be present in combination


with lymphadenopathy
169.Most accurate test sarcoidosis?: Biopsy lymph node
- granulomas are noncaseating
170.Which levels are elevated in sarcoidosis?: ACE levels: 60%
pts
Hypercalciuria: 20%
Hypercacemia: 5% Angiography
- granulomas in sarcoidosis make vitamin D - can be fatal 0.5% cases
171.What does PFT show sacroidosis?: PFTs: restrictive lung - rarely done
disease - adverse effect (allergy, renal toxicity, death)
- Decreased FEV1, FVC, and TLC with a normal FEV1/FVC ratio 178.CXR finding PE?: Usually normal
172. Drug of choice tx sarcoidosis?: Prednisone • Atelectasis: Most common
• Wedge-shaped infarction (Hampton hump), and oligemia of one
- asymptomatic hilar adenopathy does NOT need to be tx lobe (Westermark sign) are much less common
179. EKG findings PE?: 189.When are thrombolytics the right answer for PE?:
Hemodynamically unstable pt (hypotension systolic BP < 90)

Acute RV dysfunction

* no specific time limit as with MI or stroke for thrombolytics in


PE
190.When are direct-acting thrombin inhibitors (argatroban,
lepirudin) the answer?: Heparin-induced thrombocytopenia
• Usually shows sinus tachycardia (HIT) (Fondaparinaux is an inhibitor that is an alternative to
• Most common abnormality is nonspecific ST-T wave changes heparin)
(SI QIII TIII pattern wrong answer rarely seen) 191. When is aspirin the answer for PE?: NEVER
• Only 5% will show right axis deviation, RV hypertrophy or 192.Pulmonary HTN is
RBBB systolic BP > ___.
180.ABG in PE?: Hypoxia and respiratory alkalosis (high pH and diastolic BP > ___.: systolic > 25 mm Hg
low pCO2) with normal CXR diastolic > 8 mm Hg
181. V/Q scan is only first line in ___.: 193.Causes of Pulmonary HTN?: 1. Primary
- Idiopathic dysfunction of pulmonary arteries

2. Secondary causes include:


- L sided-heart failure (MCC)
- Intracardiac L-> R shunting
- Hypoxic vasoconstriction from chronic lung disease (e.g.,
COPD)
- Thromboembolic disease
194.Chronic hypoxemia of any cause can lead to ___.: Pulmonary
Pregnancy HTN
- a completely normal scan excludes a clot
195.Dyspnea, fatigue, chest pain (exertional), wide splitting S2
182.D-dimer usefulness in PE?: • Test is very sensitive (better with a loud P2 or tricuspid and pulmonary valve
than 97% negative predictive value) insufficiency. Syncope (exertional).: Pulmonary HTN
• Specificity poor. Any clot or bleeding elevates D-dimer level
196. Best initial test for pulmonary HTN?:
• Negative test excludes clot
• Positive test doesn't mean anything
183.___ is answer when pretest probability of PE is low and you
need a simple, noninvasive test to exclude thromboembolic
disease: D-dimer
184.___ good test if the V/Q and spiral CT do not give a clear
diagnosis.: LE Dopplers
185.Algorithm for PE?: Spiral CT negative -> V/Q or LE Doppler ->
negative -> withhold therapy with heparin
186. Best initial therapy PE?: Heparin

Warfarin should be started at the same time as the heparin in


order to achieve a therapeutic INR 2-3x normal as quickly as
possible
CXR + CT
Alt Heparin: Fondaparinux
- dilation of the proximal pulmonary artery w/ narrowing or
187.When can Rivaroxaban and dabigatran be used in tx PE?: "pruning" of distal vessels
Oral gets do not require INR monitoring can be used for tx of PE
- reach therapeutic effect in several hours, instead of several
days like warfarin
- used after initial therapy with LMWH
188.When is an inferior vena cava (IVC) filter the right answer
for PE?: • Contraindication to the use of anticoagulants (e.g.,
melena, CNS bleeding)
• Recurrent emboli while on heparin or fully therapeutic warfarin
(INR of 2-3)
• RV dysfunction with an enlarged RV on ECHO. In this case,
the next embolus, even if seemingly small, could be potentially
fatal
197.Most accurate test and most precise method to measure 206.Tx of obstructive sleep apnea?: 1. Weight loss and avoidance
pressures in pulmonary HTN?: of alcohol
2. Continuous positive airway
pressure (CPAP)
3. Surgical widening of airway
(uvuloplatopharyngoplasty) if this fails
4. Avoid use of sedatives
207.Severe hypoxia, poor lung compliance, noncardiogenic
pulmonary edema. Dx.: ARDS
- caused by endothelial injury at the level of alveolus, making
lung cells "leaky" so that alveoli fill up with fluid
- DEC surfactant
208. Causes of ARDS?: Idiopathic in nature
- Sepsis or aspiration
- Lung contusion/blunt trauma
- Near-drowning
- Burns or severe, acute pancreatitis
209. CXR ARDS:
Right heart or Swan-Ganz catheter
- PCWP (LA pressure) LVEDP
198.EKG and ECHO in pulmonary HTN show?: EKG: Right axis
deviation, RA, RV hypertrophy

ECHO: RA and RVH, doppler estimates pulmonary artery


pressure

V/Q scan ID chronic PE as cause of chronic pulmonary HTN


199.CBC in pulmonary HTN show?: polycythemia from chronic
hypoxia
Dense B/L infiltrates become confluent "white out"
200.Tx of Pulmonary HTN.: 1. Correct underlying cause
2. Idiopathic disease tx: Air bronchograms are common
- Inhaled/ IV prostacyclin analogues (pulmonary arterial
210. ARDS defined as having PaO2/FiO2 ratio < ___.: 300
vasodilators): epoprostenol, treprostinil, iloprost, beraprost
- Endothelin antagonists: bosentan
If pO2 (on ABG) is 70 while breathing 50% oxygen, the ratio is
- Phosphodiesterase inhibitors: sildenafil
70/0.5 or 140 = ARDS
3. O2 slows progression
211. What is the wedge pressure in ARDS?: Normal < 18 mmHg
201.What is the only curative way to treat idiopathic pulmonary
HTN?: lung transplantation
w/ normal findings on RH catherization
202.___ most commonly identified cause of obstructive sleep
212.Tx ARDS?: • Low tidal volume mechanical ventilation is the
apnea?: OBESITY
best support while waiting to see if lungs will recover
203.Most common presentation of obstructive sleep apnea?: -Use 6 mL/kg of TV
Daytime somnolence and history of loud snoring • Steroids aren't clearly beneficial in most cases
-They may help in late-stage dz in which pulmonary fibrosis
other symptoms: develops
- HA (secondary to early morning INC bicarbonate after night of
213.When is PEEP used in ARDS?: Severe hypoxia leads to high
obstructions)
FIO2 and FIO2 above 50% are toxic
- impaired memory/judgement
-> PEEP is used when pt is undergoing mechanical ventilation to
- Depression, HTN
DEC FIO2
- ED
- "Bull neck"
Maintain plateau pressure < 30 cm of water measured on
*Arrhythmias and erythrocytosis is common => RV failure and
ventilator
pulmonary HTN
214.What tx is proven to reverse ARDS?: There is NO tx proven to
204.Most accurate test obstructive sleep apnea?:
reverse ARDS
Polysomnography (sleep study)
- tx underlying cause
- multiple episodes of apnea
205.Risk factors for obstructive sleep apnea?: 1. Male gender
2. Obesity
3. Large uvula/tongue
4. Retrognathia (recession of mandible)

You might also like